Boards 2

Pataasin ang iyong marka sa homework at exams ngayon gamit ang Quizwiz!

Enzyme responsible for replication of the adenovirus genome in the host cell?

DNA dependent DNA polymerase Adenovirdae are a family of double stranded DNA viruses that uses DNA dependent DNA polymerase to replicate their genomes in the host cell nucleus

What regular activity is recommended in diabetic patients?

Daily foot inspection (looking for signs of trauma or infection)

Unconscious shifting of emotion or desires associated with a person from the past to another person in the present

Transference

Chemotactic agents of neutrophils?

IL-8, n-formylated peptides, LTB4, 5-HETE, C5a

Osteoarthritis is a chronic condition characterized by the progressive erosion of articular cartilage. Most significant risk factor?

Advancing age Other major risk factors: obesity, female sex, occupational joint loading, past trauma, abnormal joint alignment

Activation of muscarinic M3 receptors on bronchial smooth muscle by acetylcholine activates the Gq-phospholipase C-IP3 pathway which increases intracellular calcium and triggers muscle contraction. Drug that competitively inhibits muscarinic receptors on bronchial smooth muscle, leading to smooth muscle relaxation?

Atropine

3 week old baby boy with fever, irritability, poor feeding. Blood cultures grow gram negative rods that form pink colonies on MacConkey agar. Most important bacterial virulence factor for the development of the patients condition?

Capsule E. coli is a frequent cause of neonatal meningitis, second only to group B strep. E. coli strains that cause neonatal meningitis possess the K1 capsular antigen. K1 capsule is a virulence factor that allows the bacteria to survive in the bloodstream and establish meningeal infection

34 year old known hepatitis B carrier presents with progressive anorexia, nausea, and abdominal pain. His Hepatitis D RNA results come back positive. Patient's chronic infection assists what life cycle aspect of the current infectious agent?

Coating of viral particles The hepatitis B surface antigen of HBV must coat the hepatitis D antigen of the HDV before it can infect hepatocytes and multiply HDV contains a very short circular molecule of single stranded RNA

Patient is diagnosed with methicillin sensitive S. aureus osteomyelitis and antibiotics are started. Single most important measure to reduce risk of transmission to other patients?

Hand hygiene Hand hygiene is the most important measure to reduce the risk of transmission of hospital acquired infections

Long term, heavy alcohol use may be associated with alcoholic cerebellar degeneration, a syndrome characterized by cerebellar vermis atrophy that presents with?

Gait ataxia, truncal instability, and a rhythmic postural tremor of the fingers and hands

Destruction of the sub thalamic nucleus results in?

Hemiballism (involuntary flinging movements of the extremities contralateral to the lesion)

Diabetic ketoacidosis is characterized by polydipsia, polyuria, and a fruity odor to the breath/urine. DKA is associated with elevated anion gap metabolic acidosis that is usually accompanied by compensatory respiratory alkalosis. This combination yields what acid base status?

Low pH, low serum bicarbonate, low PaCO2

Medication that must be avoided in a patient with Parkinson disease who is diagnosed with a bowel perforation with peritonitis?

Metoclopramide (can exacerbate or cause de novo parkinsonian symptoms and trigger extrapyrimidal reactions due to its antagonistic effects on dopamine receptor) NEVER use in patients with Parkinson disease

SVC syndrome results from SVC compression which is usually due to a tumor. Symptoms include face and neck edema and neck vein engorgement. Location of SVC on CT?

In the region of the right heart just lateral to the ascending aorta

CSF grow M. tuberculosis with significantly decreased activity of intracellular catalase peroxidase. They slates would most likely exhibit resistance to what agent?

Isoniazid Mycobacterial resistance to isoniazid can be accomplished through non-expression of the catalase peroxidase enzyme or through genetic modification of the INH binding site on mycolic acid synthesis enzyme

Characterized by recurrent intrusive thoughts (obsessions) and repetitive time consuming rituals (compulsions) that the individual feels driven to perform

Obsessive compulsive disorder DIFFERENTIATE FROM obsessive compulsive personality disorder which involves a lifelong pattern of insistence on control, orderliness, and perfection and DOES NOT involve compulsions perfumed in response to intrusive obsessions

Most notable side effect of ethambutol?

Optic neuropathy which can manifest as decreased visual acuity, central scotoma, or color blindness (may be revered with discontinuation of the drug)

Hormones that utilize the cAMP-protein kinase A pathway upon binding to their receptor?

PTH, ACTH, TSH, ADH (V2 receptor)

Patients with down syndrome, ataxia telangiectasia and neurofibromatosis type 1 have increased risk of what malignancy?

acute lymphoblastic leukemia

Rats subjected to chronic inhalation exposure of formaldehyde develop nasal squamous cell carcinomas. Decreased activity of what gene is likely to be found in the studies of the tumor cells?

p53 p53 is a tumor suppressor gene that controls cell division and apoptosis and its inactivated in many tumors

Most common translocation in childhood B-cell acute lymphocytic leukemia?

t (12,21)

MOA of azole antifungals?

Inhibit 14 alpha demethylase (a CYP 450 enzyme)

Most commonly injured structure in rotator cuff syndrome?

Tendon of the supraspinatous muscle. Since its an abductor of the humerus, injury to its tendon causes pain with abduction of the arm.

Standard error is a measure of variability around the mean. Association with sample size?

The larger the sample size, the smaller the standard error SE=standard deviation/ square root of the sample size

37 year old presents with lower abdominal pain, fever, and chills on postoperative day 5. She underwent a c-section due to fetal concerns at 39 weeks after 20 hours of labor. Patient has a history of chlamydia that was treated with antibiotics 4 years ago. Temperature is 102. PE reveals tender lower abdomen and foul smelling vaginal discharge. Labs reveal leukocytosis with neutrophilic predominance. Most likely site of infection?

Uterus C-section is the most important risk factor for postpartum endometritis due to the introduction of microbial organisms and foreign bodies into the incised uterus. Signs and symptoms of postpartum endometritis include fever, leukocytosis, uterine tenderness, and foul smelling vaginal discharge

56 year old man with history of HTN presents with progressively worsening dyspnea. PE shows bilateral basilar lung crackles, JVD, bilateral LE edema. CXR reveals cardiomegaly and hilarious prominence. ECG shows LVH. Echocardiogram shows pulmonary artery HTN. Most likely mechanism for patients pulmonary artery hypertension?

Vasoconstriction due to pulmonary venous congestion Left sided heart disease can cause pulmonary hypertension by increasing pulmonary venous pressure and congestion. This leads to passive increase in pulmonary artery pressure which is made worse by reactive vasoconstriction and structural remodeling of the pulmonary vasculature secondary to impaired nitric oxide availability and increased endothelin expression

7 alpha helical transmembrane domains which interact strongly with the inner core of membrane phospholipids, most likely contain what amino acids?

Alanine, valine, leucine, isoleucine, phenylalanine, tryptophan, methionine, proline, glycine Transmembrane or membrane associated domains of any protein as composed of the above hydrophobic amino acids Arginine, histidine, lysine- have basic R groups and are positively charged (hydrophilic) Aspartate, glutamate- carboxylic acid R groups and are typically negatively charged (hydrophilic)

Alpha 1 antitrypsin deficiency is an autosomal codominant genetic disorder that can result in panacinar emphysema and liver disease. Most severe consequences of liver involvement?

Cirrhosis and hepatocellular carcinoma

What area of the penis is the primary site of phosphodiesterase-5 inhibitors?

Corpora cavernosa (area labeled 1 in picture) Phosphodiesterase 5 inhibitors (sildenafil) increase intracellular concentrations of cGMP in the vascular smooth muscle of the corpora cavernosa. This leads to smooth muscle relaxation and subsequent engorgement Note: area labeled 2 is the corpus spongiosum which maintains the patency of the urethera during ejaculation

CFTR gene mutations are the most common cause of congenital bilateral absence of the vas deferens. Patients with this condition have azoospermia and infertility but normal levels of FSH, LH, and testosterone. Diagnostic test that will most likely confirm diagnosis?

Elevated swear chloride levels

Patients with chronic kidney disease with mineral bone disease can be asymptomatic or develop weakness, bone pain, and fractures. Usually presents with?

Hyperphosphatemia, secondary hyperparathyroidism, and decreased calcitrol levels

Undifferentiated (anaplastic) tumors bear no resemblance to the tissue of origin. They are composed of pleomorphic (variable shape and size) cells with large, hyperchromatic nuclei that grow in a disorganized fashion. Anaplastic tumors may also contain?

Numerous abnormal mitoses & GIANT TUMOR CELLS

Behavioral disorder of childhood characterized by argumentative and defiant behavior toward authority figures. It doesn't involve the more severe violations of the basic rights of others seen in conduct disorder

Oppositional defiant disorder

Normal cells show contact inhibition in which they reproduce until the petri dish surface is covered with a single layer of cells. Malignant cells do not show contact inhibition and will continue to reproduce indefinitely creating what pattern on the petri dish?

Piles of mounds

Treatment of ascites secondary to cirrhosis involves restriction of Na+ intake combined with diuretics. Most commonly prescribed initial therapy is a combo of furosemide and spironolactone. Locate portal vein on CT

Portal vein can be identified on a cross sectional scan lying medial to (or just within) the right lobe of the liver and anterior to the inferior vena cava.

Benign tumors of skeletal muscle that most commonly occur on the head and neck of younger men. Characterized histologically by fascicles of polygonal cells with a vacuolated cytoplasm and a peripherally placed nucleus

Rhabdomyomas

Earliest sign of alcohol withdrawal?

Tremulousness Other common symptoms: GI distress, agitation, anxiety, and autonomic disturbance. Delirium tremens is the most severe manifestation of alcohol withdrawal and typically begins 48-96 hours after the last drink

Diffusing capacity in intrinsic vs. extrinsic restrictive lung diseases?

intrinsic- reduced diffusing capacity extrinsic- normal diffusing capacity

What occurs after a growth factor binds to a receptor and protein kinase B is activated?

mTOR translocates to the nucleus and to induce genes involved in cell survival, anti-apoptosis, and angiogenesis Mutations in growth factor receptors, Act, mTOR, or PTEN that enhance the activity of this pathway contribute to cancer pathogenesis & drugs targeting this pathway such as sirolimus which inhibits mTOR have shown benefit in treating certain cancers

Infant born to a 26 year old woman is found to have a protruding tongue, excessive skin at the nape of the neck, and up slanting palpebral tissues. Startle reflex is symmetric and weak. Cardiac auscultation reveals a harsh III/IV systolic murmur heart best over the left lower sternal border. Mother has a history of 2 spontaneous abortions in the past three years. Echocardiography revels VSD. Karyotype most likely to be found in this patient?

46, XX, t(14,21) Unbalanced robertsonian translocations account for a minority of down syndrome cases. Affected parent is asymptomatic because they have a normal (just rearranged) genetic component (balanced translocation). When an ovum containing the translocated chromosome and a normal chromosome 21 is fertilized with a sperm containing a normal test of chromosomes, the resultant fetus has an UNBALANCED robertsonian translocation with 46 chromosomes and 3 effective copies of chromosome 21. Another possible maternal gamete includes 0 copies of chromosome 21 which increases the rate of miscarriages in balanced robertsonian translocation carriers. Epicanthal folds, upslanting palpebral fissures, a protruding tongue, and excessive skin at the nape of the neck are characteristic of down syndrome. Birth weight and length are usually below average and hypotonia and a weak startle reflex (Moro reflex) are characteristic. Cardiac abnormalities are present in 50%

What curve on image best corresponds to lung cancer?

A- lung cancer B- breast C- colon D- pancreas E- stomach Between 1950 and 2000, rising rates of tobacco use resulted in an increase in female lung cancer incidence and mortality. Lung cancer is currently the most common cause of cancer death in both women and men in the USA

1 month old full term boy brought to ED by mother due to difficulty awakening him. She says she left the infant alone for a few minutes this morning and he rolled off the bed. Since then he's been sleepy and less active. PE shows lethargic boy with full anterior fontanelle. Bilateral retinal hemorrhages are seen on fundoscopy. What should be strongly suspected in this patient?

Abusive head trauma Abusive head trauma can be caused by vigorous shaking of an infant and results in subdural hemorrhage (due to tearing of the bridging veins) and retinal hemorrhages. Infants have larger heads, enlarged subarachnoid spaces, higher brain water content, and decreased cervical muscle tone leading to increased movement of the immature brain in relation to the skill. Posterior rib fractures are also alarming by a perpetrators grasp of the torso. Retinal hemorrhages are HIGHLY suggestive of AHT due to rupture of congested retinal veins. Pay attention to injuries inconsistent with history or developmental age (rolling over isn't typical before 4 months). Fall from a bed wouldn't result in retinal hemorrhages or a large intracranial hemorrhage . Note: retinal hemorrhages aren't seen with vitamin K deficiency

23 year old woman found to have lymphocytes that contain a mutated and functionally defective Fas gene product. What immunologic mechanism is most likely impaired as a result of this defect?

Activation induced T lymphocyte death The Fas receptor acts to initiate the extrinsic pathway of apoptosis. Mutations involving the Fas receptor or Fas ligand can prevent apoptosis of auto reactive lymphocytes, increasing the resk of autoimmune disorders.

6 year old boy presents after traumatic fall from bicycle. Further testing shows lipoprotein lipase activity measured after IV heparin administration is substantially lower than normal. If condition remains untreated, what is he at most risk of developing?

Acute pancreatitis Familial chylomicronemia syndrome (type 1 hyperlipoproteinemia) presents in childhood with recurrent episodes of acute pancreatitis, marked hypertrigyceridemia, lipemia retinalis (milky appearing retinal vasculature), and eruptive skin xanthomas (yellow papule surrounded by erythema found mainly on extensor surfaces). Patients with the disorder are not usually at increased risk for premature CAD.

Self limiting year round infection that causes acute onset of fever, cough, congestion, pharyngitis, and conjunctivitis. This pharygoconjunctival disease can occur in outbreaks among those living in close quarters (camp dormitories, military barracks)

Adenovirus

Fibrosarcomas are fibroblast based malignancy histologically characterized by?

Anaplastic spindle cells that may grow in an organized herringbone or a disorganized cellular fashion; marked disarray, pleomorphism, mitotic activity, hemorrhage, and necrosis are present

16 year old boy with progressive confusion and agitation for the past several hours. Started acting bizarre after chewing some plant seeds. Patient said bugs were crawling on his skin and he was talking to people who weren't here. Patient reports trouble seeing clearly. Frequently asks for water. Patients presentation is most similar to an overdose of what drug?

Atropine Anticholinergic intoxication causes fever, dry skin and mucous membranes, flushing, blurred vision and altered mental status. Jimson weed (Datura stramonium) is a drug of abuse that can cause anticholinergic syndrome (contains atropine, hyoscyamine, scopolamine)

Smooth muscle cell antibodies are commonly seen in patients with?

Autoimmune hepatitis

54 year old man with blood in urine and abdominal discomfort for the past 2 days. PMH includes HTN. PE is unremarkable. CT abdomen shows bilaterally enlarged kidneys with multiple cysts. Mode of inheritance of this patients condition?

Autosomal dominant ADPKD is the most common hereditary cause of renal failure in adults. A positive family history and numerous kidney and liver cysts on imaging (seen by the 3rd to 4th decade) help make the diagnosis

56 year old with advanced HIV infection presents with fever, weight loss, diarrhea, and weakness. Symptoms started several weeks ago and have progressively worsened. On PE, liver and spleen are enlarged and skin pallor is present. Labs show CD4+ count of 40 & elevated alkaline phosphatase and LDH. CXR is normal. CT of abdomen shows hepatosplenomegly and retroperitoneal lymphadenopathy. Blood cultures are positive for colonies of acid fast bacteria that grow optimally at 41 C. Patients current condition could have been prevented through prophylactic use of what medication?

Azithromycin Mycobacterium avid complex is a common opportunistic pathogen that causes disseminated disease in HIV patients. Patients with CD4 counts less than 50 should be administered prophylactic azithromycin weekly to prevent MAC infection. MAC typically presents with nonspecific symptoms of fever, weight loss, and diarrhea in an HIV patient. Marked anemia, hepatosplenomegaly, and elevated alkaline phosphatase and LDH are common with MAC due to widespread involvement of the reticuloendothelial system. MAC grows well at high temperatures and will exhibit optimum growth at 41 C. Disseminated infection is treated with Clarithromycin or azithromycin in combination with rifabutin or ethambutol.

35 year old man with nontender cervical lymphadenopathy that he first noticed while shaving. Biopsy is performed and reveals aggregates of packed follicles that obscure the normal lymph node architecture. Cytogenetic analysis would most likely demonstrate?

BCL-2 overexpression Follicular lymphoma is a B cell tumor characterized by aggregates of packed follicles that obscure the normal lymph node architecture. 90% of patients with it have t(14,18) translocation which causes over expression of antiapoptotic BCL-2 protein. Its composed predominantly of centrocytes (small cleaved cells) and fewer number of centroblasts (large noncleaved cells).

Coarse erythrocyte basophilic stippling and microcytic hypo chromic anemia are common peripheral blood smear findings in lead poisoning. High risk groups are young children ingesting paint chips and industrial workers inhaling particulate lead. What causes basophilic stippling and hypo chromic, microcytic anemia?

Basophilic stippling results from the abnormal degradation of rRNA (due to lead induced inhibition of a nucleotidase) & microcytic hypochromic anemia results from inhibit of ALA-dehyratase and the resultant reduced incorporation of iron into heme

Profound cerebral hypoperfusion may lead to global cerebral ischemia (ischemic hypoxic encephalopathy). Watershed infarcts occur between the zones of perfusion of the anterior, middle, and posterior cerebral arteries. These infarcts typically appear as?

Bilateral wedge shaped strips of necrosis over the cerebral convexity, parallel and adjacent to the longitudinal cerebral fissure

45 year old with several months of episodic right upper quadrant abdominal pain and nausea. Pain is brought on by fatty meals and subsides in a few hours. A cholecystokinin stimulation test shows slow and incomplete gallbladder emptying. Patient is most likely to have what pathologic finding?

Biliary sludge The gallbladder functions actively to absorb water from bile. Gallbladder hypomotolity causes the bile to become concentrated, promoting precipitation and accumulation of particulate material. This forms a viscous biliary sludge that can cause transient bile duct obstruction (biliary colic) and promote cholesterol gallstone formation.

Patient noted to have several subcutaneous bumps attached to achillies tendon. Father died of MI at 35 and mother suffers from RA. Her aunt was diagnosed with colon CA at 55. Best initial test in this patient?

Blood cholesterol Xanthomas are suggestive of hyperlipidemia, especially when present in conjunction with a family history of early cardiac death. Histologically they are composed of benign macrophages packed with finely vacuolated 'foamy' cytoplasm. The cytoplasm contains high levels of cholesterol, phospholipids, and triglycerides. The lipid laden macrophages are frequently enclosed by inflammatory cells and fibrotic stroma (except xanthelasmas)

Autosomal dominant condition associated with mutations in cardiac sodium or L-type calcium channels leading to characteristic ECG changes (pseudo RBBB, ST segment elevation in leads V1-V3) & increased risk of ventricular arrhythmias and sudden cardiac death

Brugada syndrome

Patient with a history of PED and GERD diagnosed with gout and started on a medication that selectively binds to an interleukin-1 inducible enzyme that is highly expressed by inflammatory cells and undetectable in the surrounding normal tissue. Most likely drug used in this patient's treatment?

Celecoxib COX-2 is an inducible enzyme unregulated during inflammation by IL-1 and TNF-alpha. Selective COX-2 inhibitors (celecoxib) decreased inflammation by inhibiting COX-2 production of pro-inflammatory arachadonic acid metabolites. Because they don't affect COX-1, they have minimal GI toxicity

Can form in the middle ear after hemorrhage & appear as bluish-black gelatinous material behind the tympanic membrane

Cholesterol granulomas

Characterized by a consistent delay in the time of sleep onset relative to the daily light dark cycle. It results in excessive daytime sleepiness in an otherwise healthy individual when social demands require waking before the delayed sleep cycle can be completed

Circadian rhythm disorder

Involves a persistent pattern of violating major societal norms or the rights of others. Behaviors include aggression toward people and animals, deceitfulness or theft, destruction of property, and serious violation of rule.

Conduct disorder Note: children with conduct disorder have a greater risk for developing antisocial personality disorder as adults; antisocial personality disorder can't be diagnosed before age 18

56 year old man with HTN, T2DM, hyperlipidemia and intermittent mild asthma. What vascular bed is most likely to carry the highest atherosclerotic burden in this patient?

Coronary arteries Atherosclerosis is a pathophysiologic process involving endothelial cell dysfunction and it develops most rapidly in areas with bends and branch points that encourage turbulent flow. The lower abdominal aorta and coronary arteries are the vascular beds most susceptible to atherosclerosis; they tend to develop atherosclerosis earliest in life and have the highest overall atherosclerotic burden

19 year old boy with indurated, painless ulcer near the glans penis without surrounding erythemal or inguinal lymphadenopathy. First line treatment for this patient has structural similarities with?

D-alanine-D-alanine Patient likely has a chancre due to primary syphillis. Penicillins work by binding and covalently inhibiting transpeptidase (the enzyme that catalyzes the cross linking step in peptidoglycan cell wall synthesis). Penicillins, structurally similar to D-alanine-D-alanine, inhibit transpeptidase by binding covalently to its active site. The result is failed synthesis of the bacterial peptidoglycan cell wall Note: vancomysin, a glycopeptide antibiotic, directly binds to D-alanine-D-alanine residues preventing the incorporation of new subunits into the cell wall. It acts at an earlier stage of cell wall formation than do penicillins

28 year old with lump in right breast. Mother died of ovarian cancer at 34 and aunt died of breast cancer at 32. Patient is diagnosed with breast cancer. She most likely inherited a mutation in a gene normally responsible for what process?

DNA repair Hereditary breast cancer is most commonly associated with mutations in BRCA1 and BRCA 2. These tumor suppressor genes are involved in DNA repair of double stranded breaks, and their mutation increases the risk of developing breast & ovarian cancer. They are inherited in autosomal dominant manner with variable penetrance. Note: HER2 mutations that cause breast cancer are acquired, not inherited.

In fetal circulation, blood from the placenta flows through the umbilical vein into the liver. At the liver, blood may either be delivered to the hepatic sinusoids or diverted to the IVC via the?

Ductus venosus

30 year old with history of IV drug use and known HIV presents with increasing abdominal dissension and anorexia. CT shows ascites and large mass surrounding the small intestine. Biopsy reveals uniform, round, medium sized tumor cells with basophilic cytoplasm and proliferation fraction (Ki-67) of > 99%. What infectious agent is most closely associated with the development of his current condition?

Epstein-Barr Virus EBV genome is identified in 50% of systemic B cell lymphomas and almost all primary CNS lymphomas occurring in the setting of HIV infection. A high mitotic index is typical of Burkitt lymphoma. Chronic EBV infection increases B cell proliferation and is thought to increase the risk of a c-myc translocation.

Dopamine agonist that can used to treat Tourettes syndrome?

Fluphenazine

A small area of the brain of a 54 year old male shows neuronal shrinkage and intense cytoplasmic eosinophilia. If the patient survives, eventually the area would most likely demonstrate?

Glial hyperplasia Proliferation of astrocytes in an area of neuron degeneration is called gliosis. It leads to the formation of a glial scar which compensates for the volume loss that occurs after neuronal death

24 year old man with episodic anemia, jaundice, and dark urine. Episodes occur after use of certain drugs or infection. Has a brother with similar episodes. Blood smear reveals bite cells. Deficiency of what enzyme results in a condition similar to this patients presentation?

Glutathione reductase G^PD causes hemolytic anemia and jaundice secondary to increased oxidative stress due to the lack of NADPH. Glutathione reductase deficiency has a similar clinical consequence as its absence results in inability to utilize NADPH to reduce glutathione

Patient with myasthenia graves is started on appropriate treatment and reports significant symptom improvement but now has abdominal cramping, nausea, sweating and diarrhea. What agent can be used to effectively control these new symptoms?

Glycopyrrolate The treatment of myasthenia gravis involves the use of a cholinesterase inhibitor, immunosuppressants, and possible thymectomy. Cholinesterase inhibitors may cause adverse effects related to muscarinic overstimulation which can be ameliorated by the use of a selective muscarinic antagonist such as glycopyrrolate, hyoscyamine, or propantheline

Androgens synthesized in the ovaries are converted to estradiol in what cell type?

Granulosa cells Androgens and progesterone are synthesized from cholesterol in the theca internal cells under the influence of LH. Androgens released form theca internal cells migrate into the nearby granuloma cells, which contain the enzyme aromatase. Aromatase converts androgens into estradiol under the influence of FSH. Theca externa is composed of a layer of smooth muscle and fibroblast cells; it servers as a connective tissue support structure for the follicle

32 year old woman with fatigue and malaise over last several days. No sick contacts. TTP is suspected. What is essential in making this diagnosis?

Hemolytic anemia TTP is one of several primary thrombotic microangiopathies resulting in microangiopathic hemolytic anemia and thrombocytopenia, which are essential for the diagnosis. Significantly reduced ADAMTS13 activity can help confirm diagnosis. Plasma exchange therapy has prompted more rapid diagnosis and is life saving.

23 year old woman suffering from chronic diarrhea now has a draining abscess. Patient has malaise and significant pain originating from the site. Temperature is 101. PE reveals mouth ulcers and an area of fluctuant with overlying erythema near the perineum. Biopsy of what area would be most helpful in diagnosing her underlying condition?

Ileum Crohn disease is a chronic, idiopathic, inflammatory condition that most frequently involves the terminal ileum and presents with abdominal pain, diarrhea, and low grade fever. Other distinguishing features are aphthous ulcers of the mouth and perianal disease (fissures, abscesses)

Electrolyte differences in the body fluids of patients with CF develop due to tissue specific variation in the functioning of the CFTR gene. Difference in sweats glands vs. respiratory epithelium?

In sweat glands mutated CFTR prevents salt reabsorption producing sweat that has higher levels of Na+ and Cl-. In respiratory epithelium, mutated CFTR causes thick, viscid secretions that are low in both Na+ and Cl-.

29 year old with severe right sided pelvic pain. Pain is sharp and came on suddenly. Its accompanied by nausea and is worse with movement. Pelvis US reveals a right adnexal mass measuring 6cm with no blood flow to the ovary. Patients condition most likely involves what structure?

Infundibulopelvic ligament Ovarian torsion typically involves twisting of the infundibulopelvic ligament, often due to the weight of a large adnexal mass. The resulting occlusion of the blood and nerve supply to the ovary results in severe acute pelvic pain and ovarian ischemia.

MOA of cyclosporine?

Inhibits calcineurin, decreasing transcription of IL-2 gene in T lymphocytes in response to antigenic stimulation

Orthostatic hypotension is defined as a decrease in systolic BP of at least 20 or decrease in diastolic of at least 10 after moving from supine to standing. This is a common cause of syncope in the elderly. A major cause of postural hypotension is?

Insufficient stimulation of vascular alpha 1 adrenergic receptors upon standing

Herpes zoster (shingles) develops due to reactivation of varicella zoster virus in the dorsal root ganglia (sensory neurons). It presents with painful vesicular rash in a dermatomal distribution. What is seen on LM?

Intranuclear inclusions in keratinocytes and multinucleate giant cells

Most significant of hydroxychloroquine (well tolerated antirheumatic drug used in RA and SLE)?

Irreversible retinopathy (patients should have regular ophthalmologic exams)

Phenomenon in which brief repetitive episodes of myocardial ischemia, followed by reprofusion, protect the myocardium from subsequent episodes of ischemia (repetitive episodes of angina prior to MI can delay cell death)

Ischemic preconditioning

64 year old man with 2 days of progressive knee pain. Says he bumped his knee on a chair and it started to cause knee swelling. Arthrocentesis shows multiple neutrophils with cocci. Most likely cause of patient's symptoms?

Joint infection The presence of polymorphonuclear leukocytes and bacteria in the joint space indicates infection. Some of the most common causative organisms of infectious arthritis are S. aureus and N. gonorrhea. Note: Children younger than 2 are especially susceptible to H. influenzae arthritis.

Hemochromatosis is an AR disease characterized by excessive GI absorption of iron. Common manifestations include?

Liver disease with hepatomegaly, DM secondary to pancreatic islet destruction, arthropathy, pituitary hormone deficiencies, hyperpigmentation, cardiomyopathy

Blunt trauma to the globe can cause orbital blowout fractures. These fractures most commonly involve which orbital walls?

Medial or inferior orbital walls due to the thin bone bordering the ethmoid and maxillary sinuses

Groups results of several trials to increase statistical power and provide an overall pooled effect estimate

Meta-analysis Note: the null value for a ratio (odds ration or relative risk) is 1 & the null value for a difference is 0 --> not significant if the two values cross the null value

What cells are primarily responsible for the uptake of Shigella?

Microfold M cells Shigellosis is an infectious disease that can be caused by a variety of Shigella species; S. sonnei is the most common cause of shigellosis in industrialized nations. Shigella invades the GI mucosa by gaining access to microfilm cells in ileal Peyer patches through endocytosis. Shigella subsequently lyses the endoscope and spreads laterally into other epithelial cells, causing cell death and ulceration with hemorrhage and diarrhea

Best way to exclude an asthma diagnosis?

Negative methacholine challenge Bronchial challenge testing is highly sensitive but nonspecific measure with a high negative predictive value that can help exclude a diagnosis of asthma. A provocative stimulus (typically aerosolized methacholine) is administered at increasing concentrations to induce bronchoconstriction. Patients with asthma are hyperresponsive to this stimulus and experience a decline in FEV1 at lower doses than non asthmatics

ANP is secreted by artrial cardiomyocytes in response to stretch induced by HTN or hyervolemia. ANP causes peripheral vasodilation and increased urinary excretion of Na+ and H2O. Drug class that prevent degradation of ANP, enhancing its beneficial hemodynamic effects in heart failure patients?

Neprilysin inhibitors (sacubitril)

38 year old Asian women with intermittent abdominal pain and bloating since childhood. Doesn't drink milk because eating anything dairy upsets her stomach. LM of a biopsy sample of this patients intestinal mucosa would most likely show?

Normal intestinal mucosa Small bowel mucosa of patients with primary lactase deficiency is normal on histology

Patient is started on a medication that causes a QRS prolongation with only minimal increase in QT interval duration. What region on the action potential curve is most affected by this drug?

Phase 0 Class IC anti arrhythmic agents (flecanide, propfenone) block the fast Na+ channels responsible for ventricular depolarization (phase 0), prolonging QRS duration with little effect on QT interval. Class IA and class III agents cause the most QT interval prolongation.

Isotretinoin is used to treat severe acne with significant scarring or patients who have failed therapy with topical tretinoin/benzoyl peroxide and antibiotics. Absolute contraindication to this medication?

Pregnancy due to risk of teratogenicity Sexually active women must be advised to use 2 forms of contraception and take monthly pregnancy tests

Regenerative nodules seen in liver cirrhosis are composed of?

Proliferating hepatocytes (located within the confines of fibrous septae)

44 year old man with history of heavy alcohol consumption with nosebleed after getting into a fight while intoxicated. PE shows distended periumbilical veins, ascites, and flapping hand tremor on wrist extension. What lab finding would be most indicative of a poor prognosis for this patient?

Prolonged prothrombin time Increased AST and ALT are indicators of hepatocellular damage, and increased alkaline phosphatase and gamma-glutamyl transpeptifase indicate biliary injury. Serum albumin levels, bilirubin levels, and prothrombin time are reflective of liver function and are of greatest prognostic significance in patients with cirrhosis.

Patient develops an infection that spreads to the psoas major muscle. Identify this muscle on CT.

Psoas muscle originates from the anterior surface of the transverse processes and lateral surface of the vertebral bodies at T12-L5. It acts primarily to flex the thigh at the hip and contributes somewhat to lateral rotation and abduction of the thigh

Relative risk equation?

RR= risk among exposed/risk among unexposed RR= [a/(a+b)]/[c/(c+d)]

Drug eluting stents contain cytostatic drugs (mTOR inhibitors- everolimus, sirolimus, zotarolimus). Purpose of this?

Reduce smooth muscle proliferation; helps prevent neointimal hyperplasia and luminal restenosis following intracoronary stent placement

Most gastric ulcers arise along the lesser curvature of the stomach, usually at the transitional zone between the gastric corpus (body) and antrum. Arteries that run along the lesser curvature and are likely to be penetrated by ulcers causing gastric bleeding?

Right & left gastric arteries

Cardiac monitor shows normal sinus rhythm. What is the most likely earliest site of electric activation?

Right atrium near the opening of the superior vena cava The sinoatrial node consists of specialized pacemaker cells located at the junction of there right atrium and superior vena cava. It is the site of earliest electrical activation in patients with sinus rhythm

54 year old man with episodic burning substernal chest pain. S4 is heard on auscultation. ECG at rest shows LVH. A myocardial perfusion scan reveals inducible ischemia on the inferior surface of the heart. What coronary artery is most likely occluded in this patient?

Right coronary artery Inferior wall of the left ventricle forms most of the inferior (diaphragmatic) surface of the heart and is suppled by the posterior descending artery. In 85-90% of individuals, the posterior descending artery derives from the right coronary artery (right dominant coronary circulation)

First line treatment for generalized anxiety disorder?

SSRI and SNRI Benzos should be limited to short term use for non depressed patients with no history of substance abuse who respond poorly to antidepressants

Neuroepileptic malignant syndrome is a life threatening adverse reaction to antipsychotic medications characterized by diffuse muscle rigidity, high fever, autonomic instability (HTN, tachycardia), and altered sensorium. Labs reveal elevated creatine kinase due to rhabdomyolysis. Treatment?

Stop the antipsychotic and provide supportive care & administer dantrolene (inhibits calcium ion release from the SR of skeletal muscle) to reduce the muscle rigidity Note: bromocriptine (a dopamine agonist) has also shown clinical benefit

Difference in lepromin skin test results for tuberculoid and lepromatous leprosy?

The lepromin skin test will be positive in patients with tuberculoid leprosy as they exhibit a strong CD4 Th1 cell mediated immune response to Mycobacterium leprae. However, patients with lepromatous leprost will test negative due to their weak Th1 cell mediated immune response

4 day old infant with abnormal movements. Had had intermittent episodes of tonic posturing over the past 3 hours and poor feeding, vomiting, and irritability for the past 2 days. Mom says diapers smell like sugar. In addition to appropriate dietary restriction, what may improve his condition?

Thiamine Branched chain alpha ketoacid dehydrogenase requires several coenzymes : thiamine, lipoate, coenzyme A, FAD, NAD. Some patients with MSUD improve with high dose thiamine, but most require lifelong restriction of leucine, isoleucine, and valine Mutations in BCKDC results in accumulation of branched chain amino acids in serum and peripheral tissues resulting in neurotoxicity that includes seizures, irritability, lethargy, and poor feeding. A metabolite of isoleucine gives the urine a distinctive sweet odor. This disorder can be life threatening if untreated as brain swelling can lead to death

Congenital deficiency of factor XII (Hageman) causes marked PTT prolongation without bleeding diathesis. Patients instead have a tendency for?

Thromboembolic complications Factor XII is a plasminogen activator and is needed to dissolve clots.

Pain purpose of blinding in clinical trials?

To prevent patient or researcher expectancy from interfering with outcome

Etopiside killed cancer cells show a high number of double stranded DNA fractures, which are due to dysfunction of?

Topoisomerase II Etoposide is a chemotherapeutic agent that inhibits the sealing activity of topoisomerase II. Treatment with etopiside causes chromosomal breaks to accumulate in dividing cells, causing cell death. Used in testicular cancer and small cell lung cancer. Note: ironotecan and topotecan inhibit topoisomerase I

Receptor is of the epidermal growth factor receptor family that has intrinsic tyrosine kinase activity. What is most likely immediate effect of the ligand binding to the receptor?

Transient dimerization Ligand binding to a tyrosine kinase receptor results in formation of receptor dimers which each phosphorylate the other, and a conformational change that exposes tyrosine kinase active sites. Hormones that use tyrosine kinase receptors: insulin, EDGF, PDGF, VEGF

Medication most helpful in treating both acute seizure disorder and bipolar disorder over long term?

Valproate Lithium and the anticonvulsants valproate, carbamazepine, and lamotrigine are mood stabilizing agents used in bipolar disorder. Of these, valproate is the most commonly used for seizure prophylaxis. It blocks voltage gated Na+ channels and enhances GABA synthesis and release. Note Carbamazepine is effective in treatment of mania but it is a CYP450 inducer that can lower the levels of contaminant medications.

64 year old man with angina is being treated with atenolol and aspirin. He reports that his symptoms have been worsening over the last week. His physician decides to add a new medication. Several days later he returns complaining of severe dizziness. On PE BP is 100/70 and HR is 38. What medication was most likely administered?

Verapamil Combined used of non-dihydropyrimidine calcium channel blockers (dilimiazem, verapamil) and beta adrenergic blockers (atenolol) can have additive negative chronotropic effects yielding severe bradycardia and hypotension

You are caring for a 78 year old man in ICU. He is taken for emergent cardiac cath. While speaking to patient he says he is afraid of dying during the procedure and asks you to pray for him. Assuming your religious beliefs are different from those of your patient, whats the most appropriate reply?

'I understand that you are afraid and that your beliefs are important. I will keep you in my thoughts during your procedure'. When an ill patient requests your prayers in an acute setting it is appropriate to offer your personal support without interjecting your personal beliefs into the interaction. The overriding goal in these situations is to 'do no harm'. This can be achieved by not disagreeing with the patient, not entering into a religious debate with them, and not displacing responsibility for the care of the patient onto others.

Patient approaches your table and has a conversation with your colleague whom you are eating with and nods briefly at you. Afterward your colleague asks you how you know her saying she is a family friend who was recently diagnosed with breast cancer. He says he has been worried about her and asks if he can assume she is a patients of yours and ask hows she's doing. Most appropriate response?

'No I cannot say whether she is my patient' Its unethical to discuss any information regarding a patient's diagnosis and treatment with another individual, including a physician who isn't involved in the patients care. Likewise, the physician should neither confirm nor deny whether the person of interest is a patient

Psoriasis presents with well demarcated, scaly erythematous plaques involving the extensor surfaces of the extremities. Frequent additional complications of psoriasis?

-Psoriatic arthritis -Yellow-brown discoloration, pitting, thickening, or crumbling -Conjunctivitis, blepharitis, uveitis

Absolute contraindications to the use of oral contraceptives?

1. Prior history of thromboembolic event or stroke 2. History of an estrogen-dependent tumor 3. Woman over 35 who smoke heavily 4. Hypertriglyceridemia 5. Decompensated or active liver disease (would impair steroid metabolism) 6. Pregnancy

Buproprion vs. Buspirone?

Buproprion- NE and dopamine reuptake inhibitor used for major depressive disorder and smoking cessation (can worsen anxiety due to activating effects) Buspirone- non-benzo anxiolytic that is second line treatment for general anxiety disorder

28 year old man with 3 month history of low back pain and morning stiffness. PE shows limited anterior flexion of the spine. X ray shows narrowing of sacroiliac joints. What is most strongly associated with his disease?

A specific human leukocyte antigen class I serotype The seronegative spondyloarthropathies include ankylosing spondylitis, reactive arthritis, psoriatic arthritis, and arthritis associated with IBD. Individuals expressing HLA B27 are at increased risk for the seronegative spondyloarthropathies.

PID is most frequently caused by N. gonorrhoeae and C. trachomatis. Severe or inadequately treated PID can result in fallopian tube scarring, which can lead to infertility. Treatment must always include coverage of both organisms with?

A third generation cephalosporin (ceftriaxone) as well as azithromycin or doxycycline

How would an arteriovenous shunt affect the left ventricular pressure volume loop?

AV shunts can be congenital or acquired; acquired forms can result form medical interventions or penetrating injuries. AV shunts increase preload and decreased after load by routing blood directly from the arterial system to the venous system, bypassing the arterioles.High volume AV shunts can eventually result in high output cardiac failure. Physical exam may reveal a pulsatile mass with a thrill on palpation. Auscultation reveals a constant bruit over the site.

40 year old male dies in MVC. Had been a 1 PPD smoker and was moderately overweight. His family history is significant for a MI in his father at 48 and a stroke in his mother at 58. What vessel is most likely to show atherosclerotic involvement in this patient at autopsy?

Abdominal aorta Atherosclerotic plaques develop predominantly in large elastic arteries (aorta, carotid, and iliac arteries) and in large or medium sized muscular arteries (coronary, popliteal arteries). In humans the MOST HEAVILY involved vessel is the abdominal aorta followed by the coronary arteries, popliteal arteries, internal carotids, and circle of willis

56 year old man presents with skin lesions as shown in the image (areas of hypopigmentation). PMH is significant for autoimmune thyroiditis and mild HTN. The hypopigmented lesions are most likely to demonstrate?

Absence of melanocytes in the skin Vitiligo is a common condition characterized by the loss of epidermal melanocytes. Histology shows a complete absence of melanin pigment. There is a significant correlation between vitiligo and autoimmune disorders. Note: individuals with albanism have melanocytes that don't produce melanin because of absent or defective tyrosinase

Develops on chronically sun exposed areas of the skin in predisposed individuals. Lesions consist of erythematous papule with central scale and a rough 'sandpaper' like texture. Premalignant lesions and have potential to progress to squamous cell carcinoma

Actinic keratosis Note: seborrheic keratoses occur in elderly individuals with a 'stuck on', deeply pigmented or flesh colored lesion with a velvety or 'greasy' surface

56 year old man with worsening lower extremity weakness. Abdominal CT shows an advanced bladder tumor compressing a nerve that passes through the obturator canal. What action is likely to be impaired in this patient?

Adduction of the thigh The obturator nerve (L2-L4) is the only major nerve that exits the pelvis through the obturator foramen. Nerve injury typically results from compression (pelvic trauma, surgery, tumor) and presents with weakness on thigh adduction and sensory loss over the distal medial thigh

Typically presents insidiously with memory loss (especially of RECENT events), executive dysfunction (difficulty planning and organizing), and visuospatial impairment (getting lost in own neighborhood). Language and behavioral abnormalities appear later in the disease course. Classic histopathologic findings include amyloid plaques (central amyloid beta core surrounded by dystrophic neurites) and neurofibrillary tangles (aggregates of hyperphosphorylated tau protein)

Alzheimer dementia

Receiver operating characteristic curves can be produced by plotting the sensitivity (true positive rate) against 1- specificity (false positive rate). The accuracy of the test is represented by?

Area under the curve

During an experiment a researcher cuts unmyelinated nerve fibers. These fibers are most likely a component of what?

Autonomic postganglionic nerve Unmyelinated nerve fibers conduct action potentials more slowly than do myelinated nerve fibers. Among primary sensory (afferent) fibers, those responsible for slow, dull, burning or visceral pain, heat sensation, and olfaction are unmyelinated. Efferent neurons axons tend to be myelinated with the notable exception of the unmyelinated fibers that originate from postganglionic autonomic neurons. Unmyelinated neurons are referred to as group C nerve fibers

Pineal tumors (pinealomas) may cause Parinaud syndrome due to mass effect on the dorsal midbrain in the superior colliculus region. Typical findings include?

Upward gaze palsy, absent pupillary light reflex, and impaired convergence

Diabetic patients often need 2 types of insulin, a basal long acting insulin and a postprandial short acting insulin. Compare different insulins.

Best basal long acting insulins are glaring and detemir (administered as once daily shots). NPH is good for about 18 hours (shots twice a day) Best short acting insulins are lispro, apart, glulisine (shots given 3 times a day with meals). They have a very rapid onset of action with peak effects coinciding with peak postprandial hyperglycemia.

Drugs that have been shown to improve long term survival in patients with HF due to LV systolic dysfunction?

Beta blockers, ARBs, ACE inhibitors, aldosterone antagonists Note: beta blockers decrease myocardial work and oxygen demand by slowing the ventricular rate and reducing contractility; they also lower the PVR (after load) by decreasing circulating levels of vasoconstriction hormones (endothelin, renin).

Aminoglycosides inhibit bacterial protein synthesis by binding the 16S ribosomal RNA within the 30S ribosomal subunit. The concurrent use of what drug can improve amino glycoside penetration into the bacterial cell?

Beta lactams (disrupt cell wall synthesis)

Agatroban has what mechanism of action?

Binds to thrombin active site Direct thrombin inhibitors (hirudin, lepirudin, agatroban) dont require antithrombin III for their action and are the drug of choice for treatment of heparin induced thrombocytopenia. Patients with HIT need ongoing anticoagulation due to the presence of or possibility of thrombosis. Upon clinical suspicion of HIT, the most important initial step is to stop all forms of heparin. HIT is caused by antibodies to heparin and platelet factor IV. Both high molecular weight heparin and LMWH should be avoided in these patients.

Pufferfish poisoning is caused by tetrodotoxin, a neurotoxin produced by microorganisms associated with the fish. MOA of tetrodotoxin?

Bings to voltage gated Na+ channels in nerve and cardiac tissue, preventing Na+ influx and depolarization. Symptoms of tetrodotoxin include dizziness, weakness, loss of reflexes, paresthesias of the face and extremities, nausea, vomiting and diarrhea. Higher exposures can cause severe hypotension and general paralysis & death can occur from respiratory failure and hypotension Treatment: supportive care & intestinal decontamination with gut lavage and charcoal

Tetrodotoxin, a potent neurotoxin found in pufferfish, functions by?

Blocking voltage gated sodium channels in nerve cell membranes; inhibits passive transport of Na+

36 year old woman diagnosed with pulmonary artery HTN is scheduled for a lung transplant. What medication is indicated for management of her condition during the waiting period?

Bosentan (competitive antagonist of endothelin receptors used for treatment of idiopathic pulmonary arterial HTN). It decreases pulmonary arterial pressure and lessens the progression of vascular and right ventricular hypertrophy

64 year old man with skin blisters on his trunk and groin. Patient had itching for the past several weeks and developed blisters a week ago. Exam shows numbers .5 to 5cm bullous skin lesions. Most likely diagnosis?

Bullous pemphigoid Characterized by autoantibodies against hemidesmosomes alongg the basement membrane of the dermal-epidermal junction. Causes the entire epidermis to separate from the dermis forming tense, subepidermal blisters

Patient who takes oral morphine sulfate for his back pain is scheduled to undergo surgical decompression. What drug may precipitate opioid withdrawal symptoms if used in this patient?

Buprenorphine Buprenorphine is a partial opioid receptor agonist that binds with high affinity but has low intrinsic activity. In patients on long term opioid therapy, buprenorphine can displace other opioids and precipitate withdrawal in opioid tolerant patients with chronic pain

Antidepressants that don't cause sexual side effects?

Buproprion and Mertazapine

Characterized by fibroblasts in a cartwheel or storiform pattern and supernummerary ring chromosomes that contain sequences from chromosomes 17 and 22

Dermatofibrosarcoma protuberans

Fever, pharyngitis, lymphadenopathy, hepatosplenomegaly, atypical lymphocytosis and positive heterophiles antibody reaction characterize infectious mononcleosis, caused by EBV. Cancers EBV is associated with an increased incidence of?

Burkitt lymphoma & nasopharyngeal carcinoma

Spinal nerve roots that innervate biceps and brachioradialis reflexes?

C5-C6

Primary TB infection is marked by initial uncheck M. Tuberculosis replication within the alveolar space and alveolar macrophages. After a few weeks what changes?

CD4 lymphocytes are stimulated to release IFN-gamma which activates macrophages and leads to control of the infection

35 year old African American female with exertion dyspnea and cough. CXR shows bilateral hilariously aden-patchy and reticular pulmonary infiltrates. Serum ACE levels and calcium levels are elevated. What cell type is most likely to predominant in the patients bronchoalveolar lavage fluid?

CD4+ lymphocytes CD4+ T helper cells are predominant type of lymphovyte found in sarcoid granulomas. Intraalveolar and interstitial accumulation of CD4+ T cells in sarcoidosis often results in high CD4+/CD8+ T cell ratios in bronchoalveolar lavage fluid. Sarcoid granulomas produce ACE and 1,25 dihydroxycholecalciferol so patients have elevated ACE and hypercalcemia. Note: CD8+ cells predominant in hypersensitivity pneumonitis; neutrophils predominate in idiopathic pulmonary fibrosis

What enzyme can be inhibited with medication to reduce a patient's risk for colon adenoma?

COX-2 Adenomatous polyps contain dysplastic mucosa and are premalignant. Regular screening with timely incision of polyps is effective for prevention of colon adenocarcinoma. Studies have linked increased COX-2 activity to some forms of colon adenocarcinoma and suggest that regular Aspirin use decreases adenomatous polyp formation.

In order for vesicles in the musculocutaneous nerve to release their contents into the synaptic cleft, what substance is required?

Calcium Calcium influx into the nerve terminall occurs following neuronal depolarization and opening of voltage gated calcium channels.

32 year old male diagnosed with Hodgkin's lymphoma. Patient began aggressive chemotherapy regimen. During one of his treatment cycles, he experiences fever, dyspnea and malaise. Blood cultures incubated for 3 hours at 37 degrees C reveals the formation of true hyphae as seen in the image. Most likely cause of patients condition?

Candida albicans Candida albicans is the most common cause of opportunistic mycosis. It can affect any organ and cause generalized candidemia. Yeasts and pseudohyphae on LM and a positive germ tube test (incubating at 37 for 3 hours reveals germ tubes) are diagnostic.

4 year old caucasian boy with difficulty walking. PMH includes frequent respiratory infections. Cultured cells from this patient demonstrate high rate of radiation induced genetic mutation. This patient is most likely to have what finding in the brain?

Cerebellar atrophy Ataxia-telangiectasia is an autosomal recessive disorder resulting from a defect in DNA repair genes. The DNA of these patients is hypersensitive to ionizing radiation. Manifestations include cerebellar ataxia, oculocutaneous telangiectasias, repeated sinopulmonary infections, and increased risk of malignancy

Statistical test used to evaluate the association between two categorical levels?

Chi-square test for independence Note: if data numerical but divided into categorical groups such as high fibrinogen (>400) and normal fibrinogen (200-400) you use chi square!

Hormone responsible for gallbladder contraction thats made from I cells in the duodenum and jejunum in response to fatty acids and amino acids?

Cholecystokinin Note: CCK is also responsible for decreasing gastric emptying and increasing pancreatic enzyme secretion by acinar cells

39 year old woman with several episodes of upper abdominal pain that is triggered by fatty foods and revolves spontaneously. Symptoms began a few months ago after an uncomplicated pregnancy. She has a history of hypertriglyceridemia which is treated with a vibrate. She undergoes cholecystectomy with multiple stones noted in the gallbladder. Decreased activity of what enzyme contributed to this patients condition?

Cholesterol 7 alpha hydroxyls Fibrate medications (fenofibrate, gemfibrizol) inhibit cholesterol 7 alpha hydroxylse which catalyzes the rate limiting step in the synthesis of bile acids. The reduced bile acid production results in decreased cholesterol solubility in bile and favors the formation of cholesterol gallstones.

Most likely cause of increased cardiac output as well as increased venous return on graph?

Chronic arteriovenous fistula A chronic arteriovenous shunt would increased CO because of increased sympathetic stimulation to the heart, decreased TPR, and increased venous return. It would also cause the venous return curve to shift to the right because the circulating blood volume is increased through renal retention of fluids and because venous pooling is reduced by the increased sympathetic tone

NADPH oxidase catalyzes the first step of the oxidative burst pathway, transferring an electron from the reducing equivalent NADPH to molecular oxygen and forming the ROS, superoxide. Deficiency in NADPH oxidase results in?

Chronic granulomatous disease (increased risk of infection with catalase positive organisms)

If HBeAg persists for several months and host anti-HBeAg remain at low or undetectable levels, you should suspect?

Chronic hepatitis B infection with high infectivity

45 year old with a week of purulent nasal discharge, headache, sore throat, and a nonproductive cough. No PMH except for episode of infectious mononucleosis at age 22. Smoke 1PPD. Temperature is 100.4. Has maxillary sinus tenderness, pharyngeal erythema, and tender anterior cervical lymphadenopathy. Leukocyte count is elevated at 58,000. Myelocytes are 30%. Blasts are 1%. Leukocyte alkaline phosphatase score is low. Most likely diagnosis?

Chronic myelogenous leukemia CML and leukemioid reaction can have presentations similar to leukocytosis, however, leukocyte (neutrophil) alkaline phosphatase level is normal or elevated in a leukemia reaction but DECREASED in CML. The definitive diagnosis of CML requires demonstration of the Philadelphia chromosome t(9,22) or BCR-ABL fusion gene or mRNA. Management typically includes a tyrosine kinase inhibitor. Note: Acute myelogenous leukemia is most common acute leukemia in adults, but mean age is 65 and patients have a significant increase in blast cells (>20%)

Ototoxic chemotherapeutic agent thought to damage hair cells in the cochlear membranous labyrinth?

Cisplatin Note: Adverse effects of other drugs Glucocorticoids- vertigo & psychosis Etopiside- alopecia, myelosupression & GI irritation Doxorubacin- myleosuppression, myocardial damage & alopecia Methotrexate- liver fatty change & myelosuppression Ondansetron- headache and constipation

45 year old man with several days of cough and fever. Temperature is 101.2. PE of right lung reveals occasional rales and decreased breath sounds at the base. CXR shows lung infiltrate, hilarious aden-patchy, and right sided pleural effusion. Lung tissue specimen shows thick walled spherules with endospores. Diagnosis?

Coccidiodes immitis C. immitis infection can be asymptomatic or can cause pulmonary disease ranging from flulike illness to chronic pneumonia. It causes disseminated disease in immunocompromised patients. Spherules (larger than RBCs) containing endospores are found in tissue samples. Culture on Sabouraud agar and serology are important in making diagnosis.

In the US bats are the main source of rabies (only transmitted by mammals). Agitation and spasms progressing to coma within weeks of exposure is strongly suggestive of rabies encephalitis. Prophylactic vaccine is recommended for individuals at high risk for exposure to rapid animals or their tissues. Features of the approved rabies vaccine?

Consists of various rhabdovirus strains grown in tissue cell culture and then INACTIVATED

22 year old with involuntary movements. Liver biopsy reveals high issue copper content. A greenish ring is apparent on eye exam. This finding is most likely due to copper deposition in what area of the eye?

Cornea (in Descemet's membrane of cornea). These rings are typically found in patients with neurologic involvement Note: iris hamaratomas (Lisch nodules are seen in NF1)

32 year old with several spots on her face for the past 3 weeks. Lesions have increased in size and are becoming more numerous. They're mildly pruritic but patients has no pain or fever. On exam multiple papular lesions are noted on the trunk and face as shown in the image (dimpled lesions). Biopsy of these lesions will most likely show?

Cytoplasmic inclusion bodies Molluscum contagiosum is characterized by umbilicate, flesh colored papules on the skin and mucous membranes. It's caused by a poxvirus, which appears in pathologic specimens as eosinophilic cytoplasmic inclusions (molluscum bodies)

Preproinsulin has a signal sequence in the N-terminal consisting of hydrophobic amino acids. A model is developed with several base pair deletions in the gene segment encoding this signal sequence. EM of the pancreatic beta cells from this model are most likely to show accumulation of the hormonal precursors in what cellular compartment?

Cytosol Proteins destined for the RER possess hydrophobic, N-terminal peptide signal sequences that identify them as such. The N terminal sequences are translated in the cytosol and are rapidly recognized by signal recognition particles which halt translation and target the ribosome to the protein pores in the RER. Removal of these signal sequences would cause inappropriate protein accumulation in the cytosol.

17 year old boy with bilateral breast enlargement. Has a longstanding history of learning disabilities. He is 95th percentile for height and weight is in the 25th percentile for age and sex. Testicles are small and firm. Labs would likely show?

Decreased testosterone, increased LH & FSH, increased estradiol Klinefelter syndrome (47, XXY) results in primary hypogonadism. In addition elevated estradiol levels results in gynecomastia & risk of malignant transformation.

Mechanism of action of Methimazole in treatment for hyperthyroidism?

Directly inhibits coupling of iodotyrosines Thioamides (methimazole, PTU) decrease the formation of thyroid hormones via inhibition of thyroid peroxidase, the enzyme responsible for both iodine organification and coupling of iodotyrosinases. PTU also decreases peripheral conversion of T4 to T3

X linked recessive myopathy that manifests with proximal muscle weakness and enlargement of the calf muscles in boys age 2-5. Most often results from frameshift deletions affecting the dystrophin gene. Dystrophin provides a stabilizing interaction between the sarcolemma and intracellular contraction apparatus, and disruption of the protein results in membrane damage and myonecrosis

Duchenne muscular dystrophy

Hormone that increases bicarbonate concentration is secreted from what location?

Duodenal S cells Secretin is produced by the S cells in the duodenal mucosa in response to stimulation by intraluminal acidity. Secretin stimulates the release of bicarbonate rich secretions from the exocrine pancreas which is the major source of acid neutralizing bicarbonate entering the duodenum.

5 year old boy with recurrent spasms in his hands. Hasn't gained height in the last 2 years. On exam he has short stature, short fingers, and a round face. Serum calcium level is low and serum phosphorous and PTH levels are elevated. Creatinine is normal. Most likely cause of patient's condition?

End organ resistance to PTH Pseudohypoparathyroidism is characterized by end organ resistance to PTH due to defects in the PTH receptor and downstream signaling pathways. Patients have hypocalcemia and hyperphosphatemia despite elevated PTH levels. Albright hereditary osteodystrophy is an autosomal dominant disorder characterized by pseudohypopatathyroidism caused by a defect in GNAS1 which codes for the alpha subunit of the Gs protein that mediates the effects of PTH and is associated with short stature and short metacarpal and metatarsal bones.

62 year old man undergoes radical prostatectomy and during the surgery the nerves within the fascia surrounding the gland are inadvertently injured. Most likely consequence of the nerve injury?

Erectile dysfunction The prostatic plexus lies within the fascia of the prostate and innervates the corpus cavernosa of the penis, which facilitates penile erection. As a result prostatectomy or injury to the prostatic plexus can cause erectile dysfunction

Dysphagia is common in patients with longstanding GERD and can be caused by impaired peristalsis or esophageal inflammation, stricture or malignancy. How do you identify esophagus on chest CT?

Esophagus is located between the trachea and vertebral bodies in the superior thorax. Its typically collapsed with no viable lumen (D in picture)

75 year old man with worsening dyspnea and fatigue on exertion for the past 6 months. Recently has had severe lightheadedness during physical activity. PE reveals a harsh ejection type systolic murmur at the base of the heart radiating to the neck. Second heart sound is diminished in intensity and a 4th heart sound is heart at the apex. Most likely cause of this patients heart condition?

Extensive valve calcification with impaired leaflet mobility Calcific degeneration of the trileaflet aortic valve is the most common cause of aortic stenosis in developed nations. Aortic stenosis is characterized by progressive aortic valve leaflet thickening and calcification, leading to restricted leaflet exclusion and mobility. Aortic stenosis murmur is usually a harsh ejection type systolic murmur best heart at the base of the heart in the 'aortic area' (second right intercostal space) with radiation to the carotid arteries. Second heart sound is diminished in intensity due to reduced mobility of the aortic leaflets and 4th heart sound may be heard due to decreased compliant of the hypertrophic myocardium

Involves the conscious and deceptive feigning or self production of physical or psychological symptoms to obtain attention and medical care from health care personnel

Factitious disorder

23 year old with Mycoplasma is treated with Azithromycin. Two months later her symptoms and anemia have resolved. Best explanation for the resolution of the anemia?

Fading of immune response against the bacteria Mycoplasma pneumoniae binds an oligosaccharide (I-antigen) on the respiratory epithelium that is also present on erythrocytes leading to the generation of cross reacting IgM antibodies (cold agglutinins). Patients with M. pneumoniae infections can develop mild, transient hemolytic anemia that resolves as IgM antibody tiers decline (6-8 weeks after infection begins)

14 year old girl with bump on chest below right breast that has been there as long as she can remember but became larger a few years ago. Bump becomes tender just before she starts her menses. There is a soft hyper pigmented, 0.5cm contender lesion inferior to the right breast. Most likely cause of this patient presentation?

Failed involution of the mammary ridge Accessory nipples are the most common congenital breast anomaly resulting from failed regression of the mammary ridge in utero. They're usually asymptomatic but can become tender along with breast tissue during times of hormonal function.

Active TB is never treated with drug monotherapy due to?

Fast emergence of mycobacterial antibiotic resistance from rapid selective gene mutations Note: Isoniazid monotherapy may be used for patients with have a positive PPD and a negative CXR (no evidence of clinical disease)

Duchenne muscle dystrophy manifests with proximal muscle weakness and atrophy. True hypertrophy of the distal muscle is noted early in the disease as muscles compensate for weak proximal ones. Muscle fibers of the distal extremities are later replaced by?

Fat and connective tissue (pseudohypertrophy)

22 year old man with acute pancreatitis is found to have very high triglyceride levels. What medication would be most helpful in preventing reoccurrences of his condition?

Fenofibrate In patients with severe hypertriglyceridemia, pancreatic lipase can cause toxic levels of FFA to be released within the pancreatic tissue, leading to acute pancreatitis. Fibrates are the most effective agents for the treatment of hypertriglyceridemia; they activate PPAR alpha which increases the synthesis of LPL

GFR can be estimated by? RPF can be estimated by?

GFR- inulin or creatinine clearance RPF- para-aminohuppuric acid (PAH) clearance FF=GFR/RPF FF=20% in healthy individuals FF= fraction of RPF that is filtered across the glomerular capillaries into bowman's space Clearance = (urine concentration x urine flow rate)/plasma concentration

46 year old woman with persistent diarrhea, weight loss, and abdominal pain. Upper endoscopy reveals post bulbar duodenal and jejunal ulcers. Patient most likely has a tumor secreting what molecule?

Gastrin Zollinger-Ellison syndrome is caused by gastrin secreting tumors involving the small intestine or pancreas. Patients typically have peptic ulcers (often beyond the duodenal bulb), abdominal pain/acid reflux, and diarrhea. The condition is frequently associated with MEN1.

Deep bleeding ulcer on posterior wall of the duodenal bulb is most likely to penetrate what artery?

Gastroduodenal The gastroduodenal artery lies along the posterior wall of the duodenal bulk and is likely to be eroded by posterior duodenal ulcers. Ulceration into the gastroduodenal artery can be a source of life threatening hemorrhage

21 year old male with mild jaundice after hiking trip. Similar episode 2 years ago after fasting for 2 days. Total bilirubin is elevated. Patient most likely suffers from?

Gilbert syndrome Gilbert syndrome is a common familial disorder of bilirubin glucuronidation in which the production of UDP glucuronyl transferases (enzymes that mediate glucuronidation of various substances) is reduced. Diagnosis is suggested in patients with no apparent liver disease who have mild unconjugated hyperbilirubinemia thought to be provoked by a classic trigger: hemolysis, fasting, physical exertion, febrile illness, stress, and fatigue. Unconjugated bilirubin persists with repeated testing but LFTs, CBC, blood smear, and reticulocyte count are normal.

23 year old man with no history of seizure presents after having a generalized tonic clonic seizure. Roommate says he's had a fever and headache for the past two days and today he wasn't making sense. MRI shows swelling of temporal lobes. CSF analysis is most likely to show/

Glucose- normal Protein- increased Cells- increased lymphocytes and erythrocytes HSV1 is most common cause of sporadic encephalitis. Viral infections of the CNS are usually characterized by elevated protein, normal glucose, and an elevated WBC count with a lymphocytic predominance. Patients with herpes encephalitis also have elevated erythrocytes in the CSF due to hemorrhagic inflammation of the temporal lobes

18 year old girl with history go T1DM presents with nausea, vomiting, and abdominal pain. She hasn't taken insulin in 2 days. PE shows fruity odor and dry MM> Labs show glucose level of 452 and high anion gap metabolic acidosis. UA is positive for ketones. Its determined that her insulin deficiency is causing increased production of gluconeogenic precursors that are subsequently converted to glucose in the liver. Enzyme most likely to provide these precursor substrates?

Glycerol kinase Glycerol produced by the degradation of triglycerides in adipose tissue can be used by glycerol kinase in the liver and kidney to synthesize glucose during gluconeogenesis.

Paroxysmal breathlessness and wheezing in a young patient that is unrelated to ingestion of aspirin, pulmonary infection, inhalation of irritants, stress, and/or exercise should raise a strong suspicion for extrinsic allergic asthma. Classic sputum findings include?

Granule containing cells (eosinophils) & crystalloid bodies (charcot-layden crystals) Eosinophils are recruited and activated by IL-5 secreted by Th2 type helper T cells

47 year old woman has a mammography performed which is highly suspicious for malignancy and a needle biopsy reveals infiltrating ductal carcinoma. Over expression of what marker is most likely to be associated with aggressive disease in this patient?

HER2 Estrogen or progesterone positivity in breast cancer indicates expected sensitivity to tamoxifen and aromatase inhibitor treatment. HER2 over expression in breast cancer suggests a more aggressive tumor that typically responds to therapy with the anti-HER2 monoclonal antibody Trastixumab. HER2 is a transmembrane glycoprotein with tyrosine kinase activity that increases cell proliferation

An outbreak in New Delhi was characterized by high incidence of fulminant hepatitis in pregnancy women who experienced mortality rate of 20%. Virus responsible for the outbreak?

Hepatitis E HEV is an undeveloped, single stranded RNA virus spread through the fecal oral route. The most concerning feature of HEV infection is the high mortality rate observed in infected pregnant women

In Von Gierke disease (glycogen storage disease type I), glucose 6 phosphatase deficiency impairs the release of free glucose generated through glycogenolysis and gluconeogenesis. Affected patients present with?

Hepatomegaly, growth failure, lactic acidosis, and fasting hypoglycemia that DOES NOT IMPROVE with fructose or galactose administration

Mechanism of action of hydroxyurea used for the treatment of sickle cell anemia?

Increases hemoglobin F synthesis Hydroxyurea increases fetal hemoglobin synthesis by an unknown mechanism and is reserved for patients with frequent pain crises. Gardos channel blockers hinder the efflux of K+ and water from the cell preventing dehydration of erythrocytes and reducing the polymerization of Hb S

Suicide risk assessment includes consideration of both risk and protective factors. Single strongest risk factor for suicide?

History of a previous attempt

Infarction of the territories supplied by the middle cerebral, posterior cerebral, or anterior choroidal arteries would cause what visual disturbance?

Homonymous hemianopsia (any unilateral visual pathway lesion beyond the optic chasm can cause this)

Cystinuria results from defective dibasic amino acid transport in the intestinal and proximal renal tubular epithelial cells. It most often presents with recurrent stone formation at a young age due to decreased reabsorption of cysteine from the urine. UA shows pathognomonic hexagonal cystine crystals and the sodium cyanide nnitroprusside test can be used to detect excess cystine in the urine. Treatment?

Hydration and urinary alkalization (acetazolamide)

74 year old woman with osteoporosis and HTN. What drug has a beneficial effect on Ca2+ homeostasis and can reduce fracture risk in this patinet?

Hydrochlorothiazide Thiazide diuretics increase Ca2+ absorption in the DCT within the nephron. Thiazides are associated with increase bone mineral density and are recommended for the treatment of HTN in patients at risk for osteoporosis. Loop diuretics increase Ca2+ loss.

Pompe disease (glycogen storage disease type II) is caused by acid alpha glucosidase (acid maltase) deficient and is characterized by?

Hypertrophic cardiomyopathy, diffuse muscular weakness, and macroglossia. Pompe disease is the only glycogen storage disease caused by defective lysosomal enzyme and the only one that presents with cardiomegaly in infancy. Periodic acid shift stain identifies glycogen.

52 year old homeless man found unresponsive. ABG shows partial pressure of O2 in arterial blood is 60 mmHg. Partial pressure of O2 in his alveoli is 71 mmHg. Most likely cause of patients symptoms?

Hypoventilation Patient has low PaO2 and low PAO2 with a normal A-a gradient. Normal PaO2 is 100 and normal PAO2 is 104. The A-a gradient is normal with alveolar hypoventilation which helps distinguish it from other forms of hypoxemia (its also normal in high altitude). This patients A-a gradient is 11 (71- 60= 11) Normal A-a gradient is between 5-15

Stages of sarcoidosis in the lung?

I- bilateral hilar lymphadenopathy II- bilateral hilar lymphadenopathy and pulmonary infiltrates (usually in upper lobes) III- lung infiltrates only IV- lung fibrosis Sarcoidosis most commonly affects young black women and presents with malaise, cough, and varied cutaneous findings including erythema nodosum. CXR reveals bilateral hilar lymphadenopathy. Transbronchial biopsy shows non-caseating granulomas is necessary for diagnosis

Refers to modeling ones behavior after someone who is perceived to be more powerful or prestigious

Identification Example: child of an abusive father who becomes a child abuser himself

52 year old man with persistent nonproductive cough and exertion dyspnea that has progressed over the past year. No past medical history and takes no meds. Spirometry shoes FVC 40% of predicted value and a FEV1 /FVC ration of 87%. CT guided lung biopsy is performed and histopathology shows dense fibrosis, fibroblast proliferation, and cyst formation which are most prominent in the sub pleural regions. Most likely diagnosis?

Idiopathic pulmonary fibrosis Progressive exertion dyspnea and dry cough, a restrictive profile on PFT, and interstitial fibrosis with cystic air space enlargement (due to alveolar collapse) are characteristic of idiopathic pulmonary fibrosis. The pathologic findings of IPF are termed usual interstitial pneumonia and show patchy involvement with dense fibrosis, 'honeycomb' changes (cystic spaces lined by hyper plastic type II pneumocytes or bronchiolar epithelium) , and fibrobalstic foci more predominant in the sub pleural and paraseptal spaces

With maternal blood types A and B, erythroblastosis fetalis and hemolytic disease of the newborn do not occur as the naturally occurring antibodies (anti-A and anti-B) are of the IgM type and cannot cross the placenta. In contrast, in type O mothers, the antibodies are predominantly what type?

IgG and can cross the placenta Unlike Rh disease, ABO disease can occur with the first pregnancy because anti-A and anti-B antibodies are formed in early life from exposure to A- or B-like antigens present in foods, bacteria, and viruses

13 year old boy underwent appendectomy. He began having burning pain at the surgical scar radiating to the suprapubic region a few days after the surgery. On exam theres loss of sensation over the right suprapubic area. Cremasteric reflex is normal. Most likely injured nerve in this patient?

Iliohypogastric iliohypogastric nerve (L1 nerve root) provides sensation to the upper suprapubic and gluteal regions and motor function to the anterolateral abdominal wall muscles. Abdominal surgery can damage/entrap the nerve and cause decreased sensation and/or burning pain at the suprapubic region Note: genitofemoral nerve arises from L1-L2 nerve roots and provides sensation to the upper anterior thigh and motor to parts of the genitala (mons pubis, cremasteric reflex) Note: ilioinguinal nerve originate from L1 and accompanies the spermatic cord through the superficial inguinal ring; it provides sensation to the upper/medial thigh and parts of the genetalia

Lac operon is regulated by 2 distinct mechanisms: negatively by binding of the repressor protein to the operator locus and positively by cAMP-CAP binding upstream from the promotor region. Constitutive expression of the structural genes of the lac operon occurs with mutations that result in?

Impaired binding of the repressor protein (Lac I) to its regulatory sequence in the operator region

Immunohistochemical evaluation shows an over expression of RANK (receptor activator of nuclear factor kappa B) on the surface of certain bone cells. Most likely effect of this finding?

Increased bone resorption M-CSF and RANK-RANKL interaction is essential for the formation and differentiation of osteoclasts. Osteoprotegerin blocks binding of RANK-L to RANK and reduces formation of mature osteoclasts. Low estrogen states cause osteoporosis by decreasing osteoprotegerin production, increasing RANK-L production, and increasing RANK expression in osteoclast precursors Denosumab is a monoclonal antibody used in the treatment of postmenopausal osteoporosis. It works in a manner similar to OPG by binding RANK-L and blocking its interaction with RANK

MOA of action of cough syncope?

Increased intrathoracic pressure during a coughing episode decreases venous return to the heart thereby transiently decreasing CO and cerebral perfusion

23 year old who was stabbed in the right upper quadrant of the abdomen. BP is 70/42. Abdomen is firm and distended. FAST is positive for blood in the right upper quadrant. H his taken for laparotomy and 1 liter of blood is evacuated from the peritoneal cavity. Brisk, nonpulsatile bleeding is seen emanating from behind the liver. Surgeon occludes the hepatioduodenal ligament, but the patient continues to hemorrhage. Source of this patients bleeding?

Inferior vena cava Occlusion of the portal triad (pringle maneuver) is a surgical technique used to distinguish the source of right upper quadrant bleeding. If hepatic bleeding persists after occlusion of the portal triad (hepatic artery, portal vein, common bile duct), the IVC or hepatic veins are likely to be injured.

40 year old with complaint of food sticking in his throat. Also has had a cough, nocturnal regurgitation, and difficulty belching. Barium study shows narrowing of the distal esophagus with dilation of the rest of the esophagus. Most likely cause of patients symptoms?

Inflammation and degeneration of neurons in the esophageal wall Achalasia is an esophageal motility disorder characterized by the absence of esophageal peristalsis in the distal esophagus and incomplete relation of a hypertensive lower esophageal sphincter. Achlasia is caused by the degeneration of inhibitory ganglion called in the myenteric (Auerbach plexus)

Involves the use of excessive thinking to avoid painful emotions or fears. Its particularly common in patients recently diagnosed with a serious medical illness

Intellectualization

44 year old man with progressive dyspnea over the past several years. PE shows prolonged expiratory phase without wheezes or rhonchi. CT shows bilateral lower lobe predominant emphysema. Further testing reveals patient has a protease inhibitor deficiency, which has lead to increased elastin fiber breakdown. Elastin fibers within alveolar walls normally allow lung to stretch during active inspiration and recoil during expiration. What explains this property of elastin?

Interchain cross-links involving lysine The rubber like properties of elastin are due to extensive demosine cross linking between elastin monomers which is facilitated by lysyl oxidase (requires copper). Patients with alpha 1 antitrypsin deficiency can develop early onset lower lobe predominant emphysema due to excessive alveolar elastin degradation

IN a normal mammalian kidney, densely immunostaining protein containing cells adjacent to the renal glomerulus are most likely?

Juxtaglomerular cells in the JGA (synthesize, store, and secrete renin)

The size of adenomatous polyps determines their malignant potential. Adenomas less than 1 cm are unlikely to undergo malignant transformation whereas those greater than 4 cm are very likely to progress to adenocarcinoma. Mutation that failitates the growth of adenomas by causing uncontrolled cell proliferation?

KAS protooncogene

24 year old wit fatigue, daytime sleepiness, and occasional headaches. Sleeps in separate room from wife because of his snoring. Hematocrit is 57%. Decreased oxygen deliver to what organ is responsible for increased hematocrit?

Kidneys Renal cortical cells sense hypoxia and respond by synthesizing and releasing erythropoietin. Erythropoietin stimulates the production of erythrocytes in the bone marrow.

Polyethylene glycol is an osmotic laxative that attracts water into the intestinal lumen, distending the intestinal wall and increasing peristalsis. This is similar to what pathologic condition?

Lactase deficiency Diarrhea with lactase deficiency is also osmotic and occurs due to accumulation of nonabsorbable lactose in the intestinal lumen. Magnesium hydroxide (and other magnesium compounds such as magnesium citrate) are other osmotic laxatives and are often used although their efficacy is less proven.

55 year old man with poorly controlled HTN and T2DM with difficulty seeing. Has had blurry vision for the last 2 weeks, and 1 day age suddenly noticed a shadow develop across the visual field of his left eye. On exam PERRL. There is an area of reduced vision in the left eye. Fundoscopic exam reveals a flame shaped hemorrhage in the left temporal hemiretina. Transmission of visual information through what neural structure will be disrupted?

Left lateral geniculate body Left temporal hemiretina receives visual information from the nasal visual field of the left eye. Damage to the left temporal hemiretina will disrupt transmission of visual information along the ipsilateral optic nerve, lateral optic chiasm, optic tract, lateral geniculate body in the thalamus, optic radiations, and primary visual cortex.

32 year old African American woman with heavy menses which have become heavier over the past year. Received a blood transfusion at the end of her last menses. Also reports pelvic heaviness. Bimanual pelvic exam shows an irregular 14 week size uterus. Hemoglobin is low. Most likely diagnosis?

Leiomyoma Leiomyomas are common, benign uterine smooth muscle tumors that can result in endometrial venous ectasia (dilation), which can lead to heavy menstrual bleeding. Submucosal/intracavitary fibroids are particularly associated with anemia. Distortion of the intrauterine cavity can result in reproductive difficulties.

Multinucleated giant melanocytes are a characteristic finding in what condition that are common pigmented lesions in elderly patients and are considered to be a type of melanoma in situ

Lentigo maligna melanoma

Syndrome that results in ENaC over expression in the collecting tubules causing increased renal sodium absorption with HTN, hypokalemia, and metabolic alkalosis

Liddle syndrome

Partly composed of lipoblasts that contain non-membrane bound cytoplasmic lipid that shifts periodically, causing scalloping of the nuclear membrane

Liposarcomas Most common soft tissue sarcoma in adults and typically present as slow growing masses in middle aged adults in deep connective tissue spaces of the trunk or lower extremities

An atrial septal defect will be associated with what PO2 readings?

Low superior vena cava PO2, moderate right atrial and right ventricle PO2, and high left atrial and left ventricular PO2 SVC- 43 RA- 65 LA-97 RV-65 LV- 97 Pulmonary artery- 65 ASD would cause you to hear a fixed splitting of S2 on exam

46 year old with 3 week history of low grade fever, weakness and neck swelling. Biopsy reveals Reed-Sternberg cell. Abnormality in what tissue is responsible for patient's symptoms?

Lymphoid tissue Reed Sternberg cells are derived from germinal center B lymphocytes and are neoplastic cells of Hodgkins lymphoma. Reed-Sternberg cells are large binucleated cells with an 'owl eyes' appearance that appear on a background of lymphocytic infiltrates and must be present to diagnose Hodgkins.

4 year old boy with high fever, nasal discharge, and cough for 3 days. His family recently returned from a trip to Mexico. On exam he has bilateral conjunctival injection and several small white spots with an erythematous base on his buccal mucosa. Records show parents refused preventative health services in favor of a 'natural' approach to childs health. Over next several days, what is likely to develop?

Maculopapular rash (cephalocaudal & centrifugal spread, spares palms and soles) Measles (rubeola) presents with fever, cough, rhinorrhea, and conjunctivitis, followed by a maculopapular rash that starts on the face and spreads downward. Koplik spots are pathognomonic for measles and characterized by tiny white or blue gray lesions on the buccal mucosa

Most common cardiac abnormalities associated with DiGeorge syndrome?

Maldevelopment of the truncus arteriosus is common in utero result in persistent truncus arteriosus, teratology of fallot, or interrupted aortic arch

Disease associated with TGF-alpha overproduction resulting in mucosal cell hyperplasia with gastric fold enlargement. Causes hypoplasia of parietal/chief cells resulting in glandular atrophy with reduced gastric acid secretion

Menetrier disease

Disease in which serum copper content is decreased & is associated with abnormally pigmented, kinky hair and hypo pigmented irises as well as abnormal development

Menkes disease

Rare type of skin cancer that appears as flesh colored or bluish red nodules on the face, head or neck. Due to proliferation of neuroendocrine receptor cells in the skin associated with sense of touch

Merkel cell carcinoma

Spleen is a large, wedge-shaped lymphatic organ that is situated in the posterior superior portion of the left abdominal cavity. Its derived from what embryological derivative?

Mesoderm While most gut tissue is derived from endodermal origin, the spleen is unique as it is derived from condensed mesenchymal tissue in the dorsal mesentery during embryologic development

53 year old male with progressively worsening anorexia and abdominal discomfort. Lost 31 pounds since onset of symptoms 4 months ago. PE reveals nontender hepatomegaly. Labs show elevated alkaline phosphatase and marginally elevated ALT. Contrast scan is shown & shows multiple hypotenuse masses in the liver. Most likely diagnosis?

Metastatic liver disease Most common malignant hepatic lesion is a metastasis from another primary site (breast, lung, colon) NOT hepatocellulr carcinoma! Liver is 2nd most common site of metastatic spread (after lymph nodes) because of its large size, dual blood supply, high perfusion rate, and filtration function of Kupffer cells.

21 year old with rhinorrhea that began 2 weeks earlier, sore throat, and sneezing. After initial improvement, he developed nasal congestion, facial pain, and persistent low grade fevers. Temperature is 100. What is most likely site of the opening structure involved in this patient?

Middle nasal meatus The maxillary sinuses are the most commonly affected sinuses of secondary infection with oropharyngeal bacteria as their path of drainage (semilunar hiatus in the middle nasal meatus) is located superior to the floor of the sinus, impairing drainage while upright. Maxillary sinuses are located inferior to each orbit

26 year old with depressed mental status brought in by police. On fear of arrest he swallowed a handful of pills as police approached him. On exam he responds to painful stimuli but is somnolent. After naloxone bolus his RR increases from 6 to 14. What additional findings would you expect upon initial assessment of this patient?

Miosis, bradycardia, hypotension (due to histamine release) Opioid intoxication presents with miosis, depressed mental status, decreased RR, decreased bowel sounds, hypotension, and bradycardia. Of these, decreased RR is the best predictor of intoxication and is a frequent cause of mortality.

Small to medium vessel vasculitis caused by circulating immunoglobulin complement complexes that precipitate on refrigeration. Associated with chronic inflammatory states (SLE, HCV) and can present with fatigue, arthralgies, myalgia, palpable purpura in the LE due to cutaneous vasculitis

Mixed cyroglobulinemia

HLA haplotypes (MHC Class I and II molecules) participate in immunologic recognition of specific antigens by T lymphocytes. MHC molecules don't undergo somatic recombination, and individuals with HLA haplotypes that cannot bind certain polypeptide fragments will be unable to?

Mount a T cell dependent (IgG, cell mediated)immune response against these antigens

Bacertia isolated from the lung tissue fail to decolarize with HCl and alcohol after staining carbolfuchsin. What cell wall component is responsible for this staining phemonemon?

Mycolic acid The acid fast stain identifies organisms that have mycelia acid present in their cell walls, including Mycobacterium and some Nocardia species. Acid fast standing is carried out by applying aniline dye (carbolfuchsin) to smear and then decolorizing with acid alcohol to reveal whether the organism present are acid fast

Number needed to treat is the number of patients the need to be treated with a medication to avoid an additional negative outcome. Equation?

NNT= 1/ARR Lower number needed to treat values represent more beneficial treatment

34 year old with progressive exertion dyspnea, LE edema, and cough. Reports frequent nocturnal episodes of breathlessness and recent hoarseness. Auscultation reveals loud 1st and 2nd heart sounds and a mid-diastolic rumble best heart at the cardiac apex. Patient's hoarseness is most likely caused by?

Nerve impingement Left atrial enlargement (caused by mitral stenosis in this case) can sometimes cause left recurrent laryngeal nerve impingement (Ortner syndrome). Neurapraxia (failure of nerve condition due to blunt injury) resulting in left vocal cord paresis and hoarseness may results. Recurrent laryngeal nerve innervates all intrinsic muscles of the larynx except the cricothyroid muscle.

Start with cohort studies in which participants are followed over time and those participating who develop an outcome of interest become cases for a case control study

Nested case control designs

Most commonly occurs in the setting of dry skin (xerosis). Use of harsh soaps and exposure to environmental irritants can exacerbate the condition. Presents with pruritic coin shaped erythematous patches

Nummular eczema

Most common location of endometriosis?

Ovaries Results in dysmenorrhea which is the most common presenting symptom. Gross pathology often reveals adnexal masses that appear as 'chocolate cysts'

35 year old with worsening SOB. Has also noted mild wheezing and sputum production recently. Has smoked 1/2 PPD for the past 5 years. Father died from lung and liver disease at a lung age. Spirometry shows reduced FEV1 and FVC. Patients lower lung lobes are most likely to demonstrate?

Panacinar emphysema Alpha 1 antitrypsin is the major serum inhibitor of neutrophil elastase. AAT deficiency can cause panacinar emphysema, which usually affects the lower lung lobes more severely.

An outbreak of a water-borne gastoenteritis with high mortality is reported in some Asian countries. Its caused by an oxidase positive, gram negative bacilli that grow well on highly alkaline selective media. Patients that would most likely require the smallest infectious dose to initiate infection?

Patients who have been on chronic omeprazole therapy The minimal infectious dose of Vibrio Cholerae infection is usually high with 10^10 organisms required to cause infection following ingestion of contaminated water and food. V. cholerae is very sensitive to gastric acidity. Any condition that decreases acidity (or increases gastric pH), such as antacid use or other conditions causing achlorhydria (gastritis) will lower the minimum infectious disease of V. cholerae by multiple orders of magnitude.

Acute onset mid chest pleuritic pain that decreases on sitting up and leaning forward is characteristic of acute pericarditis. Fibrinous and serofibrinous are the most common form. Most striking physical finding?

Pericardial friction rub Note: a pericardial knock is a brief high frequency precordial sound heart in early diastole (shortly after S2) in patients with constrictive CHRONIC pericarditis

34 year old woman with sharp chest pain that radiates to left shoulder. Pain increases on inspiration and partially relieved by sitting up and leaning forward. Was evaluated for facial rash 6 months ago. Also currently being evaluated for proteinuria. Most likely cause of her chest pain?

Pericardial inflammation Pericarditis is the most common cardiovascular manifestation associated with SLE. It presents with sharp pleuritic chest pain thats relieved with sitting up and leaning forward

36 year old male presents with fever, bleeding gums, and sore throat. Peripheral blood smear shows several myeloblasts with Auer rods. These cells contain structures that would stain positive for?

Peroxidase Auer rods are deformed azurophilic granules found in the cytoplasm of myeloblasts that stain potivively for myeloperoxidase. Auer rods are found in abundance in AML M3 (acute promyelocytic leukemia)

Zidovudine helps decrease risk of perinatal transmission by inhibiting what component of viral genome replication?

Phosphodiester bond formation Zidovudine is a NRTI used to precent maternal to fetal transmission of HIV during labor (when the mother is not virally suppressed). Its s thymidine analog that doesn't have the normal 3' hydroxyl group found on thymidine. Because new nucleotides are added to growing DNA chains only at the 3' hydroxyl group, the addition of zidovudine into DNA results in chain termination

Pleuritic chest pain is characterized by sharp, localized often severe pain that is exacerbated by coughing, breathing, or changing position. It can result from any condition that causes inflammation of the pleura (infection, pulmonary embolism, uremia). Pain arising from the mediastinal or diaphragmatic pleura will be carried by what nerve and referred to what area?

Phrenic nerve referred to the C3-C5 distribution

What parameters used to evaluate the accuracy and usefulness of diagnostic test are influenced by disease prevalence in the target population?

Positie and negative predictive values Note: sensitivity, specificity, and likelihood ratios are not prevalence dependent

30 year old man with 4 day history of progressively worsening abdominal pain and bloody diarrhea. Was started on mesalamine therapy 6 months ago after being diagnosed with UC but has been noncompliant with treatment. There is marked abdominal dissension and tenderness. Temperature is 102. BP is 100/70. HR is 130. HE is lethargic and has dry MM. Rectal exam shows guac positive maroon colored, liquid stool. Next best step in this patient's work up?

Plain abdominal x-ray Toxic megacolon is a well recognized complication of ulcerative colitis. Patients typically present with abdominal pain/distension, bloody diarrhea, fever, and signs of shock. Plain abdominal x-ray is the preferred diagnostic imaging study. Barium contrast studies and colonoscopy are contraindicated due to risk of perforation.

Urine cultures show extended spectrum beta lactamase producing E. coli. What feature is most likely to be found in this organism?

Plasmid with drug resistance gene Extended spectrum beta lactamases can be produced by gram negative bacteria, rendering cephalosporins (including third and 4th generation) and other beta lactam antibiotics (monobactams) inactive. These genes can be transmitted between organisms/species through plasmid conjugation. Carbapenems are the treatment of choice for ESBL producing organisms. Note: porin mutations can confer penicillin and aminoglycoside resistance & are particularly important in gram negative organisms such as Pseudomonas Note: Penicillin binding protein alterations are mechanism that leads to methicillin resistance in S. aureus

Secretory phase of menstrual cycle occurs between ovulation and menses from days 15-28 of the normal menstrual cycle. What do you expect endometrial sampling to reveal during this phase?

Progesterone released by the corpus luteum causes the uterine glands to coil and secrete glycogen rich mucous in preparation for embryo implantation. The endometrial stroma becomes edematous and completely traversed by tortuous spiral arteries that extend from the deeper layers into the uterine lumen

Treatment for the gram negative helical organism H. pylori?

Proton pump inhibitor, clarithromycin, amoxicillin, with or without bismuth

53 year old patient with dyspnea. Studies show that his work of breathing is increased with decreased respiratory rate. Most likely diagnosis?

Pulmonary fibrosis The work of breathing is minimized in patients with increased elastic resistance (pulmonary fibrosis, pulmonary edema, ARDS) when their respiratory rate is high and tidal volume is low (fast, shallow breaths). In contrast, patients with diseases that increase airflow resistance (asthma, COPD, fixed upper airway obstruction) breathe at a lower rate/higher tidal volume (slow, deep breaths) in order to minimize the work of breathing

5 year old with jaundice and fatigue. Has had a URI for the past 3 days. Pallor, scleral icterus, and palpable splenomegaly are seen on exam. Hemoglobin is low. Patient recovers spontaneously after a few weeks. Peripheral smear after recover reveals spherocytes. Most likely cause of patients condition?

RBC membrane cytoskeleton abnormalities Hereditary spherocytosis (autosomal dominant) results from red cell cytoskeleton abnormalities, most commonly spectrum and ankyrin. Hemolytic anemia, jaundice, and splenomegaly (spherocytes have difficulty passing through the cords of Villroth and accumulate in the spleen) are classic manifestations. Spherocytes (lack a zone of central pallor & are more densely hemoglobinized at the periphery) are present on peripheral blood smear. Infections can trigger hemolysis and lead to acute hemolytic crisis. Treatment is splenectomy.

72 year old develops mild abdominal pain and bloody diarrhea after undergoing a cholecystectomy. Surgery was complicated by an episode of hypotension that was treated with bolus IV crystalloid fluids. Patient had no previous GI bleeding. Last colonoscopy was normal. PMH of HTN, hyperlipidemia, T2DM, and MI 7 years ago. Patient smoked for 40 years and quit after MI. Colonoscopy would likely show pathology in what portion of the large bowel?

Rectosigmoid junction Patient likely has acute nonexclusive ischemic colitis. The splenic flexure (border between SMA and IMA supply) and rectosigmoid junction (border between sigmoid artery and superior rectal artery) lie between regions of perfusion of major arteries. These 'watershed' areas are susceptible to ischemic damage during hypotensive states, especially in patients with underlying arterial insufficiency. Impaired perfusion to bowel leads to ischemia and necrosis of bowel wall. Complications can include acidosis, sepsis, gangrene and perforation. Colonoscopy will reveal pale mucosa and petechial hemorrhages

Patients cervical biopsy reveals low grade dysplasia. What feature distinguishes low grade dysplasia from cervical malignancy?

Reversibility of changes Dysplasia is characterized by disruption of organized epithelial cell differentiation with marked cellular pleomorphism. Although high grade dysplasia (involving most or the entire epithelial layer) often progresses to invasive cancer, low grade dysplasia (involving a small portion of the epithelium) typically spontaneously regresses. Invasive cancer occurs when the abnormal cells penetrate the basement membrane

44 year old man with rash on left outer thigh. He is an industrial and sustained an iron burn to the area; The burn site became more inflated and a yellow crust developed. PE shows plaques and thick crusts with golden hue on left outer thigh. What organism is the most likely cause of his symptoms?

S. aureus Impetigo is a superficial infection of the skin associated with erythema and a yellow honey crust. Its most commonly caused by S. aureus and Group A Strep. In cases due to S. aureus, bullae may form due to toxin production (bullous impetigo)

Effect of TMP-SMX on Warfarin?

Slows the metabolism of warfarin by inhibiting CYP450 microsomal enzymes. At a given dose of Warfarin, the initiation or discontinuation of TMP-SMX will increase or decrease the INR, respectively

Patient is started on capsaicin cream and her pain improves. Decreased activity of what neurotransmitter is most likely associated with relief of pain?

Substance P Topical capsaicin causes excessive activation of TRPV1 (a transmembrane cation channel) causing a buildup of intracellular calcium. This causes defunctionalization of afferent pain fibers and depletion of substance P. Initial application results in a burning and stinging sensation, but chronic exposure leads to reduced pain transmission

20 year old college student brought to ED by roommate. Patient is frightened and claims that campus police are following him and plotting to kill him. Roommate says he seemed totally normal until the past few days when he began staying up all night to prepare for final exams. Patient has no known medical or psychiatric history. BP is 150/95 and pulse is 110. On exam he is sweating profusely, pacing, hyper vigilant, and has mildly pressured speech. Most likely diagnosis?

Substance induced psychotic disorder Stimulant intoxication can present with paranoid ideation and must be differentiated from primary psychiatric disorders. Physical signs of stimulant intoxication include tachycardia, hypertension, hyperthermia, diaphoresis, and mydriasis

Mechanism of resistance of Vancomycin in organisms such as vancomycin resistant enterococcus?

Substitution of D-lactate in the place of D-alanine during the process of peptidoglycan cell wall synthesis; prevents the binding of vancomycin to its usual D-alanyl-D-alanine binding site in the cell wall

Incision made during an emergency cricothyrotomy would most likely pass through what structures?

Superficial cervical fascia and cricothyroid membrane Cricothyrotomy is indicated when an emergency airway is required and orotracheal or nasotracheal incubation is either unsuccessful or contraindicated. The cricothyrotomy incision passes through the skin, superior cervical fascia (including subcutaneous fat and platysma muscle), pretracheal fascia, and criciothyroid membrane and establishes an airway between the cricoid and thyroid cartilages

35 year old mechanic with progressive right hand weakness. Frequently uses a screwdriver at work. Sensation is preserved in upper limbs. Weakness on extension of the fingers and thumb in the right hand is noted. Strength is otherwise intact throughout. Tricep reflexes are 2+/4 bilaterally symmetric. Nerve affected in this patient was most likely injured at what location?

Supinator canal Injury to the radial nerve during its passage through the supinator canal can occur due to repetitive pronation/supination of the forearm, direct trauma, or subluxation of the radius. Patients typically have weakness during finger and thumb extension ('finger drop') WITHOUT wrist drop or sensory deficits

Male pattern hair grown in a women is most commonly caused by PCOS. Combination oral conceptive pills treat hirtuism by?

Suppressing pituitary LH secretion and subsequently decreasing ovarian androgen production. They also increase sex hormone binding globulin synthesis by the liver, decreasing free testosterone levels

Student worried about her USMLE score but decides not to think about the exam until her scores arrive because 'worrying isn't going to change the result'. What type of defense mechanism is this?

Suppression Suppression is a mature defense mechanism involving conscious choice not to dwell on a particular thought or feeling Note: in suppression the individual is aware of the emotion but decides not to tend to it, whereas in repression unacceptable thoughts or feelings are blocked from entering conscious awareness

61 year old with skin dimpling of the right breast. PE shows a prominent contender skin retraction without discoloration or swelling of the right breast. Theres a 6 cm immobile firm mass in the right upper outer quadrant of the breast. This patients findings are likely due to malignant infiltration of what structure?

Suspensory ligament (Cooper ligament) Invasive breast carcinoma typically presents as an irregularly shaped adherent breast mass, most commonly in the upper outer quadrant. Malignant infiltration of suspensory ligaments of the breast cause dimpling of the overlying skin

Characterized by formation of a cavity in the cervical region of the spinal cord. Cavity (syrinx) damages the ventral white commissure, leading to bilateral loss of pain and temperature sensation that is limited to the affected levels (starting 1-2 levels below the lesion) typically located in the arms and hands, while distal sensation is preserved. Destruction of the motor neurons in the ventral horns (due to extension of the syrinx) causes flaccid paralysis and atrophy of the intrinsic muscles of the hand

Syringomyelia

Mutation in what gene is responsible for transformation from advanced colonic adenoma to adenocarcinoma?

TP53 The adenoma to carcinoma sequence is a series of gene mutations that leads to the development of colon adenocarcinoma. TP53 tumor suppressor gene mutation is part of the final step in the sequence and leads to malignant transformation of preexisting large adenomatous polyps

RNA dependent DNA polymerase that synthesizes telomeric DNA sequences (TTAGGG) that can replace the lost chromosomal ends of telomeres. Cancer cells typically contain increased activity of these enzymes to allow for continued proliferation

Telomerase

Hyperthyroidisim is characterized by increased sensitivity to catecholamines due to?

Thyroid hormone mediated up regulation of beta adrenergic receptors Beta blockers help decrease the target organ effects of thyroid hormones and certain ones (propranolol) also decrease peripheral conversion of T4 to T3

Involves symbolically nullifying an unacceptable or guilt provoking thought, idea, or feeling by confession or atonement (commonly seen in OCD)

Undoing

Benign prostatic hyperplasia is associated with an increased risk of?

Urinary tract infection Benign prostatic hyperplasia can increase resistance to urine flow in the urethra and lead to incomplete bladder emptying during micturition. The residual urine can act as a growth medium for pathogenic bacteria and increase the risk for UTI

Healthy 27 year old female presents with mild nausea that has been 'on and off' for the past 2 weeks. She has been picky about her eating due to her nausea and her diet consists mainly off eggs, fruits and vegetable. Urine beta hCG is positive. Supplementation of what substance is most likely unnecessary in this patient?

Vitamin A Large doses of preformed vitamin A (retinol) can be highly teratogenic, especially during the first trimester of pregnancy. Adverse effects include fetal microcephaly, cardiac anomalies, early epiphyseal closure, growth retardation, and spontaneous abortion

Accumulation of an abnormal prion protein is considered a cause of prion diseases. Characteristic microscopic findings are?

Vacuoles in the gray matter (spongiform encephalopathy) with no inflammatory changes Example is Crutzfeldt-Jakob

Valproate during conception increases risk of what fetal defect?

Valproate decreases levels of folic acid and increases risk of neural tube defects (meningocele, myelomeningocele)

BRAF is a protein kinase involved in activation of signaling pathways for melanocyte proliferation; the BRAF V600E mutation is seen in 40-60% of patients with melanoma. Drug that is a potent inhibitor of mutated BRAF that has significant anti-tumor effects with improved survival and long term outcomes in advanced stage disease?

Vemurafenib

45 year old man with SOB and hypoxemia who was admitted 3 days ago after MVC. Evaluation at that time revealed femur fracture and patient underwent open reduction and internal fixation of the fracture. This morning, patient suddenly became SOB and developed sharp chest pain. CXR is unremarkable and ABG shows a pH of 7.51, PaCO2 of 30, and PaO2 of 65. Primary cause of hypoxemia in this patient?

Ventilation/perfusion mismatch Pulmonary embolism is common in hospitalized and postoperative patients and classically presents with sudden onset SOB and pleuritic chest pain. It causes hypoxemia due to ventilation/perfusion mismatch; arterial PCO2 is usually normal or decreased Note: although fat embolism is also increased following a long bone fracture, patient lacks the typical skin rash and neurologic findings (confusion)

1 week old boy was born to a woman with gestational diabetes mellitus. Nursery course was uncomplicated and he was discharged at 30 hours following observation of normal feeding, voiding, and stopping. Weight, height, and head circumference are 50th percentile. PE shows II/IV harsh, holosystolic murmur best heart at the left mid to lower sternal border. Birth records show no murmur was heard by 2 different health care providers in the newborn nursery. Most likely diagnosis?

Ventricular septal defect VSD typically presents in the neonatal period after pulmonary vascular resistance has declined (4-10 days old). Clinical presentation depends on the size of the defect which ranges from an asymptomatic holosystolic 'blowing' murmur (small VSD) to heart failure (large VSD). Most are insignificant and close spontaneously.

Irreversible inhibitor of GABA transaminase, the enzyme responsible for GABA catabolism in the CNS. One of several drugs that can be used in cases of treatment resistant epilepsy

Vigabatran

Migratory thrombophlebitis, erythema and tenderness extending linearly along a limb, should raise suspicion for?

Visceral cancer A paraneoplastic syndrome of hypercoagulability may be seen in some patients with cancer, especially adenocarcinomas of the pancreas, colon, or lung. Hypercoagulability develops because adenocarcinomas produce thromboplastin like substance capable of causing chronic intravascular coagulations that can disseminate and migrate. Superficial venous thromboses may therefore appear in one site and then resolve, only to rear in another side. This is known as Trousseau syndrome (migratory superficial thrombophlebitis), an indication of visceral cancer.

Produce dextran that aid them in colonizing host surfaces, such as dental enamel and heart valves. These organisms cause subacute bacterial endocarditis, classically in patients with preexisting cardiac valvular defects after dental manipulation

Viridans streptococci

70 year old female with easy fatiguability, exertion dyspnea, frequent falls, and weight lossPE reveals symmetrically decreased vibratory sensation to the lower extremities. Her hemoglobin is 7.8 and a peripheral blood smear shows hypersegmented neutrophils. Best treatment for this patient?

Vitamin B12 Vitamin B12 and folic acid deficiencies cause similar hematologic pictures, however, neurological dysfunction is only seen in patients with vitamin B12 deficiency. If megaloblastic anemia due to vitamin B12 deficiency is mistakenly treated with folate alone, the neurologic dysfunction can worsen.

1 month old full term African American exclusively breast fed boy is brought in for routine check up. Supplementation with what substance should be recommended for this infant?

Vitamin D Breast milk content of vitamins D and K is typically insufficient for the nutritional needs of the newborn. Vitamin K is given parenterally to all newborns at birth to prevent hemorrhagic disease of the newborn. Exclusively breastfed infants require vitamin D supplementation due to lack of sunlight exposure and to prevent rickets Supplement vitamin D- dark skin pigmentation, exclusive breast feeding, lack of sunlight exposure Supplement iron- preterm/low birth weight & after 4 months of age until solid food intake provides adequate amount

Patient diagnosed with pulmonary TB started on isoniazid and rifampin several months ago. Neurologic testing demonstrates sensory ataxia and decreased pain sensation in distal extremities. Patients condition is most likely caused by?

Vitamin deficiency Isoniazid is structurally similar to pyridoxine (B6). As a result, this antibiotic increases the urinary excretion of pyridoxine, often causing a frank deficiency of vitamin B6, and also competes for vitamin B6 binding sites leading to the defective synthesis of neurotransmitters like GABA. Isoniazid induced neuropathy can usually be prevented with pyridoxine supplementation & is more common in elderly, alcoholics or those with other comorbidities (kidney or liver dysfunction). Usually occurs in the form of peripheral neuropathy

46 year old develops LE DVT ad is started on IV Heparin. Shortly after, he develops right sided weakness and facial droop. CT head shows left MCA stroke. What PE findings are likely to be present in this patient?

Wide splitting of S2 that doesn't change with respiration Paradoxical embolism occurs when a thrombus from the venous system crosses into the arterial circulation via an abnormal connection between the right & left cardiac chambers (patent foramen ovals, ASD, VSD). Atrial left to right shunts cause a wide and fixed splitting of S2 and can facilitate paradoxical embolism due to periods of transient shunt reversal (during straining or coughing).

Confounding occurs when the exposure-disease relationship is muddled by the effect of extraneous factor associated with both exposure and disease. Confounding bias can result in the false association of an exposure with a disease. Difference between confounding and effect modification?

With confounding, there is usually no significant difference between the strata (with effect modification when you stratify the data it will be statistically significant)

AN infant suffering from recurrent infections is diagnosed with an intracellular messaging abnormality. The abnormal gene codes for a cytoplasmic tyrosine kinase. Normally this tyrosine kinase is activated in CD19 cells by antigen exposure resulting in cell maturation. The abnormal gene responsible for this patients condition can be mapped to what chromosome?

X chromosome Bruton agammaglobulinemia is characterized by an intracellular messaging abnormality (defect in Bruton's tyrosine kinase) that results in recurrent bacterial infections. B cells cannot leave the bone marrow to enter the circulation, and antibodies aren't produced. Defective gene is on the X chromosome. Note: defects associated with chromosomes chromosome 7- CF, osteogenesis imperfecta, Ehlers Danlos chromosome 16- polycystic kidney disease, tuberous sclerosis chromosome 20- maturity onset diabetes of the young type 1 chromosome 22- DiGeorge syndrome

Expected arterial blood gas in high altitude exposure?

pH- 7.5 PaO2- 60 PaCO2- 20 HCO3- 15 High altitude exposure results in hypoxemia with respiratory alkalosis (due to low partial pressure of O2 in the inspired air causing hyperventilation). Over a course of a few days, chronic respiratory alkalosis sets in with a corresponding decrease in the serum bicarbonate level reflecting renal compensation

Crohn disease is characterized by patchy, TRANSMURAL inflammation of the GI tract. It can infect ANY part of the tract from the mouth to the anus. Associated with NONcaseating granulomas. Complications include?

strictures (due to bowel wall edema, fibrosis, and thickening of the muscularis mucosae), fistulas (due to penetration of ulcers through the intestinal wall), and abscesses.

Urea breath test is used to diagnose H. pylori infection. These gram negative bacteria produce urease, which transforms urea into CO2 and ammonia. What is measured in the breath test?

In the urea breath test patients consume radio labeled urea and have their breath tested for radio labeled CO2

Healthy 21 year old with a concern for developing cancer. She has a strong family history of early onset breast cancer and tissue sarcomas and a family history of Li-Fraumeni syndrome is suspected. If she has inherited this condition, germline DNA analysis would demonstrate what genetic abnormality?

Inactivating mutation affecting 1 allele In a patient with inherited inactivating mutation in a tumor suppressor gene, malignant transformation occurs when a single somatic cell subsequently acquires a second hit. This mechanism is behind most heritable cancer syndrome, including Li-Fraumeni. Note: deletion of both copies of a tumor suppressor gene occurs in malignant cells; this would be seen in a tumor cell analyzed from a Li-Fraumeni patient, however, the gremline DNA would demonstrate only a single inactivating mutation

Effect that antifungals have on Warfarin?

Increase prothrombin time/bleeding risk Flucanzaole inhibits cytochrome P450 mediated metabolism of warfarin in the liver. As a result, patients using flucanazole experience a GRADUAL increase in their steady state of warfarin and a corresponding rise in the PT over a period of several days.

54 year old man with a history of hepatitis c infection presents after several episodes of vomiting bright red blood. The next day after admission he's disoriented and can't identify the month or his current location. PE shows significant abdominal distention and gynecomastia. Repetitive flicking motions of the hands are seen when he is asked to outstretch his arms and dorsiflex his wrists. What event is most likely the precipitant of this patients altered mental status?

Increased absorption of nitrogenous substances by the gut Patient likely developed hepatic encephalopathy due to his GI bleed. Ammonia is normally produced by the GI tract as a result of enterocytic catabolism of glutamine and colonic bacterial catabolism of dietary protein. GI bleeding causes increased nitrogen delivery to the gut in the form of hemoglobin which is then converted to ammonia and absorbed into the blood. Hepatic encephalopathy is caused by increased levels of ammonia and other neurotoxins in the circulation that lead to increased inhibitory neurotransmission and impaired excitatory neurotransmitter release. Hepatic encephalopathy is frequently precipitated by a stressor (GI bleeding, infection) that increases blood ammonia levels. Treat with lactulose. Bacterial action on lactulose results in acidification of the colonic contents which converts ammonia into non absorbable ammonium ions, trapping it in the stool and increasing fecal nitrogen excretion. Note: This condition is associated with decreased BUN because the liver can't convert ammonia to urea efficiently

52 year old man with dizziness, headaches, and pruritus after showering. PE shows a ruddy complexion and mild splenomegaly. Hemoglobin, hematocrit, erythrocytes, platelets, and leukocytes are all elevated. Most likely cause of this patient's findings?

Increased bone marrow sensitivity to growth factors Polycythemia vera is a clonal myeloproliferative disease of pluripotent hemopoietic stem cells. Characteristic features include increased RBC mass, increased plasma volume, and low EPO levels. Additional manifestations can include elevated platelet and/or WBC count, thrombotic events (from blood hyper viscosity), peptic ulceration and pruritus (due to histamine release from basophils), gouty arthritis (increased cell turnover). PE shows plethoric, ruddy face and splenomegaly. Majority of patients with PV have the JAK2 V617F mutation, rendering hematopoietic stem cells more sensitive to growth factors such as EPO and TPO. Treat with serial phlebotomy as necessary to keep hematocrit less than 45%. Secondary polycythemia is associated with an increase in RBCs only & is not associated with splenomegaly

If you clamp the renal artery what physiologic change is most likely to be seen?

Increased filtration fraction Kidneys act to maintain a relatively constant GFR across a wide range of perfusion pressures. Fall in renal perfusion pressure causes efferent arteriolar contraction which helps improve GFR at the expense of renal plasma flow. Filtration fraction (GFR/RPF) therefore will increase as renal perfusion pressure drops

Patient states that he is not interested in known his diagnosis and further discussion indicates that the patient is cognitively intact without signs of mental illness. Family states that they dont want the patient knowing any further information abut his condition. Most appropriate response to family?

"I will not inform him of the results if that is his preference' Patients have the right to refuse to receive medical information. Physicians must understand and respect the beliefs of cultures that value beneficence (doing whats best for patient) and nonmaleficience (not doing harm to patient) over autonomy (patient decides what they want to do). In these cultures, family members may be expected to make medical decisions to avoid perceived harm, disrespect, or mental distress caused by disclosure to the patient

62 year old with exertion dyspnea. Patient states he can't climb 2 flight of stairs without having to stop to rest so he hardly gets out anymore. Reports he's not going to quit smoking and that he knows its bad for him. Most helpful statement the physician can make to encourage him to stop smoking?

"you enjoy smoking, but you also want to breathe better and get out more' Motivational interviewing is a type of psychotherapy used to help patients who are not ready to make a change. It involves acknowledging resistance & ambivalence, highlighting DISCREPANCIES and encouraging internal motivators for change

Attending informs patient that she needs to undergo pericardiocentesis. He is then called to assist with an urgent case and asks the intern to obtain informed consent for procedure. Intern has never performed or observed a pericardiocentesis. Best appropriate action by intern?

'Ask the supervising resident to assist in obtaining informed consent for pericardiocentesis' Informed consent should be obtained by a provider who has sufficient knowledge to give an accurate description of the intervention, risks and benefits of the treatment, and available alternative treatments and to answer all of patients questions.

Patient tells you that her and her daughter got into a argument recently and she has been trying to hard to make everything right since they recently moved to the area. Most appropriate response by the physician?

'How did that make you feel' Open ended questions allow patients to talk about the emotions or issues they feel are most important. Open ended questions are vital to obtaining a detailed history and establishing a good rapport.

Patient gives you a thank you card and asked you to accept basketball game tickets as he wants to thank you for being such a wonderful doctor. Most appropriate response?

'I appreciate your thoughtfulness and will treasure the card you have made for me. However, it would not be right for me to accept the tickets.' It is ethically problematic for physicians to accept expensive gifts as they may influence or appear to influence physician behavior. These gifts should be declined after expressing appreciation for the gesture. Accepting gifts of low monetary value given as a token of appreciation or cultural tradition and that are not intended to influence care is appropriate.

Patient with chronic back pain comes to office for appointment with a new physician. He is requesting a disability form be signed & gets angry when physician asks about his previous treatment stating 'I wouldn't be here if anything had worked'. Records review normal MRI a year ago and inconsistent adherence with rehab. Patient walks normally and there is no apparent discomfort or limitations. Most appropriate response by physician?

'I cannot sign the form without further history and assessment' Physicians are frequently asked to evaluate whether a patient is entitled to disability benefits. When interacting with demanding patients, its best to explain that the physician has a responsibility to perform a thorough assessment prior to making this determination

5 year old boy presents for well visit. Childs father says 'I think you should know that my son is playing with his sisters dolls and doesn't seem to like cars or trucks like most other boys. Is that normal?' Most appropriate response?

'I hear your concern, however many boys play with dolls rather than trucks and this is developmentally normal when exploring the world' Although children develop gender identity by age 4, experimenting with opposite gender activities is normal form of exploration. Children tend to know gender is permanent by age 5-6. In contrast, gender dysphoria is the prolonged and intense feeling that one is the opposite gender from one's birth sex. This is likely to carry into puberty and people with this condition have increased rates of psychiatric illness & need monitoring and support

Patient states 'please don't tell me what they find because if its cancer i don't want to know'. Most appropriate response to this patients request?

'Tell me more about your concerns so I can better understand why you feel this way' Patients have the right to have information withheld from them regarding their medical condition. Physicians must respect their wish not to know, but should also explore the patient concerns to better understand their preferences and maximize their involvement in subsequent medical decision making

Net filtration pressure equation?

(Hydrostatic pressure capillary -hydrostatic pressure interstitium) - (oncotic pressure capillary -oncotic pressure interstitium) Note: a negative result reflects movement from the interstitium into the capillaries and a positive result reflects movement from the capillary into the interstitium

56 year old with dehydration secondary to excess output from an ileostomy. Patient had a total colectomy with diverting ileostomy for colon cancer 5 years ago. For the past 6 months he has had increased output from the ileostomy and has been admitted twice with similar episodes of dehydration. He reports flank pain on second day of admission. X-ray shows no evidence of renal calculi. US shows a stone in the right ureter. He passes the stone and is is a diamond/rhombus shape. What is the underlying mechanism leading to stone formation in this patient?

Increased hydrogen ion excretion in the kidney Uric acid stones appear as yellow/brown diamond or rhomboid shaped crystals that are radiolucent on plain radiography. Biochemical risk factors for uric acid nephrolithiasis include low urinary pH and low urine volume with high uric acid concentration which can occur in the setting of chronic diarrhea. They can also occur in the setting of gout, high cell turnover states (lymphoproliferative disorders), and metabolic syndrome. Patients with chronic diarrhea or those who have had a colectomy have reduced bicarbonate resorption from the gut, leading to a state of chronic metabolic acidosis. The kidneys compensate by increasing excretion of H+ and resorption of bicarbonate in the collecting ducts. This increases the acidity of the urine and increases the conversion of soluble urate salts into insoluble uric acid. Stone formation is further promoted by dehydration due to production of concentrated urine.

35 year old alcoholic male with hematemesis. He has had three episodes of vomiting. Endoscopy shows longitudinal mucosal tears in the GE junction. Patients condition is most likely related to?

Increased intraabdominal pressure Mallory-Weiss tears account for 10% of cases of upper GI bleeding. They occur due to increased intraluminal gastric pressure due to retching, vomiting, or other abdominal straining

MOA lantanoprost eyedrops of improving glaucoma?

Increased outflow of aqueous humor Glaucoma is a form of optic neuropathy characterized by increased intraocular pressure associated with increased production or decreased outflow of aqueous humor. Topical prostaglandins (latanoprost) increase outflow of aqueous via the uveoscleral pathway and are the preferred treatment for open angle glaucoma. They increase pigmentation in the iris and eyelashes Muscarininc agonists increase trabecular outflow Open angle glaucoma evolves over decades with progressive loss of peripheral visual fields. It can be visualized as a pale optic disc and an enlarged optic cup Note: -Carbonic anhydrase inhibitors (dorzolamine) decrease formation in the ciliary body leading to decreased sodium and fluid transport and decreased production of aqueous humor -Alpha adrenergic agonists (bromonidine) inhibit production of aqueous humor due to vasoconstriction in the ciliary body -Beta blockers (timolol) decreased production of aqueous humor

40 year old woman with 3month history of progressive limitation of physical activity due to fatigue. Had positive tuberculin skin test 7 months ago with normal CXR and has been compliant with prescribed treatment despite bitter taste. PE shows tired woman with conjunctival and palmar pallor. CBC reveal low hemoglobin, hematocrit, and MCV. Bone marrow aspirate stained with Prussian blue is shown. Decreased activity of what enzyme explains the anemia found in this patient?

ALA synthase Patient has latent TB and has lab values and a bone marrow aspirate consistent with sideroblastic anemia due to isoniazid use. Isoniazid directly inhibits the enzyme pyridoxine phosphokinase which normal concerts pyridoxine (B6) to its active form, pyridoxal 5' phosphate. Pyrodixal 5' phosphate is a cofactor for ALA-synthase, the enzyme that catalyzes the rate limiting step in heme synthesis. Inhibition of this enzyme causes microcytic hypo chromic anemia. Iron is transported to the developing erythrocytes that cannot form heme, and its granules accumulate circumferentially around the nucleus forming ring sideroblasts. Causes of sideroblastic anemia: X-linked ALA synthase mutation, myelodysplastic syndrome, alcohol abuse, copper deficiency, medications (isoniazid, chloramphenicol, linezolid) Since pyridoxal 5' phosphate is a cofactor for numerous enzymes, pyridoxine deficiency can also lead to dermatitis, stomatitis, neuropathy, and confusion.

18 month old boy with language regression. His moods have become more unpredictable over the past 4 months with frequent tantrums. They tried to bring him in sooner, but they live in an impoverished part of the city and had financial difficulties with transportation to the office. On exam the child is quiet and maintains appropriate eye contact. Hemoglobin is 9. Most likely enzyme inhibited in this patient?

ALA-dehydratase Lead toxicity is most prevalent among IMPOVERISHED children who reside in homes built before 1978. Lead directly inhibits ferrochelatase and ALA dehydratase resulting in anemia, ALA accumulation, and elevated zinc protophorphryin (Zn combines with protophoryin IX since Fe 2+ can't). Neurotoxicity is a significant long term complication Note: pyruvate kinase deficiency is an AR condition that leads to hemolytic anemia and can present with pallor scleral icterus, and splenomegaly but does not present with behavioral regression or language deficits

4 week old boy evaluated for malrotation and volvulus. Normal rotation but constriction of the duodenum is noted. CT shows pancreatic tissue encircling the duodenum. Most likely cause of patient's condition?

Abnormal migration of the ventral pancreatic bud Annular pancreas, or pancreatic tissue encircling the descending duodenum, is caused by failure of the ventral pancreatic bud to properly migrate and fuse with the dorsal bud during the 7th and 8th week of fetal development. Annular pancreas is usually asymptomatic but may present with duodenal obstruction or pancreatiits Note: ventral bud gives rise to main pancreatic duct and uncinate process portion of the head & dorsal bud gives rise to pancreatic tail, body, and remainder of the head

15 year old with persistent fever and sore throat. PE shows anterior and posterior cervical lymphadenopathy and splenomegaly. Peripheral blood smear shows atypical lymphocytes. These atypical cells are?

Activated CD8+ cytotoxic lymphocytes Atypical lymphocytes observed in the peripheral blood smears of patients with infectious mononucleosis represent activated CD8+ cytotoxic T lymphocytes. These T-lymphocytes function to destroy virally infected B lymphocytes. They appear as cells much larger than the quiescent lymphocytes with abundant cytoplasm, an eccentrically placed nucleus and a cell membrane that appears to conform to the borders of neighboring cells

Long standing RA can affect the cervical spine causing vertebral malalignment (subluxation) that can affect the atlantoaxial joint. Extension of the neck during endotracheal intubation can worsen the subluxation leading to?

Acute compression of the spinal cord (causing flaccid paralysis with decreased or absent reflexes below the level of the compression due to spinal shock) and/or vertebral arteries. Note: the flaccid paralysis becomes spastic as the spinal shock resolves over days/weeks

63 year old with fatigue and easy bruising. No lymphadenopathy on exam. Hemoglobin is low. Platelets are low. Leukocytes high at 20,000. Peripheral blood smear shows blasts. Most likely diagnosis?

Acute myelogenous leukemia Blast cells make the diagnosis of acute leukemia likely. Auer rods are helpful in making the diagnosis of AML. Median age of AML is 65. Most present with pancytopenia- fatigue from anemia, bruising/bleeding from thrombocytopenia, infection from functional neutropenia despite leukocytosis. WBC count is usually around 15,000. Diagnosis requires the presence of 20% myeloblasts in the bone marrow. Note: chronic leukemia causes prevalence of mature cells in the peripheral blood. In CML peripheral smears show many mature granulocytes and few blasts (<2%). In CLL smears show many mature lymphocytes.

Guanosine analog drug that has significant antiviral activity against HSV1, HSV2, and VZV but has weak activity against EBV and CMV?

Acyclovir Once acyclovir enters the infected host cell, it is phosphorylated to acyclovir monophosphate, principally via a virally encoded thymidine kinase. This is the rate limiting step in acyclovir activation. Acyclovir monophosphate is then phosphorylated by cellular enzymes into the active triphosphate form which impaired viral DNA polymerase mediated replication. EBV and CMV don't produce the take thymidine kinase as HSV and VZV and as a result infected cells can't easily convert acyclovir into its active form. Famciclovir and Valaciclovir are also more effective against HSV and VZV for this reason.

2 year old boy with several episodes of vomiting and lethargy. Mom says he developed a low grade fever and runny nose 2 days ago and has since been not eating well. Has a seizure while being evaluated. PE shows mild hepatomegaly. Labs show glucose of 40 and low blood ketone levels. What enzyme is most likely deficient in this patient?

Acyl-CoA dehydrogenase Hypoglycemia after prolonged fasting with inappropriately low ketone bodies suggests impaired beta oxidation. Medium chain acyl-CoA dehydrogenase deficiency is the most common genetic defect in beta oxidation

Patients echocardiography reveals a flail posterior mitral leaflet due to choral rupture and severe mitral regurgitation. What best reflects the patients cardiac hemodynamics compared to normal heart?

Afteroload decrease, preload increase, left ventricular ejection fraction increase Regurgitant flow into the left atrium in acute mitral regurgitation leads to increased left atrial pressure and increased left ventricular end diastolic volume (preload). The low resistance regurgitant pathway also decreases left ventricular afterload with a resulting increase in ejection fraction but overall decrease in forward stroke volume (since much of the total ejection fraction is lost to regurgitation to the left atrium). Increased left atrial pressure and decreased cardiac output result in pulmonary edema and severe hypotension, respectively.

81 year old brought to ED by neighbor due to abdominal pain that comes in waves and is associated with N and V. Patient is confused. On exam she has a low grade fever and mild tachycardia. She appears in painful distress. MM are dry and there is decreased skin turgor. Imaging reveals complete small bowel obstruction. Patient undergoes laparotomy with extraction of a hard mass obstructing the ileocecal valve. Cholesterol content of the mass is 85%. What radiographic finding is most consistent with this patient's presentation?

Air in the biliary tree Gallstone ileum is a mechanical bowel obstruction that occurs when a large gallstone erodes into the intestinal lumen. Pneumobilia (air in the biliary tract) is a common finding.

5 year old girl with intense penal itching, especially at night. Microscopic analysis of tape reveals presence of eggs that are asymmetrically flattened on one side. Most appropriate treatment?

Albendazole Enterobius vermicularis infection (enterobiasis) occurs most frequently in school age children and presents with perianal pruritus. Diagnosis is made by scotch tape test. Albendazole is first line treatment with pyrantel pamoate as an alternate agent preferred in pregnant patients

The Emergency Medical Treatment and Active Labor Act was enacted by Congress to prevent hospitals from inappropriately transferring, discharging, or refusing to treat indigent patients. It states that?

All patients who presents to the ED must receive an appropriate screening medical exam and stabilization of their condition, regardless of ability to pay

Study performed on several patients diagnosed with beta thalassemia. Patinets' beta global genes were evaluated and some were found to have a missense mutation in exon 1 , a few with nonsense mutation in exon 3, and others with promotor site mutation. However, all of these patients have similar clinical manifestations of the disease. What genetic principle best explains these findings?

Allelic heterogeneity Allelic heterogeneity exists when different mutations in the same genetic locus cause similar phenotypes. The heterogeneity of disease may be explained by the fact that some mutations can cause complete loss of protein function while others cause only partial loss. Note: genetic heterogeneity- mutations of different genes produce similar phenotypes phenotypic heterogeneity- mutations in the same gene produce different phenotypes

Drug class that is useful for the treatment of both BPH and HTN?

Alpha 1 blockers (doxazosin, prazosin, terazosin) Note: patients with CAD and HF along with HTN benefit from cardioselective beta blockers & hydrochlorothizide is the first line medication for treatment of essential HTN in the general population ( but can cause elevation in glucose, uric acid, and calcium and decrease in sodium so not recommended for DM, gout, or hypercalcemic patients) & ACE inhibitors are preferred for those with DM and HTN CCBs are useful for patients with evidence of vasospasm (Raynaud, Prinzmetal angina)

43 year old male diagnosed with M. tuberculosis. Resistance testing shows resistance to streptomycin. What best explains the resistance pattern seen by these bacteria?

Altered structure of bacterial ribosomal proteins (streptomycin inhibits protein synthesis by inactivating the 30S (small) ribosomal subunit) Mechanism of resistance to TB drugs: Ethambutol- increased activity of enzymes involved in cell wall polysaccharide synthesis (ethambutol interferes with mycobacterial peptidoglycan cell wall synthesis by inhibition of arabinosyl transferase) Isoniazid- decreased activity of bacterial catalase peroxidase Rifampin- structural alteration of enzymes involved in RNA synthesis (DNA dependent RNA polymerase)

45 yer old male with 1 day history of severe dyspnea on exertion that has been occurring with progressively shorter walking distances. ABG shows PaP2 of 54 and PaCO2 of 26. Most likely cause of blood gas abnormalities seen in this patient?

Alveolar hyperventilation Arterial PaCO2 is a direct indicator of alveolar ventilation status. Hypocapnia implies ongoing alveolar hyperventilation. Upper airway obstruction, reduced ventilatory drive, respiratory muscle fatigue, and decreased chest wall compliance are possible causes of HYPOventilation and hypercapnia

65 year old women with worsening memory problems. PE shows cognitive impairment but no focal neurologic deficits. Two years later, she dies suddenly from MI. Congo red staining of brain obtained from hippocampus reveals patchy red deposits that turn yellow green under polarized light. Patient most likely suffered from?

Alzheimer disease Amyloid is an abnormally folded (insoluble) extracellular protein that has apple green bifringence when stained with Congo red and viewed under polarized light. Alzheimer disease is associated with beta amyloid deposits in the brain parenchyma (neuritic plaques) and wall of cerebral vessels (amyloid angiopathy)

77 year old is prescribed a new medication for HTN. During his follow up three weeks later, he reports bilateral leg swelling but has no other symptoms. What medication was most likely prescribed to this patient to treat his HTN?

Amlodipine Amlodipine is a dihydropyridine CCB commonly used as mono therapy or in combination with other agents for treatment of HTN. Major side effects include headache, flushing, dizziness, and peripheral edema. Peripheral edema is due to the preferential dilation of the precapillary vessels (arteriolar dilation) that leads to increased capillary hydrostatic pressure and fluid extravasation into the interstitium. ACE inhibitors and ARBs can cause postcapillary ventilation and can normalize the increased capillary hydrostatic pressure reducing the risk of peripheral edema in patients taking CCBs.

A study is conducted with 4 groups each smoking a different range of cigarettes. FEV1 is quantitatively measured in all participants. Most appropriate statistical method to compare the mean FEV1 results among all 4 groups?

Analysis of variance (ANOVA) A t-test is used to compare the difference between the means of 2 groups. ANOVA compares the difference between the means of 2 or more groups. Results of t-test and ANOVA test will be equivalent when comparing the difference between the means of 2 groups. Prediction- buzzword for regression Relationship- buzzword for correlation

Patient with a hydatid cyst dies during surgery to resect the mass. Most likely cause of patients death?

Anaphylaxis Echinococcus granulosus is the most common cause of hydatid cyst (causes unilocular lesion vs. E. multilocularis which causes multilocular lesions). Spilling of cyst contents can cause anaphylactic shock. Surgical manipulation should be performed with caution due to this. Echinococcal larvae implant within the capillaries triggering an inflammatory reaction involving monocytes and eosinophils. Some larvae encyst. Microscopic exam demonstrates an encapsulated and calcified cyst ('eggshell calcification') containing fluid and budding cells that eventually become daughter cells. Outer wall is composed of gelatinous sheets surrounded by a thick fibrous capsule.

63 yar old with exertion dyspnea that has been progressing over the last 3 weeks. Has been sleeping in a recliner with his head elevated as he gets short of breath when laying flat. Distended jugular veins are seen in the semirecumbant position on exam. An apical heave is found on cardiac exam. Pitting edema is evident in the bilateral LE. Levels of what substance will be higher in the pulmonary vein compared to the pulmonary artery in this patient?

Angiotensin II Heart failure results in stimulation of the sympathetic nervous system and the RAAS system in an attempt to maintain an effective intravascular volume. Inactive angiotensin I is converted to active angiotensin II by endothelial bound angiotensin converting enzyme in the small vessels of the lungs (higher in the vein than the artery)

Patient presents for routine check up and is found to have serology anti-hepatitis A IgM negative & anti-hepatitis A IgG positive suggests?

Anicteric viral infection as a toddler HAV infection is most commonly silent or subclinical ('anicteric') in young children but can also present as an acute, self limited illness characterized by jaundice, malaise, fatigue, anorexia, nausea, vomiting, RUQ pain, or an aversion to smoking

52 year old female complaining of episodic loss of urine for the last several months. Urodynamic evaluation is significant for detrusor instability. Appropriate treatment for this patients condition include an agent that affects what receptors?

Antagonism of muscarinic cholinergic receptors (oxybutynin) Urge incontinence, or overactive bladder syndrome, is caused by uninhibited bladder contractions (detrusor instability). It results in a sense of urgency accompanied by an involuntary loss of urine. If behavioral therapy alone is unsuccessful, pharmacologic therapy with antimuscarinic drug (targeting M3 receptors) can improve symptoms. These agents should be used with caution in the elderly, as they can cause confusion and functional decline.

Where do you expect the most commonly populated MRSA site to be in carriers?

Anterior nares In the general population at any given time, 25-30% of individuals have nasal colonization with S. aureus. The anterior nares are most common side of colonization for both methicillin sensitive and methicillin resistant S. aureus. A colonized individual can be a carrier and transmit the organism by direct contact. Nasal carriage increases the risk of infections following surgery, peritoneal dialysis, and hemodialysis.

12 year old boy with chronic headaches and visual changes. Headaches have progressively worsened over the past year and are associated with nausea. An intracranial calcified mass is detected. Its removed and on exam shows cystic spaces filled with thick, brown yellow fluid rich in cholesterol. Mass is most likely derived from cells responsible for forming what structure?

Anterior pituitary Craniopharyngiomas are suprasellar tumors found in children composed of calcified cysts containing cholesterol crystals. They arise from remnants of Rathke's pouch, an embryonic precursor of the anterior pituitary that is derived from surface ectoderm. Note: posterior pituitary is formed from outpouching of diencephalon and is derived from neuroectoderm

12 year old boy with difficulty breathing, wheezing, and nonproductive cough. He has visited ED seven times over the past year for similar symptoms. PE reveals decreased breath sounds, prolonged expiration, and end expiratory wheezes. CXR shows hyperinflated lungs without infiltrates. Treated with B-agonists and corticosteroids and significantly improves. Returns 2 weeks later requiring intubation. What therapy will most likely decrease the likelihood of such events happening in the future?

Anti-IgE antibodies Omalizumab is an effective and acceptable add on therapy for patients with SEVERE allergic asthma. Its been shown to be effective in reducing dependency on both oral and inhaled steroids.

64 year old with 3 month history of cramping pain in right half that has gradually worsened. Pain is worse when he walks and is now sometimes present at rest. Pain is worse at night and interferes with sleep unless he sleeps in a chair. Diagnosed with T2DM 15 years ago. PE shows thinning of skin at right foot with small ulcer at the tip of the right second toe. Most likely cause of patients symptoms?

Arterial atherosclerosis Atherosclerotic plaques are characterized by eccentric intimal thickening with a fibrous cap, smooth muscle proliferation, local inflammatory infiltrate, and a lipid filled core. Symptoms of peripheral artery atherosclerosis include exertion leg pain, dermal atrophy and arterial ulcers. Leg elevation at night reduces blood flow and can cause pain.

47 year old man presents with persistent fever, night sweats and fatigue. He is diagnosed with CML. While undergoing treatment, he begins to complain of headaches, scant nasal discharge, and a problem with his left eye. PE reveals tenderness over the paranasal sinuses and left sided orbital swelling and cellulitis. Mild proptosis and ptosis of the left eye are also present. Biopsy of his sinus mucosa reveals fungal hyphae branching at acute angles with septations. Most likely cause of this patients condition?

Aspergillus fumigatus Immunosuppresant patients are at risk for Aspergillus fumigatus infection. This fungus produces thin, septet hyphae with acute V-shaped branching. It only exists in the mold form and is commonly grows on decaying vegetables. It causes aspergillosis, aspergillomas, and allergic pulmonary aspergillosis. Treat invasive disease with Amphotericin B Note: Mucor and Rhizopus are NONSEPTATE and branch at wide angles & are ribbon shaped and broad

62 year old man with long history of HTN presents for routine PE. Cardiac auscultation reveals a low frequency, presystolic sound that immediately precedes the S1 heart sound and is best heard during expiration when the patient is laying on his left side. CXR reveals extensive calcification around the mitral and aortic valves. Most likely explanation for the addition heart sound?

Increased stiffness of the LV wall A low frequency late diastolic sound on cardiac auscultation that immediately precedes the S1 is most often a fourth heart sound. It occurs after the onset of the P wave on the ECG coinciding with active phase of ventricular filling. An abnormal S4 can be heard in patients with reduced ventricular compliance (hypertensive heart disease: *what this patients has*, aortic stenosis, hypertrophic cardiomyopathy) due to a sudden rise in end diastolic pressure with atrial contraction

Woman started on estrogen and progesterone therapy 2 months ago for menopausal symptoms. Effect of this medication on thyroid hormone production?

Increased total (bound plus free)T4 & T3 levels An increase in estrogen activity, as seen in pregnancy or postmenopausal estrogen therapy, increases the level of thyroxine binding globulin. This leads to an increase in total thyroid hormone levels but feedback control maintains normal levels of free (biologically active) thyroid hormone. Patients remain euthyroid and TSH returns to the normal range.

66 year old with dizziness, dysarthria, and bilateral limb ataxia that has been progressively worsening over the past 2 weeks. He has a 50 pack year smoking. CXR shows a mass in his right lung. He dies despite treatment. Autopsy shows extensive cerebellar purkinje cell degradation. What term best describes the etiology of the patients neurologic condition?

Autoimmune Paraneoplastic syndromes can result from production of hormone like substances from tumor cells or from immune reactions against tumor cells that cross react with normal cells, causing dysfunction/damage to healthy organs and tissues. Paraneoplastic cerebellar degeneration is due to an immune response against tumor cells that cross react with purkinje neuron antigens, leading to acute onset repaid degeneration of the cerebellum. Anti-Yo, anti-P/Q and anti-HU are most common antibodies in the serum, but are not always detectable Note: cerebellar metastasis can cause similar symptoms, but headaches or decreased consciousness are often present due to mass effect and autopsy would show metastatic cancer cells with surrounding inflammation rather than purkinje cell degeneration Note: if the etiology was vascular it would occur over several years and affect multiple vascular areas throughout the brain & autopsy would show areas of liquefactive necrosis (acute infarcts) and glial scarring (chronic infarcts)

Newborn undergoes abdominal US that shows kidneys are normal size, structure and location. Patient doesn't appear to be in any distress and PE is normal. If patient were to be diagnosed with renal disease lateral in life it would most likely be?

Autosomal dominant polycystic renal disease ADPKD manifests in patients 40-50 years old with enlarged kidneys, HTN, and renal failure. In newborns, the kidneys are of normal size and the cysts are too small to be detected on abdominal US. As the cysts enlarge, they compress the renal parenchyma and cause symptoms

39 year old with toxic lithium level. During emergent hemodialysis, his blood is passed along a semipermeable membrane and allowed to equilibrate with a dialysate solution. What would be most likely to increase the rate of drug removal?

Increasing the surface area of the membrane Diffusion speed across a semipermeable membrane increases with higher molecular concentration gradients, larger membrane surface areas, and increased solubility of the diffusing substance. Diffusion speed decreases with increased membrane thickness, smaller pore size, higher molecular weights, and lower temperatures

5 weeks old boy with rapid breathing and tiring with feeds. Was born at home after an uneventful pregnancy. Mother declined all prenatal testing and ultrasounds. Cardiovascular exam is notable for a hyperdynamic precordium, a mid-diastolic rumble at the left sternal border, and a 3/6 holosystolic murmur in the apex that radiates to the left axilla. Echocardiogram shows defects in the lower part of the interatrial septum and the inter ventricular septum. Condition is most likely associated with what genetic condition?

Autosomal trisonomy A complete AV canal defect is comprised of an ASD, VSD, and a common AV valve. Its the most common congenital anomaly associated with Down syndrome. Due to failure of endocardial cushion fusion. Significant left to right shunting and AV valve regurgitation lead to excessive pulmonary blood flow and symptoms of HF (tachypnea, poor feeding). Auscultatory findings of AV valve regurgitation (holosystolic, best heard at apex) and increased pulmonary venous return (mid-diastolic rumble) are characteristic.

Difference in serology in a patient who has received HBV vaccine and someone who is recovered from the virus?

Individuals who are successfully immunized against HBV will have protective anti-has antibody. Individuals who recover from HBV infection will have anti-has AND anti-nbc antibodies. Presence of circulating HBsAg indicates active infection

Anterior horn cells of the spinal cord demonstrate cell body rounding, peripheral displacement of the nuclei and dispersion of Nissl substance to the peripheral of cells. These findings most likely indicate?

Axonal reaction The changes in the body of a neuron after the axon has been severed are called axonal reaction. This process reflects an increased protein synthesis that facilitates axon repair. Enlarged, rounded cells with peripherally located nuclei and dispersed finely granular Nissl substance are seen. Axonal reaction becomes visible 24-48 hours after the injury. Maximal changes are see at 12 days after the injury. Note: transient, profound hypoxia, ischemia, or toxic injury can lead to loss of neurons --> irreversible neuronal injury manifests with shrinkage of the neuronal body, deep eosinophilia of the cytoplasm, pyknosis of the nucleus, and loss of Nissl substance

23 year old man diagnosed with chronic hepatitis and corneal changes that are visible to the naked eye. Patient is most likely to have what condition?

Basal ganglia atrophy Kayser-Fleischer ring is an ophthalmologic finding most strongly associated with Wilson's disease. Caused by granular deposition of Copper within Descemet's membrane in the cornea. Its seen most frequently in patients with neuropsychiatric complications. Basal ganglia atrophy is typically present in these patients. First line treatment is copper chelators such as D-penicillamine and trientine

24 year old woman with three day history of fever, dyspnea, and productive cough of yellow sputum. Temperature is 102. She has bronchial breath sounds and crackles in the right lower lung field. Labs show normal hemoglobin and platelets but elevated leukocytes (54,000) with high neutrophils, bands, myelocytes, and metamyelocytes. Leukocyte alkaline phosphatase is elevated. Most likely additional finding on patients blood smear?

Basophilic oval inclusions in mature neutrophils Leukemia reaction is a benign leukocytosis (>50,000) that occurs in response to an underlying condition such as severe infection/hemorrhage, malignancy (leukemia) or acute hemolysis. Leukocyte alkaline phosphatase levels are normal or increased. Peripheral smear can show increased bands, early mature neutrophil precursors (myelocytes) and granules (Dohle bodies) in the neutrophils. The blue color in Dohle bodies is due to ribosomes bound with right endoplasmic reticulum. They are commonly seen in toxic systemic illness but can also occur with burns or myelodysplasia. Other findings of systemic inflammation include increased bands (left shift), toxic granulation, and cytoplasmic vacuoles

28 year old with hyperthyroidism. PE shows bilateral conjunctival redness and severe proptosis. Appropriate therapy is begun and on a follow up visit 4 weeks later her eye symptoms have improved & exam reveals a small decrease in proptosis without any redness. The drug that improved her ocular symptoms most likely did do by affecting?

Inflammatory infiltration Graves ophthalmopathy is caused by stimulation of orbital fibroblasts by thyrotropin receptor antibodies and cytokines released by activated T cells. Excess deposition of extracellular glycosaminoglycans and inflammatory infiltration lead to expansion of extra ocular muscles and retro-orbital tissues. Glucocorticoids improve Graves ophthalmopathy by decreasing the severity of inflammation and reducing the excess extra ocular volume.

18 year old with severe right groin pain and difficulty bearing weight on the RLE. Was hospitalized 6 months ago for similar pain in his proximal humerus but was discharged without a definitive diagnosis. On PE he has nontender hepatosplenomegaly. Radiograph shows marked osteopenia with cortical collapse of the proximal right femur. Enzyme deficiency is found to be the cause. Most likely enzyme thats deficient?

Beta-glucocerebrosidase Gaucher disease is the most common lysosomal storage disease that results in glucocerebroside accumulation. Clinical features include splenomegaly which can lead to pancytopenia due to rapid splenic destruction of blood cells and skeletal destruction, which can lead to severe bone pain and avascular necrosis

34 year old woman evaluated for several months of hearing loss. Has noticed a disturbing ringing noise in her left ear. Neurologic exam shows left sided facial numbness, asymmetric smile, and decreased afferent and efferent corneal reflex responses in left eye. She has diminished hearing on the left. Location of intracranial mass?

Between the cerebellum and lateral pons Acoustic schwannomas are commonly located at the cerebellopontine angle and arise from CN VIII. Patients may have ipsilateral sensorineural hearing loss/tinnitus and vertigo (CN VIII), loss of facial sensation (CN V), and facial paresis (CN VII). Bilateral acoustic neuromas are associated with NF2.

14 year old girl with 5th percentile height and 25th percentile weight. She has a short and thick neck, broad chest, and shortened 4th metacarpals bilaterally. A murmur is head on cardiac auscultation. What would most likely be seen on echocardiogram?

Bicuspid aortic valve Turner syndrome is associated with congenital anomalies of the aorta, and the most common defect is a bicuspid aortic valve. A nonstenotic bicuspid aortic valve can manifest as an early systolic, high frequency click over the right 2nd interspace. Bicuspid aortic valves are at risk for stenosis, insufficiency, and infection.

Healthy volunteer is given a slow IV infection of PAH (para-aminohippuric acid). Concentration of this substance is most likely to be lowest in what nephron segment?

Bowman's space PAH is primarily secreted into the nephron by the proximal tubule, but some is also freely filtered by the glomerulus. PAH is not reabsorbed by any portion of the nephron. Therefore, tubular fluid concentration of PAH is lowest in Bowman's space.

53 year old man with 2 week history of progressive fatigue and exertion dyspnea. Gets SOB after walking a few blocks. Cardiac auscultation reveals a murmur best heart when patient sits up and leans forward. Patient's murmur would most likely be loudest at what point on cardiac cath graph?

C Aortic regurgitation causes a decrescendo diastolic murmur with maximal intensity occurring just after closure of the aortic valve, when the pressure gradient between the aorta and left ventricle is the highest. It is best heart when patient is LEANING FORWARD. The pressure tracing for aortic regurgitation is characterized by loss of the aortic dicrotic notch, steep diastolic decline in aortic pressure (because blood is regurgitating back into LV), and high peaking systolic pressures (LV tries to compensate for regurgitant back flow by increasing SV, resulting in higher systolic BP with combined low aortic diastolic pressure leading to wide pulse pressure)

65 year old with a large left lung mass. Recently developed left shoulder pain, persistent hiccups and dyspnea. Compression of nerve arising from what location is most likely causing his recent symptoms?

C3-C5 Phrenic nerve arises from C3-C5 segments of spinal cord and innervates the ipsilateral hemidiaphragm. Intrathoraic spread of lung cancer can affect the phrenic nerve causing hiccups and diaphragmatic paralysis with dyspnea. Brachial plexus involvement can cause pain in the distribution of the C8, T1, and T2 nerve roots. Involvement of recurrent laryngeal nerve can cause hoarseness.

32 year old with history of HIV with worsening headache and confusion. Patient had been acting inappropriately over the last 2 days. Today he assaulted a coworker because she was 'staring' at him. Patient takes antiretroviral medications and last CD4 count was 400. Temperature is 100.9. Patient appears lethargic. MRI reveals temporal lobe findings as shown in the image (edema/hemorrhage in the temporal lobe). What test is most likely to establish the diagnosis in this patient?

CSF PCR for herpesvirus HSV1 is the most common cause of sporadic encephalitis. The temporal lobe is the main region of the brain affected. Patients frequently have fever, headache, seizures, altered level of consciousness, and symptoms of temporal lobe dysfunction (olfactory hallucinations, personality changes, psychosis) HSV1 --> olfactory tract --> olfactory cortex (temporal lobe) Treat with IV Acyclovir

24 year old woman with 3 week history of progressive fatigue. PE shows splinter hemorrhages involving the nail beds. Further evaluation will most likely reveal additional abnormalities in what part of her PE?

Cardiac auscultation Microemboli from the valvular vegetations of bacterial endocarditis are the most common cause of subungual splinter hemorrhages. The presence of these lesions necessitates careful cardiac auscultation to detect possible new onset regurgitant murmur. Progressive fatigue may be the presenting symptom of subacute bacterial endocarditis. Note: Janeway lesions (small, macular erythematous or hemorrhagic contender lesions on the palms and soles) are also a sign of micro embolism

32 year old with HIV presents with lethargy, fever, and frontal headache. India ink staining of CSF shows encapsulated yeast. Patient is hospitalized and appropriate treatment is started. Several days later, he develops impaired renal function and hypokalemia. Antimicrobial agent most likely responsible for these side effects act on what fungal cell structure?

Cell membrane Amphotericin B is a polyene macrolide anti fungal medication commonly used to treated Cryptococcus infections. It binds ergosterol (a sterol unique to fungi) creating pores in fungal cell membranes. Nephrotoxicity is this drugs major adverse effect, affecting 80% of patients. Other common side effects are hypokalemia, hypomagnesemia, and renal tubular acidosis Note: treatment of C. neoformans is with combination of Amphotericin B with flucytosine followed by long term fluconazole FLucytosine is a pyrimidine analog whose metabolites impair fungal DNA replication and RNA synthesis. Human cells are unable to synthesize active metabolites. Adverse effects include colitis and bone marrow suppression due to release of active metabolites by gut flora

75 year old man presents after high speed MVC. Resuscitation was unsuccessful. No significant PMH. At autopsy, heavy calcifications of the aortic valve are seen. What most likely preceded the aortic valve changes seen?

Cell necrosis Patient's aortic valve calcifications most likely represent dystrophic calcification affecting an aging aortic valve. Dystrophic calcification occurs in damaged or necrotic tissue in the setting of normal calcium levels. Thought to be the result of endothelial and fibroblast death secondary to chronic hemodynamic stress or atherosclerotic inflammation. Release of cellular degradation products into the valvular interstitium promotes calcification and thickening of the valve leaflets and annulus. These changes are benign in elderly adults 9aortic sclerosis) but over time progressive valvular stiffening can lead to outflow obstruction (calcific aortic stenosis) Metastatic calcification occurs in normal tissue in the setting of hypercalcemia. Calcium deposition in this process typically occurs in more alkaline tissues involved in acid excretion such as the kidneys, lungs, systemic arteries, and gastric mucosa.

65 year old male complains of bilateral visual difficult that has progressed over the past year. Fundoscopy reveals small yellow retinal lesions clustered in the macula. What would you expect on visual field examination?

Central scotomas A scotoma is a visual field defect that occurs due to the pathologic process that involves part of the retina or the optic nerve resulting in a discrete area of altered vision surrounded by zones of normal vision. Lesions of the macula cause central scotomas. Patient is likely to have macular degeneration. Macular degeneration is characterized by progressive loss of central vision due to deposition of fatty tissue (drusen) behind the retina (dry MD) and neovascularization of the retina (wet MD) Note: arcuate scotoma- damage to particular region of optic nerve head bitemporal hemianopsia- compression of medial optic chiasm binasal hemianopsia- compression of lateral optic chiasm homonymous hemianopia- transection of contralateral optic tract

Chest xray in acute decompensated heart failure with secondary pulmonary edema shows?

Cephalization of the pulmonary vessels, perihilar alveolar edema (batwing distribution), and blunting of the costophrenic angles due to pleural effusions. Kerley B lines (short horizontal lines perpendicular to the pleural surface that represent edema of the interlobular septa) may also be present. Cardiomegaly is also common. Patients typically present with progressive cough, dyspnea, orthopnea, and/or LE edema. PE reveals tachycardia, tachypnea, accessory muscle use, and bilateral crackles or wheezing. Common precipitationg factors are myocardial ischemia/infarction, severe HTN, valvular heart disease, arrhythmias (afib), drug use (cocaine)

Patient given isoflurane to achieve the desirable depth of CNS depression in preparation to undergo a major surgical procedure. An increase in what parameter is likely to happen during the anesthesia in this patient?

Cerebral blood flow Almost all volatile anesthetics increase cerebral blood flow. Its an undesirable effect as it results in increased ICP. Other important effects of inhalation anesthetics are myocardial depression, hypotension, decreased hepatic flow, respiratory depression and decreased renal function Note: halothane and sevoflurane have bronchodilation properties and are preferred for patients with asthma

In addition to affecting the PIP, MCP, MTP, wrists, ankles, knees, and elbows, rheumatoid arthritis can affect what area in longstanding disease?

Cervical spine (leads to severe pain and disability due to spinal instability with potential radiculopathy/cord compression). Note: hips and lumbosacral spinal are typically spared

Caused by chronic infection by Trypanosoma cruzi and causes secondary achalasia (megaesophagus) due to destruction of the submucosal (Meissner) and myenteric (Auerbach) plexus. Other manifestations include nonischemic cardiomyopathy & megacolon

Chagas disease Trypanosoma cruzi is a slender C or U shaped flagellated parasite with darkly staining nucleus and kinetoplast. Parasitosis related inflammation and immune mediates cross reactivity between the parasite and the enteric ganglia lead to destruction of the plexi. Patients have a higher risk for esophageal cancer.

Not admitted to NICU shortly after being born to a 28 year old woman who had poor prenatal care. Pulse Ox shows 85% on room air. PE is significant for orbital hyertelorism, a submucous cleft palate, and bided uvula. Echocardiogram reveals RVH, pulmonary stenosis with VSD, and overriding aorta. Patients diagnosis is confirmed by FISH. These findings are most consistent with what mechanism?

Chromosome microdeletion 22q11.2 micro deletion results in DiGeorge syndrome (cardiac anomalies, hypo plastic or absent thymus, and hypokalemia) and velocardiofacial syndrome (cleft palate, cardiac anomalies, dyspmorphic facies). Fluorescence in situ hybridization is the 'gold standard' for detecting a microdeletion. Defective neural crest migration into derivatives of the 3rd and 4th pharyngeal pouches results in maldevelopment of the thymus and parathyroid as well as subsequent T cell deficiency and hypocalcemia.

65 year old with progressive weight loss, jaundice, and anorexia over the last 3 months. Urine has bene dark and stools have been pale. Abdominal exam shows an enlarged but contender gallbladder. Strongest predisposing factor to patients current condition?

Cigarette smoking Smoking is the most important risk factor for pancreatic cancer. Other risk factors include advanced age, chronic pancreatitis, and genetic predisposition (Peutz Jeghers syndrome). Pancreatic adenocarcinoma should be considered in any patient with painless obstructive jaundice (elevated bilirubin, dark urine, pale stool) and weight loss. Courvioser (painless palpable gallbladder in a jaundiced patient) can also raise suspicion. Tumors of the pancreatic head can produce obstructive symptoms due to compression of the CBD whereas those in the body/tail tend to produce midepigastric pain due to invasion of the splanchnic plexus.

64 year old with chronic progressive loss of peripheral vision and elevated intraocular pressure. Patient started in timolol ophthalmic drops. Target of this medication?

Ciliary epithelium Open angle glaucoma is characterized by progressive loss of peripheral vision from elevated intraocular pressure. Timolol and other nonselective beta blockers work by diminishing the secretion of aqueous humor by the ciliary epithelium. Acetazolamide, a carbonic anhydrase inhibitor, also decreases aqueous humor secretion by the ciliary epithelium. Prostaglandin F2alpha (latanoprost, travoprost) and cholinomimetics (pilocarpine, carbachol) decrease the intraocular pressure by increasing the outflow of aqueous humor.

68 year old man with thigh and leg pain that worsens with exertion. PMG of DM and HTN. Smokes 2PPD. PE shows weak dorsalis pedis pulses bilaterally. Further evaluation confirms peripheral artery disease involving both lower extremities. What drug would best provide symptomatic improvement due to direct dilation of arteries and inhibition of platelet aggregation?

Cilostazol Symptomatic management of PAD includes graded exercise program and cilostazol. Cilostazol is a phosphodiesterase inhibitor that inhibits platelet aggregation and acts as a direct vasodilator. Cliostazol increases platelet cAMP levels which decreases platelet aggregation by preventing platelet shape change and granule release. Patients with PAD should also receive an anti platelet agent (ASA or clopidrogrel) for secondary prevention of coronary heart disease and stroke.

63 year old woman is concerned about wrinkles around her eyes that make her look old. A decrease in what is most likely responsible for patients complaint?

Collagen fibril production Photoaging is a product of excess exposure to ultraviolet A wave lengths and is characterized by epidermal atrophy with flattening of rate ridges. In addition, there is decreased collagen fibril production, increased collagen cross linking, an increased degradation of collagen and elastin in the dermis Note: UVB wavelengths are absorbed in the upper dermis and contribute to sunburn and increased risk of malignancy

21 year old woman with long history of episodic headaches, dizziness, and gait imbalance. Headaches are in occipital region and are characterized as dull, and last for several hours. PE reveals gait ataxia. MRI of break shows low lying cerebellar tonsils extending below the plan of the foramen magnum into the vertebral canal. Most likely cause of patients condition?

Congenital malformation (chiari type 1 malformation) Chiari malformations are congenital disorders that result from underdevelopment of the posterior fossa, causing parts of the cerebellum and medulla to herniate through the foramen magnum. Chiari type I (low lying cerebellar tonsils) is relatively benign and presents during adulthood with occipital headache and cerebellar dysfunction. Chiari type II (displacement of the vermis and tonsils and medulla) is more severe form that affects neonates and is often associated with lumbar myelomeningocele and non-communicating hydrocephalus due to aqueductal stenosis

18 year old female had heart murmur discovered during routine check up. Runs daily and is concerned murmur is a sign of heart disease and will prevent her from pursuing athletic activities. Has no family history of sudden cardiac death. Auscultation reveals mid systolic click followed by a short late systolic murmur at the cardiac apex. Murmur disappears with squatting. Patients condition is most likely related to an abnormality involving what type of tissue?

Connective tissue Cardiac auscultation in patients with MITRAL VALVE PROLAPSE typically reveals a non ejection (mid systolic) click and mid to late systolic murmur of mitral regurgitation. MVP is often caused by defects in mitral valve connective tissue proteins that predispose to myxomatous degeneration of the mitral leaflets and chordae tendinae. Squatting from a standing position increases venous return and LV volume, helping to bring the leaflets into a more normal anatomic arrangement.

58 year old with 1 day history of fever, headache, and vomiting. He underwent kidney transplant last year. Temperature is 103. Neck stiffness is present on exam. Gram positive rods with tumbling motility at room temperature are seen on CSF microscopy. Patients infection most likely transmitted by?

Contaminated food Listeriosis is most commonly transmitted through food ingestions and can cause meningitis in immunocompromised adults. Listeria is a gram positive rod with tumbling motility. It grow well in cold temperatures and can contaminate refrigerated food. Ampicillin is antibiotic of choice.

34 year old with SOB, dizziness, palpitations, and flushed skin after accidental exposure to chemical fumes. Patient is tachypneic and has a reddish discoloration of skin without cyanosis. Patient is immediately instructed to inhale amyl nitrite from a safety kid. Success of the antidote is dependent upon its ability to convert hemoglobin into a form with increased affinity for what substance?

Cyanide Cyanide positioning presents with reddish skin discoloration, headache, tachypnea, tachycardia, nausea/vomiting, confusion, and weakness. Labs reveal lactic acidosis and narrowing of the venous arterial PO2 gradient. Nitrites are oxidizing agents that are effective in treating cyanide poisoning due to their ability to induce methemoglobinemia. Methoglobin contains ferric (Fe3+) rather than ferrous (Fe2+). Cyanide binds to cytochrome c oxidase in the mitochondria and inhibits it. Cyanide binds to ferric iron more avidly than to mitochondrial cytochrome enzymes, diminishing cyanide's toxic effect. Hydroxycolbamin (B12 precursor) and sodium thiosulfate are also antidotes for cyanide positioning.

25 year old woman with difficulty breathing for the past hour. On PE there is prolonged expiration and scattered expiratory wheezes bilaterally. Several minutes after appropriate treatment is administered the patient begins to feel better. Treatment most likely improved this patient's symptoms by increasing the intracellular concentration of what substance?

Cyclic AMP Emergent relief of an acute asthma exacerbation is most quickly achieved with inhaled beta 2 adrenergic agonists. Beta 2 agonists bind to Gs protein coupled receptors and cause Gs protein dissociation, activation of AC, and production of cAMP as an intracellular messenger. Cyclic AMP activates protein kinase A which relaxes bronchial smooth muscle cells by inhibiting myosin phosphorylation and lower intracellular Ca2+ concentrations resulting in bronchodilation .

Mitochondrial enzyme that activates caspases and indirectly bring about cell death through intrinsic pathway apoptosis?

Cytochrome c

Patient with history of recently diagnosed PUD presents with acute gout attack and is started on a new medication that provides significant relief of symptoms. He returns a week later with persistent diarrhea and nausea. The drug used in this patient to treat gout affects what cell structure?

Cytoskeleton Colchicine inhibits tubulin polymerization into microtubules and can be used for acute treatment and prophylaxis of gout. Important side effects are nausea, abdominal pain, and diarrhea. May be administered initially at first signs of gout flare and can be administered again an hour later. May be used for prophylaxis while initiating a urate lowering therapy such as allopurinol. Avoid in patients who are elderly or have severe renal dysfunction

Novel chemotherapeutic drug that inhibits DNA synthesis is developed. Cancer cell lines are exposed to isotopically labeled CO2 and incubated in enriched growth media. The carbon isotopes become incorporated into the nucleobases of newly synthesized nuclear DNA. The first reaction in this process occurs in what cellular structure?

Cytosol CO2 is utilized in the de novo synthesis of pyrimidine and purine nucleobases. Carbomyl phosphate synthetase II is an enzyme that catalyzes the initial step in de novo pyrimidine synthesis and is located in the cytosol. Note: Carbamoyl synthetase I of the urea cycle is located in the mitochondria

24 year old with irregular bleeding for the past 2 months. Pregnancy test is positive. US reveals a central heterogeneous mass in the uterus with numerous discrete anechoic spaces. Patient undergoes D and C and pathology reveals edematous chorionic villi. Staining with p57 is negative. Most likely cause of patients condition is proliferation of?

Cytotrophoblasts Complete molar pregnancy is an abnormal proliferation of cytotrophoblasts and syncytiotrophoblasts without fetal tissue. Due to paternal imprinting, staining with p57 is negative. 'Snowstorm' pattern on ultrasound. Classic sonographic finding of central heterogeneous mass with numerous discrete anechoic spaces corresponds histologically with trophoblastic hyperplasia and edematous chorionic villi

57 year old male with history of alcoholic cirrhosis with altered mental status. When asked to extend his fingers as if stopping traffic, the patient makes rhythmic flapping movements. He is started on rifamixin. Most likely MOA of this drug when used to treat the patient's current condition?

Decreased intraluminal ammonia production Hepatic encephalopathy is a neurologic complication of cirrhosis due in part to the livers inability to convert ammonia to urea. A primary source of ammonia is degradation of nitrogen products by intestinal bacteria. GI bleeding can precipitate HE as hemoglobin breakdown leads to increased nitrogen products in the gut. Excess dietary protein intake is another common trigger. Others include infection, sedatives, and metabolic derangements. Treatments for hepatic encephalopathy include lactulose (increases conversion of ammonia to ammonium) and rifamixin which is a nonabsorable antibiotic that alters GI flora (decreases intraluminal ammonia production and absorption) RIfamixin is sometimes used for travelers diarrhea, as it inhibits bacterial RNA synthesis through binding with DNA dependent RNA polymerase

32 year old man started on infliximab for treatment of refractory Crohn disease. Ten days later he develops joint pain and a pruritic skin rash. Skin biopsy shows scattered areas of fibrinoid necrosis and neutrophil infiltration involving his small blood vessels. Most likely finding to accompany this patients condition?

Decreased serum C3 Serum sickness is a type III hypersensitivity reaction to nonhuman proteins (chimeric monoclonal antibodies, venom antitoxins) characterized by vasculitis resulting from tissue deposition of circulating immune complexes. Clinical findings include fever, pruritic skin rash, arthralgia, and low serum C3 and C4 complement levels

51 year old woman with asymmetry of her face when she smiles, puffs out her cheeks, and closes her eyes tightly. There is also effacement of the right nasolabial fold and her mouth is drawn toward the left side. What additional finding is most likely associated with this patients condition?

Decreased tearing from the right eye In addition to unilateral facial paralysis, patient's with Bells palsy may experience decreased tearing, hyperacusis, and/or loss of taste sensation over the anterior 2/3 of the tongue.

21 year old with several brief back to back generalized tonic clonic seizures and never fully regains consciousness between episodes. IV Lorazepam is administered and seizures stop. IV infusion of another drug is started simultaneously to prevent seizure reoccurrence. Most likely MOA of the second drug?

Decreases Na+ current in cortical neurons Status epileptics is a single seizure lasting >5 minutes or the occurrence of multiple discrete seizures with incomplete recovery of consciousness between episodes. The initial treatment includes IV Lorazepam and phenytoin given concurrently. Phenytoin is a long acting anticonvulsant that inhibits neuronal high frequency firing by reducing the ability of Na+ channels to recover from inactivation. phenytoin is given with Lorazepman to prevent reoccurrence of seizure activity regardless of response to Lorazepam

8 year old boy diagnosed with absence seizures. Mechanism of action of the medication used in this patient?

Decreases the Ca2+ current in thalamic neurons Ethosuximide, the first line treatment for absence seizures, prevents neuronal burst firing by blocking thalamic T-type Ca2+ channels. Side effects include nausea, vomiting, fatigue, and hyperactivity. If side effects aren't tolerated or if drug is ineffective, then valproic acid is used as second line treatment for absence epilepsy. Phenytoin, carbamazepine, and valproic acid inhibit high frequency firing by reducing the ability of Na+ channels to recover from inactivation.

46 year old woman with progressive dyspnea for the past 2 days. Patient had to sleep sitting up last night. The apical impulse is hyper dynamic. Cardiac auscultation reveals diminished S1 and an apical holosystolic murmur radiating to the left axilla. Diffuse pulmonary crackles heard bilaterally. What would increase the ratio of forward flow volume to regurgitant flow volume in this patient?

Decreasing systemic vascular resistance In patients with mitral regurgitation, LV afterload is determined by the balance of resistance between forward flow (aortic pressure) and regurgitant flow (left atrial pressure). A reduction in systemic vascular resistance increases the ratio of forward to regurgitant blood flow and improves cardiac output. Pharmacologic vasodilators (nitroptrusside) help to increase forward cardiac output and reduce pulmonary congestion in patients with MR.

36 year old man with a 3 month history of a pruritic rash over the elbows and knees. Also has a prolonged history of episodic abdominal discomfort, flatulence, and voluminous greasy stools. Skin exam shows papulovesicualr skin rash in groups with erosion and excoriations. Most likely skin diagnosis?

Dermatitis herpitiformis (characterized by erythematous pruritic papules, vesicles, and bull that appear bilaterally and symmetrically on the extensor surfaces) Histologically dermatitis herpitaformis is characterized by microabscesses containing fibrin and neutrophils at the tips of the involved papillae. The overlying basal cells become vacuolated and coalescing blisters form at the tips of the involved papillae. Pathogenesis involes formation of IgA antibodies against gliadin that cross react with epidermal transglutaminase. This condition is strongly associated with celiac disease in which you would expect to see increased intestinal intraepithelial lymphocytes, variable loss of villus height, and crypt hyperplasia (causing malabsorption) Gluten free diet improves both of these conditions.

Benign growths composed of broad fascicles of rounded fibroblasts with abundant cytoplasm and a significant surrounding collagenous matrix that infiltrates adjacent soft tissue

Desmoid tumors

23 year old man in MVC. Several days after hospitalization his ECF and ICF volumes are decreased and osmolarity is increase. What condition most likely caused these changes?

Diabetes insipidus Hyperosmotic volume contraction is caused by loss of free water (with retention of electrolytes). It can occur in patients with diabetes insipidus or as a result of decreased fluid intake/excessive sweating

26 year old girl presents with mom for concern of excessive weight loss. Patient has lost 10 pounds over last 2 months, but still thinks she appears overweight and is worried about gaining weight. Mom has hypothyroidism. On exam patient appears anxious. Thyroid is small and without palpable nodules. There is a fine tremor of outstretched hands. DTRs are 3+ diffusely. HR is 105 and regular. Skin is warm and moist. Labs show low TSH, elevated free T4, and undetectable thyroglobulin levels. Most likely to be seen on thyroid biopsy?

Diffuse atrophy of the thyroid follicles with decreased colloid Exogenous hyperthyroidism is characterized by high free T4, suppressed TSH, and low/undetectable thyroglobulin. It can occur with surreptitious levothyroxine abuse, use of animal sourced thyroid supplement, and erroneous dosing of thyroid replacement therapy. Over time, lack of TSH stimulation causes thyroid follicles to become atrophic.

1 day old diagnosed with hyperphenylalanemia by newborn screening. He's placed on a phenylalanine restricted diet with tyrosine supplementation. Several months later labs reveal infant has a normal phenylalanine level. Careful exam reveals some neurological abnormalities, including axial hypotonia and microcephaly. Further work up is notable for elevated prolactin and a cofactor deficiency is suspected. Most likely deficient enzyme?

Dihydrobiopterin reductase Disorders involving impaired tetrahydrobiopterin (BH4) levels, the most common being dihydrobiopterin reductase deficiency, account for 2% of phenylketonuria cases. BH4 is an important cofactor for both phenylalanine hydroxylase and tyrosine hydroxylase. Although phenylalanine levels can be corrected with dietary restriction, downstream deficiencies of dopamine, norepinephrine, epinephrine, and serotonin lead to progressive neurologic deterioration

13 year old female with urine discoloration. Was treated for facial impetigo three weeks ago. UA shows hematuria, mild proteinuria, and occasional RBC casts. Renal biopsy is most likely to demonstrate?

Discrete subepithelial humps on EM Histological picture of PSGN: -Enlarged hypercellular glomeruli on LM -'Lumpy bumpy' granular deposits of IgG & C3 on IF -Electron dense depots on the epithelial side of the BM on EM

Colonization of the gastric antrum with D shaped gram negative bacteria is associated with a decreased number of somatostatin producing antral cells. Depletion of these cells from the gastric antrum is most likely to cause what condition?

Duodenal ulcers H. pylori in antrum --> decreased somatostatin from antral D cells --> increased gastrin --> increased parietal cell H+ secretion H. pylori produced urease which converts urea to ammonia which alkalinizes the surrounding pH but injures the gastric epithelial cells. This is compounded by localized inflammation due to immunologic response against the bacteria. H pylori also releases cytosine that inhibit duodenal production of bicarbonate. Increased acid load is emptied into the under protected duodenum producing ulcers Note: GASTRIC ulcers occur when H. pylori colonize the gastric body. Chronic inflammation leads to multifocal atrophic gastritis and reduction in the number of acid producing parietal cells (which are mostly located in the gastric body). Delta cells of the antrum are not markedly reduced therefore acid secretion is often low to normal. Gastric ulcer formation results from direct mucosal damage caused by bacterial products (ammonia, cytosine) and the resultant inflammatory response. Its associated with metaplasia and malignancies (gastric lymphoma, adenocarcinoma) unlike astral predominant disease.

The flow rate of pancreatic exocrine secretions increases in response to the hormone secretin. Difference in fluid secretion composition in regards to flow rate?

During high flow rate, bicarbonate concentration increases while chloride concentration drops, where as low flow rate the opposite is true. A chloride-bicarbonate exchanger on the apical surface of the ductal cells helps maintain this inverse relationship

Lesions of the jugular foramen (due to tumors, trauma, infection) can result in jugular foramen (Vernet) syndrome which is characterized by CN IX, X, XI dysfunction. Symptoms include?

Dysphagia, loss of taste posterior 1/3 tongue, hoarseness, loss of gag reflex on the ipsilateral side, and deviation of the uvula to the normal side, sternocleidomastoid & trapezius muscle paresis

28 year old with infertility. Experiences pain with deep vaginal penetration. Cycles are accompanied by moderate to severe lower abdominal pain but are regular. Pelvic exam shows a normal sized retroverted uterus. Posterior vaginal fornix is very tender to palpation. Patients condition most likely involves?

Ectopic endometrial tissue Endometriosis referes to the presence of endometrial glands and stroma outside the uterus. It may be asymptomatic or present with dysmenorrhea, dyspareunia, and/or infertility. Risk factors are nulliparity, early menarche, and prolonged menses. Ectopic endometrium responds to hormonal influences of the menstrual cycle. Bleeding and shedding of this extrauterine endometrium leads to blood formation in these ectopic locations and the blood eventually undergoes hemolysis, resulting in inflammation. Local inflammation is followed by adhesion formation which distorts structure and function. Adhesions involving the uterosacral ligament an results in a fixed, retroverted uterus.

Increased creatinine after starting on an ACE inhibitor is best explained by the effect of the drug on what portion of the nephron?

Efferent arteriole ACE inhibitors reduce angiotensin II levels and cause efferent arteriole dilation decreasing GFR and filtration rate. This can precipitate ARF in patients with reduced intrarenal perfusion pressure (renal artery stenosis, CHF, hypovolemia, CKD)

Patient with sunburn. What is most likely to happen within his skin cells as a result of this sunburn?

Endonuclease nicking of the damaged DNA strand Pyrimidine dimers are removed by nucleotide excision repair. Pyrimidine dimers are formed in DNA as a result of ultraviolet light exposure. They are recognized by a specific endonuclease complex that initiates the process of repair by nicking the damaged strand on both sides of the pyrimidine dimer. The damaged segment is then excised, and replacement DNA is synthesized by DNA polymerase. Mutations that impair the components involved in nucleotide excision repair cause xeroderma pigmentosum, a condition characterized by severe photosensitivity and the development of skin cancers at a young age. Note: base excision repair (used for deamination of bases that can occur spontaneously or due to chemical exposures) is removal of abnormal bases by glycosylation without disruption of the phosphodiester backbone and then replacement with the correct base Note: DNA mismatch repair occurs during normal DNA replication

25 year old with newly diagnosed seizures presents with fever and skin rash. She was started on phenytoin 4 weeks ago. Temperature is 102. There is a diffuse confluent erythema involving 60% of the body, palpable generalized lymphadenopathy, and symmetrical facial swelling. What laboratory finding is most likely to be present in this patient?

Eosinophilia Drug reaction with eosinophila and systemic symptoms syndrome typically occurs 2-8 weeks after exposure to high risk drugs such as anticonvulsants (phenytoin, carbamazepine), allopurinol, sulfonamides (sulfasalazine), and antibiotics (minocycin, vancomycin). Patient develop fever, generalized lymphadenopathy, facial edema, diffuse skin rash, eosinophilia and internal organ dysfunction. Clinical improvement occurs over several weeks following withdrawal of the drug. Thought to be due to drug induced herpes virus reactivation followed by clonal expansion of T cells that cross react with the drug

57 year old male with suspected bacterial pneumonia is admitted and given cefrtiaxone and azithromycin for treatment. Soon after first dose of cheftriazone he complains of difficulty breathing, abdominal cramps and lightheadedness. Current BP is 70/50 & HR is 120. PE reveals diffuse maculopapular rash. What drug should be administered next to this patient?

Epinephrine Anaphylactic shock is characterized by vasodilation, increased vascular permeability, and bronchoconstriction. Epinephrine counteracts these physiological mechanisms and is the drug of choice for treatment of anaphylaxis. Note: diphenhydramine is a first generation antihistamine that competitively inhibits peripheral H1 receptors in the GI tract, blood vessels, and respiratory tract; may be used for treatment of anaphylaxis AFTER patient has been stabilized with epinephrine

27 year old woman with 1 day history of skin rash. Two weeks ago, she had an episode of recurrent genital lesions which began with small papule and subsequently became vesicular with ulceration, causing, and eventual healing. She has small linear erosions in the oral mucosa & erythematous round papules and target lesions with a dusky central area. Most likely diagnosis?

Erythema multiforme Erythema multiforme is a cell mediated (predominately CD8+) inflammatory disorder of the skin characterized by red papule that evolve into target lesions. Its most commonly associated with infections, particularly herpes simplex virus. Can also be associated with mycoplasma, medications (sulfonamides), malignancy, and collagen vascular diseases. Its a systemic immune response to a LOCALIZED infection.

61 year old found unresponsive on the sidewalk. Paramedics note breath smells of alcohol and he has vomitus in his mouth. He dies and autopsy shows foci of hemorrhage and necrosis in the mammillary bodies and gray matter surrounding the 3rd and 4th ventricles. Patients brain findings are most likely associated with a decrease in what?

Erythrocyte transketolase activity Chronic thiamine (B1) deficiency impairs glucose utilization in the CNS. This occurs due to decreased activity that use thiamine as a cofactor (pyruvate dehydrogenase, branched chain alpha ketoacid dehydrogenase, alpha ketoglutarate dehydrogenase, transketolase). Thiamine deficiency can be diagnosed if baseline erythrocyte transketolase activity is low but increases after addition of thiamine pyrophosphate.

13 year old is found to have a hemoglobin mutation that changes the partial pressure of oxygen at which hemoglobin is 50% saturated to 20mmHg. In comparison, normal hemoglobin becomes 50% saturated with oxygen at 26 mmHg. What sequelae will patient most likely develop as a result of her mutation?

Erythrocytosis P50 refers to the partial pressure of oxygen at which hemoglobin is 50% saturated. Hemoglobins with high oxygen affinity have a decreased P50 that is represented by a leftward shift of the oxygen dissociation curve. The reduced ability to release oxygen within the peripheral tissues leads to renal hypoxia, increased EPO synthesis, and compensatory erythrocytosis

56 year old with recent onset lethargy, confusion, and vomiting. Has been seeing a psychiatrist for the last several months for depression following the death of his wife. Labs show low pH, decreased PaCO2. Sodium is 136. Chloride is 96. Bicarb is 8. Several hours later he develops back pain, gross hematuria, and oliguria. An overdose of what substance is most likely responsible for his symptoms?

Ethylene glycol Sudden onset back pain, hematuria, and oliguria in a patient with high anion gap metabolic acidosis is suggestive of ethylene glycol poisoning. Ethylene glycol is metabolized to glycolate which is cytotoxic to the renal tubules and causes acute tubular necrosis. Glycolate is further metabolized to oxalate which contributes to the oliguric renal failure by precipitating in the kidneys and causing tubular obstruction. Urine microscopy will show envelope or dumbbell shaped calcium oxalate crystals

13 month old boy with an undescended right testicle. A round mass is palpated superior to the scrotum, medial to the right mid-inguinal point. Orchiplexy, the replacement and fixation of the testis in the scrotum is recommended to family. During the procedure, the malpositioned testis will be pulled through a physiologic opening in what structure?

External oblique aponeurosis Patient's undescended testicle is lodged in the inguinal canal (its palpable media to the deep inguinal ring) and must be mobilized through the superficial inguinal ring and stitched into place in the scrotum. Superficial inguinal ring is formed by an opening in the external oblique muscle aponeurosis. Note: the deep inguinal ring is a physiologic opening in the transversals fascia bounded by the transversus abdomis laterally and the inferior epigastric vessels medially; its located superior to the mid inguinal point (midway between the ASIS and pubic tubercle)

Psychiatric patient discharged home on olanzapine. What study should be performed at 3 month check up?

Fasting glucose and lipid panel As a class, second generation antipsychotics are associated with metabolic adverse effects (weight gain, dyslipidemia, hyperglycemia, increased risk of DM). Olanzapine and clozapine carry the greatest risk. Patients taking second generation antipsychotics require routine monitoring of metabolic parameters including weight, waist circumference, blood pressure, fasting glucose, and lipid profile. Note: second generation ziprasidone is associated with prolongation of the QT interval and requires routine ECG monitoring; second generation antipsychotic risperidone has been associated with a higher risk of elevated prolactin level

59 year old with fatigue and generalized itching that developed slowly over the past year. Exam shows mild hepatomegaly. Labs show increase in alkaline phosphatase, with moderate elevations in bilirubin and hepatic transaminases. Liver biopsy reveals dilated bile canaliculi with green brown plugs and yellow green accumulations of pigment within the hepatic parenchyma. Patient is at risk of developing what complication?

Fat soluble vitamin deficiency Fatigue, pruritus, hepatomegaly, and elevated alkaline phosphatase suggest cholestatic liver disease. Cholestasis can be due to hepatocellulr dysfunction or intrahepatic or extra hepatic biliary obstruction. Both obstructive and non obstructive cholestasis are characterized by the deposition of bile pigment within the hepatic parenchyma, often with green plugs in the dilated bile canaliciuli. When its prolonged, the reduction in bile flow causes intestinal malabsorption of fats and fat soluble vitamins while require bile salts for digestion

43 year old man comes to ED with 3 day history of persistent headaches. Has a history of HTN but has had poor follow up. Serum creatinine is 1.4. Bilateral papilledema is noted. BP is 224/115. Appears mildly confused during exam. Patient is started on an IV medication that causes arteriolar dilation, improves renal perfusion, and increases natriuresis. What agent was most likely administered?

Fenoldopam Fenoldopam is a selective peripheral dopamine-1 receptor agonist. It is given IV to lower BP in hypertensive emergency, especially in patients with renal insufficiency. Fenoldopam causes arteriolar dilation, increases renal perfusion, and promotes diuresis and natriuresis. Dopamine-1 receptor stimulation activates adenylate cyclase and raises intracellular cAMP resulting in vasodilation of most arterial beds with decrease in BP. Note: Hydralazine is a direct arteriolar vasodilator with with no significant effect on renal perfusion or naturesis; its not used in hypertensive emergency as its associated with reflex sympatheticc activation, resulting in increased HR and contractility and Na+ and fluid retention

24 year old G1 P0 presents for prenatal visit during 3rd trimester. Earlier in pregnancy she was involved in a minor car accident in which she was a passenger. Pregnancy has been otherwise noncomplicated. She has noticed a new breast lump since her last visit. On exam a 3cm mobile, contender and firm mass is palpated in the lower outer quadrant. Shortly after delivery the mass is decreased in size but remains firm, mobile, and nontender. Most likely diagnosis?

Fibroadenoma Fibroadenoma is a common estrogen sensitive (increases in size in pregnancy or in the luteal phase of menstrual cycle) benign breast tumor in young women. Typical presentation is a solitary, nontender, firm, well demarcated spherical and mobile nodule corresponding to an encapsulated proliferation of stromal and ductal elements Note: Fat necrosis is an inflammatory response to trauma/invasive procedures and presents as an irregular, firm mass accompanied by ecchymosis thats not affected by estrogen. Galactocele is a milk filled cyst due to a blocked breast duct (these do not occur before delivery). Fibrocystic disease is most common cause of palpable nodular masses in the breast. Presents as poorly defined, diffuse increase in cysts and fibrotic tissue throughout the breast. Lesions are numerous and painful with maximal tenderness during premenstrual phase of cycle

48 year old male with severe epigastric abdominal pain and vomiting after an episode of binge drinking. Four weeks later he is found to have a palpable upper abdominal ass and a cystic lesion visualized on CT. The inner walls of the lesion are most likely lined with?

Fibrous and granulation tissue Pancreatic pseudocyst is a common complication of acute pancreatitis. Its a collection of fluid rich enzymes and inflammatory debris. Its walls consist of granulation tissue and fibrosis. Unlike true cysts, pseudocysts are not lined by epithelium Note: Serous pancreatic neoplasms- glycogen rich cuboidal epithelium Mucinous cystic neoplasms- columnar mucinous epithelium Papillary variant of pancreatic adenocarcinoma- atypical cells forming papillary projections

34 year old woman with fever, dysuria, and flank pain. Urine culture grow colonies of motile, gram negative rods demonstrating green metallic sheen on eosin methylene blue (EMB) agar and hemolysis on blood agar. Most important bacterial factor responsible for this patients condition?

Fimbrial antigen E. coli is one of the dominant components of the normal flora in the intestinal tract of humans and animals. It causes 80% of all UTIs. P fimbriae are the most important virulence factor that uropathogenic E. coli express. Without P fimbriae, E. coli wouldn't be able to bind to uroepithelial cells and infect the bladder, ureters, and kidneys & the bacteria would simply be washed away on urination E. coli is facultative anaerobic and able to ferment both glucose and lactose. It grows well on blood, McConkey, and EMB agar plates. On EMB, organisms that ferment lactose (such as E. coli) bind to dye in the agar and produce colonies that have a distinct green metallic sheen. Most strains of E. coli demonstrate beta hemolysis on blood agar as well.

64 year old man with BPH is started on a new medication and after 6 months of therapy his urinary symptoms improve and his prostate volume deceases by 20%. Medication most likely responsible for the decreased prostate volume in this patient?

Finasteride 5 alpha reductase inhibitors (finasteride, dutasteride) block the conversion of testosterone to dihydrotestosterone in the prostate. These drugs reduce prostate volume in patients with BPH and relive the fixed component of bladder outlet obstruction.

39 year old HIV positive male with facial puffiness in the morning. Labs reveal proteinuria (3.5 g/day). Renal biopsy shows glomerular epithelial cell enlargement and vacuolization as well s varying degrees of glomerular capillary wall collapse. Many renal tubules show cystic dilation and are filled with proteinaceous material. Patient most likely suffers from?

Focal segmental glomerulosclerosis Collapsing glomerulopathy is a variant of FSGS characteristic of HIV associated nephropathy. The usual FSGS lesions are seen in addition to collapse and sclerosis of the whole glomerular tuft, glomerular epithelial cell proliferation and hypertrophy, and marked tubular injury with accompanying microcyst formation. FSGS is also common with heroin addiction and sickle cell disease.

26 year old involved in motorbike accident. Found to have a maxillofacial injury. While recovering from surgery, patient develops difficulty chewing food. On exam, jaw deviates to the right side when the patient is instructed to open his mouth. Nerve that has been injured in this patient exits the skill though what foramina?

Foramen ovale The third (mandibular) branch of the trigeminal nerve exits the skull through the foramen oval and innervates the muscles of mastication, including the master, the medial and lateral pterygoids, and temporalis muscles. Mandible deviates toward the paralyzed side. Masseter, medial pterygoid, termporalis: close jaw Lateral pterygoid: opens jaw

Rats exposed to carbon tetrachloride suffer rapid and extensive liver damage. LM exam of affected liber specimens show fatty change and hepatocyte necrosis. These changes are the result of?

Free radical injury P450 microsomal oxidase system play an important role in detoxification. In carbon tetrachloride poisoning, however, it produces free radicals that start a vicious cycle of hepatic injury

54 year old brought by daughter due to acting strange for the past 2 years. Makes inappropriate sexual jokes, has little regard for social rules, is often irritable, and is borderline aggressive at times. Patient denies that his personality is different from norma. When speaking to patient, physician notices patient has minimal verbal output and repeats 'it is what it is' when asked about his strange behavior. On PE he appears unkempt, but not other significant findings. Most likely has a condition involving?

Frontal cortex Frontotemporal dementia (Pick's disease) is associated with pronounced atrophy of the frontal love. It manifests initially with changes in personality, social behavior, and language that progress over time to a more global dementia with obvious neurocognitive deficits.

14 year old girl with elevated serum glucose, low serum bicarbonate, elevated anion gap, and elevated serum ketones. She is started on high flow IV fluids and infusion of regular insulin. As a result of this intervention, what substance is expected to increase sharply in liver cells?

Fructose 2,6 biphosphate Insulin increases intracellular concentrations of fructose 2,6 biphosphate promoting PFK-1 activity and the formation of fructose 1,6 biphosphate (glycolysis)

77 year old with fever and confusion. Urine culture grows Pseudomonas. Further analysis reveals that the isolated organism produces an enzyme thats located on the cytoplasmic surface of the cell membrane and catalyzes transfer of acetyl groups to exogenous substances. The bacteria are most likely resistant to what drug?

Gentamicin Aminoglycoside (gentamicin) resistance is most commonly due to antibiotic modifying enzymes. These enzymes add chemical groups to the antibiotic, which diminishes its ability to bind to the 16S ribosomal RNA within the 30S ribosomal subunit. Most of these antibiotic modifying enzymes arise via transfer of plasmids or transposons rather than chromosomal mutations

32 year old female with feeling of something stuck in her throat. Symptoms have been present for one hour and are worse when she tries to swallows alive. PE is normal. Barium swallow is unrevealing. Upper GI endoscopy reveals no abnormalities. Most likely diagnosis?

Globus sensation Globus sensation is the feeling of a 'lump in the throat' without any accompanying physical, endoscopic, or radiologic findings of esophageal obstruction

12 year old being evaluated for DM. He has a strong family history of the disease but his relatives have been able to maintain good glycemic control with dietary modification alone without need for anti diabetic medication, despite developing the disease early in life. Oral glucose tolerance test shows fasting blood glucose of 107 and a 2 hour postprandial glucose of 204. Patient most likely has a deficiency in what enzyme

Glukokinase Gluokinase is preferentially expressed in the liver and pancreatic beta cells where it plays an important role in glycogen synthesis and insulin secretion, respectively. Mutations that impair expression of glucokinase raise the glucose threshold required for insulin release, leading to maturity onset diabetes of the young. Presents as non-insulin dependent diabetes in lean young adults. Hyperglycemia is mild and doesn't require medication, and is not associated with an elevated risk of long term diabetic complications.

76 year old woman with T2DM was found unresponsive. Daughter reports they had a busy day yesterday with many activities that required extended walking and patient ate less dinner than usual last night because she felt tired. She only takes one oral medication for DM. Blood glucose is 44. Patients condition rapidly improves after administration of IV bolus of dextrose, but she becomes confused again several hours later. Repeat blood glucose is 49. What medication is most likely causing her current condition?

Glyburide Sulfonyureas increase insulin secretion by pancreatic beta cells independent of blood glucose concentration. Long acting sulfonyureas (glyburide, glimepiride) have a high incidence of hypoglycemia, especially in the elderly Note: glipizide is short acting and has a significantly lower incidence of hypoglycemia

Mechanism of action of terbinafine?

Inhibition of squalene epoxidase Terbinafine is used for the treatment of dermatophytosis. It inhibits synthesis of fungal membrane ergosterol by suppressing the enzyme squaline epoxidase Note: bind to ergosterol- amphotericin, nystatin blocks synthesis of beta 1,3-D-glucan- caspofungin inhibits fungal protein synthesis- flucytosine binds polymerized microtubules & disrupts mitotic spindle-griseofulvin

71 year old man is administered a vaccine containing inactivated viral components that he normally receives every year. He is subsequently exposed to the virus though natural infection. Patients prior exposure to the vaccine is most likely to result in?

Inhibition of viral entry into the cells Inactivated versions of influenza vaccines (IM injection) function by inducing neutralizing antibodies against the hemagglutinin antigen in selected viral strains. Upon subsequent exposure to the influenza virus through natural infection, these antibodies inhibit binding of hemagluttin to sialylated receptors on the host cell membrane Inactivated (killed or component) viral vaccines predominantly generate a humoral immune response instead of a cell mediated immune response. Live-attenuated viral vaccines (such as nasal influenza) enter the MHC class I antigen processing pathway to generate CD8+ cells that kill infected cells & can therefore generate a strong cell mediated immune response in addition to providing humeral immunity

16 year old girl with history of anemia since childhood is evaluated for a genetic abnormality after several members of her family are also found to be anemic. Its determined she has a point mutation in the beta globin gene which has resulted in the substitution of guanine with cytosine in the MRNA sequence just upstream of the +1 site (its located three bases upstream from AUG). What cellular process is most likely impaired due to this patient's gene mutation?

Initiation of translation This patient has beta thalassemia due to an inherited beta globin gene mutation in the kozak sequence. The kozak consensus sequence occurs on eukaryotic mRNA and is defined by the sequence gccgccRccAUGG in which R is either adenine or guanine. This sequence helps initiate translation at the methionine start codon (AUG). R being a purine plays a role in the initiation process and having cytosine present instead will interfere with initiation of translation

34 year old man with several episodes of coffee ground emesis. Says his mouth feels dry and becomes dizzy on standing. Has recently taken ibuprofen for a back injury sustained several weeks ago. BP is 90/60 and pulse is 120. Skin is cool and clammy and there is mild epigastric tenderness. Increased level of what molecule is most likely to be seen in the vascular smooth muscle of this patient?

Inositol triphosphate Patient has coffee ground emesis, orthostatic hypotension and tachycardia and likely has a brisk upper GI hemorrhage due to excessive ibuprofen use. His sympathetic tone has increased in an effort to prevent circulatory failure from hypovolemia, causing vasoconstriction (cool, clammy skin) and tachycardia. NE and epinephrine are the two major mediators of the sympathetic response and stimulate adrenergic cell surface receptors. Stimulation of alpha 1 adrenergic receptor by catecholamines results in Gq activation and initiation of the PIP second messenger system. Phospholipase C releases IP2 and DAG from membrane phospholipids. IP3 liberates Ca2+ from the ER promoting smooth muscle contraction and vasoconstriction

62 year old man with left arm clumsiness. PE reveals motor weakness involving left arm and leg. Has slurred speech and drooping of the left lower face. On passive extension of the left arm, there is initial resistance followed by a sudden release of tension as extension is continued. Lesion affecting what brain structure is most likely responsible for this condition?

Internal capsule Patients with INTERNAL CAPSULE stroke typically have pure motor weakness affecting the contralateral arm, leg and lower face. Patient has 'clasp knife' spasticity which is characterized by initial resistance to passive extension followed by a sudden release of resistance. This form of spasticity is seen with UMN lesions and results from lack of upper motor neuron inhibition on the spinal stretch reflex arc. Upper motor neuron lesions can affect any part of the pyramidal motor system including the corticospinal tracts of the spinal cord, medulla, pons, midbrain, internal capsule, and pre central gyrus (primary motor cortex). Note: lesions in the external segment of globus pallidus result in decreased motion and lesions in the internal segment of the globes pallidus result in excessive movement; putamen is involved in initiating movement and lesions in it can result in contralateral tremor, bradykinesia, and rigidity

During apoptosis activated cascades degrade several intracellular proteins including translation initiation factors. Although the cells are dying, they continue to synthesize many new proteins required for apoptosis. This can be best explained by what feature of mRNA?

Internal ribosome entry site Eukaryotic translation is initiated when the small ribosomal subunit binds the 5' cap of mRNA (facilitated by initiation factors) and scans for the AUG start codon within the Kozak consensus sequence. Internal ribosome entry is an alternative method used by apoptotic cells whereby a distinct nucleotide sequence (usually in the 5' untranslated region) allows translation to begin in the middle of mRNA.

47 year old man with worsening SOB over the last several moths. PE demonstrates wheezing and poor air movement. Liver transaminases are elevated. Liver biopsy is performed and LM after treatment with diastase and PAS staining reveals red pink granules. Most likely cause of patients SOB?

Intraalveolar septa destruction Alpha 1 antitrypsin is a serum protein that, through the inhibition of neutrophil elastase, reduces tissue damage caused by inflammation. Histologically, AAT deficiency (autosomal codominant disorder) is associated with reddish-pink, periodic acid schiff positive granules of unsecreted polymerized alpha 1 antitrypsin in the periportal hepatocytes. These globules resist digestion by distastase. Liver disease develops due to intrahepatocyte accumulation of polymerized AAT molecules and can progress to cirrhosis and hepatocellulr carcinoma. Most patients are homozygous for the Z allele which causes decreased secretion of AAT secondary to abnormal protein folding. Most patients develop severe panacinar emphysema due to unchecked destruction of alveolar walls which contain large amounts of elastin. Smoking plays a synergistic role.

21 day old boy with palpable swelling in his neck. Feeds well but appears comfortable only when held sideways. Favors looking to the right and cries when head is turned to the left. There is firm swelling on the left side of the neck that doesn't move when he swallows. What condition was most likely present prenatally?

Intrauterine malposition Congenital torticollis is typically noted by 2 to 4 weeks of age after which the child prefers to hold the head tilted to one side. A soft tissue mass may be palpable in the inferior 1/3 of the SCM. Plagiocephaly and facial asymmetry may be seen in severe cases. Its most commonly the result of malposition of the head in utero or birth trauma. Most cases resolve with conservative therapy and stretching exercises.

If a gene sequence is hybridized with its complementary MATURE mRNA, loops of unbound DNA will appear representing regions of the gene that code for?

Introns These DNA loops form because their complementary mRNA sequences (introns) have been spliced out post transcriptionally

A researcher is studying macrophage antigen presentation to CD4 T lymphocytes. What HLA class II component is digested by macrophages during antigen processing?

Invariant chain Each class II molecule is a heterodimer composed of an alpha chain and a beta chain. The invariant chain brings the alpha and beta chains together to form a stable complex in the RER. The invariant chain then leaves the RER, passes though golgi, and enters endocytic compartment in which its proteolytically degraded and an external protein is inserted between the alpha and beta chains. During antigen processing in macrophages, the invariant chain is removed from the MHC invariant complex and replaced by an external protein. The MHC-peptide complex (containing an alpha chain, beta chain, and external protein) is then exposed at the cell surface and recognized by CD4 cells

73 year old man with blood in urine. Cystoscopy reveals a mass in the bladder. Biopsy confirms malignancy. Most important prognostic marker in this patient?

Involvement of the muscular layer Majority of bladder carcinomas are urothelial (transitional cell) carcinomas in which the neoplastic cells resemble those of normal bladder epithelium. Painless gross hematuria is the main presenting symptom. Tumor stage is the most important feature in determining prognosis and therefore tumor penetration of the bladder wall is the major determinant of prognosis

Patient undergoes partial gastrectomy with gastrojejunostomy. Long term supplementation with what substance will most likely be necessary in this patient?

Iron Iron absorption occurs predominantly in the duodenum and proximal jejunum. Bypass of this segment of small bowel by gastrojejunostomy results in iron deficiency anemia. The post surgical decrease in gastric acidity also diminished iron absorption and may contribute to iron deficiency. Note: malabsorption of vitamin B12, folate, fat soluble vitamins (especially vitamin D) & calcium can also be observed following gastric bypass procedures

17 year old has a miscarriage. PE shows a longitudinal vaginal septum and duplication of the cervix. Patient also has hypo plastic great toes and thumbs. MRI of the pelvis reveals uterus didelphys. Patients mother is found to have similarly shortened first digits and a history of several miscarriages. Mutation in what gene is most likely responsible for patients findings?

HOXA13 Homeobox (How) genes encode transcription factors that guide the pattern of embryo development along the rostro-caudal, limb, and genital axes. Mutations in HOXA13 can cause hand foot genital syndrome, a rare dominantly inherited syndrome marked by distal limb defects including hypo plastic first digits and genitourinary malformations Note: PAX genes are important for embryologic specification of certain tissues; PAX6 is the key regulatory gene for eye and brain development. Mutations in PAX6 are associated with ocular defects such as absent iris (aniridia)

Patients homozygous for HbC have mild chronic hemolytic anemia, whereas those homozygous for HbS have more severe anemia complicated by vasoocclusive pain episodes. What property best accounts for the differing clinical severity between these two conditions?

HbS allows hydrophobic interaction among hemoglobin molecules HbS contains valine in place of glutamic acid at the 6th potation of the beta globe chain. This promotes hydrophobic interaction among Hb molecules and results in HbS polymerization and erythrocyte sickling. RBC sickling is promoted by low oxygen levels, increased acidity, and dehydration. HbC has a glutamic acid replaced by a basic polar (positively charged) lysine residue. Even though lysine has the opposite charge of glutamic acid, it still has a charge so there isn't a hydrophobic interaction between hemoglobin molecules and no polymerization/sickling

Neoplastic lesions of the CNS can promote vasogenic edema by disrupting the blood brain barrier. Can result in mass effect with midline shift. Increased vascular leakage into the cerebral interstitium contributes to elevated ICP which presents with?

Headache, nausea/vomiting, and papilledema Note: cytotoxic edema is an increase in intracellular fluid within neurons, glia, and endothelial cells that happens when cell injury impairs the Na+/K+ ATPase & typically occurs following an ischemic insult (cerebral infarct) PICTURE IS VASOGENIC EDEMA Obstruction of CSF flow occurs with tumors involving the 3rd ventricle and resulting in enlargement of the lateral ventricles

What hemodynamic changes do you expect after taking nitroglycerin?

Heart rate- increased EDLV volume- decreased Sublingual nitroglycerin is used as a first line agent for the rapid relief of angina symptoms. The anti-ischemic effect of nitrates is mediated by systemic vasodilation (primarily venodilation) with a decreased in left ventricular volume and wall stress. The result is decreased myocardial oxygen demand despite a reflexive increase in HR.

9 year old boy with beta thalassemia major presents for routine red cell transfusion. Has received numerous blood transfusions and chelation therapies. Recent liver biopsy showed Kupffer cells with coarse yellow brown cytoplasmic granules. Granules are composed of?

Hemosiderin Since iron is utilized poorly in patients with thalassemia and can't be excreted actively, the ferritin micelles accumulate in macrophages of the reticuloendothelial system. The resulting iron storage complex is known as hemosiderin. Patients undergoing regular transfusions should undergo routine chelation therapy to reduce overall iron land with the body and improve survival. Eventually the iron burden will overwhelm the reticuloendothelial cells capacity to sequester iron resulting in parenchyma iron overload in liver, myocardium, skin, and pancreas (bronze diabetes)

Patient is concerned as he is prescribed a high dose of isosorbide dinitrate therapy, compared to his sublingual nitroglycerin. Most likely reason for using a high dose of oral nitrate?

High first pass metabolism Isosorbide dinitrate has a low bioavailability due to extensive first pass hepatic metabolism prior to release in systemic circulation. Sublingual nitroglycerin is absorbed directly from oral mucosa into the venous circulation and has a higher bioavailability.

62 year old woman with severe fatigue & progressive exertion dyspnea for the past 6 months. Reports several episodes of food getting stuck in her throat, especially when attempting to swallow large chunks of large solid food. On PE, she has has mucosal pallor and her tongue appears smooth and red. Fingers are spoon shaped. Supplementation of what substance should improve this patients condition?

Iron Plummer Vinson syndrome is characterized by dysphagia (esophageal web formation) and iron deficiency anemia. Findings associated with iron deficiency anemia include koilonychia (spoon shaped nails) and a shiny red tongue. Most symptoms resolve following iron supplementation

9 month old girl presents after a 2 minute generalized seizure. Has had a fever for the past 12 hours and has been very fussy. Temperature is 103. 3 days later, she is afebrile but develops a maculopapular rash on her trunk. Infection most likely due to?

Human herpesvirus 6 Roseola infanatum classically presents with fever for 3-5 days followed by a blanching maculopapular truncal rash. Rash starts on trunk and spreads to face and extremities. Its the most common cause of febrile seizures. Treatment is supportive.

72 year old man with severe chest tightness and dyspnea that began twenty minutes ago at a family dinner. Never had similar symptoms in the past. PMH of HTN, hyperlipidemia, T2DM, and prostate cancer. Smoked 1PPD for 20 years and stopped smoking ten years ago. CT scan shows a saddle pulmonary embolism. Risk factor that most likely contributed to patient's current condition?

Hypercoagulabiltiy Saddle pulmonary embolism straddles the bifurcation of the main pulmonary artery. Venous thromboembolism (pulmonary embolism or DVT) arises due to the Virchow triad of endothelial injury, venous stasis, and hypercoagulable state. Malignancy causes a hypercoagulable state and is a strong risk factor for venous thromboembolism

Electrolyte ECG changes?

Hyperkalemia: peaked T waves (earliest change), loss of P wave, prolonged PR interval, widening of QRS, slowed HR (can progress to sine wave pattern --> ventricular fibrillation or asystole) Hypokalemia: QT interval prolongation, U waves, ST segment depression (can progress to torsades) Hypocalemia: QT interval prolongation Hypercalcemia: QT interval shortening, T wave flattening/inversion in severe cases

Drug induced antineutrophil cytoplasmic antibodies associated vasculitis, which presents with constitutional symptoms, athralgias/arthritis, cutaneous vasculitis, is most commonly linked to what medications?

Hyperthyroid medication (PTU, methimazole) & hydralizine

Dopamine beta hydroxyls is responsible for the synthesis of NE form Dopamine. A congenital deficiency of beta hydroxyls will impair synthesis of NE and E and lead to impaired sympathetic adrenergic activity. Usually presents with?

Hypotension, ptosis, hypothermia in children. Postural (orthostatic) hypotension, exercise intolerance, nasal congestion, and ejaculatory difficulties in adults.

LM of brain tissue taken from an experimental animal shows neurons with shrunken nuclei, no detectable Nissl substance and intensely eosinophilic cytoplasm. Findings described most likely indicate?

Irreversible cell injury A neuron that is responding to irreversible injury is called a 'red neuron'. Characteristic changes become evident in 12-24 hours after the injurious event and include shrinkage of the cell body, eosinophila of the cytoplasm, pyknosis of the nucleus (shrunken and basophilic), and loss of Nissl substance and nucleolus. Nissl substance is basophilic granules that represent Rerouted and is responsible for protein synthesis. Axonal reaction (loss of axon) is characterized by enlargement of the cell body, eccentric nucleus, enlargement of the nucleus, dispersion of Nissl substance Neuronal atrophy (progressive degenerative disease) is characterized by loss of neurons and functional groups of neurons & reactive gliosis

24 year old presents for postpartum follow up 2 weeks after vaginal delivery. She is concerned as she has not yet lactated. Reports excessive fatigue and significant weight loss. Also reports nausea, loss of appetite, and postural dizziness. No headaches, visual problems, cold intolerance, constipation, or polyuria. Hemoglobin, sodium, cortisol, free T4, and TSH are all low. Pathologic process most likely responsible for patients symptoms?

Ischemic necrosis High estrogen levels during pregnancy cause enlargement of the pituitary gland without proportional increase in blood supply. Peripartum hypotension can cause ischemic necrosis of the pituitary leading to panhypopituitarism (sheehan syndrome). Patients commonly develop failure of lactation due to deficiency of prolactin.

45 year old woman with 2 weeks go persistent back pain. Also reports excessive fatigue, anorexia, and unintentional weight loss over the past several months. No PMH but there is a history of cancer in the family. Imaging reveals lytic lesion in T10. Analysis of the biopsy sample shows cell lineage with abnormally high replicative potential and a genetic study reveals a single nucleotide substitution causing an activating mutation. What gene is most likely involved in this patients disease?

KRAS Most tumors possess multiple cytogenetic abnormalities. Activation of protooncogenes results in stimulation of cellular proliferation. Inactivation of anti-oncogenes (tumor suppressors) eliminates oversight of the cell cycle. APC, BRCA1, TP53, and RB all all tumor suppressors and would be inactivated in neoplastic cells

6 year old boy is found to have significantly decreased intestinal absorption of lysine, arginine, ornithine, and cysteine. If his condition is left untreated what complication is he at greatest risk of developing?

Kidney stones Cystinuria is an autosomal recessive disorder caused by defective transportation of cystine, ornithine, arginine, and lysine (COLA) across the intestinal and renal tubular epithelium. Recurrent nephrolithiasis is the only clinical manifestation. UA shows pathognomonic hexagonal cystine crystals. Risk factors for cysteine precipitation include a low urine pH, presence of a preexisting crystal nidus and urine supersaturation

54 year old man with severe fatigue and dyspnea. Has a long history of progressively worsening heart failure that has been resistant to treatment with medications. Patient dies and autopsy reveals dense, thick fibrinous tissue in the pericardial space between the visceral and parietal pericardium. Most likely sign that would have been detected during PE of this patient?

Kussmal sign Constrictive pericarditis is a chronic condition in which the normal pericardial space is replaced by a thick, fibrous shell that restricts ventricular volumes and eventually causes heart failure. Impaired right ventricular filling leads to increased JVP and often results in a positive Kussmaul sign (rise in JVP during inspiration because the restricted RV can't accommodate the inspiratory increase in venous return). There may also be a pericardial knock, a sharp accentuate sound , which occurs earlier in diastole than the S3 heart sound. Note: restrictive cardiomyopathy causes an S3 heart sound but constrictive pericarditis reduces ventricular compliance via an external force and causes pericardial knock

45 year old is started on high intensity atorvastatin therapy, along with anti platelet therapy and appropriate medications to control BP. Four weeks lateral his cholesterol has decreased from 200 to 140. What is most likely increased as the result of the therapy?

LDL receptor density Treatment with statins causes hepatocytes to increase their LDL receptor density, leading to increased uptake of circulating LDL

Cytokine responsible for isotope switching from IgM to IgE?

IL-4 I-4 is produced by Th2 subset of helper T cells. It facilitates growth of B-cells and Th2 lymphocytes, and stimulates antibody isotope switching to IgE. Note: IL-5 stimulates IgA production and eosinophil activity & is important for host defense against parasites

Description of optimal cannulation site for the femoral vein?

Immediately medial to the femoral artery The femoral triangle (lateral to medial) consists of the femoral nerve, femoral artery, femoral vein, and deep inguinal nodes/lymphatic vessels. Cannulation of the femoral vein should occur approximately 1cm below the inguinal ligament and just medial to the femoral artery pulsation.

35 year old woman with sudden onset numbness of the left arm and face. No weakness, but has had several days of generalized headache, dyspnea on exertion, and easy fatiguability. Light touch sensation is decreased in the left UE and lower left face. Strength and reflexes are normal. Hemoglobin & platelets are low. BUN and creatinine are elevated. PT and PTT are normal. Blood smear shows numerous shistiocytes. UA is positive for hematuria and proteinuria. Most likely primary pathogenesis of patients current condition?

Impaired cleavage of von Willebrand factor Patient has new onset neurological symptoms, anemia with schistocytes, thrombocytopenia, and acute kidney injury which suggests TTP. Left arm and lower facial sensory loss are consistent with a pure sensory lacunar stroke & exertion dyspnea and fatigue are due to anemia. Thrombotic thrombocytopenic purpura results from impaired function of VWF cleaving protease, ADAMTS13, resulting in uncleaved vWF multimers that are significantly more prothrombotic and cause diffuse microvascular thrombosis, microangiopathic hemolytic anemia, and thrombocytopenia. Normally circulating vWF produced by megakarocytes & endothelial cells is released as large vWF multimers that are cleaved to their regular size by ADAMSTS13. PT and PTT are normal as the coagulation cascade doesn't play much of a roll.

Effect of nitroglycerin infusion on LVEDP, peripheral venous capacitance, and SVR?

LVEDP- decreased Peripheral venous capacitance- increased SVR- decreased Nitrates are primary venodilators and increased peripheral venous capacitance, thereby reducing cardiac preload and LVEDV and LVEDP. Nitrates also have a modest effect on arteriolar dilation and cause a decrease in systemic vascular resistance and cardiac afterload

Patient with chest tightness when he walks uphill or climbs stairs is started on a medication that is metabolized to S-nitrosothiols in the vascular smooth muscle cells and improves significant symptom relief. What portion of the cardiovascular system is most susceptible to the medication prescribed to this patient?

Large veins Pharmacologic nitrates (nitroglycerin, isosorbide mononitrate, isosorbide denitrate) are metabolized to nitric oxide and S-nitrothiols in vascular smooth muscle cells leading to an increase in cGMP that stimulates vasodilation. Large veins are predominantly affected leading to increased venous capacitance and reduced venous return (preload) which decreases LV wall stress and myocardial oxygen demand to relieve anginal symptoms Note: nitroprusside dilates large arteries and veins and is used in HTN emergencies

64 year old female with headache followed by sudden onset of blindness in right eye. Was recently admitted to hospital for fever, pelvic and shoulder girdle pain, as well as weight loss. Condition is most likely caused by?

Large vessel arteritis Polymyalgia rheumatics occurs in more than half of patients with temporal arteritis. Its characterized by neck, torso, shoulder, and pelvic girdle pain and morning stiffness. Fatigue, fever, and weight loss may also occur. Monocular vision loss is a common complication of temporal arteritis. Patients also presents with focal pain over temple and jaw/tongue claudication. LM reveals granulomas in the media of the arteries consisting of mononuclear infiltrates and multinucleate giant cells. Changes in arteries are segmental with abnormal areas interspersed by segments of normal appearing arterial wall

ECG shows ST elevation in leads I and aVL. Cardiac enzymes are elevated. Emergent cardiac catheterization will most likely show occlusion of what artery?

Left circumflex artery Leads I and aVL correspond to the lateral limb leads on ECG. Therefore, ST elevation (acute MI) or prominent Q waves (old MI) in these leads are indicative of infarction involving the lateral aspect of the ventricle which is supplied by the LCX artery (originates from left main coronary artery)

Orthopnea, supine dyspnea that is relieved by sitting up, is a specific sign of?

Left sided heart failure Bilateral LE edema and congestive hepatomegaly are most specific for right sided HF. Left sided HF can also a productive cough with pink frothy sputum due to rupture of the bronchial veins and exertional wheezing or chest tightness, but these are nonspecific signs seen in many disorder.

64 year old with occasional dizziness. Symptoms usually occur while playing table tennis. Smoked 1PPD for the past 40 years. He undergoes doppler US of the left vertebral artery which reveals retrograde (caudal) flow instead of normal anterograde flow. Most likely artery that is occluded based on these findings?

Left subclavian Subclavian steal syndrome occurs due to severe stenosis of the proximal subclavian artery, which leads to reversal in blood flow from the contralateral vertebral artery to the ipsilateral vertebral artery. Patients may have symptoms related to arm ischemia in the affected extremity (exercise induced fatigue, pain, paresthesias) Note: Brachiocephalic artery stenosis/occlusion can cause subclavian steal syndrome but patients develop retrograde flow through the RIGHT vertebral artery rather than the left

17 year old boy with skin rash that has been present for one day. One week ago he had a sore throat that improved with oral penicillin therapy. Skin exam shows several violaceous, raised, nonblanchable lesions distributed over the bilateral lower extremities. Most likely diagnosis?

Leukocytoclastic vasculitis Cutaneous small vessel vasculitis is associated with medication (penicillins, cephalosporins, sulfonamides, phenytoin, allopurinol) use and typically presents with palpable purpura in the lower extremities. Characteristic histopathologic findings include marked perivascular inflammation of small blood vessels with fibrinoid necrosis and a predominance of neutrophils and fragmented neutrophilic nuclei (leukocytoclastic vasculitis)

8 year old boy with recurrent episodes of SOB and wheezing. PE reveals mildly prolonged expirations. CBC shows eosinophilia. Physician prescribes pharmacologic receptor antagonist and symptoms improve substantially. Drug most likely acts at the receptor for?

Leukotriene D4 Numerous substances are thought to play a role in the pathogenesis of allergic asthma, but only leukotrienes LTC4, LTD4, and LTE4 and acetylcholine have pharmacologic receptor antagonists that offer clear therapeutic treatment. Zafirlukast and Montelukast are LTD4 receptor antagonists that may offer long term control of atopic asthma by increasing airway caliber and reducing mucosal inflammtion Inhaled ipratropium is an antimuscarinic agent that blocks M3 receptors in airway smooth muscle and submucosal glands. It may offer short term relief of bronchoconstrcition in allergic asthma. HISTAMINE ANTAGONISTS LACK EFFICACY IN ALLERGIC ASTHMA

53 year old woman with cellulitis is started on antibiotics. Three days later she becomes agitated and delirious and develops severe abdominal cramps and diarrhea. Medications include metformin, lisinopril, paroextine. Temperature is 102.6. BP is 180/100. HR 120 and regular. On exam she is diaphoretic, tremulous, and pupils are dilated. Bilateral hyperreflexia and clonus are present. What antibiotic was likely given to patient to treat her cellulitis?

Linezolid Serotonin syndrome is characterized by a triad of autonomic instability, altered mental status, and neuromuscular irritability. It may develop when a antidepressant MAOI or a non antidepressant with MAOI activity (LINEZOLID) is combined with serotonergic medication such as SSRI, SNRI, or TCA

'Moldy' grains in some regions of China contain a substance that causes a specific G to T mutation in the gene p53. The population in these regions is expected to be at increased risk of what disease?

Liver cancer High levels of dietary aflatoxin exposure is associated with a G:C --> T:A transversion in codon 249 of the p53 gene, a mutation thought to greatly increase the risk of developing hepatocellular carcinoma Combination of chronic aflatoxin exposure and HBV infection significantly increases the risk of hepatocellular carcinoma

45 year old with progressive weakness and fatigue over the last year. Genetic testing is performed and a loss of expression mutation is identified in a gene coding for a protein found on the basolateral surface of hepatocytes and enterocytes that is known to interact with the transferrin receptor. What condition is patient at risk of developing?

Liver cirrhosis and hepatocellular carcinoma HFE protein mutations are the most common cause of primary hemochromatosis. HFE interacts with the transferrin receptor normally to form a complex that functions as a sensor of iron stores. Mutations cause the detection of falsely low iron levels. Inactivation of the HFE protein results in decreased hepcidin synthesis by hepatocytes and increased DMT1 expression by the enterocytes, leading to iron overload. When body iron levels exceed 20g patients develop micro nodular cirrhosis, DM, skin pigmentation. Patients with hemochromatosis are at increased risk hepatocellular carcinoma, CHF, testicular atrophy/hypogonadism.

What drug property of Methadone is helpful in treating patients with opioid abuse?

Long half life Methadone is a drug of choice for maintenance treatment of opioid abuse. It is a potent, long acting FULL mu opioid agonist with food bioavailability and can be given once daily oral dosing. Its prolonged effects suppress withdrawal symptoms and cravings, allowing for more productive patient functioning. Adverse effects are QT interval prolongation and respiratory depression with lethality in overdose Note: buprenorphine is a PARTIAL agonist used alone or with naloxone (an opioid antagonist) & is less likely to cause respiratory depression and mortality in overdose compared with methadone

33 year old homeless man with recent weight loss, odynophagia, and progressive cough. He's found to be HIV positive and CD4 count is 45. Patients condition improves with treatment, but 4 weeks later he is found to have significantly elevated fasting blood glucose. What medication is most likely responsible for his hyperglycemia?

Lopinavir Protease inhibitors are HIV antiretroviral medications that inhibit cleavage of the polypeptide precursor into mature viral proteins. Their side effects as a class include hyperglycemia, lipodystrophy and drug drug interactions due to inhibition of CYP450. Note: Rifampin increases CYP450 activity and will decrease protease inhibitor serum levels; replace rifampin with rifabutin in the treatment of TB in patients on protease inhibitors

56 year old woman with a 2 day history of high fever, headache, mild confusion, and dry cough. She also has mild abdominal discomfort and watery diarrhea. Recently returned from cruise to Hawaii. Has smokes 1PPD for over 20 years. Temperature is 104. Pulse 85. BP 104/63. lung exam reveals lower lobe crackles with no wheezing. CXR shows bilateral lower lobe interstitial infiltrates. What additional finding is most likely to be present in this patient?

Low serum sodium Legionella pneumophilia is a facultative intracellular gram negative bacillus that can cause a systemic infection. Symptoms include high fever, cough, confusion, diarrhea. Most common lab abnormality is hyponatremia and sputum gram stain shows many neutrophils but few or no organisms.

43 year old with progressively worsening neurologic symptoms. Brain MRI reveals a well circumscribed PARASAGITAL tumor. Patient most likely presented with what symptoms?

Lower limb sensory loss and hemineglect Patient with progressive neurologic signs and a well circumscribed parasagittal tumor likely has a meningioma. Meningiomas are common adult intracranial tumors that typically arise in regions of dural reflection (falx cerebri, tentorium cerebelli). Tumor is compressing patients right parietal lobe. Lesions involving the primary somatosensory cortex (located in post central gyrus of the parietal lobe) typically result in contralateral sensory loss, whereas damage to the parietal association cortex (particularly the non dominant hemisphere) cause contralateral hemineglect due to impaired visuospatial processing.

Researcher recruits a number of patients with a history of calcium oxalate kidney stones. What is most likely to be seen in affected patients compared to healthy individuals in this study?

Lower urinary citrate Renal calculi occur due to an imbalance of the factors that facilitate and prevent stone formation. Increased urinary concentrations of calcium, oxalate, and uric acid promote salt crystallization whereas increased urinary citrate concentration and high fluid intake prevent calculi formation. Citrate binds to free calcium preventing its precipitation and facilitating its excretion. Potassium citrate is often prescribed to prevent recurrent calcium stones. Hypocitaturia occurs in setting of chronic metabolic acidosis (distal tubular acidosis, chronic diarrhea) due to enhanced renal citrate reabsorption. Note: diet high in Ca2+ reduces risk of renal stone formation as dietary calcium is ingested with oxalate in food and forms insoluble calcium oxalate salts in the intestinal lumen resulting in decreased gut absorption and therefore reduced renal excretion of oxalate. Note: high urinary pH is associated with calcium phosphate and ammonium magnesium phosphate but NOT calcium oxalate stones

F508 is the most common mutation in the CFTR protein in patients with CF. This mutation leads to protein mid folding and failure of glycosylation, followed by proteasome mediated degradation and significantly decreased number of CFTR proteins. Drugs that can potentially help these patients by restoring the CFTR proteins to the membrane and also enhance protein function (chloride transport) at the membrane, respectively?

Lumacaftor and ivacaftor

47 year old woman with rash on her chest for 2 months. Rash is itchy and skin feels rough to touch. Patient has a history of surgery for bilateral silicone breast implants 10 years ago. PE reveals induration and swelling over the right breast. Further exam reveals right axillary lymphadenopathy without breast masses. Most likely cause of this patients skin changes?

Lymphatic obstruction Peau d'orange is an erythematous, itchy breast rash with skin texture skin changes analogous to an orange peel. This is the key dermatologic presentation of inflammatory breast cancer and is caused by cancerous cells obstructing the lymphatic drainage due to spread to the dermal lymphatic spaces. Differentiate from mastitis via lack of fever.

A microsatellite is a short DNA sequence containing a repetitive pattern of nucleotides; its prone to mutation due to slippage of DNA polymerase. Microsatellite instability occurs as a result of defective DNA mismatch repair and is the genetic basis for what disease?

Lynch syndrome (hereditary nonpolyposis colorectal cancer)

CD8+ lymphocyte receptors recognize foreign proteins on the epithelial cell surface. Foreign proteins are presented on the epithelial surface by MHC molecules. These MHC molecules comprise what components?

MHC class I heavy chain and B2-microglobulin

23 year old woman with one day of fever, vomiting, diarrhea, and muscle pains. Symptoms developed rapidly over the course of the day. Patient is confused and unable to answer questions. Temperature is 102.4. A diffuse erythematous, macular rash resembling sunburn is noted on exam. Pelvic exam reveals a tampon in the vagina. Activation of what cells is most likely responsible for patients condition?

Macrophages and T lymphocytes Superantigens (toxic shock syndrome toxin) interact with MHC molecules on APCs and the variable region of the T lymphocyte receptor to cause nonspecific, widespread activation of T cells. This results in the release of IL-2 for the T cells and IL-1 and TNF from macrophages. This immune cascade is responsible for the manifestations of toxic shock syndrome

62 year old with cough and dyspnea. She expectorates copious amounts of pale tan colored fluid. CXR shows pulmonary infiltrate. Histological exam shows columnar mucin secreting cells that ling the alveolar spaces without invading the storm or vessels. Patients condition is best characterized as?

Malignant neoplasm Adenocarcinoma in situ (formerly known as brionchioalveolar carcinoma) is a subtype of lung adenocarcinoma that accounts for less than 10% of lung cancers. It consists of well differentiated dysplastic columnar cells that line the alveolar septa without vascular or stromal invasion. The tumor has a tendency to undergo aerogenous spread and can progress to invasive disease if not treated.

Its determined that beta oxidation of fatty acids is inhibited in cells in animals with high carbohydrate, high protein diet for a prolonged period of time. An increase in what substance is most likely responsible for this observed effect?

Malonyl-CoA High ATP during fasting state inhibits isocitrate dehydrogenase in TCA cycle causing citrate to build up in the mitochondria. Citrate is transferred to the cytosol via citrate shuttle and cleaved by ATP citrate lyase to form acetyl CoA. Cytosolic acetyl-CoA carboxylase converts acetyl-CoA to malonyl CoA during the rate limiting step of de novo fatty acid synthesis. Malonyl-CoA also inhibits the action of mitochondrial carnitine acetyltransferase, thereby inhibiting beta oxidation of newly formed fatty acids.

Unresponsive 62 year old man is brought to ED after being in an MVC. He is obtunded but responds to painful stimuli. BP is 160/90. After initial treatment and stabilization, he develops severe tachypnea and decreased oxygenation. CXR shows pulmonary edema. He is intubated and given O2 but dies. What drug could have caused this patients condition?

Mannitol Mannitol is an osmotic diuretic that works by increasing plasma and tubular fluid osmolality. Increased plasma and fluid osmolality causes extraction of water from the interstitial space into the vascular space or tubular lumen, with subsequent diuresis. In the brain, water redistribution from the tissues into the plasma helps reduceredistriburionedema and ICP in the setting of cerebral edema. One of the more severe toxicities of aggressive osmotic diuretics is pulmonary edema. Its due to the rapid rise in volume that can increase the overall hydrostatic pressure in the vasculature. Also causes dilutional hyponatremia, metabolic acidosis, and hyperkalemia.

26 year old with lower abdominal discomfort. She has never been pregnant. Sexually active with husband and they use condoms for contraception. Smokes 1/2 PPD. Mother was diagnosed with breast cancer at 52 and grandfather died of colon cancer at 77. BMI is 30. US of abdomen shows right sided adnexal mass that is removed. Histologic findings are shown. Most likely diagnosis?

Mature teratoma Teratomas are the most common subtype of germ cell tumor. Ovarian teratomas occur most frequently in females age 10-30. They are divided into mature (cell lines of >1 germ layer, commonly including hair, teeth, and skin) and immature types. Note: unlikely carcinoma as the median age of diagnosis of ovarian carcinoma is 60. Complex cysts, defined by the presence of cystic and solid components are commonly seen.

68 year old woman with severe right hip pain after a fall. She is unable to move her right hip and the right leg appears shower than the left. Pelvic x-ray reveals a right femoral neck fracture. Injury of what artery is most likely to lead to osteonecrosis in this patient?

Medial circumflex The medial femoral circumflex artery and its branches (ascending cervical and retinacular) provide the majority of the blood supply to the femoral head and neck. Injury to these vessels due to displaced femoral neck fracture can cause osteonecrosis of the femoral head. Note: obturator artery gives rise to artery of ligamentum trees which supplies a minor portion of the femoral head; this vessel is important in children because it supplies the region of the femoral head proximal to the epiphyseal growth plate, but its of minimal significance in adults

5 year old with new onset gait instability and gait ataxia. MRI shows midline posterior fossa mass. Biopsy of mass reveals sheets of primitive cells and many mitotic figures. Most likely diagnosis?

Medulloblastoma Medulloblastoma is the second most common neoplasm of childhood (pilocytic astrocytoma is most common). Its located in the cerebellum, often at the vermis and consists of sheets of small, round, blue cells with scant cytoplasm. Like other PNET (primitive neuroectodermal) tumors, medulloblastomas are poorly differentiated and have a bad prognosis Note: on microscopic exam of a pilocytic astrocytoma, pilocytic astrocytes and rosenthal fibers are seen (low grade tumors & have a better prognosis than medulloblastoma) Note: ependyomas are 3rd most common brain neoplasm in children; arise in the walls of the ventricles and can hamper CSF outflow and cause hydrocephalus; on microscopic exam they form gland like structures called 'rosettes'

Young male presents with scattered skin lesions. Has a strong family history of skin tumors. Biopsy of a lesion demonstrates dermoepidermal junction aggregates of large cells that fuse with adjacent nests. The individual atypical cells have irregular nuclear contours, hyperchromasia, and are round or spindle shaped. A mutation of chromosome 9p21 is identified in atypical cells. Most likely predisposed to developing what type of cancer?

Melanoma Dysplastic nevus syndrome manifests with numerous dysplastic nevi in a young person who has a family history of melanoma in more than three first degree relatives. This syndrome is associated with mutations in the CDKN2A gene on chromosome 9p21.

56 year old who was recently diagnosed with colon cancer suffers from generalized edema. Urine protein excretion is 4.5 grams over 24 hours. Kidney biopsy shows glomerular capillary wall thickening without an increase in cellularity. When the sample is stained with methenamine, irregular spikes protruding from the glomerular basement membrane are seen. Patient most likely suffers from what condition?

Membranous glomerulopathy Membranous glomeulopathy is one of the most common causes of nephrotic syndrome in adults. It can occur secondary to underlying malignant tumors (lung and colon), DM, immunologic disorders (SLE), infections (HBV, HCV, malaria, syphilis), or certain mediations (gold, penicillamine, NSAIDs). Diffuse increased thickness of the glomerular basement membrane on LM (without increased cellularity), 'spike and dome' appearance on methanamine silver stain, and granular deposits on immunoflourecence are are diagnostic.

45 year old woman had an episode where she abruptly los consciousness and had rhythmic jerking movements of the limbs followed by a period of confusion. She has had mild daily headaches associated with nausea that have progressed over the past 2 years. Headache are worse at night and occasionally awaken her. No head trauma, visual changes, weakness, numbness, difficulty swallowing or speaking. Intracranial mass is discovered on imaging and shows a whorled pattern of cellular growth that forms nests & calcified psommoa bodies. Most likely diagnosis?

Meningioma Meningiomas are slow growing, well circumscribed benign intracranial tumors typically found at the cerebral convexities in adults. Characteristic histopathologic features include a whorled pattern of cellular growth that forms nests, which may calcify into round, esosinophilic laminar structures called psammoma bodies

43 year old with dyspnea that awakens him at night. Symptoms began 1 year ago after a severe URI and since then he has also had an episodic cough. Older brother has atopic dermatitis. Lung auscultation reveals bilateral normal breath sounds with no wheezing or crackles. FEV1 is 88% of predicted. FEV1/FVC ratio is 84%. Diffusion capacity for carbon monoxide is 95% of predicted. Administration of what medication would be most useful during further diagnostic evaluation of this patient?

Methacholine (inhaled muscarinic cholinergic agonist that induces bronchoconstriction) Asthma is characterized by reversible airway obstruction, and lung function tests may be normal between exacerbations. Bronchoprovocation can be used to aid diagnosis in patients with normal spirometry; methacholine is administered and followed by serial spirometry. Patients with asthma demonstrate hyperresponsitvity to the stimulus leading to FEV1 reductions at lower doses than in those without asthma Bronchodilator administration can help diagnose patients who have an obstructive value on spirometry at baseline. Spirometry is performed before and agfter administration of bronchodilator & reversal of airflow obstruction therapy suggests asthma while irreversible obstruction suggests an alternative diagnosis (COPD, bronchiolitis)

45 year old woman with rheumatoid arthritis. A multi drug treatment is started and soon after she develops painful mouth ulcers and nausea. LFTs show new AST and ALT elevations. What medication is most likely responsible for the adverse effects seen in this patient?

Methotrexate Methotrexate is the preferred disease modifying treatment for patients with moderate to severe RA. Significant adverse effects include stomatitis, bone marrow suppression, pulmonary fibrosis, and liver function abnormalities

What do you expect to see on laboratory testing with a genetic defect that results in decreased beta globin chain synthesis?

Microcytosis Beta thalassemia results in hypochromic, microcytic anemia due to decreased beta globin chain synthesis. Unpaired alpha chains precipitate within red cells and cause membrane damage, leading to ineffective erythropoiesis and extravascular hemolysis. Other common findings include anisopoikilocytosis (variation in size and shape), target cells, teardrop cells, nucleated red cell precursors, and basophilic stippling

32 year old with several day history of high grade fever, dyspnea and fatigue. Temperature is 102.9. PE reveals new holosystolic murmur with blowing quality best heart over cardiac apex. Skin shows nontender lesions on sole of the feet. What do these lesions most likely represent?

Microemboli to skin vessels Patients presentation is suggestive of mitral regurgitation due to infective endocarditis. Janeway lesions are nontender, macular, and erythematous lesions typically located on the palms and soles of patients with acute infective endocarditis and are the result of septic embolization from valvular vegetations

34 year old woman with 1 week history of mood swing, difficulty concentrating, and hand tremors. Initially attributed symptoms to viral illness she had a few weeks earlier. Has had pain in the front of her neck that radiates to her ears and is worse with swallowing. Thyroid gland is very tender on palpation. Thyroid scan reveals decreased radioiodine uptake. ESR is 1-5. What pathological change would most likely be on biopsy of patients thyroid gland?

Mixed, cellular infiltration with occasional multinucleated giant cells Subacute granulomatous (de Quervain) thyroiditis is characterized by painful thyroid enlargement and usually follows a viral illness. Biopsy shows mixed inflammatory infiltrate with macrophages and multinucleated giant cells. Self limited and resolves in less than 6 weeks. Patient may have brief hypothyroid phase before returning to euthyroid state.

20 year old diagnosed with narcolepsy. Best treatment?

Modafinil (a nonamphetamine stimulant) Treatment also includes scheduled daytime naps.

Scientists studying hypoferfusion to kidney apply a clip to a pig's right renal artery which significantly reduces blood flow to the kidney. After 6 months, they perform a right nephrectomy and exam the glomeruli. What cell type would be most likely to undergo hyperplasia as a result of the lip placement?

Modified smooth muscle cells of the afferent arteriole (juxtaglomerular cells) Renal artery stenosis causing significant renal hypo perfusion will result in decreased GFR and subsequent activation of the RAAS system. This leads to increased renin release by modified smooth muscle (juxtaglomerular) cells within the afferent glomerular arterioles. Chronic renal hypo perfusion can cause hyperplasia of the juxtaglomerular apparatus.

Patient with pruritic sin rash on both lower extremities that is worst at the anterolateral right ankle. Hasn't used any new products. Has stayed inside since slipping and falling on icy sidewalk 4 weeks go. On PE the skin over her extremities is dry and cracked. Best advice for this patient?

Moisturizers should be applied after bathing Patient with an itchy rash and dry, cracked skin has typical features of xerosis (asteatotic dermatitis, 'winter itch'). Its most common in older patients during winter months when indoor heaters lower the humidity of air. Patients with this condition have defects in the stratum corneum permeability barrier. Bathe in lukewarm water and moisture immediately after bathing with creams or ointments to prevent excessive skin water loss since bathing strips away from of the parts of the permeability barrier.

34 year old with an enlarged lymph node in his anterior cervical chain. Biopsy reveals an abnormal lymph node architecture and numerous lymphocytes. What, if present, would be most consistent with malignancy in this patient?

Monocloncal T cell receptor gene rearrangements LAD can either be reactive hyperplasia or malignant transformation. Benign lymph node enlargement in response to antigenic stimulation is associated with polyclonal proliferation of lymphocytes. A monoclonal lymphocytic proliferation is strong evidence of malignancy.

36 year old HIV negative man being screened for potential kidney donor for his mom. Screening comes back positive for Hepatitis B surface antibody and CMV antibody IgG. Patient most likely had what condition in the past?

Mononucleosis like illness CMV is typically associated with subclinical infection in the immunocompetent, with the occasional individual developing a mononucleosis like syndrome (fever, malaise, myalgia, atypical lymphocytosis, elevated liver transaminases) that is heterophiles antibody (mono spot) negative. In the immunocompromised, primary or reactivated CMV infection can result in severe retinitis, pneumonia, esophagitis, colitis, or hepatitis.

45 year old woman with unintentional 15 pound weight loss over the past 6 months. She experiences lower abdominal pressure and feeling full very quickly and has some epigastric pain. PE shows bilateral adnexal fullness. Pelvic US shows bilateral complex ovarian masses with solid and cystic components. CT shows stomach wall thickening and ovarian masses. Most likely to be seen on histologic exam of the ovaries?

Mucin secreting signet ring cells Epigastric pain and gastric thickening is concerning for gastric cancer. A Krukenberg tumor is a gastric tumor that has metastasized to the ovary and can present with unintentional weight loss, epigastric pain, and adnexal masses. Histologically, the metastatic timor cells have large amounts of mucin with apically displaced nuclei resulting in a signet ring appearance. Gastric signet ring cell cancer is particularly infiltrative and spreads to the ovaries by lymphatic invasion and peritoneal seeding.

Feature of eukaryotic DNA replication that allows it to be quick and effective despite its large size and complexity of the genome compared to that of the prokaryotic organisms?

Multiple origins of replication

A new antimicrobial agent that inhibits bacterial glycosyltransferase enzyme is developed. What bacteria is most likely to be resistant to this drug?

Mycoplasma hominis The enzyme glucosyltransferase is a crucial component of peptidoglycan synthesis as it adds glycan molecules to the growing paptidoglycan chain. Peptodiglycan is the main component of the bacterial cell wall in both gram positive and gram negative organisms. Inhibition of glucosyltransferase results in caps in the bacterial cell wall with loss of bacterial shape and cell lysis from osmotic stress. All organisms in the Mycoplasma genus, including ureaplasma urealyticum, lack peptidoglycan cell walls and are therefore resistant to agents that target the cell wall (such as penicillins, cephalosporins, carbapenems, and vancomycin). Mycoplasma infections can be treated with anti ribosomal agents (tetracyclines, macrolides)

15 year old boy with fever, headache, malaise, and cough for 3 weeks after returning from summer camp. Lung auscultation is unremarkable. CXR shows nodular infiltrates. Patients blood samples are collected for analysis in anticoagulated (edentate disodium) tubes which are transported submerged on ice. When taken out, turbidity and clumping are seen. Warming the tube to body temperature leads to rapid dissolution of the blimps. Most likely organism responsible for his condition?

Mycoplasma pneumoniae The organism attaches to the respiratory epithelium using I antigen (surface antigen) that is also present on the plasma membrane of red blood cells. Infection with Mycoplasma can result in the formation of cold agglutinins which are IgM antibodies that bind red blood cells and cause clumping/agglutinations at low body temperatures. Cold agglutinins are usually asymptomatic but some patients develop intravascular hemolytic anemia that resoles in 6-8 weeks when the IgM antibody titers fall. Other illnesses resulting in cold agglutinin formation include infectious mononucleosis and certain hematologic malignancies

53 year old woman presents for check up. Mom suffered from HTN and was diagnosed with breast CA at 68 and died 4 years later. Dad had DM. Her BP is 140/85 and her HR is 80. PE is normal. Fasting glucose is 140 on more than 2 occasions. Patient is most likely to die from?

Myocardial infarction Patients with non coronary atherosclerotic disease, DM, or CKD are at the same risk of cardiovascular events (MI, stroke) as patients with known coronary heart disease. Coronary heart disease is the most common cause of death in patients with DM. CVA accounts for 10% of total mortality in patients with T2DM vs. 40% from coronary heart disease. Note: her risk of breast cancer is not significantly increased since her mother was at an advanced age when the cancer occurred

4 year old who just recovered from severe staphylococcal pneumonia. Has a history of recurrent lymphadenitis and skin infections. Dihydrorhodamine flow cytometry testing reveals an absence of the green fluorescence that is characteristic of normal neutrophils. Patients condition is most likely due to the impaired activity of what enzyme?

NADPH oxidase Chronic granulomatous disease is most commonly due to an X linked mutation affecting NADPH oxidase. Deficiency of this enzyme leads to an inability of neutrophils to form the oxidative burst necessary to kill organisms in their phagolysosomes. CGD can be diagnosed by absence of normal blue and fluorescent green pigment produced by the nitroblue tetrazolium test and dihydrothodamine flow cytometry test, respectively.

During process of T lymphocyte maturation, T ell receptors of many lymphocytes demonstrate a very high affinity interaction with MHC molecules expressed on thyme medullary epithelial and dendrites cells. What process do lymphocytes undergo at this time?

Negative selection The process of negative selection in T cell maturation is necessary for eliminating T cells that bind to self MHC self antigens with overly high affinity. This process occurs in the thymus medulla after positive selection occurs in the thymus cortex. If these cells were permitted to survive, they would induce immune and inflammatory reactions against self antigens leading to autoimmune disease

76 year old man with 2 month history of progressive back pain. Pain is constant and nagging and unrelieved by rest and position changes. Especially bad at night & interferes with sleep. Patient has taken ibuprofen without relief. PMH of HTN and osteoarthritis. Used IV drugs when younger but not for 30 years. Type of process most likely responsible for his back pain?

Neoplastic Clinical features that suggest a malignant cause of back pain include occurrence at night, not relieved with rest or analgesics, advanced age, and systemic symptoms. Common malignancies with a propensity for bony metastasis include prostate, breast, kidney, thyroid, lung

59 year old with diplopia that started several hours ago. PMH of DM with poor glycemic control. PE shows right sided ptosis with right pupil in the inferolateral position. PERRL bilaterally. Patients condition is most likely due to what etiology?

Nerve ischemia Diabetic mononeuropathy often involves CN III. Its caused by predominately central ischemia which affects the somatic nerve fibers (that innervate the EOM) but spares the peripheral parasympathetic fibers (responsible for pupillary constriction and accommodation). Symptoms include ptosis, down and out gaze, and normal light and accommodation reflexes. In contrast to diabetic CN III mononeuropathy, pupillary dysfunction is an easy manifestation of external nerve compression due to injury of the peripherally located parasympathetic fibers

Small cell carcinoma is strongly associated with smoking and is usually centrally located. This tumor arises from the primitive cells of the basal layer of the bronchial epithelium. Immunohistochemical stains are usually positive for?

Neuroendocrine markers (chromogranin, neuron specific enolase, synaptophysin)

43 year old male with recent onset oliguria and high serum creatinine level. Has been seen several times for an intranasal ulcer that has failed to heal. Patients condition is most likely associated with antibodies against what cell type?

Neutrophils Necrotizing vasculitis of the upper and lower respiratory tract (nasal ulcerations, sinusitis, hemoptysis) and RPGN (producing a satiable degree of renal failure) is characteristic of granulomatosis with polyangitis (Wegener's). This disease is associated with c-ANCAs which may target neutrophil proteinase 3 Note: Goodpasture syndrome presents with hemoptysis, focal pulmonary consolidations, and a glomerulonephritis that may rapidly progress to renal failure but upper respiratory tract ulceration is NOT characteristic

When an experimental drug is applied to cells expressing a receptor, there is an immediate change in transmembrane calcium, sodium, and potassium flow secondary to opening of the receptor channel. This is most likely what type of receptor?

Nicotinic cholinergic receptor The autonomic nervous system utilizes three types of signal pathways: cAMP, IP3, and ion channels. Nicotinic receptors are ligand gated ion channels that open after binding acetylcholine. This results in IMMEDIATE influx of Na+ and Ca2+ into the cell and outflow of K+ out of the cell alpha1- IP3 pathway Beta1- cAMP pathway beta2- cAMP pathway M1- IP3 pathway M2- cAMP pathway (decrease cAMP with activation) M3- IP3 pathway

75 year old man found unresponsive at home. Patient has history of poorly controlled HTN. BP is 240/120 and pulse is 104. He has extensor posturing and pinpoint pupils. CT head reveals acute pontine hemorrhage with associated mass effect. Patient dies and autopsy reveals disruption of all pigmented neurons in the posterior rostral pons at the lateral floor of the fourth ventricle. These neurons normally produce?

Norepinepherine Locus ceruleus is a paired brainstem nucleus located in the rostral pons near the lateral floor of the 4th ventricle and functions as the principal site for NE synthesis in the brain. It projects to virtually all parts of the CNS and helps control mood, arousal (reticular activating system), sleep wake states, cognition, and autonomic function. Patients with pontine hemorrhage typically present with coma due to disruption of the reticular activating system. Other features include total paralysis with extensor posturing due to corticospinal and corticobulbar tract injury and pinpoint pupils secondary to descending sympathetic tract damage

Autopsy of 78 year old man demonstrates decreased LV cavity size and sigmoid shaped ventricular septum. Light microscopy shows increased collagen content within the ventricular wall. Some myocardial cells have brownish peri nuclear cytoplasmic inclusions. Changes described are most consistent with what condition?

Normal aging Normal morphological changes in the aging heart include decrease in LV chamber apex to base dimension, development of sigmoid shaped ventricular septum with the basilar portion bulging into the LV outflow tract (due to decrease in chamber length), myocardial atrophy with increased collagen deposition & extracellular amyloid deposition, and accumulation of cytoplasmic lipofuscin pigment within cardiomyoctes

62 year old man with water diarrhea. He has also been having episodes of dyspnea and wheezing. Wife has noticed that sometimes he becomes very flushed and red. CT scan reveals mass lesions in the right liver lobe and ileum. Surgery is scheduled but what medication can be used to relive his symptoms prior to surgery?

Octreotide Carcinoid syndrome may accompany extra intestinal metastases of gastrointestinal carcinoid tumors. Octreotide is a synthetic somatostatin analog used to control the symptoms of carcinoid syndrome.

34 year old woman with diplopia, clumsiness of left hand, and numbers of right foot. Had numbness in the right hand six months ago that spontaneously resolved. On exam she has left eye adduction slowing and left arm power of 4/5 with mild spasticity. Sensation to pinprick is decreased over the right foot. CSF shows mononuclear cell pleocytosis and oligoclonal banding. What pathologic finding is most likely to be present in patients brain?

Oligodendrocyte depletion Multiple sclerosis is an autoimmune demyelinating disease that results from oligodendrocyte depletion (and relative sparing of axons). Within the lesions, inflammatory infiltrates of lymphocytes and macrophages are seen surrounding oligodendrocytes and myelin sheaths. Oligoclonal bands are highly sensitive but not specific in patients with MS.

35 year old with difficulty seeing out of her right eye. Started having blurry vision and pain with eye movement 2 days ago. Had an episode of double vision that resolved after a few weeks 2 years ago. Has an afferent pupillary defect on exam. MRI of brain reveals multiple ovoid white matter lesions with perpendicular orientation to the lateral ventricles. Condition is most likely caused by inappropriate immune response against what cell type?

Oligodendrocytes (myelin producing cells of CNS) Pathogenesis of multiple sclerosis involve autoimmune response (T cell and antibody mediated) targeting oligodendrocytes and myelin in the CNS leading to slowed nerve condition with neurological defect. T cells abnormally react to antigens on myelin, releasing inflammatory cytokines that promotes infiltration of destructive macrophages/microglia

Kluver-Bucy syndrome can result from bilateral damage to the temporal lobes, particularly the amygdale. Potential causes include HSV encephalitis and TBI. Symptoms?

Oral fixation, hyperphagia, hypersexuality, visual agnosia (inability to interpret sensations), placidity (calm), and amnesia

Patient started on long term daily inhaled medication to control his asthma. What should be advised to reduce the risk of adverse drug effects in this patient?

Oral rinsing Inhaled glucocorticoids improve symptoms and reduce the risk of severe exacerbation in patients with asthma but can cause oropharyngeal candidiasis. Using a spacer and rinsing the mouth after use can reduce the risk of side effects

56 year old with decreased hearing in both ears for the past six months thats been getting worse. For past 12 years he worked in a factory where he has to shout to communicate with workers and doesn't wear hearing protection. Audiogram shows high frequency sound loss. Most likely abnormal?

Organ of Corti Prolonged exposure to loud noises causes hearing loss due to damage in the stereociliated hair cells of the of the organ of corti. High frequency sound is always lost first. Sound pathway: TM --> oval window via ossicles --> basilar membrane -->. bending of cilia hair cells against tectorial membrane --.> hyper polarization and depolarization of auditory nerve

Patient given a medication to help with weight loss that can cause abdominal discomfort, flatulance, and diarrhea as it inhibits enzymes that break down fat in the intestine. What drug?

Orlistat Orlistat inhibits intestinal lipase, which leads to decreased intestinal fat absorption. It produces modest weight loss and is associated with significant GI side effects (abdominal discomfort, flatulance, diarrhea)

Patients with urea cycle disorder typically have discrete episodes of vomiting, tachypnea, and confusion/coma secondary to hyperammonemia (a metabolic emergency). Most common disorder of the urea cycle that is characterized by hyperammonemia and elevated urinary orotic acid?

Ornithine transcarbamylase deficiency Metabolic decompensation is often triggered by illness (viral URI, AOM), fasting, or increased protein intake. Note: uridine monophosphate synthetase is part of the pyrimidine synthesis pathway and its deficient also results in orotic acid buildup, but not hyperammonemi; characteristic findings are delayed growth and megaloblastic anemia

Viruses with segmented genomes are capable of genetic shifts through reassortment. Reassortment (genetic shift) involves exchange of genomic segments, a far more dramatic process (causes epidemics or pandemics)than the point mutations responsible for genetic drift. What viruses are segmented and can thus undergo genetic shift?

Orthomyxoviruses, reoviruses, bunyaviruses, and arenaviruses

23 year old with recurrent severe nosebleeds is found to have pink spider like lesions on his oral and nasal mucosa, face, and arms. Most likely diagnosis?

Osler-Weber-Rendu syndrome Also called hereditary hemorrhagic telangiectasia. Autosomal dominant condition marked by presence of telangeictasias in the skin as well as the mucous membranes of the lips, oropharynx, respiratory tract, GI tract, and urinary tract. Rupture of these telangeictasias can cause epistaxis, GI bleeding, or hematuria.

The development of respiratory distress, diffuse neurologic impairment (confusion) and upper body petechial rash (due to thrombocytopenia) within days of a severe long bone fractures is characteristic of the fat embolism syndrome. What its the stain that results in a dense black reaction that can be used to demonstrate fat?

Osmium tetroxide

56 year old with lung cancer is hospitalized with confusion and fatigue. Sodium is 118. If her electrolyte abnormalities are corrected too quickly, what would she be most likely to develop?

Osmotic demyelination syndrome Rapid correction of hyponatremia may lead to osmotic demyelination of the axons in the central part of the pons. This condition is called central pontine myelinolysis. It manifests with spastic quadriplegia due to demyelination of the corticospinal tracts and pseudobulbar pasly due to demyelination of the corticobulbar tracts of CN IX, X, XI (manifests as head and neck muscle weakness, dysphagia, and dysarthria) Note: cerebral edema results from the rapid correction of HYPERnatremia.

76 year old African American with diffuse lower extremity bone pain and weakness. Symptoms began 2 months ago and have progressively worsened. Eats prepackaged processed foods. Spends little time outdoors. On exam, he has tenderness to firm palpation of the tibia bilaterally. Patient most likely has impaired mineralization of what bone component?

Osteoid Osteomalacia is characterized by decreased mineralization of newly deposited osteoid and presents with bone pain, muscle weakness, and increased fracture risk. Its most often caused by vitamin D deficiency and resulting hypophosphatemia. Decreased GI calcium absorption due vitamin D deficiency causes increased PTH which causes phosphate wasting. The low phosphate impairs mineralization of newly deposited osteoid matrix on Haversian canal and trabecular surfaces

6 year old boy with history of 5 bone fractures, 3 of them occurring after only minimal trauma. On PE he has small, malformed teeth and blue sclera. Patients condition is associated with impairment of what process?

Osteoid production of osteoblasts Osteogenesis imperfecta results from defective synthesis of type 1 collagen by osteoblasts. Clinical findings include a history of fracture after only minimal trauma, blue sclerae, and small malformed teeth. In most patients, osteogenesis imperfecta is transmitted by autosomal dominant inheritance. Some patients are susceptible to bruising and hearing loss.

Pagets disease is associated with what bone cancer?

Osteosarcoma Radiographic findings of osteosarcoma include destruction of the normal trabecular bone pattern, mixed radio dense and radiolucent areas, periosteal new bone formation, lifting of the cortex, and cowman's triangle. Adjacent soft tissue usually demonstrates ossification in a 'sunburst pattern'

3 month old boy has a right sided white pupillary reflex. Father was diagnosed with retinal neoplasm during childhood. This child is at greatest risk of developing what neoplasm?

Osteosarcoma Familial retinoblastoma occurs as a result of mutations of each of the two Rb genes ('two hits'). These patients have increased risk of secondary tumors especially osteosarcomas later in life. Germline mutation of Rb tumor suppressor gene on chromosome 13 is present in every cell of the body. Malignancy doesn't occur until a somatic second mutation occurs in the retinal cells ('second hit' that leads to malignancy) Hereditary- 1 inherited and 1 acquired mutations Sporadic- 2 acquired mutations Note: children with sporadic retinoblastoma are NOT at risk for other malignancies; sporadic retinoblastoma occur due to TWO acquired somatic mutations affecting only the retinal cell lineage

12 year old girl with pain and swelling of elbow for the past week. Had knee pain the preceding week as well. Temperature is 102. Elbow are swollen and tender with ROM. A new holosystolic murmur is heard on cardiac auscultation. ASO titers are elevated. What complication is patient at greatest risk of dying from?

Pancarditis The primary cause of morbidity in acute rheumatic fever is heart failure from severe pancarditis. The pathogenesis of acute rheumatic fever is autoimmune. Mitral stenosis develops years or decades after the original illness. Joint involvement is usually transient.

Common cause of innocent murmur that is heard after the first day of life when the patent ductus arteriosus is closed. Murmur results from relative hypoplasia of branch pulmonary arteries compared to the relatively large main pulmonary artery. Auscultation may reveal a low grade, mid systolic, high pitched, or blowing ejection murmur that is best heard in the pulmonary area with radiation to the axilla and back

Peripheral pulmonary stenosis (pulmonary branch stenosis)

34 year old female with a 2 day history go reduced sensation in her legs. She has had significant fatigue especially with exercise recently. Six months ago she had an episode of blurring vision that resolved spontaneously after a few days. Exam indicates decreased pain and light touch perception in both legs below the level of the umbilicus. Mild bilateral spasticity is noted. What pathologic brain lesion is most specific for this patient's disease process?

Perivenular inflammatory cells Multiple sclerosis is an immune mediated disorder of the CNS characterized by focal demyelination (plaques). Histologically the plaques contain foci of perivenular inflammatory infiltrates made up primarily of auto reactive T lymphocytes and macrophages. Patchy demyelination occurs followed by astrocyte hyperplasia (glial scarring).

65 year old diagnosed with acute subcortical infarct in the left internal capsule. Six months later what finding would most likely be observed in the affected pyramidal tracts of this patient?

Persistent myelin debris Wallerian degeneration refers to the process of axonal degeneration and the breakdown of the myelin sheath that occurs distal to a site of injury. Axonal regeneration doesn't occur in the CNS due to the persistence of myelin debris, secretion of neuronal inhibitory factors, and development of dense glial scarring

Rare autosomal dominant disease caused by mutations in the serine threonine kinase 11 (STK 11) gene on chromosome 19 in which individuals develop pigmented mucocutaneous macule and multiple hamartomatous polyps in the GI tract

Peutz-Jeghers syndrome

45 year old man with persistent cough and dyspnea on exertion. Lung biopsy reveals fine carbon particles within the patients respiratory bronchioles and alveolar ducts. What defense mechanism is most directly responsible for clearing particles from this portion of the respiratory tract?

Phagocytosis The pneumoconioses are diseases resulting from the inhalation of fine dust particles that reach the respiratory bronchioles and alveoli. Particles that ledge in this region are normally cleared by alveolar macrophages. High particulate burden can cause the excessive release of cytokines from macrophages, resulting in progressive pulmonary fibrosis

Urinary analgesic that provides symptomatic relief of dysuria in UTIs?

Phenazopyridine

Refers to confection of a host cell by 2 viral strains, resulting in progeny visions that contain nucleocapsid proteins from one strain and the unchanged parental genome of the other strain. Because theres no change in underlying viral genomes (no genetic exchange), the next generation of visions revert to their original, unmixed phenotypes

Phenotypic mixing Note: reassortment and recombination cause progeny to contain the genomic change; transformation (uptake of naked DNA) doesn't cause genomic change in progeny

What medication should be administered first in preparation for removal of a pheochromocytoma?

Phenoxybenzamine Phenoxybenzamine is a long acting alpha adrenergic blocker used in preoperative medical management of pheochromocytoma. Administer a beta blocker after phenoxybenzamine as use of beta blocker prior can cause severe vasoconstriction and lead to hypertensive crisis

43 year old male with seizure disorder and mental illness presents with gingival hyperplasia and messed up teeth. What medication is most likely responsible for his teeth and gum symptoms?

Phenytoin Gingival hyperplasia is common side effect of phenytoin and is sometimes reversible when phenytoin is withdrawn. Phenytoin causes increased expression of PDGF. When gingival macrophages are exposed to increased amounts of PDGF, they stimulate proliferation of gingival cells and alveolar bone. Phenytoin toxicity mainly effects the cerebellum and vestibular system, causing ataxia and nystagmus Other adverse effects of phenytoin: -nystagmus and ataxia -coarsening of facial features, hirsutism -megaloblastic anemia -increases cytochrome P450 oxidase system -fetal hydantoin syndrome if taken during pregnancy

46 year old hospitalized with DKA. In the morning, the on call physician hands off the patient to the day shift physician who arrives an hour late due to an emergency at his nighttime moonlighting job. Handoff information includes instructions to check electrolytes and check anion gap. Physician performs inpatient rounds and then sees his usual clinic outpatients in the afternoon but neglects to check the patient's anion gap. Patient develops worsening acidosis requiring intubation and respiratory distress. Most likely cause of this adverse outcome?

Physician fatigue Sleep deprivation in physicians often causes cognitive impairment, resulting in medical errors. Although mandated resident work hour limitations are in place, it is the responsibility of all physicians to self regulate their workloads to promote patient safety.

20 year old with recurrent episodes of jaundice. PE shows pallor, icterus, and mild splenomegaly. Labs show low hemoglobin, increased LDH, elevated indirect bilirubin. Negative direct Coombs test. When patient's RBCs are incubated in a hypotonic solution with glycerol, hemoglobin is released. The control sample does not release hemoglobin. Patient is at greatest risk for developing?

Pigmented gallstones (complication of any hemolytic anemia; in chronic hemolysis, the increased bilirubin from lysed RBCs precipitates as calcium bilirubinate forming pigmented stones in the gallbladder) Hereditary spherocytosis (AD inheritance) results from red cell cytoskeleton abnormalities, most commonly spectrum and ankyrin. Increased osmotic fragility on acidified glycerol lysis testing confirms the diagnosis. Spherocytes demonstrate increased osmotic fragility in this test due to their decreased surface area to volume ratio. Hemolytic anemia, jaundice, and splenomegaly are classic manifestations. Complications include pigmented gallstones and aplastic crises from parvovirus B19 infection. Treat with splenectomy. Note: autoimmune hemolytic anemia presents similarly but would be associated with a positive direct antiglobuln (Coombs) test.

Drug capable of causing pupillary constriction in a denervated eye?

Pilocarpine Direct muscarinic cholinergic agonists can cause pupillary constriction in the denervated eye. Indirect cholinergic agonists (such as physostigmine) need acetylcholine release from the parasympathetic postganglionic neurons and denervation doesn't allow for this.

14 year old with nausea, vomiting, generalized headache, ataxia, and visual complaints. MRI reveals cystic tumor in the cerebellum. Biopsy reveals well differentiated neoplasm comprised of spindle cells that have hair like glial processes and are associated with microcysts & admitted with Rosenthal fibers and granular eosinophilic bodies. Most likely diagnosis?

Pilocytic astrocytoma A cystic tumor in the cerebellum of a child is most likely a policytic astrocytoma. Biopsy will show a well differentiated neoplasm comprised of spindle cells with hairlike glial processes that are associated with microcyte. These cells are mixed with rosenthal fibers and granular eosinophilic bodies.

46 year old woman with T2DM has an oral anti diabetic medication that induces differentiation of preadipocytes into adipocytes and increases GLUT4 expression on the adipocyte membrane aded to her regimen. What agent was most likely added?

Pioglitazone Thiazolidinones are PPAR gamma agonists that increase transcription of insulin responsive genes. They exert their glucose lowering effects by decreasing insulin resistance. Main side effects are fluid retention and weight gain

Begins as a solitary pink or brown scaly plaque with central clearing on the trunk, neck, or extremities (herald patch). Its followed by development of an ovoid maculopapular rash with lesions classically oriented in an oblique direction along the skin tension lines on the back (christmas tree pattern)

Pitryriasis rosea

34 year old woman delivers a 9 pound infant & the placenta is removed in pieces via manual extraction. Has a history of c-section. After removal of the placenta, profuse vaginal bleeding occurs. Bleeding continues despite uterine massage and administration of uterotonic medications. Most likely cause of ongoing vaginal bleeding?

Placenta accreta Postpartum hemorrhage is an OB emergency and can result from abnormal placentation. Prior uterine surgery can impair decidualization, resulting in myometrial invasion by villous tissue and placenta that is abnormally adherent to the myometrium (placenta acreta). Uterine atony is the most common cause of postpartum hemorrhage, but lack of response to uterotonic agents and removal of the placenta in pieces suggests placenta accreta. Placenta accreta is normally diagnosed prenatally and managed with c-section followed by hysterectomy.

57 year old male with fatigue and low energy. Reports intermittent back pain that responds to ibuprofen. Creatinine is elevated. Plasma protein electrophoresis reveals a high peak corresponding to gamma-globulins. Most likely diagnosis?

Plasma cell neoplasm Finding of a high peak in the gamma-globulin region on serum protein electrophoresis (SPEP) usually represents an M protein consisting of an overproduced monoclonal immunoglobulin. MM causes an M protein peak on SPEP as well as anemia (weakness), lytic bone lesions (back pain, pathologic fractures), and renal insufficiency (related to amyloid deposition and hypercalcemia).

21 year old with 'frothy' appearance to his urine. Also reports easy fatiguability and anorexia. Had a URI several weeks ago. Symmetric putting edema of bilateral LE on exam. What is most likely decreased in this patient?

Plasma oncotic pressure Frothy or foamy urine may be caused by proteinuria. Heavy proteinuria, as in nephrotic syndrome, can cause regional or generalized interstitial edema because the decrease in serum albumin and total protein concentration lowers the plasma oncotic pressure and increases net plasma filtration in capillary beds

2 day old boy develops abdominal dissension and refuses to breastfeed. Exam shows significant abdominal distention with palpable intestinal loops. Rectum has no stool and tone is normal. During exam, child has several episodes of dark green emesis. X-ray of abdomen shows air fluid levels and small bowel dilation. Patient taken for laparotomy which shows inspissated green, fecal mass obstructing the distal ileum. Infant recovers. Most likely cause of mortality in this patient?

Pneumonia Meconium ileum is a distal small bowel obstruction due to abnormally dehydrated meconium in a patient with cystic fibrosis. Persistent, treatment resistant infectious pneumonias, bronchiectasis, and cor pulmonate account for most deaths due to CF. Note: differentiate with Hirschsprung's disease in which patients present with increased rectal tone, 'squirt sign' (forceful expulsion of stool after rectal examination) and obstruction at the rectosigmoid region. Enterocolitis (fever, abdominal pain/distension, and fever) is most common cause of mortality in these patients

46 year old man presents after an episode of large volume hematemesis. Was siting at work when he suddenly felt nauseous and started vomiting bright red blood. PE reveals palpable spleen. Endoscopy shows bleeding varicose. Liver biopsy performed 2 days later is normal. Most likely cause of patients condition?

Portal vein thrombosis Portal vein thrombosis causes portal HTN, splenomegaly, and varicosities at portocaval anastomoses. It doesn't cause histologic changes to hepatic parenchyma. Ascities is uncommon as the obstruction is presinusoidal. Note: In Budd-Chiari syndromen there is occlusion of the hepatic vein which drains blood from the liver and portal circulation into the systemic circulation. Liver biopsy shows centrilobular congestion and fibrosis

65 year old with hoarseness for the past 2 weeks. Cheered loudly during his grandsons soccer game last month but hasn't strained his voice since. Underwent coronary artery stunting 2 years ago following an episode of chest pain. CT reveals dilation of the aortic arch. What additional finding is most likely to be seen in this patient?

Posterior cricoarytenoid muscle dysfunction Recurrent laryngeal nerve branches off the vagus nerves within the thorax and innervate all of the intrinsic muscles ('arytenoids') of the larynx that are responsible for sound production, except for the cricothyroid (supplied by external laryngeal nerve). Aneurysmal dilation of the aorta can damage the left recurrent laryngeal nerve as it loops below the aortic arch, resulting in hoarseness due to paralysis of most of the intrinsic laryngeal muscles on that side Note: other innervations Internal branch of superior laryngeal nerve- sensation over the supraglottic area External branch of superior laryngeal nerve- cricothyroid muscle Omyhyoid muscle (depresses hyoid bone and larynx)- ansa cervicali (C1-C3) Stylohyoid muscle (elevated hyoid bone and larynx) - CN VII (facial)

39 year old nulliparous woman with pelvic pressure and constipation that has worsened over past year. She finds it easier to defecate while pushing the vaginal canal with her fingers to get the stool out but still has an uncomfortable sensation after evacuation. Menses occurs every 28 days without heavy flow or severe pain. No pain with intercourse. Exam shows irregularly enlarged uterus and normal rectal tone. Most likely etiology of patients constipation?

Posterior subserosal uterine leiomyoma Subserosal leiomyomas (fibroids) often cause irregular uterine enlargement, resulting in pelvic pressure. Fibroids arising from the posterior subserosal surface of the uterus can cause constipation due to pressure on the colon

Hormone thats concentration increases the most after ovulation?

Progesterone Of all hormones influencing the menstrual cycle, progesterone increases the most in concentration after ovulation. During the latter half of the menstrual cycle, the corpus lutem secretes high levels of progesterone, which thickens the endometrium and prepares it to receive and nourish a blastocyst

33 year old woman with idiopathic pulmonary HTN complicated by Eisenmenger syndrome. She is pregnant and aware that pregnancy poses a major risk to herself and the fetus due to hemodynamic changes of gestation. She consents to pregnancy termination with mifepristone and misoprostol. MOA of mifepristone in this situation?

Progesterone antagonist Progesterone is necessary of implantation and maintenance of pregnancy. Mifepristone is a progesterone antagonist that is used with misoprostol (prostaglandin E1 agonist) to terminate 1st trimester pregnancy. Mifepristone binds to progesterone receptors with greater affinity that the natural hormone, resulting in apoptosis and necrosis of the uterine decidua. Misoprostol causes cervical softening and uterine contractions leading to expulsion of the pregnancy

Form of parkinsonism resulting from neurodegeneration of the midbrain and frontal subcortical white matter. Presents with rapidly progressive gait dysfunction and falls, executive function loss, and vertical gaze palsy. Brain biopsy show deposits of abnormally phosphorylated tau proteins

Progressive supranuclear palsy

Patient with soft tissue mass involving the hypothalamus and pituitary stalk. What pituitary hormone is most likely to be elevated in this patient?

Prolactin Release of prolactin is under inhibitory control by dopamine secretion from the hypothalamus. Disruption of dopaminergic pathways or blockade of dopamine D2 receptors can cause hyperprolactinemia

23 year old with heart palpitations. Gentle neck massage just below the angle of the right mandible provides immediate improvement of his condition. Mechanism responsible for the improvement of this patients condition?

Prolonged atrioventricular node refractory period Carotid sinus massage leads to an increase in parasympathetic tone causing temporary inhibition of SA node activity, slowing of conduction through the AV node, and prolongation of the AV nodal refractory period. Its useful vagal maneuver for termination of paroxysmal supra ventricular tachycardia .

44 year old woman with several 'near collisions; over the past month while driving at night. Also has generalized pruritus. On PE, skin is diffusely coarse and dry. What process is most likely responsible for these findings?

Prolonged biliary obstruction Vitamin A deficiency causes night blindness and hyperkeratosis. Deficiency of this fat soluble vitamin can develop in patients with biliary disorder, exocrine pancreatic insufficiency or intestinal malabsorption. This patient is a middle aged woman with generalized pruritus and signs of vitamin A deficiency who likelyy has underlying primary biliary cholangitis. Bile acids are important for solubilization and absorption of fats & patients with primary biliary obstruction due to PBC may develop malabsorption of fat soluble vitamins & fats.

Tardive dyskinesia is a medication induced movement disorder characterized by abnormal involuntary movements of the mouth, tongue, trunk, extremities. Its due to?

Prolonged exposure to dopamine blocking agents causing the up regulation and super sensitivity of dopamine receptors

Most effective strategy for preventing catheter associated UTI?

Prompt removal of catheter when no longer indicated DURATION OF CATHETERIZATION is MOST significant risk factor for UTI. UTIs are common in hospitalized patients with indwelling urinary catheters. Risk for UTI can be reduced by avoiding unnecessary catheterization, using sterile technique when inserting the catheter and removing the catheter ASAP. Common symptoms of catheter associated UTI are fever, pyuria, delirium, suprapubic/flank pain, and CVA tenderness. Catheters should only be changed for specific indications (active infection, obstruction) as routine replacement doesn't prevent infection.

Epinephrine increases systolic blood pressure (alpha 1 and beta 1) and heart rate (beta 1) and either increases (high dose affects alpha 1) or decreases (low dose affects beta 2) diastolic blood pressure. Pretreatment with what drug eliminates the beta effects of epinephrine (vasodilation and tachycardia) leaving only the alpha effect (vasoconstriction)?

Propanalol (nonselective beta agonist) Relative strength of epinephrine = B1 = B2 > alpha 1 In combination with epinephrine, propranolol counteracts epinephrine's B2 mediated vasodilation and B1 mediated heart rate increase while leaving the alpha 1 (vasoconstriction) intact. This increases diastolic BP and causes a slight reflexive reduction in HR. Isoproterenol- nonselective B-adrenergic agonist that wound potentiate effects of epinephrine Phentolamine- nonselective alpha blocker that would cause profound fall in BP and reflex tachycardia (would potentiate epinephrine effects) Phenylepherine- nonselective alpha agonist that increases BP (which may be associated with reflex bradycardia) but with epinephrine administration HR would rise due to unopposed B1 stimulation

54 year old missionary traveling alone becomes lost and goes 24 hours without food or water. His urine osmolarity is 1150. The majority of the total amount of water filtered by this individual's glomeruli is reabsorbed in what portion of the nephron?

Proximal tubule Regardless of the patient's hydration status, the majority of water reabsorption in the nephron occurs in the proximal tubule passively with the reabsorption of solutes (no concentration or dilution of urine occurs in this segment)

57 year old male with severe right sided eye pain and ipsilateral headache. Repots nausea and seeing 'halos' around objects. After initial treatment with the appropriate medication the severity of his pain decreases & he experiences increased diuresis with highly alkaline urine. Drug used to treat this patient's eye condition acts on what nephron segment?

Proximal tubules Patient has acute angle closure glaucoma and was treated with Acetazolamide. Carbonic anhydrase is found in high concentrations in the PT and is responsible for catalyzing reactions necessary for NaHCO3 reabsorption. Acetazolamide is a diuretic that works by inhibiting carbonic anhydrase, which effectively blocks NaHCO3 and water reabsorption in the PT resulting in urinary bicarbonate wasting. Carbonic anhydrase inhibitors are used to relieve intraocular pressure in open angle and angle closure glaucoma. Carbonic anhydrase in the eye tissue modulates HCO3- formation in aqueous humor & inhibition of this enzyme decreases HCO3- & aqueous humor formation. Common side effects include somnolence, paresthesias and urine alkalization. Rare side effects include metabolic acidosis, dehydration, hypokalemia, hyponatremia.

Psoas muscle inflammation causes abdominal pain thats exacerbated by the iliopsoas test. Its carried out by having the patient flex their hip against the examiner's resistance. Common causes of posts inflammation include appendicitis and psoas abscess. Identify psoas on CT.

Psoas muscles are located immediatley lateral to the vertebral bodies Note: the quadrates lumborum runs from the 12th rib to the iliac crest & helps to extend and lateral flex the vertebral column and fixes the 12th rib during inspiration

Excision of tender hemorrhoids that originate below the dentate line is planned. Anesthetic agent to be used most likely blocks sensory impulses carried by what structure?

Pudendal nerve External hemorrhoids which originate below the dentate line are covered by modified squamous epithelium and have cutaneous (somatic) nervous innervation from the inferior rectal nerve, a branch of the pudendal nerve. Internal hemorrhoids originate above the dentate line and are covered by columnar epithelium. They have autonomic innervation from the inferior hypogastric plexus, which is only sensitive to stretch (not pain, temperature, or touch)

31 year old female with exertion SOB and chest tightness over the last several months. PE shows right ventricular heave on chest palpation and bilateral pitting edema of the ankles. ECG shows RVH. Most likely cause of patients condition?

Pulmonary endothelial dysfunction Pulmonary endothelial dysfunction is characteristic of pulmonary arterial HTN.

72 year old man with diarrhea and vomiting for past 24 hours & has not taken in much fluid since becoming sick. On exam mucous membranes are dry and BP is 90/60. What changes do you expect in RPF, GFR, and FF?

RPF- very decreased GFR- decreased FF- increased Hypovolemia results in reduced RPF and GFR. This leads to compensatory efferent arteriolar vasoconstriction which raises the FF and maintains GFR near normal (but still decreased) levels. As RPF continues to decline, increasing glomerular oncotic pressure will eventually overwhelm the compensatory increase in hydrostatic pressure, leading to a precipitous drop in GFR & renal failure.

Most important risk factor for primary thyroid malignancy?

Radiation exposure (radiotherapy, fallout from nuclear accidents or detonations)

26 year old female with acute onset diplopia. Has had episodes of blurry vision involving her right eye that occurred 6 months ago and 2 years ago.Episodes were associated with deficits in color vision and pain is worse with eye movement. Patient recovered majority of visual acuity after a few months in both cases. On exam she has mild right disk optic atrophy and relative afferent pupillary defect in right eye. There is slowed and impaired adduction of the left eye with right lateral gaze. Convergence testing reveals normal adduction in both eyes. Patient's diplopia is most likely caused by what mechanism?

Reduced saltatory condition Multiple sclerosis is an autoimmune disorder of the CNS characterized by recurrent epodes of demyelination leading to reduced saltatory conduction (reduced 'jumping' of the impulse between nodes). Demyelination results in condition block, but over time the Na+ channels eventually redistribute form the nodes of Ranvier to across the naked axons and surviving oligodendrocytes partially demyelinate neurons, allowing for some restoration of signal transmission (explaining relapse and remitting). Intranuclear opthalmoplegia and optic neuritis (transient blurred vision and eye pain with movement) are common manifestations. Loss of axons and neurons occurs over time in chronic multiple sclerosis plaques.

Primary mechanism of pregnancy prevention with combined oral contraceptive pill?

Reduction in serum gonadotropin levels All hormone containing contraceptives prevent pregnancy through the actions of progestins. The main mechanism of action of contraceptives with systemically active progestins (combined OCPs including pill, transdermal patch, and vaginal ring & progestin implants/injection) is inhibiting ovulation by decreasing FSH and LH synthesis in the anterior pituitary. Since LH is required to stimulate ovulation, ovulation is inhibited and so is fertilization. Note: locally acting progestins (progestin only pill, IUD) thicken the cervical mucous inhibiting sperm penetration. Women still ovulate.

12 year old girl with HTN. She has had several episodes of fever and abdominal pain which her parents have been able to treat with OTC antibiotics. Renal US shows dilated calyces with overlying cortical atrophy bilaterally mostly in the upper and lower poles. Most likely cause of patients condition?

Reflux nephropathy Patients history of recurrent fever and abdominal pain along with imaging findings of renal scarring indicate recurrent pyelonephritis. Vesicoureteral reflux is caused by retrograde urine flow from the bladder into the ureter (when the ureters enter the bladder at a more perpendicular angle rather than the normal oblique angle). The hydrostatic pressure of refluxing urine along with infections due to ascending bacteria causes inflammation. The compound papillae in the upper and lower poles of the kidney are most susceptible to reflux induced damage, which appears as dilated calyces with overlying renal cortical scarring. Vesicoureteral reflux can lead to loss of nephrons and secondary HTN if uncorrected.

Patients taking high doses of loop diuretics are predisposed to hypokalemia and contraction alkalosis. Early signs of hypokalemia include muscle weakness, cramps, myalgia, and fatigue. Prompt treatment is necessary to prevent?

Respiratory depression and ventricular arrhythmias

38 year old man undergoing treatment for AML. He reports fevers, lethargy, severe right sided headaches. Says the pain is throbbing and located behind his right eye & associated with nasal stuffiness. PE reveals right proptosis and periorbital tenderness. Biopsy of maxillary sinus mucosa reveals broad, nonseptate hyphae that branch at wide (90 degree) angles. Most likely cause of patients condition?

Rhizopus species Mucormycosis is an opportunistic infection caused by Rhizopus, Mucor, and Absidia species. Classic clinical picture is paranasal sinus involvement in a diabetic or immunosuppressed patient. Fungi form nonseptate hyphae that branch at right angles. These fungi exist in mold form only. Treat with surgical debridement and amphotericin B.

61 year old with difficulty walking and frequent falls. Has a history of chronic alcohol abuse. On PE he has broad based gait and is unsteady. He is unable to walk in a straight line while touching the heel of one foot to the toe of the other with each step. Unable to perform heel to shin testing. Finger to nose test is normal. Symptoms are most likely the result of damage to a derivative of what structure?

Rhombencephalon Chronic alcohol consumption can cause degeneration of the cerebellar vermis, resulting in truncal ataxia (broad based, unsteady gait) and lower extremity incoordination. Cerebellum is derivative of the rhombencephalon. Note: Finger to nose testing is normally unaffected as its mediated by the cerebellar hemispheres.

43 year old female with reduced energy and fatigue is found to have hypochromic, microcytic anemia. Iron supplementation is prescribed. Several weeks later, peripheral blood smear demonstrated numerous enlarged RBCs that appear blue on Wright Giemsa stain. Bluish color of these RBCs is best explained by presence of?

Ribosomal RNA Increased bone marrow erythropoiesis results in an accelerated release of immature (large) RBCs (reticulocyte) into the bloodstream. Reticulocytes contain bluish cytoplasm and reticular precipitates of residual ribosomal RNA.

During thoracentesis a needle is inserted along the upper border of the 10th rib of the midaxillary line. What structure is most at risk of being injured?

Right hepatic lobe Thoracentesis should be performed above the 8th rib in the midclavicular line, 10th rib along the midaxillary line, and 12th rib along the posterior scapular or paravertebral line. Insertion of a needle lower than these points increases the risk of penetrating the abdominal structures, and insertion of the needle on the inferior margin of the rib risks striking the subcostal neurovasulcar bundle

53 year old man with severe low back pain. Pain started 2 weeks ago without history of trauma and has been keeping him awake at night. Not relived with OTC medication. Pain has acutely worsened over the past 2 days. Exam shows pain in the low back region with flexing of the back and raising of the legs. Pinprick in the perianal area doesn't cause rapid contraction of the anal sphincter. What nerve root is most likely involved in this patients condition?

S4 Saddle anesthesia and loss of the anocuteneous reflex are symptoms of caudal equina syndrome, which is associated with damage to the S2-S4 nerve roots. Pudendal nerve impairment causes loss of anal sphincter reflex. Other symptoms include low back pain radiating down one or both legs, bowel and bladder dysfunction, and loss of ankle jerk reflex with weakness of plantar flexion of feet.

35 year old african american female with arthralgia, tender, deep nodules on her legs. Hepatomegaly on exam. CXR reveals enlarged hilar lymph nodes and bloodwork reveals elevated ACE level. Skin PPD shows no induration after 48 hours. Liver biopsy would show?

Scattered granulomas Erythema nodosum, arthralgia, hilar lymphadenopathy, and elevated ACE levels are common findings in sarcoidosis. There may be evidence of liver involvement on histologic exam in up to 75% of cases. Needle biopsy of liver frequently demonstrates scattered noncaseating granulomas. Liver granulomas affect the portal triads to a greater degree than the lobular parenchyma. Note: major cause of morbidity in sacrcoidosis is pulmonary fibrosis which can progress to for pulmonale

25 year old who recently cut off all communication with friends and spends most time in her room with new tarot cards. Patient states she's trying to interpret signs from God so she can resist the power of the devil. Over the past 2 months she's heard the devils voice and suspects her internal organs have been replaced by someone elses. On exam she is disheveled, makes poor eye contact and has a flat affect. Most likely diagnosis?

Schizophreniform disorder

30 year old Asian male with exertional calf pain and painful foot ulcers demonstrates hypersensitivity to intradermally injected tobacco extract. What pathologic process is most likely responsible for his condition?

Segemental vasculitis extending into contiguous veins and nerves Thromboangitis obliterans (Buerger's disease) is usually seen among heavy cigarette smokers with onset before 35 and is associated with hypersensitivity to intradermal injections of tobacco extracts. The segmental thrombosis vasculitis often extends into contiguous veins and nerves encasing them in fibrous tissue. Histologically there is an acute and chronic inflammation of the arterial walls, often with thrombosis of the lumen which can undergo organization and canalization. The inflammatory process can eventually encase all three structures (arteries, veins, nerves) in fibrous tissue. Can result from direct endothelial cell toxicity from tobacco products or from hypersensitivity to them. More common in Israel, Japan, India. Calf, foot, or hand intermittent iclaudication may be seen as well as superficial nodular phlebitis and cold sensitivity (Raynaud's). Severe distal pain even at rest can result and may be due to neural involvement. Later complications include ulcerations and gangrene of distal extremities. TREATMENT IS SMOKING CESSATION

Estimated incidence of carbomyl phosphate synthetase I deficiency is about 1 in 800,000 newborns. If a decision is made to test all newborns for this disease, then this initial test should be designed to have a high ..... ?

Sensitivity The sensitivity of a test refers to the ability to correctly identify those with the disease. A highly sensitive test should always be considered over a highly specific test when screening for life threatening illnesses, where the identification of every person with the disease is important. Fewer cases of disease are missed with highly sensitive tests. Given a test with a high sensitivity a negative result helps rule OUT a diagnosis (SnNout). Note: confirmatory tests with high specificity are often used on patients who test positive on a screening test to ensure that the patient actually has the disease. Given a test with high specificity, a positive results would help rule IN the diagnosis (SpPin).

In an experiment pre-treatment with TNF-alpha results in decreased insulin mediated glucose uptake. This effect of TNF-alpha is most likely mediated though up regulation of what process?

Serine residue phosphorylation Phosphorylation of serine and threonine residues of insulin receptor and insulin receptor substrate by serine kinase leads to insulin resistance. This type of phosphorylation can be induced by TNF-alpha, catecholamines, glucocorticoids, and glucagon

34 year old man with infertility. Had normal puberty and has normal libido and erectile function. PE shows normal secondary sexual characteristics and bilaterally descended testes that are slightly smaller than normal. Serum concentrations of LH and testosterone are normal. Testicular biopsy shows that the testosterone concentration in the seminiferous tubules and epididymis is abnormally low. Dysfunction of what cells is most likely causing patient's findings?

Sertoli cells Androgen binding protein is synthesized by the Sertoli cells of the seminiferous epithelium and secreted into the seminiferous tubule lumen. ABP maintains the high local concentration of testosterone necessary for normal sperm production and maturation

45 year old man with worsening muscle pain and weakness for past 6 months. Carrying heavy items such as groceries has been difficult and prolonged activity causes muscle cramps. Had a 10 pound weight gain during this time. Pulse is 56. PE shows muscle weakness over the shoulder and hip girdles. DTRs show long relaxation time. Splitting and peeling of nails is present. Serum creatinine kinase is elevated. What test is most likely to identify the cause of patients muscle weakness?

Serum TSH level Patients fatigue, weight gain, bradycardia and brittle nails make hypothyroidism the most likelyy cause of his myopathy. Hypothyroid myopathy presents with myalgia, cramping, decreased exercise tolerance, proximal muscle weakness, elevated serum creatinine kinase, and associated features of hypothyroidism (delayed relaxation of DTRs). Myoedema, focal muscle contraction at the site of percussion, is a characteristic finding and is caused by impaired reuptake of calcium by the SR. Diagnosis can be confirmed with an elevated TSH level. Hypothyroid myopathy and elevations in CK can be the first manifestation of hypothyroidism and an precede other signs of hypothyroidism by months to years.

Marker that reflects the activity of osteoblasts?

Serum level of bone specific alkaline phosphatase Note: tartrate resistant acid phosphatase, urinary hydroxyproline, and urinary deoxypyridinoline reflect osteoclastic activity & urinary deoxypyridinoline is the most reliable of the three

Eighty percent of acute pancreatitis cases are caused by gallstones and chronic alcoholism. Less common causes account for the remaining 20%. What test should be considered during work up of a patient with acute pancreatitis who does not drink alcohol and doesn't have gallstones?

Serum triglycerides Inherited or acquired hypertriglyceridemia can cause acute pancreatitis if the serum level of triglycerides exceeds 1000 mg/dL. The concentration of FFA exceeds the binding capacity of albumin and leads to direct injury to the pancreatic acinar cells

2 week old girl undergoes hemoglobin electrophoresis and results are: Hemoglobin F 70% Hemoglobin A 20% Hemoglobin S 10% Patient's mother has sickle cell trans and a maternal cousin has sickle cell anemia. Patient is well appearing without pallor or splenomegaly. Most likely to be true about this patient?

She has relative protection from Plasmodium falciparum (not immune though, and should still receive prophylaxis if traveling to endemic country) Hemoglobin electrophoresis of sickle cell trait shows both HbA and HbS with the amount of HbA greater than the amount of HbS. Patients with sickle cell trait usually have normal hemoglobin, reticulocyte, and RBC index values and are asymptomatic. They can develop hematuria, priapism, and increased incidence of UTI. Splenic infarction at high altitudes has also been reported. Life expectancy is the same as the general population.

A new test is devised that uses latex particles coated with beta hCG. A woman adds her urine into a glass containing anti-hCG antibodies. Once she adds the reagent containing the latex particles to the urine, she observes agglutination. What is the most accurate statement about the test results?

She is not pregnant In agglutination inhibition test, the presence of agglutination is considered a negative result. When beta hCG coated latex particles are added to the solution after the urine, latex agglutination won't occur if the urine contains B-hCG because no anti-hCG antibodies will remain to react with the B-hCG coated latex particles. When urine lacking B-hCG is introduced to the B-hCB antibodies, the addition of B-hCG coated latex particles to the solution result in agglutination of these latex particles due to the interaction between them and the antibodies.

What organism needs as few as 10-500 cells to cause infection?

Shigella Shigella is able to invade tissues and is highly adapted to surviving the acidity of the stomach and bacteriostatic action of bile. Disease begins with watery diarrhea and progresses to fever, abdominal pain, dysentry, and tenesmus (painful rectal spasm associated with urge to defecate, but little passage of stool) Other organisms that can cause diarrhea with small inoculum are Campylobacter jejuni, E. histolytica, and Giardia lamblia

Coronary angiography of a 69 year old female with chronic atypical chest pain shows extensive atherosclerosis and near total occlusion of the LAD artery. The absence of myocardial necrosis and scarring despite vessel occlusion in this patient can be explained by what feature of the occluding plaque?

Slow growth rate (patient likely developed collaterals from the RCA to distal vessels normally perfused by the LAD) The major determinant of whether a coronary plaque will cause ischemic myocardial injury is the rate at which it occludes the involved artery. A slowly developing occlusion would allow for formation of collaterals that could prevent myocardial necrosis. A thin fibrous cap, a rich lipid core, and active inflammation in the atheroma would all decrease plaque stability and thus potentially promote rapid coronary occlusion via superimposed thrombosis if the plaque were to rupture.

42 year old found dead at home. Pathological exam shows complete thrombotic occlusion of left main coronary artery and diffuse atherosclerotic valvular disease characterized by multiple arthromas. Along with ah lipid core, these atheromas have a fibrous cap formed from dense deposit of collagen. What cells are directly responsible for synthesizing this fibrous cap?

Smooth muscle cells Vascular smooth muscle cells are the only cells within the atherosclerotic plaque capable of synthesizing structurally important collagen isoforms and other matrix components. Progressive enlargement of the plaque results in remodeling of the extracellular matrix and vascular smooth muscle cell death promoting the development of vulnerable plaques with an increased propensity for rupture

30 year old woman with T1DM is being evaluated for anorexia, weight loss, and decreased insulin requirement. PE shows generalized hyperpigmentation. ACTH stimulation test fails to elicit a significant increase in serum cortisol levels. What changes in lab values would most likely be present in this patient?

Sodium- decreased Potassium- increased Chloride- increased Bicarbonate- decreased Patients with T1DM are prone to developing other autoimmune endocrinopathies including Hashimoto's thyroiditis, Graves' disease, and primary adrenal insufficiency (Addison's disease). Electrolyte abnormalities in patients with primary adrenal insufficiency include hyponatremia, hyperkalemia, hypercholeremia, and non anion gap metabolic acidosis. Autoimmune adrenalitis results from autoantibody production against all 3 zones of the adrenal cortex. Since the main defect is in the adrenal gland, ACTH doesn't increase serum cortisol. Since glucocorticoids increase insulin resistance, decreased cortisol levels lead to lower insulin requirements in patients with adrenal insufficiency. Decreased aldosterone leads to decreased Na, increased K+, and increased H+. The decreased H+ excretion causes a non anion metabolic acidosis with low HCO3- levels. This causes increased in Cl- retention which occurs in order to maintain electrical neutrality of ECF. Note: exogenous serum cortisol increases with administration of ACTH in patients with secondary and tertiary adrenal insufficiency. These patients have an intact adrenal glans and can release aldosterone in response to angiotensin II so they don't develop mineralocorticoid deficiency, hyperkalemia, or metabolic acidosis.

How is Eranercept best characterized?

Soluble receptor decoy protein Etanerccept is a fusion protein linking soluble TNF-alpha receptor to the Fc component of human immunoglobulin G1 & therefore reduces the biological activity of TNF-alpha by acting as a decoy receptor. Suffix indications: -mab: monoclonal antibody -cept: receptor molecule -nib: kinase inhibitor Note: Infliximab is a monoclonal antibody targeted against TNF-alpha used in the treatment of autoimmune diseases such as RA and Crohns Certolizumab is a pegylated humanized monoclonal antibody that targets TNF-alpha that lacks the Fc region (preventing complement activation and cell mediate toxicity) Imatinib- small molecule tyrosine kinase receptor inhibitor treats Philadelphia chromosome positive CML and KIT positive GI stromal tumors

48 year old woman with intermittent ear discharge for the past 2 years. Also noticed decreased hearing in her right ear recently. PMH significant for obesity, hyperlipidemia, seasonal allergies, and diet controlled DM. Otoscopy shows small perforation in right TM and a pearly mass behind the membrane. Conduction hearing loss is noted in the right ear. Most likely cause of patient's aural mass?

Squamous cell debris Cholesteaomas are collections of squamous cell debris that form a round PEARLY mass behind the tympanic membrane. Cholesteatomas can be congenital or may occur as an acquired primary lesion or following infection, trauma, or surgery of the middle ear. They can cause hearing loss due to erosion into the auditory ossicles. Primary cholesteatomas are a result of the chronic negative pressure in the middle ear causing retraction pockets in the TM that become cystic and the squamous cell debris accumulate. Secondary cholesteatomes occur after squamous epithelium migrates to or is implanted in the middle ear ('skin in the wrong place')

Tetrology of fallot is the most common congenital cyanotic heart disease. Children with uncorrected defects typically squat during cyanic 'tet' spells because?

Squatting increases the SVR and cardiac after load temporarily reducing right to left shunting by forcing more blood through the stenotic pulmonary artery Tet spells are period of agitation, tachypnea, syncope, seizure and possible death TOF: pulmonary stenosis, RVH, overriding aorta, VSD

3 month old male with slowly growing bright red skin lesion on buttock. Most likely diagnosis?

Strawberry (capillary) hemangioma Strawberry hemangiomas are benign vascular tumors of childhood. They appear during the first weeks of life, initially grow rapidly, and typically regress by age 5-8. They are composed of capillaries separated by connective tissues Note: CHERRY hemangiomas are most common benign vascular proliferations in adults. They are small bright red cutaneous papule that appear in ADULTHOOD & don't regress spontaneously. They are composed of dilated capillaries and post capillary venules in the papillary dermis Note: CAVERNOUS hemangiomas may be present at birth or develop later in life. They are soft BLUE compressible masses. LM shows large dilated vascular spaces Note: Cystic hygromas are benign tumors that consist of dilated lymphatic spaces lined by endothelium. Most commonly on neck and are lobulated, compressible and usually transilluminate

36 year old male with severe left leg pain after suffering a minor laceration to his leg 2 days earlier. There is erythema of the overlying skin and his leg is swollen and tender. Temperature is 103. BP is 85/45. Patient is taken to operating room and a large area of necrotic tissue is excised. Gram stain of the necrotic tissue shows gram positive cocci and culture on a blood agar plate grows small colonies with wide zone of surrounding hemolysis. Coagulase and catalase tests are native and pyrrolidonyl arylamidase is positive. What pathogen is most likely responsible for the patients condition?

Streptococcus pyogenes Necrotizing fasciitis is a severe infection of the subcutaneous tissue and deep fascia that is a surgical emergency. The infection is often polymicrobial, but monomicrobial cases due to Steptoccocus pyogenes can also occur. S. pyogones is pyrrolidonyl arylamidase (PYR)-positive. Strep pyogenes forms small colonies with a wide zone of beta hemolysis and is sensitive to bacitracin (which has been widely replaced by the PYR test). Typically there is a sudden onset of severe pain and swelling at a site of trauma or recent surgery. Patients quickly become hypotensive and develop septic shock. Its treated with aggressive surgical debridement of all necrotic tissue along with broad spectrum epimeric antibiotics.

65 year old female with poor memory, urinary incontinence, and gait abnormalities is found on MRI to have enlarged ventricular system and minimal cortical atrophy. Urinary incontinence in this patient is most likely caused by?

Stretching of the descending cortical fibers Normal pressure hydrocephalus occurs in elderly patients. It causes triad of ataxic gait, urinary incontinence, and then dementia. These symptoms are explained by the distortion of periventricular white matter. Bladder control is influenced by descending cortical fibers that run in the distended paraventricular area. Later, loss of cortical inhibition on the sacral micturition center causes the development of urge incontinence. Voluntary relaxation of the urethral sphincter remains intact

10 year old boy recently immigrated to USA presents with ataxia, myoclonus, and visual problems. Parents say he started acting strangely and having difficulty with school several months ago. Brain biopsy is obtained and an RNA virus containing hemagluttin is cultured from the tissue sample. Most likely diagnosis?

Subacute sclerosing pan encephalitis SSPE is a rare complication of measles infection that occurs several years after apparent recovery from initial infection. Oligoclonal bands of measles virus antibodies are found in the CSF of these patients. Causes various neurological deficits and dementia and is relentlessly progressive and fatal

72 year old man presents with involuntary movements of his right arm that started several hours ago. He was watching television when his arm 'threw the remote control across the room'. PE shows wild large amplitude flinging movements affecting the proximal muscles of his right arm. What area is most likely injured in his brain?

Subthalamic nucleus Subthalamic nucleus plays an important role in the modulation of basal ganglia output. Damage to this structure (due to lacunar stroke for example) may result in contralateral hemiballism, characterized by wild, involuntary, large amplitude flinging movements of the proximal limbs (arm and/or leg) on one side of the body

47 year old alcoholic homeless man with pins and needles sensation in his legs. On exam, patients has glossitis and angular stomatitis. Abdominal exam reveals hepatomegaly. Labs show very low urinary riboflavin excretion. Activity of what enzyme is most likely directly impaired in this patient?

Succinate dehydrogenase Riboflavin is a precursor of the enzymes FMN and FAD. FAD participates in the TCA cycle and ETC by acting as an electron acceptor for succinate dehydrogenase (complex II) which converts succinate to fumarate. Symptomatic riboflavin deficiency is rare in the USA but can occur in chronic alcoholics and severely malnourished. Symptoms include angular stomatitis, cheilitis, glossitis, seborrheic dermatitis, eye changes (keratitis, corneal neovascularization) and anemia

A patients liver begins to synthesize large quantities of glucose from source molecules such as alanine, lactate, and glycerol. As part of this process, phosphenolpyruvate is formed from oxaloacetate in a reaction that requires a specific nucleoside triphosphate as a cofactor. What reaction directly synthesizes this cofactor?

Succinyl CoA --> Succinate GTP is synthesized by succinyl CoA synthetase during the conversion o succinyl CoA to succinate in the citric acid cycle. During gluconeogenesis, phosphenolpyruvate carboxykianse uses GTP to synthesize phosphenolpyruvate from oxaloacetate

49 year old man with longstanding HTN presents with severe SOB. BP is 260/144. IV furosemide and continuous nitroprusside infusion are started along with noninvasive positive pressure ventilation and he experiences improvement of his symptoms. The next morning he seems confused and lethargic and the nitroprusside infusion rate is found to be higher than recommended. A medication that acts as a donor of what element would help reverse this patients condition?

Sulfer Cyanide toxicity can occur in patients treated with nitroprusside. Cyanide toxicity presents with altered mental status, seizures, cardiovascular collapse, lactic acidosis, and bright red venous blood. Antidotal treatment of cyanide toxicity can be achieved by 3 different strategies: direct binding of cyanide ions (hydroxycolblamin), induction of methemoglobinemia (sodium nitrite), and use of detoxifying sulfer donors (sodium thiosulfate) Cyanide is normally metabolized in the tissues by rhodanese, an enzyme that transfers a golfer molecule to cyanide to form thiocyanate which is less toxic and excreted in the urine. Cyanide overdose depletes the available sulfur donors allowing cyanide to accumulate in toxic amounts. Sodium thiosulfate provides additional surfer groups for rhodanse, enhancing cyanide detoxification.

CT scan of a patients abdomen reveals an irregular mass in the third portion of the duodenum that is infiltrating beyond the gut wall. If this mass continues to enlarge, what structure is most likely to be compromised in this patient?

Superior mesenteric artery The 3rd part of the duodenum courses horizontally across the abdominal aorta and inferior vena cava at the level of the 3rd lumbar vertebrae. The superior mesenteric vessels lie anterior to the duodenum at this location Note: 1st portion of the duodenum is the only portion that is NOT retroperitoneal.

4 year old boy presents after a seizure. Had a fever, cough, and runny nose for the past 2 days. He was alert when paramedics arrived and has been drinking fluids and talking to his parents. Temperature is 104.5. Appears tired but is alert and cooperative. Next best step in management?

Supportive care only Febrile seizures are the most common neurologic disorder affecting children and are a benign sequelae of fever. Children who experience a febrile seizure are at risk for reoccurrence but have low risk of developing epilepsy. Supportive care (antipyretics) can improve comfort during fever but doesn't prevent future seizures. Antipyretics inhibit synthesis of PGE2 which reduces the thermoregulatory set point in the hypothalamus

53 year old with dull ache in right shoulder. Physician asks patients to abduct arm to 90 degrees to the side and flex them 30 degrees forward with thumbs pointing to floor. She then applies downward force to his arm which elects pain in his right shoulder and reveals right sided weakness. Tendon of what muscle is most likely injured?

Supraspinatous Supraspinatous muscle assists in abduction of the arm (first 15 degrees) and stabilization of the glenohumeral joint. Supraspinatous tendon is vulnerable to injury due to impingement between the acromion and the head of the humerus. Supraspinatous tendinopathy is most common cause of rotator cuff syndrome

Foam stability index evaluates?

Surfactant functionality To perform this test, multiple wells containing mixtures of ethanol and amniotic fluid are shaken and then examined to find the highest value well that contains a ring of stable foam

Acute stress disorder involves intrusive memories, flashbacks, avoidance, sense of detachment from reality, negative mood, impaired concentration and hyperarousal following exposure to a life threatening event. Its distinguished from PTSD by?

Symptom duration (symptoms last more than three days but less than one month; PTSD symptoms have to be present for more than one month) Note: in adjustment disorder intrusive recollections (distressing memories, nightmares, flashbacks) and avoidance of external reminders aren't present

27 year old male with severe left knee pain and swelling for the past 2 days. No trauma. Temperature is 101. On exam left knew is swollen, erythematous, warm, and tender with restricted ROM. Next best step?

Synovial fluid analysis Synovitis is characterized by pain, erythema, swelling, and reduced range of motion in a joint. Acute synovitis may represent serious pathology (septic arthritis), especially if accompanied by a fever or leukocytosis; should be evaluated urgently with a synovial fluid analysis. Delayed diagnosis of septic arthritis can lead to loss of the joint and long term disability, and even death

16 year old girl with heat intolerance and excessive axillary sweating. Surgical interruption of what structure is most likely to help this patients condition?

Thoracic sympathetic trunk Sweating is mediated by cholinergic postganglionic fibers of the sympathetic nervous system. Ablation of the thoracic sympathetic trunk at the level of T2 can be used to treat severe causes of axillary hyperhidrosis. Other options are systemic anticholinergic mediations (oxlybutinin) or local anticholinergic injectable mediations (botulinum toxin) Note: ablation of other structures to reduce sympathetics stellate ganglion- sweating of hands & Raynauds superior cervical ganglion- head and neck thoracic splanchnic nerves- abdomen

In a media that is folate deficient cell proliferation is minimal with the majority of cells undergoing apoptosis. A substance is added to the folate deficient media which prevents apoptosis and permits proliferation of the proerythroblasts. Most likely substance added to the medium?

Thymidine Folate deficiency inhibits the synthesis of nucleus acids, particularly the formation of deoxythymidine monophosphate (dTMP). This leads to defective DNA synthesis that characteristically causes increased apoptosis of hemopoietic cells and megaloblastic anemia. Thymidine supplementation bypasses this enzyme and can reduce erythroid cell apoptosis

Electrolyte changes associated with ischemic tissue injury?

Tissue ischemia increases the extracellular K+ concentration via reduced functionality of the membrane Na+/K+ ATPase and subsequent leakage of K+ from affected cells. Ischemia may reduce extracellular Na+, Ca++, and HCO3- concentrations. Cytoplasmic Ca++ accumulation is a hallmark of ischemic injury

13 year old boy with an empty right scrotal sac and a mass palpated in the right inguinal canal. Parents are advised the mass should be removed because?

To reduce the risk of malignancy In undescended testes, the seminiferous tubules atrophy if uncorrected due to the higher body temperatures, resulting in decreased fertility and increased risk of malignancy. Orchiopexy (surgical placement of the testes in the scrotal sac) can minimize damage and decreased the risk of testicular cancer (although it doesn't eliminate it)

8 year old Middle Eastern immigrant with low grade fever and skin rash. Rash started on face and rapidly spread down his body. Boy recently returned from month long trip to Yemen, where he visited relative. PE shows generalized, fine, pinkish, maculopapular rash and tender lymphadenopathy bilaterally behind the ears. Most likely virus to cause patients disease?

Togavirus (rubella) In a susceptible child, a febrile maculopapular rash that begins on the face and speaks to the trunk and extremities is suggestive of rubeola (measles) or rubella (German measles). Additional finding of post auricular lymphadenopathy indicates rubella is the most likely etiology. Compared to rubeola, the rash of rubella typically spreads faster and doesn't darken or coalesce. Postauricular and occipital lymphadenopathy are common in rubella. Roseola (HHV-6) causes exanthem subitem (roseola infanatum) which is characterized by a transient maculopapular rash that appears for a few days on the chest and trunk once the patient's fever subsides

Patients takes isosorbide dinitrate early in the morning and again in the afternoon, but doesn't take an evening dose. Such a pattern of drug administration is intended to decrease?

Tolerance development Patients taking maintenance nitrates need to have a nitrate free period every day to avoid tolerance of the drug. Usually this is timed to occur at night when the patient is sleeping and cardiac work is the least

65 year old male with a painless neck mass. PE reveals right sided submandibular lymphadenopathy. Biopsy is consistent with squamous cell carcinoma. Most likely primary site of the cancer?

Tongue The vast majority of cancers in the head and neck are squamous cell carcinomas which are strongly associated with the use of alcohol and tobacco. Other common sites of origin besides the ventral tongue are the floor of the mouth, lower lip, soft palate, and gingiva.

44 year old homeless man with history of alcohol abuse brought to the hospital after being found unresponsive. BP os 90/60 and pulse is 110. He is only responsive to pain and has dry MM. Patient is initially treated with IV fluids and his mental status slowly improves, but then he develops decreased urine output and flank pain. Renal biopsy is performed and reveals marked ballooning and vacuolar degeneration of the proximal renal tubules & normal glomeruli; multiple oxalate crystals are observed in the tubular lumen. Most likely pathogenesis of this patient's renal failure?

Toxic renal injury Ethylene glycol is rapidly absorbed from the GI tract and metabolized to glycolic acid (toxic to renal tubules) and oxalic acid which precipitates calcium oxalate crystals. Ethylene glycol is found in automobile antifreeze, engine coolants, and hydraulic brake fluids and may be used as a substitute for alcohol in alcohol abusers. Ethylene glycol ingestion leads to toxic, acute tubular necrosis with vacuolar degeneration and ballooning of the proximal tubular cells. Patients initially manifest with signs of ethanol intoxication and then signs of renal failure (anorexia, oliguria, flank pain) develops 24-72 hours later. Typical clinical findings induce high anion gap metabolic acidosis, increased osmoloar gap, and calcium oxalate crystals & tubular casts in the urine. Note: prolonged hypotension & severe infection can lead to ischemic acute tubular necrosis which also presents with renal failure, oliguria, and anion gap metabolic acidosis, however, oxalate crystals in the tubular lumen aren't seen

Protein is abundant in arginine and lysine residues. It contains a 30 amino acid alpha helical segment that consists of repeated leucine residues at every 7th position. Protein analyzed most likely represents?

Transcription factor Basic region leucine zippers are a class of eukaryotic transcription factors that are composed of two alpha helical proteins that combine to form a dimer. Each dimer has a DNA binding region composed of basic amino acids (lysine, arginine) and a leucine upper dimerization domain that contains repeated leucine residues every 7th position

Homeobox genes code for what type of proteins?

Transcription regulators Homeobox genes code for transcription factors that bind to regulatory regions on DNA, altering the expression of genes involved in the segmental organization of the embryo along the crania-caudal axis. Homeobox gene mutations interrupt this developmental process, often resulting in severe abnormalities such as skeletal malformations and improperly positioned limbs and appendages

Colony of bacteria resistant to gentamicin and a colony resistant to ampicillin are mixed on an agar plate containing both antibiotics. Bacterial growth is noted after 24 hours. The same experiment is repeated with addition of DNAse to the agar plate. The enzyme is functional in the culture medium but isn't able to penetrate the bacterial cell membrane. During second experiment, no bacterial growth is observed. What is most likely mechanism of bacterial survival in the first experiment?

Transformation Transformation is the process by which bacteria take up naked DNA from their environment and incorporate it into their own genomes. Conjugation-DNA passed by direct cell-cell interaction Transduction- bacteriophage mediated transfer Transposons- transfer of information from one location in the genome to another (moves a gene from a chromosome to a plasmid to ready it for conjugative transfer)

68 year old male with gradual reddening of his urine for the past 3 months. Prior to retirement, he worked at a rubber manufacturing plant for 35 years. Serum creatinine is normal. Most likely to be found on further work up?

Transitional cell carcinoma of the bladder Typically presents as gross hematuria in an elderly man. A history of smoking or occupational exposure to rubber, plastics, aromatic amine-containing dyes, textiles, or leather increases the risk of developing transitional cell carcinoma

72 year old man with fatigue, orthopnea, and exertion dyspnea. Exam reveals bibasilar crackles, elevated jugular venous pressure and bilateral LE pitting edema. LVED pressure-volume curve shows an increased pressure (decreased compliance). What is most likely cause of this patients condition?

Transthyretin deposition (transthyretin is a protein tetramer produced in the liver that acts as a carrier for thyroxine and retinol that when mutated can misfiled and produce amyloid protein) Diastolic heart failure is caused by decreased ventricular compliance and is characterized by normal LV ejection fraction, normal LV end diastolic volume, and elevated LV filling pressures. HTN, obesity, and infiltrative disorders (transthyretin-related amyloidosis, sarcoidosis) are important causes of diastolic heart failure Note: systolic dialated HF would shift the curve to the right due to thinning of the ventricular wall (increased volume and increased compliance)

Wolff Parkinson white syndrome is characterized by symptomatic paroxysmal supraventricular tachycardia (atrioventricular reentrant tachycardia) due to the presence of an accessory condition pathway. During NSR, the presence of this accessory pathway causes ventricular pre-excitation which can be identified on an ECG by?

Triad of shortened PR interval, early upslope of the QRS complex (delta wave) and a widened QRS interval

Patient diagnosed with pancreatitis. Inappropriate activation of what substance likely initiates this condition?

Trypsinogen Pancreatic zymogens are normally converted into their active form by trypsin in the duodenal lumen. Premature cleavage of trypsinogen to trypsin within the pancreas leads to uncontrolled activation of these zymogens, causing pancreatic auto digestion and acute pancreatitis.

52 year old woman with a 2 month history of a skin rash that worsens with sun exposure. Her family says she has been irritable and has episodes of disorientation. Drinks a bottle of gin daily. She has poor nutrition intake and intermittent diarrhea. On exam she has well demarcated, hyper pigmented scales rash on the hands, forearms, and upper chest. Cause of most of her symptoms is determined to be the lack of the precursor vitamin for synthesis of NAD+ coenzyme. The compensatory pathway to synthesize this coenzyme uses what precursor?

Tryptophan Niacin (vitamin B3) can be synthesized endogenously from tryptophan and is essential component of nicotinamide adenine dinucleotide (NAD) and NADP. A deficiency of this vitamin results in pellagra, which is characterized by dermatitis, diarrhea, and dementia.

32 year old woman has been depressed for the past three months and is started on a first line pharmacologic treatment. Two days later she is brought to the hospital after her sister found her to be agitated and confused & also found an empty pill bottle. Patient is tremulous, has abdominal cramps, and diarrhea. Temperature is 102. BP is 210/120. HR is 120 and regular. On exam her pupils are dilated and she is diaphoretic. Neurologic exam reveals bilateral hyperreflexia and inducible ankle clonus. Increased CNS activity cause by a derivative of what amino acid is responsible for this patients condition?

Tryptophan Serotonin syndrome is characterized by confusion, agitation, tremor, tachycardia, HTN, clonus, myoclonus, muscle rigidity, hyperthermia, diaphoresis, and hyperreflexia. Any drugs that increase serotonin levels, including combinations of SSRIs, triptans, or MAO inhibitors and high doses of SSRIs can precipitate this syndrome. Tryptophan is a precursor for serotonin. Cyproheptadine is an antihistamine with anti-serotonergic properties that can be used in the treatment of serotonin syndrome.

Mesothelioma is a rare neoplasm that arises from the pleura or peritoneum. Its strongly associated with asbestos exposure. Hemorrhagic pleural effusions and pleural thickening are characteristic. Histopathology reveals?

Tumor cells with numerous long, slender microvilli and abundant tonofilaments Note: adenocarcinoma shows glandular or papillary elements and has short and plump microvilli distinguishing it from mesothelioma

Power of a study is related to what type of error?

Type II (probability of concluding that there is no difference between groups when one truly exists) Power = 1- B, where B is the type II error rate

Rates of hepatocellular carcinoma in some developing countries are many times higher than in Western nations. A successful hepatocellulr carcinoma prevention strategy in these countries would focus on incorporating?

Universal vaccination Universal vaccination of children against HBV would likely cause a steep decline in the worldwide incidence of hepatocellular carcinoma. The countries with the highest rate of HBV infection have more than 85% of all hepatocellulr carcinoma cases. Individuals in the countries usually have chronic HBV infection acquired through vertical transmission at birth which increases risk of developing HCC by early to mid adulthood 200 fold (presents 20-40 year olds) vs. in the USA it presents in individuals over 60 and is much less common

35 year old man with confusion and lethargy. ABG shows pH of 7.59, pCO2 of 49, and pO2 of 85. What lab study would be most useful in diagnosing the cause of his acid base abnormality?

Urine chloride Metabolic alkalosis is characterized bt a high arterial blood pH, HCO3-, and pCO2. Vomiting/nasogastric suctioning and thiazide/loop diuretic use can cause volume and Cl- depletion, resulting in metabolic alkalosis thats saline responsive. In contrast, hyperaldosteronism leads to metabolic alkalosis that is saline unresponsive. Determining the patients volume status and measuring the urinary chloride concentration can help identify the cause of metabolic alkalosis.

22 year old man with recurrent blistering on the back of his hands and forearms for the past several years. Usually develops small itchy spots, but lately has had large blisters that heal with hyper pigmentation after rupturing. PE shows vesicles and erosions on dorsal of both hands. Most likely deficient enzyme in this patient?

Uroporphyrinogen decarboxylase Patient has porphyria cutanea trade, the most common disorder of heme synthesis. Enzyme deficiencies in the early steps of porphyrin synthesis cause neuropsychiatric manifestations without photosensitivity, whereas late step derangements (after PBG conversion) lead to photosensitivity due to the accumulation of porphyrinogens that react with oxygen on excitation by UV light. Photosensitivity manifests as vesicle and blister formation on sun exposed areas as well as edema, pruritus, pain, and erythema.

6 day old with inability to feed for 12 hours. Parents infant is unable to open mouth and has held hands clenched all day. PE shows increased muscle tone throughout, arching of back, and dorsiflexed feet. Umbilical cord is covered with a clump of soil which mother says is customary in their village to speed cord separation. In addition to hygienic umbilical cord care, what is the most effective strategy to prevent this condition?

Vaccination of pregnant women Neonatal tetanus can be prevented by hygienic delivery and umbilical cord care and universal immunization of women who are pregnant or may become pregnant. Immunized mothers provide passive immunity via transplacental IgG, protecting infants until they receive active immunization (vaccination) around 2 months (when their immune system is mature enough to mount a proper memory immune response).

78 year old with fever, cough, and SOB. CXR reveals a lower lobe consolidation. Sputum microscopy shows gram positive diplococci. What would have been most helpful in preventing this patients lung infection?

Vaccination with bacterial polysaccharide Steptococcus pneumoniae vaccination reduces the risk of invasive disease and is recommended for young patients and the elderly. The pneumococcal polysaccharide vaccine (23 serotypes) is an unconjugated vaccine that induces a T cell independent humoral immune response. In contrast, the pneumococcal conjugate vaccine(13 serotypes) contains polysaccharide attached to a protein antigen which creates a robust T cell mediated humoral response

15 year old with 3 months of periodic, sudden onset, jerking movements involving both arms. These contractions usually occur early in the morning soon after waking up and are aggravated by sleep deprivation. Movements are neither suppressible nor preceded by an urge to make a movement (rules out Tourettes). Patient has never lost consciousness and has no medical problems. Best initial treatment?

Valproic acid Patient has myoclonic seizures which is a form of generalized epilepsy so treatment should be broad spectrum. Broad spectrum anticonvulsants (lamotrigine, levetiracetam, topiramate, valprooic acid) successfully treat most seizure types (focal or generalized onset). Narrow spectrum anticonvulsants (carbamazepine, gabapentin, phenobarbital, phenytoin) are generally favored for focal onset seizures, although they can be effective against focal seizure that evolve to bilateral convulsive (tonic clonic) seizures

74 year old man presents for follow up for HTN. He reports he can tolerate a moderate level of activity. BP is 145/75. Auscultation findings at the base of the heart reveal s crescendo-decresendo murmur peaking in mid systole. Most likely cause of this patients physical findings?

Valvular calcification The murmur of valvular aortic stenosis is typically an ejection or mid systolic murmur of crescendo-decrescendo configuration with maximum intensity over the right second interspace and radiation to the neck and carotid arteries. The most common cause of aortic stenosis in elderly patients (>70) is degenerative calcification of the aortic valve leaflets

10 year old boy with restlessness and involuntary jerking. Patient had a sore throat 3 months ago. On PE, patient has rapid, irregular, jerking movements involving his face, arms, and legs. Patient is at greatest risk of developing what condition?

Valvular heart disease Sydenham chorea presents with involuntary, rapid, jerking movements involving the face, arms, and legs. It occurs 1-8 months after group A streptococcal infection and is one of the major clinical manifestations of acute rheumatic fever. It is caused by a delayed onset autoimmune reaction involving anti-streptococcal antibodies that cross react with the basal ganglia. Patients with this condition have a high risk of chronic valvular disease.

62 year old with mitral valve replacement one month ago with low grade fevers, malaise, and dyspnea. Blood cultures grow gram positive cocci in clusters that are catalase positive and coagulase negative. Empiric treatment should include what antibiotic?

Vancomycin Initial empiric treatment of coagulase negative staphylococcal infection should include Vancomycin due to widespread methicillin resistance, especially in nosocomial infections. If susceptibility results indicate a methicillin susceptible isolate, vancomycin can be switched to nafcillin or oxacillin

Partial agonist of nicotinic acetylcholine receptors that can assist patients with cessation of tobacco use by reducing withdrawal cravings and attenuating the reward effects of nicotine.

Varenicline Because its a partial agonist, it helps reduce the symptoms of nicotine withdrawal by mildly stimulating the receptor & its partial agonist activity only causes limited downstream release of dopamine, resulting in less stimulation of the reward pathways than nicotine

Nephrogenic dibetes insipidus is characterized by increased urine output and decreased urine osmolality that doesnt respond appropriately to water deprivation. Its associated with increased serum levels of what substance?

Vasopressin (due to resistance in the renal collecting ducts/inability of the kidney to mobilize eaquaporin 2 water channels)

Glomerulonephritis, a positive skin rash, and arthralgias in a young woman are suggestive of SLE. Up to 30% of patients with SLE have antiphospholipid antibodies, which can cause paradoxical aPTT prolongation and a false positive RPR/VDRL. Patients with antiphospholipid antibodies are at risk for?

Venous and arterial thromboembolism and recurrent pregnancy loss Note: the antiphospholipid antibodies prolong the PTT in vitro but they cause a HYPERcoagulable state in vivo

Rivaroxaban is an oral anticoagulant that directly inhibits factor Xa. Its used for?

Venous thromboembolism treatment and stroke prophylaxis in patients with atrial fibrillation

ARDS is characterized by hypoxic respiratory failure and bilateral pulmonary edema; its often associated with pneumonia and sepsis. Widespread inflammation of the pulmonary endothelium leads to leakage of fluid into the alveoli. Continued perfusion of fluid filled alveoli causes?

Ventilation perfusion mismatch with right to left physiologic shunting and hypoxemia

60 year old woman with persistent profuse mucoid diarrhea. Her diarrhea hasn't decreased despite not eating much for the past 2 days. Has not had age appropriate colon cancer screening. Labs show hypokalemia and microcytic anemia. Colonoscopy reveals a 2.5cm cauliflower like mass in the sigmoid colon. Mass is resected and histopathology reveals long glands with finger like projections. Most likely diagnosis?

Villous adenoma Adenomatous polyps are either tubular, villous, tubulovillous, depending on their histologic appearance. Villous adenomas tend to be larger, sessile, and more severely dysplastic than tubular adenomas (more likely to undergo malignant transformation). Villous adenomas can cause a secretary diarrhea from increased mucin production; patients develop hypoproteinemia and hypokalemia as mucin is a potassium rich glycoprotein

55 year old man with abdominal discomfort and black stools. His symptoms resolve, but several weeks later he returns with fever, weight loss, and muscle pains. Smokes 1PPD for 30 years and has used IV drugs in the past. Muscle biopsy demonstrates transmural inflammation of the mid sized arteries with areas of amorphous eosin staining material wall necrosis. Areas of internal elastic lamina disruption are also present. Most likely predisposing factor for this patients current condition?

Viral hepatitis Polyarteritis nodosa is a multisystem vasculitis characterized by episodic ischemic symptoms in various organs with sparing of the lungs. Biopsy shows segmental transmural inflammation with fibrinoid necrosis. Commonly associated with Hepatitis B. Treat with Prednisone and cyclophosphamide and ACE inhibitor for HTN.

Unvaccinated 20 month old is brought in with rash. Three days ago she developed a fever along with a cough, congestion, and red eyes. Rash appeared on face yesterday and spread to her trunk, arms, and legs today. Temperature is 103. She is lethargic and ill appearing. PE shows conjunctival injection and a diffuse maculopapular, erythematous rash. Deficiency of what vitamin is associated with high rate of complications from this patients condition?

Vitamin A Measles is spread by contact or aerosolized respiratory droplets. The classic exanthema starts on the face and spreads in a cephalocaudal and centrifugal pattern. Erythematous blanching, maculomapular lesions often progress to a deep red or brown non blanching coalesced rash. Rash spares palms and soles. Acute measles depletes vitamin A stores leading to a risk of keratitis, corneal ulceration, diarrhea, pneumonia, and encephalitis. Vitamin A supplementation reduces recovery time and hospital stay and is recommended in all patients with measles

35 year old with headache and vomiting. Has a history of psychatric illness and is known to practice eccentric dietary habits. PE findings reveals papilledema, dry skin, and hepatomegaly. Most likely cause of patients condition?

Vitamin A overuse (causes intracranial HTN, skin changes, and hepatosplenomegaly) Individuals who consume more than 10 times the daily value are at risk of toxicity and may suffer hepatic injury so severe as to cause cirrhosis. Teratogenic effects of excessive vitamin A include microcephaly, cardiac anomalies, and fetal death (especially in first trimester) Note: large doses of vitamin E are associated with higher mortality rates due to hemorrhagic stroke in adults and higher rates of necrotizing enterocolitis in infants

17 year old girl with occasional gingival bleeding when brushing her teeth. Platelet count is normal. Further evaluation reveals that the patient's platelets don't aggregate appropriately in response to ristocetin. When normal plasma is assed to the solution of patient platelets and ristocetin, appropriate platelet aggregation occurs. What is most likely deficiency in this patient?

Von Willebrand factor Ristocetin aggregation test measures in vitro vWF-dependent platelet aggregation. Ristocetin activates GP Ib receptors on platelets and makes them available for vWF binding. When the vWF level is decreased, there is poor platelet aggregation in the presence of riotocetin. When normal plasma that contains vWF is added, appropriate aggregation occurs. VWF deficiency also leads to a functional deficiency of VIII. This results in prolonged bleeding after tooth extraction and other minor surgeries. PTT may be normal or prolonged depending on the level factor VIII deficiency. Desmopressin stimulaties VWF release from the endothelium Note: in Bernard Soulier syndrome (hereditary deficiency of GP Ib receptors) is characterized by thrombocytopenia, enlarged platelets, and mucocutaneous bleeding; platelet aggregation to ristocetin will also be abnormal, but because vWF levels are normal addition of normal plasma won't correct aggregation Note: Glanzmann thrombasthenia is a hereditary deficiency of GP IIb-IIIa receptors which manifests with mucocutaneous bleeding; platelet aggregation in response to ristocetin is normal as levels of vWF and GP Ib receptors are normal

29 year old with several months of left elbow pain. Pain is worse with activity and is limiting her exercise. Further evaluation reveals her condition is due to tendon injury. A long acting glucocorticoid injection is administered at the most tender spot, at the lateral epicondyle of the humerus. What action is most likely performed by the affected muscle?

Wrist extension Lateral epicondylitis (tennis elbow) is caused by overuse of the extensor carpi radialis brevis and is characterized by angiofibroblastic tendinosis (excess fibroblasts and neovascularization) at its origin on the lateral elbow.

16 year old with primary amenorrhea. She has never menstruated. Exam shows Tanner stage 5 breast and vagina that ends in a blind pouch. A bulge is palpated in the left groin. Serum testosterone is elevated. LH level is high and FSH is normal. Most likely cause of this patients condition?

X-linked defect of the androgen receptor Complete androgen insensitivity is characterized by male genotype and a female phenotype with external female genetalia and a vagina ending in a blind pouch. Serum testosterone levels and LH are high while FSH is normal. Cryptorchid testes secrete testosterone and anti-mullerian hormone from the Y chromosome stimulates regression of the mullerian ducts. Masculizaination doesn't occur due to androgen resistance. Testosterone is peripherally converted to estradiol which results in breast formation. Note: mullerian genesis causes primary amenorrhea because the uterus, fallopian tubes, and proximal vagina fail to develop. LH, FSH, and estrogen levels are normal as the hypothalamic-pituitary-ovarian axis is normal.

Wave on jugular venous tracing that would be absent in atrial fibrillation?

a wave (corresponds to right atrial systole and would be absent due to failure of coordinated atrial muscle contraction)

23 year old asymptomatic Italian American male with microcytic anemia and elevated hemoglobin A2. Has been taking iron supplements as prescribed but lab values are unchanged. Peripheral smear shows hypochromia and poikilocytes, including spherocyte and target cells. Initial step in the pathogenesis of this patients disorder?

mRNA formation (deficiency of beta globin chain synthesis) Microcytic anemia and increased hemoglobin A2 with hypochromia and poikolocytes including spherocytes and target cells. This constellation of findings in an asymptomatic adult of Medetrerranian heritage suggest beta thalassemia minor. This thalassemia is often incorrectly attributed to iron deficiency, but iron supplements do not benefit these patients as they are not iron deficient. Beta thalassemia affects beta chain production and can be caused by a variety of DNA mutations affecting transcription, processing, and translation of beta global mRNA. Most common these mutations cause abbarent precursor mRNA splicing or premature chain termination during mRNA transition. In some cases, point mutations prevent RNA polymerase from binding to the promotor region. Resulting beta global deficiency occurs in the setting of normal heme and alpha chain synthesis, leading to increased formation of hemoglobin A2 and in some patients hemoglobin F. DNA replicates normally despite mutations in beta global gene, its not until the DNA is transcribed into mRNA that hemoglobin production becomes abnormal. Note: alpha thalassemia is more common in individuals from Southeast Asia.

42 year old man with fever and persistent sore throat. Temperature is 101. There are several bruises on his trunk and blood is oozing from intravenous catheter venipuncture sites. Fibrinogen level is low. Bone marrow biopsy shows predominant of immature myeloid cells with azurophilic needle shaped cytoplasmic granules. Chromosomal analysis is most likely to show?

t (15,17) Acute promyelocytic leukemia (M3 AML) can present with persistent infection and coagulopathy causing hemorrhagic signs and symptoms. Bone marrow biopsy classically reveals promyelocytes with intracytoplasmic Auer rods. APL is associated with t(15,17) chromosomal translocation that causes fusion of retinoid acid receptor alpha gene and promyelocyte leukemia gene. Treat with all trans retinoic acid Note: inv(16) is present in the M4 eosinophilic subtype of AML

2 year old girl brought in because of concerns about her development. She says about 40 words and doesn't string words together. Mom says she is worried. Most appropriate response by physician?

'Your child may have a language disorder and could benefit from further assessment' At age 2, children should have a vocabulary of 50-200 words and be using 2 word phrases. Parents concerns about delayed milestones should be validated; they should be reassured that children often catch up but may need help. Further evaluation and regular monitoring are required. Language is the most commonly delayed milestone

Healthy couple bring their three years old sun for evaluation of an eczematous rash. On exam child shows signs of intellectual disability and gain abnormality and has a musty odor. Likelihood couples next child will be affected with same disease?

1/4 Intellectual disability, gait or posture abnormality, eczema, and a musty odor in a toddler are signs of PCU. Most infants with PKU are born to 2 heterozygous carrier parents. Probability carrier parents will transmit an AR disease such as PKU is 1/4

On cardiac auscultation, a snap followed by a rumbling diastolic murmur is heard over the cardiac apex. The snap most likely occurs nearest what point on the cardiac pressure-volume loop?

4 (when the mitral valve opens) Murmur described is mitral stenosis. On the loop, mitral valve opening occurs at the point between isovolumetric relaxation and diastolic filling & this is where the snap would be loudest.

Tetracyclines (tetracycline, doxycycline, minicycline) and aminoglycocides (gentamicin, neomycin, amikacin, tobramycin, streptomycin) inhibit prokaryotic synthesis at the 30S ribosomal subunits whereas erythromycin and chloramphenicol inhibit protein synthesis at?

50S ribosomal subunit

The resting membrane potential is the difference in the electrical charges across the cell membrane under steady state conditions. The ions that are most permeable to the cell membrane make the largest contribution to the resting membrane potential. In general what ions are responsible for the resting potential?

A high potassium efflux and some sodium influx are responsible for the resting potential which is typically about -70 mV

3 year old girl with several months of fatigue and difficulty walking. She ambulates normally at first but then rapidly becomes weak and tired. Exam shoes decreased power in all extremities. Cardiac exam reveals 1/6 systolic murmur and S3 gallop. Glucose is 37. Creatine kinase is 304. UA is negative for ketones. Muscle biopsy shows low carnitine content. What substance has deficient synthesis in this patients disease?

Acetoacetate Patient with myopathy, cardiomyopathy, hypoketotic hypoglycemia in the setting of decreased muscle carnitine content is consistent with primary carnitine deficiency. Carnitine deficiency impairs FA transport form the cytoplasm into the mitochondria, preventing beta oxidation of fatty acids into acetyl CoA. This leads to cardiac and skeletal myocyte injury (due to lack of ATP from TCA cycle) and impaired ketone body production by the liver during fasting periods.

55 year old woman with persistent cough and recent unintentional weight loss. Never smoked and no history of exposure to industrial pollutants. PE reveals decreased breath sounds and dullness to percussion in the left lung base. Imaging shows irregular mass in the lower lobe of left lung and a left pleural effusion. Most likely malignancy?

Adenocarcinoma Most common lung cancer in general population is adenocarcinoma. Its also the most common subtype in women and nonsmokers. Its located peripherally and consists of tumor cells that form glandular or papillary structures. Note: squamous and small cell carcinomas have a strong association with smoking. Small cell is the most aggressive and is treated with chemo and radiation. The other 3 can be treated with surgery if they are localized

23 year old with restlessness in her legs and inability to lie or sit still. Was diagnosed with schizophrenia a month ago and has been taking medication. Most likely diagnosis?

Akathisia Akathisia is an extrapyramidal side effect of antipsychotic medication characterized by inner restlessness and an inability to sit or stand in one position. Akathisia is frequently misdiagnosed because the restlessness is misinterpreted as worsening psychotic agitation. Patient's antipsychotic dose is often increased rather than decreased, exacerbating the akathisia. Treatment includes decreasing antipsychotic dose if possible or treating with a beta blocker/benzo.

58 year old man with chronic hepatitis c with abdominal fullness, vomiting, and yellow discoloration of eyes and skin. Labs reveal elevated transaminases and total bilirubin. US reveals multiple solid nodules within the liver parenchyma. Colonoscopy is normal. Near total hepatectomy is performed and shows a large hepatic mass and multiple satellite lesions. What serum marker would be most useful for monitoring patient's disease reoccurrence?

Alpha-fetoprotein Patient with chronic hepatitis c, large hepatic mass, and multiple satellite lesions has HCC. Alpha fetoprotein is normally produced by the fetal liver and yolk sac during gestation. Its often moderately elevate din patients with chronic liver disease, such as viral hepatitis, but it can be strikingly elevated in patients with HCC, and a sudden rise in AFP is a sign patient with chronic liver disease may be harboring HCC. However, AFP doesn't correlate with the size or stage of HCC.

68 year old male with several weeks of dyspnea, cough, and malaise. Has difficulty climbing a flight of stairs due to SOB. History of RA, hypothyroidism, GERD, and difficulty to control atrial fibrillation with rapid ventricular response. Temperature is 100.6. PE shows bilateral inspiratory crackles. CXR shows patchy areas of interstitial infiltration throughout the lungs. Most likely medication responsible for his symptoms?

Amiodarone Slowly progressive dyspnea, fever, cough, and patchy interstitial infiltration on radiograph are characteristic features of inflammation and fibrosis in the pulmonary interstitium caused by amiodarone induced interstitial pneumonitis. The condition is usually reversible with a reduction in dose or discontinuation of the drug. Other reactions to oral ayidarone are arrhythmias, hepatic injury, thyroid abnormalities, and bluish grey discoloration of the skin. Other medications associated with interstitial lung disease: nitrofurantoin, methotrexate, bleomycin.

Enoxaparin binds to what structure in the blood?

Antithrombin III Enoxaparin is a LMWH that functions like heparin in that it binds and activates AT III. Activated AT III binds to factor Xa and stops factor Xa from converting prothrombin to thrombin. Due to its fewer number of molecules, LMWH acts primarily on factor Xa, not thrombin

Most common cause of death in Marfan syndrome?

Aortic disease Early onset cystic medial degeneration of the aorta predisposes to aortic dissection, the most common cause of death in these patients

Patient with sudden onset chest pain. History of DM, uncontrolled HTN, and smoking .No EKG abnormalities. CXR shows cardiomegaly, abnormal aortic contour, and widened superior mediastinum. Diagnosis?

Aortic dissection Risk factors are uncontrolled HTN and Marfans syndrome

Causes an increase in the total stroke volume with abrupt distension and rapid falloff of peripheral arterial pulses, resulting in wide pulse pressure. Leads to bounding peripheral pulses and head bobbing with each heartbeat.

Aortic regurgitation

Progesterone is the primary hormone responsible for stimulating the endometrium so that its suitable for implantation. Progesterone withdrawal causes the endometrial cells to undergo what process?

Apoptosis (resulting in menstrual bleeding)

Primary ovarian insufficiency occurs in women less than 40 and presents with amenorrhea and an elevated FSH level. It may result from follicular depletion through accelerated atresia, which is what type of process?

Apoptotic

Several months to years after an ischemic brain infarction the necrotic area appears as a cystic cavity surrounded by a wall composed of dense fibers formed by?

Astrocytic processes (glial scar) Note: neuropathology coronal sections are oriented with the right side of the brain on the examiners right side

Anti-smooth muscle antibodies are seen in what condition?

Autoimmune hepatitis (commonly presents in middle aged women as chronic progressive hepatitis)

Results from a defect night DNA helicase gene RecQL3 and is characterized by short stature, phosptensitivity, erythema and telangiettasias. Patients are predisposed to GI and lymphoproloferative malignancies

Bloom syndrome

Tacrolimus and Cyclosporine are common immunosuppressive agents used to prevent rejection of transplanted solid organs, and in inflammatory disorders (severe psoriasis). Major side effect?

Calcineurin inhibitor nephrotoxicity is dose related and manifests as a rise in serum BUN and creatinine levels as well as increased BP Note: long term use can lead to obliterative vasculopathy, tubular vacuolization, and glomerular scarring

Due to mutation of the genes responsible for myelin synthesis. Affected patients present with distal muscle weakness, sensory loss, and atrophy of the calf muscles (stork leg deformity)

Charcot Marie Tooth Disease

Presents with fever, night sweats and weight loss & lymphadenopathy. Peripheral smear typically reveals numerous small smudged lymphocytes

Chronic lymphocytic leukemia

Symptoms of intranasal cocaine abuse?

Chronic nasal discharge, atrophic nasal mucosa, thinning of the nasal septum leading to perforation, oropharyngeal ulcers, and osteolytic sinusitis. Effects are mediated primarily by local vasoconstriction

Most important respiratory component in clearing small particles that become lodged in the terminal bronchioles?

Ciliated cells Most inhaled particles that lodge in the bronchial tree are removed via proximal transport by ciliated epithelial cells (mucociliary clearance). Mucous secreting cells are present to the level of the larger bronchioles, after which club cells become the prominent secretory cell type. Terminal bronchioles are covered by ciliated cuboidal epithelium and club cells with help mucociliary clearance in this region

Occurs when 2 different strains of a mutant organism are able to produce wild type offspring. Typically, both parents have homozygous mutations in different genes within the same pathway. When they are crossed, the offspring inherits 1 normal allele from each parent allowing them to bypass both of the metabolic blockages and display the wild type phenotype

Complementation

28 year old with raised pigmented lesion on leg. No itching or pain. Doesn't know how long its been there. Has used tanning machines a few times in the past but denies excessive sun exposure. Father was recently diagnosed with skin cancer. Biopsy of lesion reveals uniform round cells at the basal positron of the epidermis that extend into the underlying dermis. The cells contain inconspicuous nuclei and show no mitotic activity. Most likely diagnosis?

Compound melanocytic nevus Compound nevi are benign proliferations of melanocytes that involve both the dermis and epidermic. These lesions appear as slightly raised papule with uniform pigmentation and symmetrical sharp borders

Most effective strategy for decreasing adverse outcomes and preventing avoidable readmissions during the transition from hospital to outpatient care?

Comprehensive discharge checklist A discharge checklist detailing medication changes and follow up appointments can significantly facilitate a patients transition from the hospital and improve adherence to outpatient treatment. Individuals who experience smooth transition from the inpatient to the outpatient setting are at a low risk for early rehospitalization

Rotavirus destroys small intestinal enterocytes resulting in loss of digestive enzymes produced by these cells which causes osmotic diarrhea. Bowel epithelial stem cells located in what location are responsible for there regeneration of the mucosal epithelium?

Crypts of Liberkuhn

67 year old male with low grade fevers and fatigue. No history of heart disease. Temp is 101.1. On exam there is a diastolic murmur at the LSB. Contender erythematous macule are noted on his hands and feet. Blood cultures show gram positive cocci that are catalase negative and able to grow in hypertonic (6.5%) saline and bile. Patients medical history is most likely to include what type of procedure in the past month?

Cystoscopy Enterococcus is a component of the normal colonic and urogenital flora and is capable of growing in hypertonic saline and bile. It is gamma hemolytic, catalase negative, and pyrrolidonyl arylamidase positive. Genitourinary instrumentation or catheterization has been associated with enterococcal ENDOCARDITIS (what patient has)

Can occur in transplant patients and usually presents with odynophagia or dysphagia that can be accompanied by fever or burning chest pain. Endoscopy typically shows linear and shallow ulcerations in the lower esophagus, and histology usually shows enlarged cells with intranuclear inclusions

Cytomegalovirus esophagitis

Patient asks physician if he would consider going out for dinner sometime. Physician finds the patient attractive and would like to accept the invitation. Most appropriate response to the patient's request?

Decline the invitation, explaining that dating a patient is always unethical Romantic and sexual relationships with current patients is always unethical. Relationships with former non-psychiatric patients may be acceptable on a case by case basis provided the physician patient relationship is terminated well before hand

20 year old with intermittent episodes of self resolving jaundice that isn't provoked by any particular circumstances or events. Labs show elevated direct bilirubin. Liver biopsy shows abundant pigment inclusions in the lysosomes of otherwise normal hepatocytes. The pigment is composed of polymers of epinephrine metabolites. Most likely cause of this patients jaundice?

Defective hepatocellular excretion of bilirubin glucuronides Dubin-Johnson syndrome is a benign autosomal recessive disorder characterized by defective hepatic excretion of bilirubin glucuronides across the canalicular membrane, resulting in direct hyperbilirubinemia and jaundice. Grossly, the liver appears black due to impaired excretion of epinephrine metabolites which appear as dense pigments within lysosomes.

65 year old man with worsening right knee pain. Reports locking of the knee during which he can't move the joint for several minutes. Right knee shows joint swelling and limited ROM. Arthroscopic evaluation reveals free floating tissue fragments in the joint space. What is an important step in the pathogenesis of this patients current condition?

Degradation of cartilage type II collagens and proteoglycans Osteoarthritis is characterized by the degradation of type II collagen and proteoglycans within articular cartilage. Major contributing factors include excessive biomechanical stress and increased intraarticular metalloprotease activity. In advanced OA, apoptosis of chondrocytes can lead to fragility and fragmentation of the articular cartilage, resulting in formation of loose bodies.

Melanoma often has an early horizontal growth phase with low metastatic potential followed by a nodular, vertical growth phase with significantly increased risk of metastasis. Most important prognostic indicator in malignant melanoma?

Depth of invasion (breslow thickness) A- asymmetirc B- irregular borders C- color variability D- diameter greater than .5 to 1cm E- evolution in size and appearance over time

65 year old with sudden onset headache. History includes lung adenocarcinoma with abdominal metastasis. Initial non contrast CT reveals small temporal lobe hemorrhage. Over the next 24 hours he becomes progressively obtunded and develops tonic clonic seizure. Repeat head CT reveals acute hematoma and expansion with brain herniation. What finding would you expect to be seen on repeat neurological exam?

Dilated pupil Expanding space occupying lesions within the temporal lobe can cause elevated ICP with transtentorial herniation of the uncus. Uncal herniation often compresses the ipsilateral third CN as it exits the midbrain resulting in oculomotor nerve palsy with a fixed dilated pupil (due to preganglionic parasympathetic fiber damage)

35 year old man with progressive fatigue, dyspnea on exertion, and LE edema for the last 2 weeks. Preceding these symptoms he had an episode of fever, runny nose, and myalgia that resolved after several days. Patient has no other PMH, no significant FH, and takes no meds. PE reveals JVD, bibasilar crackles on lung auscultation, and 2+ pitting edema involving the LE. What ECG findings is most likely to be seen in this patient?

Dilated ventricles with abnormal systolic ventricular function Dilated cardiomyopathy results from direct damage to cardiomyocytes leading to myocardial contractile dysfunction (systolic dysfunction), volume overload, and ventricular dilation (eccentric hypertrophy). Viral myocarditis is a common cause of dilated cardiomyopathy and should be suspected in a young patient who develops HF following symptomatic viral prodrome. An inadequate immune response to the viral infection allows virus to infect and persist inside cardiomyocytes resulting in damage due to dire cytotoxic effect or destructive autoimmune reaction.

New diagnosis in DSM-5 characterized by persistent irritability and frequent, developmentally inappropriate temper outbursts

Disruptive mood dysregulation

6 year old female with ALL undergoes allogenic bone transplant. Three months after the procedure her WBC count is 4500 and a serum sample reveals XY composition. What best explains this finding?

Donor karyotype When a patient receives a bone marrow transplant, the donor stem cells engraft in the bone marrow and produce all of the hematopoietic cell lines for the recipient. This, the recipients peripheral blood cells will be genetically different from the rest of the body's cells

34 year old woman with double vision. Shining into her right eye causes constriction of her right pupil, but not the left pupil. Shining light into her left eye causes construction of only her right pupil. What additional PE finding is likely to be found in this patient?

Drooping of the eyelid The pupillary light reflex is assessed by shining light in an eye and observing the response in that eye (direct) and the opposite eye (consensual). The optic nerve is responsible for the afferent limb and the oculomotor nerve is responsible for the efferent limb. Patients lesion involves left oculomotor. Symptoms of oculomotor palsy are ptosis, down and out pitiless, dilated pupil, and loss of accommodation.

23 year old with fever, chills, headache, and myalgias. She was diagnosed with syphillis earlier in the day and received an IM injection of benzathine penicillin G, and several hours later her current symptoms began. Most likely cause of this patients current symptoms?

Drug induced bacterial disintegration The Jarisch-Herxheimer reaction is an acute inflammatory reaction that occurs within hours of treated for spirocheteal infections. The rapid lysis of spirochetes releases inflammatory bacterial lipoproteins into the circulation and causes acute fevers, rigors, and myalgias

One patient receives IM inactivated polio vaccine and the other patient receives a live attenuated oral vaccine. One month after the vaccine, the levels of what poliovirus antibody will differ most between these two patients?

Duodenal luminal IgA The live attenuated oral (Sabin) poliovirus vaccine produces a stronger mucosal secretory IgA immune response than does the inactivated poliovirus (Salk) vaccine. This increase in mucosal IgA offers immune protection at the site of viral entry by inhibiting attachment to the intestinal epithelial cells.

Active massive PE can presents with syncope and obstructive shock due to occlusion of the pulmonary vasculature. Hemodynamic parameters typically show?

Elevated central venous pressure, elevated SVR and reduced CO

Prostatitis is characterized by constitutional and urinary symptoms with prostate gland tenderness/bogginess/swelling/spongy on digital rectal exam. Its most commonly caused by?

Enteric organisms similar to those found in UTI (E. coli, enterococci)

Menopause occurs on average at age 51 and is diagnosable retrospectively after 12 months of amenorrhea. An elevated serum level of what hormone confirms the diagnosis?

FSH

3 month old boy that turns blue every time he cries. When the episodes last a long time he gets agitated and gasps for breath. What is most likely responsible for his symptoms?

Fallot tetrology TOF is characterized by cyanosis that worsens with feeding, crying, or exercise. Older children with TOF squat to improve pulmonary blood flow

Risk of myopathy with statins is increased with contaminant use of what drug class?

Fibrates Primary side effects of statins include hepatitis and myopathy; fibrates (particulalry gemfibrozil) can impair hepatic clearance of statins, increasing the risk of severe myopahty Note: increased myopathy is also likely with concurrent use of niacin or ezetimibe but to a lesser extent

Medication most effective in treating bulimia nervosa?

Fluoxetine Combine with nutritional rehab and psychotherapy for first line treatment of bulimia nervosa!

Type of anemia that typically develops in alcoholics?

Folic acid deficiency anemia, a megaloblastic anemia that can develop within weeks. Peripheral smear shows macrocytosis, ovalocytosis, and neutrophils with hyperhsegmented nuclei

What occurs in a frontal eye field lesion?

Frontal eye field regions are located bilaterally in the prefrontal cortex. The right eye field generates conjugate gaze movements to the left and the left eye field generates conjugate eye movements to the right. When a frontal eye field is affected by a lesion, the influence of the contralateral eye field predominates and the eye deviate toward the lesion

Patient is homozygous for a point mutation in the beta-globing gene resulting in a glutamic acid to valine substitution at position 6. Sequencing would most likely reveal what changes?

GAG --> GTG or GAA --> GTA Sickle cell disease is the result of a missense mutation that causes valine to replace glutamic acid in position 6 in the hemoglobin beta-globin chain. RNA contains the pyrimidine base uracil (which wouldn't be seen in the beta globin gene itself, only in the mRNA transcript) whereas DNA contains the base thymidine.

43 year old HIV positive man complaining of progressive visual impairment. Last CD4+ count was 47. Opthalmoscopic exam reveals inflammatory vascular sheathing and associated hemorrhage. What medication is best initial therapy for this patient?

Ganciclovir Most common cause of retinitis in HIV patients is CMV which develops secondary to hematogenous spread of CMV to the eye. Most frequently affects AIDS patients with a CD4 count less than 50. Most common complication is retinal detachment due to tearing of thin, atrophic scar tissue that forms in areas of prior inflammation. Treatment is with Ganciclovir which is a guanosine nucleoside analog that has good activity against CMV DNA polymerase.

Reason that host antibodies do not confer effective immunity against HCV?

Genetic variations created during HCV replication result in marked variety in the antigenic structure of HCV envelope proteins. The production of host antibodies lags behind that of new mutant HCV stains preventing infected individuals from mounting an effective immune response. As production of host antibodies against a strain begins, that strain dies off, and a new one takes its place. HCG RNA dependent RNA polymerase has no 3' to 5' exonuclease proofreading activity which leads to multiple genotypes/subtypes

35 year old presents with skin rash. Was treated in hospital for Pneumocystic pneumonia and painful oral ulcers 6 months ago. Frequently uses illicit drugs. PE shows a widespread rash consisting of violet colored lesions. Most likely cause of patients current condition?

HHV8 Kaposi's sarcoma typically presents as blue-violet or brownish skin plaques on the extremities and mucous membranes of HIV positive patients. This patients clinical history of pneumocystic pneumonia and painful oral ulcers in a illicit drug user is highly suggestive of HIV infection. Kaposi tumor arises from primitive mesenchymal cells and is strongly associated with HHV8. Lesions can develop on the face and genitals as well and in late stage disease spread to the lungs and GI tract. Histologic exam of the lesions shows spindle and endothelial cell proliferation, red blood cell extravasation, and inflammation.

Cystic hygromas are most commonly associated with chromosomal aneuploidy (turner syndrome, trisomies). They manifest as soft compressible masses that transilluminate and are most commonly found in what area?

Head and neck, typically on the left side

Liver tumor that can regress with discontinuation of oral contraceptives?

Hepatic adenoma

63 year old with muscle weakness and severe cramping in LE. PMH includes HTN for which he was started on clorothalidone and amlodipine 4 weeks ago. Most likely cause of his muscular symptoms?

Hypokalemia Thiazide diuretics decrease intravascular fluid volume, which stimulates aldosterone secretion and leads to increased excretion of K+ and H+ in the urine. This results in hypokalemia and metabolic alkalosis. Significant hypokalemia can lead to muscle weakness, cramps and possible rhabdomyolysis.

What cytokine is produced exclusively by lymphocytes?

IL-2 IL-2 is exclusively produced by T lymphocytes and functions to stimulate the growth and differentiation of T cells, B cells, NK cells, and macrophages

Anatomical landmark that helps locate the optimal site for needle insertion?

Iliac crest Optimal location for needle insertion during a lumbar puncture is the L3/L4 or L4/L5 space as this is well below the spinal cord termination site (L1 in adults). L4 vertebral body lies on a line drawn between the highest points of the iliac crest

32 year old male undergoes right nephrectomy. What percentage of his baseline GFR will his GFR be immediately after surgery and six weeks after surgery?

Immediately after surgery 50% Six weeks after surgery 80% Heminephrectomy results in compensatory hyper filtration and hypertrophy of the nephrons in the remaining kidney. Post-nephrectomy, the total GFR increases from 50% to 80% within several weeks

3 week old boy who was born at full term with lethargy for a day. Boy didn't receive vaccinations or medications after birth due to parental preference of a 'natural' newborn period. PE shows large anterior bulging fontanel. Eyes appear driven downward and patient is not able to track upward. ICH is confirmed on head CT. Most likely cause of this infants condition?

Impaired clotting factor carboxylation Vitamin K deficiency results in impaired clotting factor decarboxylation. Newborns are at risk for vitamin K deficiency due to poor transplacental transfer of vitamin K and low content in breast milk. All newborns should receive vitamin K prophylaxis to prevent bleeding complications. ICH is a potentially fatal complication often presenting with signs of increased ICP (altered mental status, enlarging head circumference, bulging fontanel, downward driven eyes) Note: if the patient received vitamin K injection at birth then abusive head trauma would be the most likely diagnosis Note: germinal matrix fragility is responsible for intraVENTRICULAR hemorrhage in premature infants (germinal matrix starts involuting around 28 weeks gestation and disappears by full term)

Most common cancers (aside from skin cancer) in women in order of incidence & mortality?

Incidence- breast, lung, colon Mortality- lung, breast colon

The normal cardiovascular response to exercise involves increased CO and vasodilation in exercising muscles. This results in what changes?

Increased HR and end diastolic volume & decreased end systolic volume and SVR

39 year old with reduced taste sensation, burning sensation in her mouth, and lip dryness. Labs show low hemoglobin and low MCV. Most likely cause of her symptoms?

Iron deficiency Iron deficiency causes a hypochromic, microcytic anemia. Affected individuals may present with typical symptoms of anemia (weakness, fatigue, headache, irritability) as well as glossal pain, dry mouth, atrophy of the tongue papillae, alopecia, and pagophagia (craving for ice)

The great saphenous vein can be accessed at what site to perform a CABG?

Just inferolateral to the pubic tubercle The great saphenous vein is a superficial leg vein that originates on the medial side of the foot, courses anterior to the medial malleolus and then travels up the medial aspect of the leg and thigh. It drains into the femoral vein within the region of the femoral triangle, a few centimeters inferolateral to the pubic tubercle

Patient with colon adenocarcinoma. Therapy with monoclonal antibodies that bind to EGFR on the malignant cells is considered. An activating mutation in what molecule will most likely make this therapy ineffective?

KRAS protein Activating mutations in the KRAS gene lead to constitutive activation of the EGFR pathway, promoting increased cell proliferation and growth. Tumors harboring these mutations are resistant to treatment with anti-EGFR drugs (cetuximab, panitumumab)

TNF-alpha inhibitors (Entaracept, infliximab, adalimumab) impair cell mediated immunity. All patients beginning treatment with TNF-alpha inhibitors should be evaluated for?

Latent TB

Pulmonary artery occlusion pressure is measured at the distal tip of the pulmonary artery catheter after an inflated balloon occludes blood flow though a pulmonary artery branch and closely corresponds to the pressure in what locations?

Left atrial and left ventricular end diastolic pressure

Stress urinary incontinence is defined as involuntary urine loss with increased intraabdominal pressure. Pelvic floor strengthening (kegal exercises) targets what muscles to improve support around the urethra and bladder?

Levator ani (pubococcygeus, iliococcygeus, puboectalis)

Caused by an autosomal dominant mutation in the tumor suppressor gene TP53; leukemia, sarcomas, and tumors of the breast, brain, and adrenal cortex are most common

Li-Fraumeni syndrome Note: p53 plays an essential role in maintaining the integrity of the human genome by causing ells with mutant DNA to arrest in the G1/S stage of the cell cycle until damage is repaired

Formula for maintenance dose?

Maintenance dose= [Steady state plasma concentration x Clearance]/ [bioavailability fraction] For medications administered IV the bioavailability is 1 Make sure to account for time interval that medication is administered! For example if maintenance dose is calculated to be 2mg/min but the drug is administered every 6 hours then: 2mg/min x 60min/hour x 6 hours = 720 mg

64 year old with cramps abdominal pain while passing stool for the past 2 days. Also noticed some blood in his stool. Had an MI 6 weeks ago. PE shows decreased bowel sounds. Right femoral and dorsalis pedal pulses are weak. What findings are likely to be seen on colonoscopy?

Mucosal hemorrhage and patchy areas of necrosis Early ischemic colitis is characterized by mucosal hemorrhage and patchy areas of necrosis. Eventually the bowel wall thickens and becomes edematous, and transmural infarction develops

Damage to the nucleus ambiguous characteristically produces?

Myoclonus

62 year old man with progressive exertion dyspnea. PMH of pneumonia and HTN. 45 pack year smoking history. On exam breath sounds are decreased bilaterally and wheezes are heard. CT scan of chest shows dilated airspaces. What cell type most likely contributed to the development of this patients lung condition?

Neutrophils Heavy smoking history, exertion dyspnea, and CT findings of dilated airspaces are suggestive of centroacinar emphysema. Pathogenesis of centroacinar emphysema associated with chronic, heavy smoking predominantly involves the release of proteinases, especially elastase, from infiltrating neutrophils and macrophages Note: type I pneumocytes are destroyed by the acing wall demage, they don't contribute to it

How does PTH act on osteoclasts to increase bone resorption?

OsteoBLASTS have PTH receptors. PTH causes osteoblasts to increase the production of RANK-L and M-CSF; these 2 factors stimulate osteoclastic precursors to differentiate into bone resorbing mature osteoclasts

Involves recurrent UNEXPECTED panic attacks characterized by an abrupt surge of fear accompanied by physical and cognitive symptoms. Individuals experience persistence worry about future attacks and typically will restrict their activities as a result

Panic disorder

Virus is isolated that replicated in the nucleus using host enzymes. Infecting virus most likely belongs to what family?

Papillomaviridae (small non enveloped isocahedral viruses with circular double stranded DNA genomes) The papillomaviruses replicate within the host cell nucleus utilizing host cell DNA and RNA polymerases. Viral proteins inactivate p53 and Rb proteins causing the cell to enter the S phase and induce viral genome replication by cellular DNA polymerases Note: DNA viruses (except poxvirus) replicate in the nucleus & RNA viruses (except orthomyxovirus and retroviruses) replicate in the cytoplasm). Poxiruses are unique among DNA viruses that they don't replicate in the host cell nucleus

Injury to the ling thoracic nerve along the lateral chest wall may occur due to penetrating trauma or surgical procedures (axillary lymph node dissection, tube thoracostomy). Damage to this nerve causes?

Paralysis of the serrates anterior muscle (inability to raise the arm over the head and protrusion 'winging' of the medial border of the scapula when an outstretched arm is pushed forward against resistance)

65 year old man with SOB and cough for 2 months. Has had an 11 pound unintentional weight loss. Has smoked 2 PPD for the last 50 years. CXR shows right lung mass with enlarged mediastinal lymph nodes. Calcium level is elevated. Bone scan is negative for focal lytic lesions. What else is most likely to be elevated in this patient?

Parathyroid hormone related peptide Humoral hypercalcemia of malignancy is caused by secretion of PTHrP which closely resembles PTH at the bioactive amino terminal region. PTHrP causes increased bone resorption and decreased renal excretion of Ca2+

16 year old with vaginal bleeding. LMP was 12 weeks ago. Had a miscarriage last year that required D and C. Urine pregnancy test is positive. Transvaginal US shows an intrauterine gestational sac without fetal cardiac activity. D and C is performed which shows fetal tissue, focal trophoblastic hyperplasia, and some enlarged villi interspersed with normal villi. Most likely diagnosis?

Partial mole A partial mole will have a triploid karyotype (69,XXX or XXY) and contain fetal tissue with some edematous villi with focal trophoblastic proliferation and normal appearing villi. Patients present with vaginal bleeding and a prior miscarriage, prior molar pregnancy, and extremes of maternal age are risk factors

Superior gluteal nerve injury results in weakness and paralysis of the gluteus medius, gluteus minimus, tensor fasciae latae muscles. Signs on PE?

Pelvis tilts downward toward the contralateral side (positive Trendelenburg sign) & patients lean toward the ipsilateral side when walking to help stabilize pelvis (gluteus medius gait)

Purpose of prescribing chloroquine with primaquine?

Prevent disease relapse Chloroquine is treatment of choice for uncomplicated malaria contracted in chloroquine sensitive geographic region. It eliminates susceptible erythrocytic forms of Plasmodium species. Primaquine is added in the treatment of infections with P. vivax and P. oval to eradicate the intrahepatic stages (hypnozoites) of these malarial species, which are responsible for relapses

43 year old with advanced HIV presents with recent onset headaches and personality changes. Imaging shows solitary mass within the temporal lobe. Biopsy reveals uniform cells containing EBV virus genome. Most likely?

Primary CNS lymphoma Primary CNS lymphomas occur in immunocompromised patients such as those suffering from AIDS. These tumors arise from B cells and are universally associated with EBV. They are high grade tumors with poor prognosis. Commonly positive for B cell makers CD20 and CD79a. Clinical presentation is nonspecific and can involve mental status change, seizures, progressive neurologic focal deficits.

First line medication for nausea and vomiting in pregnancy?

Pyridoxine (vitamin B6)

Exposure to Toxicohedron species (poison ivy and oak) causes a type IV hypersensitivity reaction. Clinical clues that suggest this diagnosis?

Recent outdoor activity, involvement of exposed skin, and streaky erythematous lesions

10 year old boy that has required several blood transfusions due to anemia. Exam reveals conjunctival pallor and moderate splenomegaly. Hemoglobin is low. Enzyme assays demonstrate low pyruvate kinase activity. Most likely cause of patients splenomegaly?

Red pulp hyperplasia Pyruvate kinase deficiency causes hemolytic anemia due to failure of glycolysis and resultant failure to generate sufficient ATP to maintain erythrocyte structure. In this case, hyperplasia results from increased work of the splenic parenchyma which must remove these deformed erythrocytes from the circulation

Facilitating interview techniques are useful in obtaining information and establishing rapport. Technique that involves repeating what the patient has said to confirm understanding and convey that the patients concerns are head?

Reflection

Pattern of airway resistance along the bronchial tree?

Regional airway resistance within the first 10 generations of the bronchi contributes to most of the total airway resistance of the lower respiratory tract. Resistance is maximal between the 2nd - 5th generation airways (because of highly turbulent flow), including the segmental bronchi. In contrast, airways less than 2mm in diameter (bronchioles) contribute less than 20 percent of the total airway resistance (because the summated cross sectional area massively increases slowing airflow velocity).

It is found that a drug inhibits the codon 'UAA' in a bacterial mRNA fragment. Molecule that is the most likely target of this drug?

Releasing factor 1 Releasing factors recognize stop codons (UAA, UAG, UGA) and terminate protein synthesis. They facilitate release of the polypeptide chain from the ribosome and dissolution of the ribosome-mRNA complex

Viral bronchiolitis usually presents in children less than 2 with a low grade fever, cough, tachypnea, and increased work of breathing; examination shows diffuse wheezes and crackles, tachypnea, retractions, and hypoxemia. Most commonly caused by?

Respiratory syncytial virus

MOA of retinoids in treating acne?

Retinoids are vitamin A analogs that bind nuclear receptors which function as transcription factors. In patients with acne, retinoids decrease sebum production and increase cellular turnover and the shedding of cells from the stratum corneum. This reduces hyperkeratinization, opens blocked pores, and prevents formation of microcomedomes

19 year old woman presents after being in an MVC. History of recurrent pneumonia & celiac disease. She receives a blood transfusion with O negative packed RBCs. During the transfusion, she develops facial swelling, generalized hives and SOB. Most likely diagnosis?

Selective IgA deficiency Selective IgA deficiency is the most common primary immune deficiency and can present with recurrent sinopulmonary and GI infections as well as autoimmune disease. Patients with severe IgA deficiency can have anaphylaxis during transfusion of blood products that contain small amounts of IgA, as they can form IgE antibodies against IgA. IgG and IgM serum levels are normal.

4 year old girl with painful rash affecting the right lower extremity. Dad states the girl has had no medical problems other than some colds and a couple of generalized rashes when she was younger. PE shows grouped vesicles with underlying erythema along the lower thigh. Most likely diagnosis?

Shingles Herpes zoster results from the reactivation of latent VZV in the dorsal root ganglion. It presents as an eruption of vesicles with surrounding erythema isolated to a single dermatome accompanied by severe neuropathic pain. It can be seen at ANY AGE after initial varicella infection, although adults over 50 are at greatest risk

Tissues supplied by what artery would be affected by compression of the splenic artery?

Short gastric The splenic artery originates from the celiac artery and gives off several branches to the stomach and pancreas (pancreatic, short gastric, left gastroepiploic) before reaching the spleen. Due to poor anastomoses, the gastric tissue supplied by the short gastric arteries is vulnerable to ischemic injury following splenic artery blockage

Trazodone is a highly sedating antidepressant commonly used to treat insomnia. Proapism is a rate but serious adverse effect and therefore Trazodone should be used with caution in patients that have conditions that are known to predispose to priapism such as?

Sickle cell disease & multiple myeloma

Individual effects of Simvastatin and cholestyramine on cholesterol synthesis?

Simvastatin decreases hepatic cholesterol production, while cholestyramine increases hepatic cholesterol and bile acid synthesis. Combination therapy results in synergistic reduction in plasma LDL

44 year old man is found to have high free thyroxine, low TSH, and positive thyrotroptin receptor antibodies. What additional finding would be most specific to an underlying cause of his hyperthyroidism?

Skin thickening and induration over the shin Pretibial myxedema and Graves opthalmopathy are specific features of Graves disease. They are caused by an autoimmune response directed against the TSH receptor that results in the accumulation of glycosaminoglycans within the affected tissues

Clavicle is the most commonly fractured bone and most clavicle fractures occur in young children. Muscle that displaced the medial fragment of the clavicle superiorly after a fracture?

Sternocleidomastoid Note: the weight of the arm and the pectorals major cause inferior displacement of the lateral fragment of the clavicle

An apical bronchogenic carcinoma that invades the brachial plexus (Pancoast tumor) can cause Horner syndrome through compression of?

Superior cervical ganglion Horner syndrome is characterized by interrupted sympathetic nerve supply to the head, neck, and eye

25 year old female with arthralgia in hands for last several months. Pain frequently involves her wrists and proximal finger joints bilaterally. Erythrocytes, platelets, and leukocytes are low. UA reveals proteinuria and RBC casts. Most likely diagnosis?

Systemic lupus erythematous Pancytopenia is common in patients with SLE. It frequently occurs due to the formation of antibodies against blood cells (type II hypersensitivity). Anemia characterized by spherocytosis, positive direct Coombs test, and extravascular hemolysis. Antibodies directed against platelets form causing their destruction. Lupus nephritis is caused by immune complex deposition within the glomerulus (type II hypersensitivity). Histopathology is variable depending on the location of the immune complexes, but diffuse proliferative glomerulonephritis is the most common pattern.

Expected PFT changes as a result of normal aging?

TLC- unchanged FVC- decreased RV- increased Diffusing capacity- decreased FEV1- decreased Aging is associated with steady decreases in chest wall compliance (from rib calcification & increased thoracic curvature from osteoporosis and osteoarthritis) but increases in lung compliance due to loss of elastic recoil. his results in marked increase in residual volume, a decreased forced vital vapidity, and relatively unchanged total lung capacity.

24 year old with fever, malaise, anorexia, and bilateral parotid swelling and a swollen tender scrotum. Was treated by a holistic doctor as a child because his parents wished to avoid exposing him to thiomersal. Two months after initial presentation he complains that he doesn't feel like having sex. What treatment is most likely to help this patient?

Testosterone Mumps induced orchitis can result in Leydig atrophy and decreased testosterone production & testosterone therapy can be indicated if serum testosterone concentration is low. Note: Danazol- androgen used to suppress FSH and LH from pituitary (used in endometriosis, hemophilia, ITP)

6 year old boy with bleeding after a dental extraction this morning. History is significant for painful swelling of his knee joints after minor trauma. He is diagnosed with nhemarthrosis. Hemostasis can be achieved currently by what agent?

Thrombin Bleeding after a tooth extraction and history of hemearthrosis are suggestive of hemophilia. Decreased levels of factor VIII and IX lead to failure to convert prothrombin to thrombin and deficient thrombus formation. The addition of thrombin to the blood of a patient with hemophilia results in clotting. Note: thrombin mediates cleavage of fibrinogen to fibrin, the main component of thrombi so without thrombin fibrinogen administration would not be helpful

Patients started on warfarin develop a transient hyper coagulable state due to the short half life of protein c. The hyper coagulability is further exaggerated by preexisting protein C deficiency and can result in?

Thrombotic occlusion of the microvasculature with skin necrosis

Myasthenia graves is caused by circulating antibodies directed against acetylcholine receptors of the NMJ. Autoantibody binding leads to receptor degradation and complement mediated damage of the postsynaptic membrane, producing fluctuating weakness that worsens over the course of the day and most commonly affects the extra ocular muscles (diplopia, ptosis). Most patients also have?

Thymic abnormalities (thymoma- white arrow)

53 year old man with double vision. Has had difficulty walking down stairs because he sees duplicates of every step. Doesn't have problems walking up the stairs. Having rouble reading. A lesion affecting what structure is most likely responsible for this patients visual symptoms?

Trochlear nerve The trochlear nerve innervates the superior oblique muscle, which causes the eye to internally rotate and depress while adducted. Trochlear nerve palsy is typically traumatic or idiopathic and presents with vertical diplopia that worsens when the affected eye looks down and toward the nose (walking downstairs, up close reading). Patients may compensate by tucking the chin and tilting the head away from the affected eye.

The carotid sinus is a dilation of the internal carotid artery located just above the bifurcation of the common carotid artery. The carotid sinus reflex has an afferent limb that arises from the baroreceptors in the carotid sinus and travels to the vagal nucleus and medullary centers via the glossopharyngeal nerve. The efferent limb carries parasympathetic implies via what nerve?

Vagus nerve Note: The vagus nerve is the afferent limb for the nerve fibers originating from the aortic arch baroreceptors and the efferent limb for the carotid sinus reflex

Family members affected by Marfan syndrome may present with a wide range of clinical symptoms despite harboring the same FBN1 mutation due to what phenomenon?

Variable expressivity (condition in which a distinct genotype has varying phenotypic manifestations) Note: penetrance is defined as the proportion of people with a disease genotype who express the disease phenotype; when some individuals express the genotype but not the corresponding phenotype, the disease is said to show incomplete penetrance

68 year old man on Warfarin develops a rapid decrease in INR after starting rifampin. Rifampin induces the synthesis of CYP2C9 leading to increased Warfarin metabolism. What would be the expected change in enzyme kinetics?

Velocity is primarily dependent on the concentrations of enzyme, substrate, and product. Higher concentrations of enzyme and and substrate promote higher reaction veocities. Enzyme induction increases the expression of an enzyme but doesn't change its affinity for the substrate (Km). Rifampin increases CYP2C9 concentration, increasing the hepatic warfarin metabolism and a rise in Vmax. An increase in the amount of enzyme available in a particular system will cause an increase in Vmax but no change in Km. This would cause a decreased in the y-intercept (1/Vmax) and no change in the x-intercept (-1/Km) in a Lineweaver Burke plot.

Relationship between GFR and creatinine?

When the GFR is normal, relatively large decreases in GFR result in only small increases in serum creatinine. Conversely, when the GFR is significantly decreased, small decrements in GFR produce a large chain in serum creatinine. Every time GFR halves, the serum creatinine doubles.

Triad of Wolff-Parkinson White ECG findings?

-Shortened PR interval -Widening of WRS interval -Slurred and broad initial upstroke of the QRS complex (delta wave) WPW is caused by an accessory pathway that bypasses the AV node, causing pre-excitation of the ventricles. Patients with WPW pattern can develop symptomatic arrhythmia (atrioventricular nodal tachycardia) due to reentry of the electrical impulses through the accessory conduction pathway (WPW SYNDROME- manifests with intermittent palpitations, sensation of racing heart, and lightheadedness or syncope)

2 mechanisms by which beta blockers lower blood pressure?

1) Reducing myocardial contractility & HR 2) Decreasing renin release by the kidney Beta blockers inhibit release of renin from renal juxtaglomerular cells through antagonism of beta 1 receptors on these cells. Inhibition of renin release prevents activation of the RAAS pathway, which results in decreased vasoconstriction and decreased Na+ & H20 retention

At normal plasma concentrations of glucose, the renal tubules reabsorb the entire filtered load of glucose as this is below the maximum tubular reabsorption ability. At higher plasma concentrations, glucose is excreted when the filtered amount exceeds the transport maximum. Serum concentration at which glycosuria aka the threshold of glucose is?

200 mg/dL

Atrophic gastritis can result in profound hypochlorrhydria, inadequate IF production, B12 deficiency, and elevated methylmalonic levels. Reticulocyte count increases dramatically once B12 replacement therapy is initiated in an individual with pernicious anemia. The reticulocyte count rises within 3 to 4 days and peaks around one week before stating to decline. In contrast hemoglobin and erythrocyte count levels rise more gradually and make take up to how many weeks to normalize?

8

An acute transmural MI marked by ST elevation and subsequent Q wave formation is most likely the result of?

A fully obstructive thrombus superimposed on a ruptured atherosclerotic coronary artery plaque Note: a lesser degree of occlusion by a thrombus superimposed on an acute plaque change would cause unstable angina & a stable atheromatous lesion without an overlying thrombus (but obstructing >75% of the lumen) would cause stable angina

18 year old undergoes umbilical hernia repair and receives general anesthesia without any operative complications. Four days later he develops fever, lethargy, nausea and vomiting. Scleral icterus is present. Abdomen is distended with mild right upper quadrant tenderness. Labs show markedly elevated AST, ALT, and bilirubin. Liver biopsy is consistent with centrilobular hepatic necrosis. What drug was used during surgery that caused his condition?

A halogenated inhaled anesthetic (halothane, enflurane, isoflurane, sevoflurane, desflurane) These drugs are metabolized by the hepatic cytochrome P450 system by which they are converted to reactive intermediates that can cause hepatocellular damage (halothane hepatitis). Patients often present with acute hepatitis days after medication exposure, and liver biopsy may show centrilobular hepatic necrosis (indistinguishable from viral hepatitis). Treatment is supportive.

Left ventricular free wall rupture usually occurs within 5-14 days after an initial myocardial infarction and presents with sudden onset of chest pain, profound shock, and rapid progression to death. Morphologically it appears as?

A slit like tear in the infarcted myocardium with a preference for the LV due to high systolic pressures

Mutation of what gene is the first step in the classic adenoma to carcinoma sequence?

APC tumor supressor gene (its found in most causes of sporadic colon cancer and in all patients with familial adenomatous polyposis) Note: microsatellite instability pathway is characterized by mutations in DNA mismatch repair genes and is implicated in the development of hereditary nonpolypopsis colorectal cancer (Lymch syndrome)

Clinical manifestations of opioid withdrawal?

Abdominal pain, nausea, vomiting, diarrhea, piloerection, hyperactive bowel sounds, lacrimation, dilated pupils, yawning, diaphoresis. NOT life threatening.

65 year old with iron deficiency anemia. Has had no cough, abdominal pain, melena, or change in bowel habits, but does report 11 pound weight loss in the past 2 months. Rectal exam reveals guac positive brown stool. 3cm mass is found on colonoscopy. Biopsy shows pleomorphic cells with large, dark nuclei forming irregular, crowded glands, some of which contain mucous. Imaging reveal multiple mass lesions involving the liver and lungs. Patient's neoplasm most likely originated from what location?

Ascending colon Colon adenocarcinoma is the most common GU malignancy. Right sided lesions (ascending colon) are most likely to bleed (usually occult) and cause iron deficiency anemia. They tend to grow as large, bulky masses that protrude into the colonic lumen due to relatively large caliber of the ascending colon Left sided lesions tend to be smaller than right sided lesions & present with obstructing symptoms (altered bowel habits, constipation, abdominal distension, nausea, vomiting). They often infiltrate the wall of the colon, encircling it and narrowing the lumen

64 year old man with flank discomfort and red urine. Has a history of HTN & T2DM. Three months ago, he had an ischemic stroke and has mild residual right sided weakness. LDH is elevated. Urine microscopy shows many red blood cells. CT abdomen shows a wedge shaped right kidney lesion. Most likely cause of patients symptoms?

Atrial fibrillation The simultaneous development of stroke, intestinal or foot ischemia, and renal infarction should raise suspicion for embolic phenomena. These emboli may arise from left atrial or ventricular clots or valvular vegetations, etc.

Ramapril is most likely to cause an adverse effect in patients with what preexisting condition?

Bilateral renal artery stenosis Patients with evidence of diffuse atherosclerosis may have bilateral renal artery stenosis. GFR is dependent on angiotensin iI mediated efferent arteriolar vosoconstriction in RAS. ACE inhibitors or ARBs cause efferent arteriolar dilation that causes GFR to fall and can lead to the development of acute renal failure in these patients. Renal function must be monitored closelyy after starting ACE inhibitors/ARBs in patients with evidence of diffuse atherosclerosis

Proteosome inhibitor used in the treatment of multiple myeloma and waldenstrom macroglobulinemia?

Bortezomib

Alcohol induced cardiomyopathy is a dilated cardiomyopathy that can occur with chronic, excessive ethanol intake. Its characterized by?

Contractile dysfunction, cardiomyocyte hypertrophy, myocardial fibrosis, and biventricular enlargement leading to HF

33 year old smoker with worsening exertion dyspnea. Labs reveal markedly decreased levels of serum alpha 1 antitrypsin levels. PFT pattern most likely to be present in this patient?

Decreased FEV1/FVC ration, increased total lung capacity, decreased diffusing capacity for carbon monoxide (due to destruction of alveoli and adjoining capillary beds) Alpha 1 antitrypsin deficiency results in panacinar emphysema. Alpha 1 antitrypsin inhibits proteolytic enzymes released by inflammatory cells (elastase) reducing tissue damage from the inflammatory cascade. Cigarette smoking and the associated pulmonary inflammation that it induces greatly increase the risk of early onset emphysema in patients with alpha 1 antitrypsin deficiency due to the body's inability to inhibit tissue proteolysis. Note: the diffusing capacity of obstructive diseases varies based on the etiology; in asthma the diffusing capacity is increased due to increased pulmonary capillary blood volumes

34 year old female with vulvar itching and vaginal discharge. Pelvic exam reveals erythematous vulva and thick white discharge that adheres to vaginal wall. Microscopic exam reveals budding cells. Most likely underlying cause of her current condition?

Decreased number of gram positive bacteria in the vagina (lactobacilli) Antibiotics suppress the normal vaginal flora and facilitate Candida overgrowth. Antibiotic use is the most common cause of Candida vaginitis. Other potential causes include pregnancy, systemic glucocorticoid use, DM, and immunosuppression

An organism is isolated form a 26 year old with an underlying infection. The organism has a haploid genome with an inner and outer membrane but does not have a nuclear membrane. The organism is found to be sensitive to some but not all penicillin antibiotics. Base on this data what organism was most likely isolated from this patient?

E. coli Prokaryotes are microorganisms with haploid genomes consisting of a single chromosome or DNA strand. NO nuclei are present. A prokaryotic cell with a cell wall sandwiched between the outer and inner plasma membranes is most likely a gram negative bacteria. E. coli are sensitive to some penicillins.

7 year old with diffuse rash. She developed a lace like erythematous rash this morning on her stomach, back and extremities. 2 days ago, she developed red, flushed cheeks. She also had congestion, headache, and a low grade fever for 3 days earlier this week. Last week, her younger brother had similar symptoms that resolves spontaneously. The infectious agent most likely responsible for her disease replicates in progenitors of what cell line?

Erythrocytes Erythema infectiosum (5th disease) is caused by parvovirus B19 ( SINGLE STRANDED DNA virus) and presents with a nonspecific syndrome (malaise, fever, congestion) followed by a classic 'slapped-check' facial rash and a lacy reticular body rash. Parovirus is highly tropic for erythroid precursor cells and replicates predominantly in the bone marrow. These cells express blood group P antigen (globoside) which is the cellular receptor for parvovirus B19.

18 month old boy who was exclusively breastfed until 5 months and has since had a varied diet of fruits vegetables, and meats with continued breast feeding. PE reveals frontal bossing and bilateral tibial and femoral bowing. What abnormality is most likely to be seen in this patient?

Excess unmineralized matrix Risk factors for rickets: exclusive breast feeding, inadequate sun exposure. increased skin pigmentation (dark skinned individuals), maternal vitamin D deficiency Clinical manifestations: craniotabes ('ping pong ball' skull), delayed fontanel closure, frontal bossing, hypertrophy of the costochondral joints ('rachitic rosary'), widening and thickening of wrists, femoral and tibial bowing Rickets is characterized by an excess of unmineralized osteoid matrix and epiphyseal cartilage due to vitamin D deficiency.

PE of a girl shortly after birth shows a head circumference at the 90th percentile, mid face hypoplasia, and hymenal and femoral shortening. Hands have shortened digits and space between third and 4th digits. Parents have no abnormalities. Most likely cause of this patients presentation?

FGFR3 gene mutation Achondroplasia is an AD disorder caused by a GOF mutation of the FGFR3 gene. Results in restricted chondrocyte proliferation in growth plate cartilage and decreased endochondral ossification. 90% of cases are de novo and the rest are inherited from an affected parent.

32 year old man with refractory peptic ulcer disease. PPI did not result in improvement, he doesn't use NSAIDs, and H. pylori testing is negative. He undergoes partial gastrectomy. Significant enlargement of the gastric rural folds are noted on gross exam of tissue. Microscopy of the gastric mucosa shows parietal cell hyperplasia. What stimuli is responsible for parietal cell proliferation in this patient?

Gastrin Patients with Zollinger-Ellison syndrome develop PUD and parietal cell hyperplasia with gastric fold enlargement due to gastrin hyersecretion

Most important in making a diagnosis of tetanus?

History and physical exam Diagnosis of tetanus is clinical. Its based on history of penetrating would in a patient who hasn't been vaccinated, clinical features consistent with tetanus such as truisms, sardonic smile and muscle spasms and high clinical suspicion. C. tetani wouldn't only be found locally within the wound

53 year old with nocturnal cough undergoes endoscopy with esophageal biopsy. LM of the specimen shows elongation of the lamina propria papillae with several eosinophils and neutrophils scattered within the squamous epithelia. He's started on pantoprazole and after 2 weeks his cough symptoms resolve. Most likely cause of his symptoms?

Gastroesophageal junction incompetence GERD is caused primarily by gastroesophageal junction incompetence and can be associated with extra esophageal symptoms (nocturnal cough, hoarseness) in the absence of heartburn. Acidic gastric contents irritate the esophageal mucosa, leading to characteristic histologic findings that include basal zone hyperplasia, elongation of the lamina propria papillae, and scattered eosinophils

Pauci-immune rapidly proliferative glomerulonephritis frequently occurs as a manifestation of antineutrophil cytoplasmic antibodies associated with vasculitides (granulomatosis with polyangitis, microscopic polyangitis). Its characterized by?

Glomerular cresent formation with the absence of immunoglobulin or complement deposits. In addition to renal failure, pulmonary (cough, hemoptysis) and URI symptoms (mucosal ulceration, chronic sinusitis) are common due to underlying vasculitis

72 year old woman with persistent headaches, fatigue, diffuse muscular pain. She is treated with prednisone leading to marked rapid improvement of her symptoms. What pathologic process is most likely responsible for her condition?

Granulomatous inflammation of the media Giant cell arteritis is characterized by granulomatous inflammation of the media and predominately involves medium to smaller branches of the carotid artery, especially temporal artery. Resulting symptoms respond promptly to glucocorticoid therapy. GCA confers a risk of thoracic aortic aneurysms. Biopsy reveals granulomatous inflammation centered on the media with intimal thickening, elastic lamina fragmentation, and giant cell formation (without distinct granulomas)

43 year old man with following ABG readings: PH 7.25 pCO2 70 HCO3- 25 Most likely sufferers from what condition?

Heroin overdose Hypoventilation causes an increase in arterial pCO2 and a decrease in the serum pH (respiratory acidosis). Acute respiratory acidosis presents with a low pH, high pCO2, and a normal to mildly increased HCO3- because renal compensation requires at least 24 hours of persistent respiratory acidosis. Chronic respiratory acidosis is characterized by a low normal pH, high pCO2, and a high HCO3-(>30) Normals: pH 7.4 pCO2 40 HCO3- 24

58 year old man undergoes echocardiogram which reveals hypokinesia of the anterior wall of the LV and LVEF of 35%. He undergoes CABG and repeat echocardiogram 10 days after the surgery shows that hypokineasia is no longer evident and LVEF has increased to 50%. Best explantation for the changes in cardiac contractility and wall motion seen in this patient?

Hibernating myocardium Hibernating myocardium refers to the presence of LV systolic dysfunction due to reduced coronary blood flow at rest that is partially or completely reversible by coronary revascualrization. It is a new equilibrium that prevents myocardial necrosis. Myocardial metabolism and function are reduced to match a contaminant reduction in coronary blood flow Note: ventricular remodeling can occur with long term myocardial hibernation but would take weeks to months (not days) to recover following reperfusion therapy

72 year old woman was lying on the floor in her home for the past 2 days due to a hip injury & being unable to get up or reach the phone. She didn't eat or drink anything during that time. Labs show a serum glucose of 72 and positive urine ketones. Increased activity of what enzyme is most likely contributing to both the serum and urine lab findings?

Hormone sensitive lipase HSL is found in the adipose tissue where it functions to drive the breakdown of stored triglycerides into FFA and glycerol. During times of starvation, this enzyme provides substrates for hepatic gluconeogenesis and ketone body formation. HSL is activated in response to stress hormones (catecholeamines, glucagon, ACTH) and its inhibited by insulin.

Deficiencies in either CD40L (T cell) or CD40 (C-cell) leads to?

Hyper-IgM syndrome (recurrent sinopulmonary, GI, or opportunistic infections)

Results from defective immunoglobulin class switching due to a defect in CD40 ligand-CD40 interaction. Absence of CD40 ligand (T cell) is the most common cause and is inherited in an X-linked recessive pattern. Absence of CD40 (B cell) is less common and is autosomal recessive. Clinical features include recurrent sinopulmonary, GI, and opportunistic infections (Cryptosporidium, Pneumocystis)

Hyperimmunoglobulin M syndrome

Nondisjunction is the failure of chromosome pairs to separate properly during cell division. This could be due to a failure of homologous chromosomes to separate in meiosis 1 or a failure of sister chromatids to separate during meiosis II or mitosis. Difference between these on restriction fragment length polymorphism?

If a patient has three distinct chromosomes than the problem occurred during meiosis I when the homologous chromosomes separated. If problem occurred during meiosis II then the patient would have only two bands, but one that is darker and thicker (twice the normal amount) indicating failure of sister chromatids to separate Note: majority of cases of down syndrome arise due to nondisjunction during maternal meiosis I

Imidazoquinoline nucleoside analog that acts as an agonist of TLR 7 and * and leads to activation of nFkB. Its used as an immunomodulator in the treatment of genital or perianal warts/condyloma acuminate and actinic keratosis

Imiquimod

Lithium can cause nephrogenic diabetes insipidus by?

Impairing aquaporin-2 water channels and reducing responsiveness to endogenous an exogenous vasopressin

71 year old man with 2 month history of progressive exertion dyspnea. He is is unable to walk half a block without resting and can't climb stairs without stopping ever few steps. Has also had difficulty sleeping due to repeatedly waking up at night feeling like he is suffocating. PE shows bilateral LE edema and distended neck veins. Cardiac auscultation reveals an extra heart sound. Auscultatory findings most likely reflect?

Increased LV end systolic volume The third heart sound (S3) is a low frequency sound occurring immediately after S2 that is commonly associated with increased ventricular end systolic volumes. S3 frequently occurs in the setting of mitral regurgitation and systolic heart failure (dilated cardiomyopathy)

Acute intermittent porphyria is caused by deficiency in phorphobilogen deaminase and is characterized by intermittent episodes of abdominal pain with neurological manifestations following exposure to an offending medication. Urine from patients during attacks will classically darken when exposed to sunlight. No photosensitivity. ALA synthase inducted when combined with the inherited deficiency causes excessive metabolite accumulation. What induces ALA synthase?

Increased progesterone (OCPs, puberty), cytochrome P450 inducers, alcohol use, smoking, diet changes (fasting)

Patient with a friable mass in the rectum. Imaging studies show several lesions suspicious of metastases in the right and left lobes of the liver, but no lesions in the lungs or bones. What route of venous draining best explains the distribution of metastatic disease in this paint?

Inferior mesenteric vein Venous drainage of the rectum above the dentate line occurs through the superior rectal vein which drains into the portal venous system via the inferior mesenteric vein. Structures below the dentate line drain to the inferior and middle rectal veins which drain into the systemic circulation via the internal iliac vein

32 year old woman experiences 3 episodes of DVT in a six year period. She has a history of pulmonary embolism as well. PTT is within normal limits and remains unchanged when activated protein C is added to her plasma. Most likely cause of patient's problem is?

Inherited Inherited causes of hyper coagulability should be considered in patients younger than 50 who present with thrombosis and no obvious explanation for an acquired prothombotic state. Factor V Leiden mutation, which causes factor Va resistance to inactivation by activated protein C, may account for 30% of cases of atypical venous thrombosis Note: patients with antiphospholipid antibody syndrome typically have a prolonged baseline PTT due to the presence of lupus anticoagulants

79 year old admitted for exacerbation of COPD. Wide is concerned due to multiple hospitalizations over the last year for various issues relating to his comorbidities. What topic is most essential to address during the rest of the admission process?

Inquiring about the presence of advanced directives and the patient's wishes for end of life care Advance care planning for end of life issues should ideally be initiated during outpatient visits with the PCP. However, physicians should readdress these decisions during the hospital admission process to ensure medical personnel adhere to the patients specific wishes in the even that the patient becomes incapacitated

During fasting, a high baseline plasma insulin level in the setting of normoglycemia is a sign of insulin resistance in the peripheral tissues. 2 most important pathophysiologic mechanisms for the development of T2DM?

Insulin resistance and progressive beta cell failure

Giant cell arteritis is characterized by granulomatous inflammation of the media, with fragmentation of the internal elastic laminas of the medium and small branches of the carotid artery. Prompt institution of therapy in this patient with glucocorticoids reduces the risk of what complication?

Ischemic optic neuropathy (leads to irreversible blindness)

21 year old woman with postcoital bleeding. Has been sexually active with her male partner for 6 months and they do not use condoms. Never had an STI. Smokes 1PPD. Cervix has an area of erythema surrounding the external os and scant clear discharge. Pap smear is shown in the picture. What best describes this finding?

Koilocyte HPV infection causes cutaneous genital warts as well as benign and malignant intraepithelial neoplasia. Koilocytosis is a hallmark sign of HPV infection. Koilocytes are pyknotic, superficial or immature squamous cells with a dense, ireegaulrly staining cytoplasm and perindclear halo like clearing

Occurs when subjects with a rapidly progressive form of a disease are less likely to be detected by screening compared to those with a slowly progressive disease. This bias tends to overstate the beneficial effects of screening on length of survival and mortality

Length time bias

22 year old with abdominal pain that started around bellybutton and then moved to right lower abdominal quadrant. An inflamed, hyperemic, and edematous appendix is seen after laparoscopy. What most likely initiated this patients condition?

Lumen obstruction Obstruction of the lumen of the appendix is the first event in the pathogenesis of acute appendicitis. Fecaliths, hyperplastic lymphoid follicles, foreign bodies, or tumors may cause the obstruction. RLQ abdominal pain, nausea, vomiting, diarrhea, and fever are the typical manifestations of appendicitis.

54 year old man with fever, shaking chills, and cough productive of copious sputum. Was prescribed oral antibiotics by PCP after CXR revealed RLL infiltrate. Patient hasn't been taking medication as prescribed and his symptoms have worsened. Repeat CXR reveals a round density with an air fluid level in the RLL. Most important contributor to the observed lung lesion in this patient?

Lysosomal content released by neutrophils Abscess formation is largely driven by neutrophil recruitment and activation leading to the release of cytotoxic granules that kill bacteria but also cause liquefying necrosis of surrounding tissue. Most cases are due to aspiration of anaerobic bacteria from the oropharynx, but lung access can develop in the setting of untreated pneumonia

Autopsy of 14 year old boy who died in MVC shows several minimally raised yellow spots on the inner surface of the abdominal aorta. Rest of cardiovascular findings and autopsy are unremarkable. He had no known medical problems and there is no family history of CVD or SCD. Most likely predominant cell type in these lesions on microscopy?

Macrophages Fatty streaks are the earliest lesions of atherosclerosis and can be seen as early as the second decade of life. They appear as a collection of lipid laden macrophages (foam cells) in the intimate that can eventually progress to atherosclerotic plaques with advancing age.

61 year old obese woman with peripheral edema. UA shows significant proteinuria. Her serum contains IgG4 antibodies to the phospholipase A2 receptor (PLA2R), a transmembrane protein abundant on podocytes. Most likely diagnosis?

Membranous nephropathy Idiopathic membranous nephropathy is associated with circulating IgG4 antibodies to phospholipase A2 receptor which might play a role in the development of the disease

4 year old girl developed acute onset colicky abdominal pain, vomiting, and loose bloody stools during a family vacation. Was treated with supportive care and began feeling better. A few days later parents are concerned because she only urinated once in the past 10 hours and the urine was red. PE shows conjunctival pallor. Hemoglobin is low, platelets are low, and creatinine is high. UA shows proteinuria and hematuria. What mechanism is most likely cause of patients condition?

Microthrombi in small blood vessels HUS is a common cause of acute renal failure in children. Its characterized by the triad of microangiopathic hemolytic anemia, thrombocytopenia, and acute kidney injury (oliguria/anuria, hematuria, increased creatinine). Most cases develop following a diarrheal illness caused by Shiga-toxin producing organisms (E. Coli 0157:H7, Shigella dysenteriae)

Initial treatment of pneumothorax is focused on removing air from the pleural space to allow lung re-inflation. Effective options for large pneumothoraces are?

Needle aspiration or chest tube placement Patients with suspected tension pneumothorax (contralateral tracheal deviation, hypotension, distended neck veins) should undergo immediate decompression to prevent further clinical deterioration

CBC differential is expected to show an increase in what cell type in a patient treated with prednisone?

Neutrophils (due to 'demmargination' of neutrophils previously attached to the vessel wall) Lymphocyte, monocute, basophil, and eosinophil counts are decreased

Cells that are the primary mediators of disease in COPD?

Neutrophils, macrophages & CD8+ T lymphocytes They release enzymes and proteases such as neutrophil elastase that cause alveolar damage/destruction (emphysema), reduced ciliary motion, and increased mucous secretion by goblet cells (chronic bronchitis)

25 year old with HIV is started on a drug that is structurally unrelated to nucleosides and is not phosphorylated intracellularly. The drug acts within the cytoplasm of the infected cells to inhibit the synthesis of viral DNA from the RNA template. Most likely used agent?

Nevirapine, Efavirenz, or Delaviridine Nonnucleoside reverse transcriptase inhibitors are antiretroviral drugs that don't require activation via intracellular phosphorylation. Adverse effects include hepatic failure, Stevens Johnson syndrome, and toxic epidermal necrolysis

5 year old girl who's grandmother died 2 months ago and lived in the family home. She says she sees her grandmother at night and likes to speak to her before she goes to bed. For a few weeks following the death, the girl would become tearful and frequently ask when her grandmother would be coming back. Most likely diagnosis?

Normal behavior Hallucinations of recently deceased relatives in children are part of a normal grief reaction and may not be indicative of major psychiatric illness.

Patient is visibly annoyed when told physician is running behind schedule. He is given several forms to fill out and politely agrees. When paperwork is reviewed it only contains patients signature even though he had time to fill them out. Patient sarcastically says 'Im sure your education you can take take a good history'. When physician asks if anything is wrong patient states "Oh nothing, I am fine.'. What best describes this patients behavior?

Passive aggression (expression of angry feelings in a non-confrontational manner) Backhand compliments, sarcasm and denial of being angry are examples of passive aggressive behavior.

10 year old immigrant from Eastern europe presents with exertion dyspnea and fatiguability. Parents state he was diagnosed with a congenital heart disease in infancy for which they refused treatment but they don't recall the details/diagnosis. PE shows toe cyanosis and clubbing but no finger abnormalities. All extremity pulses are full and equal. Most likely diagnosis?

Patent ductus arteriosus Lower extremity clubbing and cyanosis without blood pressure or pulse discrepancy are pathognomonic for large patent ductus arteriosus complicated by Eisenmenger syndrome (reversal of shunt flow from left to right to right to left). Cyanosis most pronounced in the LE because the PDA delivers unoxygenated blood distal to the left subclavian artery. Severe coarctation of the aorta can cause LE cyanosis if the PDA remains patent, but it occurs in infancy. Blood pressure discrepancy and pulse delay between uppeer and lower extremities is present. Right to left shunting in patients with large septal defects (VSD & ASD) and tetralogy of fallot results in whole body cyanosis.

65 year old male admitted after acute ST elevation MI. He experiences chest pain that is sharp and increases with coughing and swallowing and radiates to his neck on day 4 of hospitalization. Temperature is 101. Most likely cause of patients chest pain?

Pericardial inflammation overlying the necrotic segment of the myocardium In contrast to angina, the chest pain of pericarditis is sharp and pleuritic and may be exacerbated by swallowing or relieved by leaning forward. Early onset pericarditis develops in about 10-20% of patients between days 2 and 4 following a transmural MI. It represents an inflammatory reaction to cardiac muscle necrosis that occurs in the adjacent visceral and parietal pericardium. Usually resolves in 1 to 3 days with Aspirin. Late onset MI pericarditis (Dressler's syndrome) begins one week to a few months following the MI and affects less than 4% of patients & is thought to be an autoimmune polyserosisitis

12 year old boy with headaches and gait instability. Symptoms began several months ago and have progressively worsened. MRI reveals a mass in the cerebellum. Most likely diagnosis?

Pilocytic astrocytoma Imaging reveals a well demarcated lesion composed of CYSTIC and solid components (medullablastoma is purely solid) Microscopic exam shows pilocytic astrocytes with bundles of GFAP positive hairlike processes and classic rosenthal fibers (eosinophilic intracytoplasmic inclusions). Due to slow growing (benign) nature, majority of patients can be treated with surgical resection and have a favorable prognosis

10 year old boy is found to have a single missense mutation in the gene coding for cystathione beta synthase enzyme. What is most likely explanation for this patient's genetic defect affecting multiple tissues?

Pleiotropy Pleiotropy describes instances where multiple seemingly unrelated phenotypic manifestations result from a single genetic mutation. Most syndromic genetic illnesses, including homocystinuria, exhibit pleiotropy Note: Locus heterogeneity- ability of one disease or trait to be caused by mutations in multiple different genes Polyploidy- more than 2 complete sets of homologous chromocomes (ex: 69, XXX) Genetic linkage- alleles located near one another on the same chromosome inherited jointly Segregation- each offspring inherits half of each parents genetic composition

40 year old man with chronic constipation presents with pain on defecation. Pain is sharp and severe but subsides within minutes of having a bowel movement. Often sees spots of bright red blood on the toilet paper. Rectal exam is most likely to show a tear at what location?

Posterior midline distal to the dentate line Anal fissures are longitudinal tears in the mucosa distal to the dentate line. They are usually due to passage of hard stool in patients with chronic constipation. Most fissures occur at the posterior midline due to the decreased flood flow in this area. Fissures in other areas may be due to less common causes (IBS, malignancy, infection).

78 year old with acute MI. 3 days later she develops sudden onset SOB due to pulmonary edema. Surgery is performed and reveals posteriomedial papillary muscle rupture. This funding suggests compromised blood flow through what coronary artery?

Posteror descending Papillary muscle rupture is a life threatening complication that typically occurs 3-5 days after MI and presents with acute mitral regurgitation and pulmonary edema. The posteromedial papillary muscle is supplied solely by the posterior descending artery, making it susceptible to rupture. Note: the anterolateral papillary muscle is supplied by both the LCX and LAD arteries

58 year old man noticed blood in his urine over last few days. CT shows right sided renal mass and biopsy reveals rounded and polygonal cells with abundant clear cytoplasm. Patients lesion most likely originated from what portion of the kidney?

Proximal renal tubules Most renal cell carcinomas (also known as clear cell carcinomas) originate from the epithelial cells of the proximal renal tubules. Clear cell carcinoma is the most common type of kidney tumor. Its easily recognizable on macroscopic and microscopic exam due to its high lipid content. On gross exam, the tumor appears as a golden yellow mass. On light microscopic exam the tumor cells are composed of cells with abundant clear cytoplasm and eccentric nuclei. Mean age is 60-70. Risk factors are smoking and obesity. Note: renal oncocytomas are rare and originate from collecting duct cells; oncocytes are large well differentiated neoplastic cells that contain numerous mitochondria. Note: transitional cell carcinoma is responsible for 90% of tumors involving the renal pelvis; it arises from renal pelvis lining, tends to be multifocal in nature and often forms papillary tumors composed of urothelium supported by a thin fibrovascular stalk

53 year old woman diagnosed with metastatic brain tumor. Several of her family members also have a history of cancer. Its determined she is predisposed to malignancy as a result of genetic alteration in a protein that functions to transduce signals from the cell membrane to the nucleus by activating other molecules. Most likely protein defective?

Ras Ras is an oncoprotein component of the MAP kinase signal transduction system. The activated Ras protein transmits a stimulus from the receptor on the cell surface to the nucleus, prompting mutagenesis. Up to 20% of all human tumors contain mutated Das that have reduced GTPase activity and remain in an excited GTP bound state.

What female pelvic ligament is most likely to be found in the inguinal canal?

Round ligament. The round ligament of the uterus is a vestige of the inferior portion of the gubernaculum (superior portion persists as the proper ligament). It projects form the uterus through the inguinal canal into the labia majora.

21 year old with impaired balance, tremor and difficulty speaking over the last several months. He is found to have elevated serum transaminases. Viral hepatitis serologies are negative. Sibling was diagnosed at a young age with progressive neurologic disease. What diagnostic study would be most helpful for confirming diagnosis?

Slit lamp examination Wilson disease is an autosomal recessive condition caused by mutation of ATP&B gene characterized by excessive copper deposition into tissues (liver, basal ganglia, cornea). It typically presents in young adults with liver disease, neuropsychiatric symptoms (gait disturbance, dysarthria, depression, personality changes) and Keyser-Fleischer (green-brown copper deposits in Descemet membrane of cornea) rings on slit lamp exam. Diagnosis confirmed by presence of low serum ceruloplasmin, increased copper urinary excretion, and elevated hepatic copper on liver biopsy. Treatment is with chelators (D-penicillamine, trientine) & zinc (interferes with copper absorption)

Aldosterone antagonist commonly used to treat HF that has significant antiandrogenic effects and can cause gynecomastia, decreased libido, and impotence

Spironolactone Note: eplerenone is a more selective aldosterone antagonist with fewer side effects

68 year old with chewing difficulty and persistent headaches of recent onset. On PE there is tenderness over the patients temples. You proceed with artery biopsy. The morphologic changes observed in this patients arteries are most similar to those in which condition?

Takayasu arteritis Takayasu arteritis and temporal arteritis involve arterial vessels of different sizes and locations (aorta and proximal aortic arterial branch involvement vs. distal carotid artery branch involvement, respectively), and have different clinical presentations. Even so, they share a common pathologic morphology, consisting of granulomatous inflammation of the media. Takaysu arteritis occurs in females less than 40. Patients can have lower BP and pulses in the upper extremities compared to the LE and cold or numb fingers

36 year old woman with symptoms of carpal tunnel has attempted treatment with conservative measure, but has had no significant improvement and is referred for surgery. Incision of what structure is most likely to improve this patients symptoms?

Transverse carpal ligament (flexor retinaculum) Carpal tunnel is defined by the carpal bones on the dorsal surface and the transverse carpal ligament on the volar surface. It contains the tendons of the flexor digitorum profundus and superficialis and flexor pollicis longus, in addition to the median nerve. Compression of the nerve is what produces carpal tunnel syndrome.

21 year old female with facial flushing, headache, nausea, vomiting, and abdominal cramps after drinking alcohol at a party. She was started on a new oral medication one week ago by her PCP, but can't remember what it was or why it was prescribed. She is sexually active with her boyfriend. She is most likely being treated for what condition?

Trichomonas vaginitis Metronidazole is used to treat trichomonal vaginitis and bacterial vaginosis. It can cause disulfiram like effects (abdominal cramps, nausea, headache) when combined with alcohol due to acetaldehyde accumulation

33 year old man with three days of SOB, profound fatigue, and chills. Has a history of IV drug use. Admits to relapsing last week even though he was involved in a Methodone program. Temperature is 104. BP is 75/40. He dies and lung biopsy shows hemorrhagic infarcts. Most likely diagnosis?

Tricuspid valve endocarditis Pulmonary infants are typically hemorrhagic (red) and wedge shaped in the periphery of the lung. IV drug users are at increased risk of tricuspid valve endocarditis, which can cause multiple septic pulmonary infarcts due to embolization of tricuspid valve vegetations.

Normal pressure hydrocephalus is the result of decreased CSF resorption by the arachnoid granulations. It may be idiopathic or secondary to brain trauma or subarachnoid hemorrhage which result in scarring of the arachnoid granulations. Classic triad consists of progressive gait difficulties, cognitive disturbances, and urinary incontinence. Brain imaging reveals?

Ventricular enlargement out of proportion to sulci enlargement

85 year old man treated with IV NE for hypotension. What cellular change would be a direct response to this medication?

cAMP increase in cardiac smooth muscle cells NE stimulates cardiac B1 adrenoreceptors, which increases cAMP concentration within cardiac myocytes and leads to increased contractility, conduction, and HR. Peripheral vasoconstriction occurs via stimulation of alpha 1 adrenoreceptors in vascular smooth muscle cells and activation of an IP3 signaling pathway. NE predominately affects a1, a2, and B1 with little influence on B2 receptors

45 year old male with fatigue that has been gradually worsening over the past 4 months. Has had difficulty keeping up with the labor involved with his job as a construction worker. ROS reveals constipation and decreased concentration. PE reveals an oversight male with conjunctival pallor who has difficulty getting onto the exam table due to SOB. MVC and hematocrit are low. Ferritin, iron, and iron binding capacity levels are all normal. Most like to be seen on patient's peripheral blood smear?

Basophilic stippling Lead poisoning in adults typically occurs through occupational exposure. Symptoms include colicky abdominal pain, constipation, lead lines on the gum (blue pigmentation), peripheral neuropathy (wrist/foot drop), and anemia, Labs show microcytic anemia with normal iron studies & peripheral smear may reveal basophilic stippling (due to ribosome aggregation because of inability to degrade RNA). Lead inhibits ALA-dehydratase and ferrochelatase in the heme synthesis pathway. SOB on exertion occurs due to decreased oxygen delivery to the peripheral tissues. Note: ring sideroblasts are formed from percipitation of iron granules in developing erythrocytes due to defects in the heme synthesis pathway, including enzymatic inhibition by lead position, but they are only seen in the BONE MARROW NOT the peripheral smear

7 year old boy with 2 day history of colicky abdominal pain now developed bloody stools. Mother states urine appeared red today. PE reveals palpable skin lesions on his buttocks. What additional findings are most consistent with this patients condition?

Joint pain Henoch-Schonlein purpura generally affects young children and is classically preceded by a URI. The IgA mediated hypersensitivity (leukocytoclastic) vasculitis commonly causes abdominal pain, joint pain, lower extremity palpable purpura, and hematuria. Most common small vessel vasculitis in children and classically affects boys 2-10 years old.

3 day old boy with poor feeding, emesis, lethargy for past 24 hours. Tachycardia and tachypneic. Appears dehydrated and abdomen is distended. Vomited during exam which has green tint. On laparotomy, fibrinous bands are seen extending from the cecum and right colon to the retroperitoneum, causing extrinsic compression of the duodenum. Most likely failed embryologic process?

Midgut rotation around the superior mesenteric artery Intestinal malrotation results when the midgut undergoes incomplete embryological counterclockwise rotation. It can present as intestinal obstruction (due to compression by the adhesive bands) and midgut volvulus (intestinal ischemia due to twisting around blood vessels)

72 year old woman diagnosed with uterine cancer undergoes hysterectomy. Following surgery, she compliments her surgeon saying she is a 'brilliant doctor who saved my life'. Several days later patient is discharged by the resident who explains the surgery was delayed due to an energy. Patient is angry and says surgeon is 'terrible and only in it for the money'. Most likely explanation for patient's behavior?

Splitting Splitting is a defense mechanism that involves experiencing the self or others as all bad or all good. Its commonly seen in patients with borderline personality disorder

23 year old woman with ectopic pregnancy. A single dose of methotrexate is administered to treat her condition. What substance is likely to accumulate in embryonic tissues as a result of treatment?

Dihydrofolate polyglutamate & folic acid Methotrexate is a folic acid antagonist used to treat ectopic pregnancy. It competitively inhibits the enzyme dihydrofolate reductase (DHF) which catalyzes the synthesis of tetrahydrofolate. DHF reductase inhibition causes the intermediate DHF to accumulate intracellularly.

A result is considered statistically significant if the 95% confidence intervals does not cross the null value (1) which corresponds to a p value of?

Less than 0.05

Effect of Verapamil administration on the cardiac pacemaker cells?

Slowed spontaneous depolarization Class IV antiarrhythmics (verapamil, dilitiazem) are commonly used to prevent recurrent nodal arrhythmias (paroxysmal supra ventricular tachycardia). They work by blocking Ca2+ channels in slow response cardiac tissues, slowing phase 4 (spontaneous depolarization) and phase 0 (upstroke). This reduces impulse conduction velocity in the SA and AV nodes.

68 year old man with abdominal pain and nausea for 2 days. ECG shows absent P waves and an irregular rate and rhythm. CT of the abdomen reveals colonic wall thickening and no enhancement of IC contrast. UA shows acidic urine. Lactic acid in venous blood is 5.6. pH is 7.25. Bicarb is 12. PaCO2 is 29. Renal metabolism of what amino acid is most important for maximizing acid excretion in this patient?

Glutamine Patient has acute ischemic colitis which is most likely due to embolic disease related to his atrial fibrillation. Ischemic bowel undergoes anaerobic metabolism causing lactate accumulation in the blood that leads to anion gap metabolic acidosis. Acidosis stimulates renal ammoniagenesis, a process by which renal tubular epithelial cells metabolize glutamine to glutamate, generating ammonium that is excreted in the urine and bicarbonate that is absorbed into the blood. This process is responsible for the vast majority of renal acid excretion in chronic acidotic states.

71 year old woman went shopping yesterday and couldn't find her way back home. Started forgetting things like where she put her keys two years ago and began to have problems 'remembering the right words'. How she is having trouble with activities of daily living. PMH is unremarkable. Brain MRI is most likely to show atrophy of what structure?

Hippocampus (area of the brain demonstrating the greatest degree of atrophy in Alzheimer's disease) Atrophy is most pronounced in the temporoparietal lobes and hippocampus. Advanced disease is associated with diffuse brain atrophy. Hippocampal atrophy on brain MRI is highly suggestive of Alzheimer's disease.

37 year old at 28 weeks gestation with loss of urine with cough. Most likely mechanism for patient's urinary incontinence?

Increased intraabdominal pressure Common symptom during pregnancy is stress urinary incontinence, which occurs due to increased intraabdominal pressure form the gravid uterus that exceeds the pressure point that maintains closure of the urethral valve. Pregnancy hormones decrease urethral tone and relax the pelvic floor muscles (elevator ani, coccygeus) which maintain closure of the urethral valve. Kegal exercises strengthen the pelvic floor muscles and can improve symptoms of stress urinary incontinence.

6 year old Caucasian boy with recurrent episodes of sinusitis. Physician suspects patient has an inherited disorder even though patient had a negative sweat chloride test and orders nasal transepithelial potential difference measurements. In this test, isotonic solution of NaCl is applied to the nasal mucosal surface and the electrical potential overlying the mucosa is compared with that of the interstitial fluid. Results show a transepithelial potential difference that is more negative than normal. Patient's nasal mucosa is most likely to demonstrate what physiologic change?

Increased sodium absorption In cystic fibrosis, impairment of the CFTR protein reduces chloride secretion and increases sodium absorption by the respiratory epithelia, resulting in dehydrated mucous. When saline is applied to the nasal mucosa, the increased Na+ absorption in patients with CF causes a more negative transepithelial potential difference (due to Cl- being retained in the lumen and Na+ being absorbed), which can be used to diagnose CF

Zinc finger motifs are composed of chains of amino acids bound together around a zinc atom via linkages with cysteine and histidine residues. They recognize specific DNA sequences and are used by many transcription factors to bind DNA and alter activity of target genes. Examples of receptors that contain zinc finger binding domains?

Intracellular receptors that bind steroids (estrogen, aldosterone, cortisol), thyroid hormone, and fat soluble vitamins that act directly as transcription factors

28 year old African American man is given prophylactic vaccinations before a business trip to Africa. Five days later he presents with jaundice and dark urine. Peripheral smear shows RBCs with dark inclusions when stained with supra vital stain. Patients condition most likely has what inheritance pattern?

X-linked recessive Patient has classic history of hemolytic anemic due to medication induced oxidative stress. Antimalarial drugs prescribed for travelers to affected regions (Africa) are notorious instigators of hemolysis in patients with G6PD. Labs show anemia, reticulocytosis, indirect hyperbilirubinemia, and low haptoglobin. Peripheral smear shows RBCs with Heinz bodies (hemoglobin that has denatured/precipitated from oxidative stress). Glucose-6-phosphate dehydrogenase deficiency has an X-linked recessive inheritance. Males are affected and female are generally carriers. Remember: most structural abnormalities follow an AD pattern while most enzyme deficients follow an AR recessive pattern; for example hereditary spherocytosis is AD

Half life equation?

half life = (0.7 xVd)/CL Half life is a measure of how quickly a drug with first order kinetics is eliminated from the body. A drug is almost completely eliminated after 5 half life intervals. Half life 0 --> 0% eliminated half life 1 --> 50% eliminated half life 2--> 75% eliminated half life 3 --> 87.5% eliminated half life 4 --> 93.75% eliminated half life 5 --> 96.875% eliminated

52 year old man with advanced pancreatic cancer that is unresectable. Physician previously discussed prognosis at length. Patient feels too weak to play with his children but says ' I am a survivor and I know I can beat this, Ill do whatever I can to be around for my kids'. Most appropriate response to patient at this time?

'We can hope for the best, but lets focus on how to maximize the remaining time with your children'. Patients faced with terminal illness may experiences stages of grief, including denial, anger, bargaining, depression, and acceptance. Denial that doesn't impair relationships or interfere with patient care should NOT be confronted (can have a detrimental effect on patient's ability to cope with the stress of terminal illness)

Urine culture grows E. coli. The organism is found to have a methyltransferase that methylates ribosomal RNA. The enzyme most likely confers resistance to what antibiotic?

Aminoglycocides Aminoglycocides work by interfering with the 30S ribosomal subunit and causing the cell to misread messenger RNA, thereby halting protein synthesis. An important mechanism of resistance is the methylation of the aminoglycocide-binding portion of the ribosome, which inhibits the ability of amino glycoside to interfere with protein translation

54 year old male with dizzy spells when walking his dog in the morning. No chest pain, SOB, weakness, or headache, Cardiac auscultation reveals murmur at right sternal border. Most likely cause of patients symptoms?

Bicuspid aortic valve Bicuspid aortic valve (calcified) is a common cause of aortic stenosis in the USA. Class auscultatory finding in patients with aortic stenosis is a harsh, crescendo-decrescendo systolic ejection murmur heard best in the right second intercostal space with radiation to the carotids. Other causes not as common in USA are calcification of a normal valve and rheumatic heart disease.

48 year old man with 6 month history of cough and fatigue. Also experiencing fatigue and weight loss. Patient is a carpenter who smokes 1PPD. Exam shows left lower lobe crackles. HIV negative. CXR reveals pulmonary infiltrate in lower lobe of left lung. Results from KOH reveal large years with single, broad based bud with thick walls. Most likely cause of patients condition?

Blastomyces dermatitidis In half of immunocompetent individuals, blastomycosis may present as a lung infection or cause flu like illness (fever, chills, myalgia, headache, nonproductive cough) or pneumonia (fever, cough, pleuritic chest pain). Pulmonary blastomycosis is diagnosed by finding typical yeast forms. In immunocompromised patients, it can cause disseminated disease. Patients experience systemic systems (fever, weight loss, night sweats), lung involvement (cough, dyspnea), skin lesions (papule, pustules, ulcers, verrucous lesions) and bone bain (caused by lytic lesions)

7 year old boy with a history of recurrent granulomatous skin infections and a prior episode of Aspergillus pneumonia undergoes partial hepatectomy to treat a poorly draining liver abscess. Genetic analysis reveals inactivating mutation affecting a structural component of a neutrophil oxidase enzyme. Patient most likely has increased vulnerability to infections caused by what pathogen?

Burkholderia cepacia, S. aureus, Serratia marcescens, Nocardia, Aspergillus Chronic granulomatous disease results from a genetic defect in NADPH oxidase. Normally, NADPH oxidase participates in the killing of microbes within neutrophil and macrophage phagolysosomes. Patients with CGD develop recurrent bacterial and fungal infections that are predominantly caused by 5 catalase organisms listed above as they are able to destroy their own hydrogen peroxide (phagocytes cannot generate potent microbicidal agents such as hypochlorite from the H2O2)

23 year old male with 3 weeks of malaise and fatigue. Temperature is 101.2. Cardiac auscultation reveals an apical holosystolic murmur radiating to the axilla, which aunt present in the past. Labs show creatinine of 2.3. Mild proteinuria and microscopic hematuria with red cell casts are present on UA. Most likely pathogenesis of patient's renal findings?

Circulating immune complex mediated injury Most likely cause of fever and fatigue with a new onset cardiac murmur is infective endocarditis. Diffuse, proliferative glomerulonephritis secondary to circulating immune complex deposition may complicate infective endocarditis and can result in acute renal insufficiency. Can be seen as capillary wall thickening with subendothelial or subepithelial deposit formation. Hypercellularity similar to that seen with PSGN or MPGN is seen on LM.

54 year old man with abdominal pain, chronic diarrhea, and recent weight loss. Has had episodic abdominal pain in the past but lately has become persistent and worsens after eating. Consumes alcohol regularly. Abdominal x-ray reveals calcifications in the epigastric area. Most likely cause of his diarrhea?

Digestive enzyme deficiency Diarrhea, weight loss, and epigastric calcifications in a patient with chronic alcoholism suggest chronic pancreatitis with resulting pancreatic exocrine insufficiency and malabsorption. Alcohol induces secretion of protein rich fluid which can precipitate in the pancreatic ducts, forming ductal plugs that can calcify. Ductal obstructions by these concretions cause exocrine insufficiency due to atrophy of the pancreatic acing cells and pancreatic fibrosis. Exocrine insufficiency leads to malabsorption with diarrhea/steatorrhea.

43 year old woman with occasional chest discomfort over the past year. Describes the pain as a midline pressure squeezing sensation that last 15 minutes and is sometimes associated with diaphoresis. No history of HTN or DM but is an active smoker. Exercise stress testing shows good exercise capacity with no ischemic signs or symptoms. Ambulatory ECG monitoring shows transient ST segment elevations in leads I, aVL, V1-v4 during an episode of chest pain at night. What medication would most likely provoke this patients chest pain?

Dihydroergtoamine (commonly used to treat migraines; constricts vascular smooth muscle via stimulation of both alpha adrenergic and serotonergic receptors) Prinzmetal (varient) gaina is characterized by episodic, transient attacks of coronary vasospasm (at rest and at night) producing temporary transmural myocardial ischemia with ST segment elevation. Possible triggers are cigarette smoking, cocaine/amphetamines, and dihydroergotamines/tripatans. Treatment includes tobacco/drug cessation and vasodilator therapy (nitrates, CCBs)

54 year old with exertion dyspnea and fatiguability. On PE, gait is unstable when her eyes are closed and there is impaired vibratory sensation in the LE. Marked pallor of the conjunctivae, nail beds, and palms is present. What lab test would help inform diagnosis in this patient?

Elevated methylmalonic acid Vitamin B12 deficiency presents with megaloblastic anemia (impaired DNA synthesis) and neurologic deficits (impaired myelin synthesis). Characteristic neurologic findings are subacute combined degeneration of the dorsal columns and lateral cortical spinal tract. Elevations of methylmalonic acid and homocysteine occur due to decreased metabolism of these molecules.

37 year old with SOB. No evidence of obstructive or interstitial lung disease or venous thromboembolism. 2/6 holosystolic murmur at lower sternal border that increases on inspiration is present. Cardiac cath results reveal an elevated mean pulmonary pressure and a normal PCWP. Parmacotherapy blocking what substance will most likely benefit this patient?

Endothelin Endothelin is a potent vasoconstrictor that also promotes smooth muscle proliferation (typically elevated in patients with PAH). Endothelin receptor antagonists (bosentan, ambrisentan) lower pulmonary arterial pressure and improve dyspnea in patients with pulmonary artery hypertension. Due to right ventricle enlargement, a holosystolic murmur of functional tricuspid regurgitation is often present. Pulmonary arterial hypertension results from proliferative vasculopathy of the pulmonary arteriolar smooth muscle which may occur in the setting of a hereditary genetic mutation (BMPR2 mutation), connective tissue disease, or HIV infection.

45 year old woman with long history of joint pain, swelling, and stiffness. On PE her PIP and MCP joints are swollen and tender bilaterally & her fingers appear slightly deformed. Subcutaneous nodules are palpated near the elbow. Labs are obtained. Serum will most likely contain IgM antibodies against what substance?

Fc portion of human IgG Rheumatoid arthritis results from an immune response directed against autoantigens in the joints. Infiltrating CD4+ T cells secrete cytokines that promote inflammatory synovitis. They also stimulate B cells to produce rheumatoid factor (IgM antibody specific for Fc component of IgG) and anti-citrullinated protein antibodies that contribute to chronic inflammation and joint destruction

32 year old woman with SOB. Her condition deteioriates during exam and is emergently intubated. Shortly after intubation she goes into cardiac arrest and dies. She was not complacent with her medication. Autopsy is performed and shows hyperinflated lungs, airway mucous plugging, and cellular infiltration of the bronchial wall. Long term use of what drug would have best prevented the cellular reaction in this patient's airways?

Fluticasone Her presentation is suggestive of persistent bronchial asthma. Corticosteroids have the strongest and most predictable effects on the inflammatory components of asthma & are used for chronic management (inhaled) and during acute exacerbations (systemic steroids). Inhaled corticosteroids form the cornerstone of chronic therapy for patients with persistent asthma and can reduce the number and severity of acute asthma exacerbations.

65 year old with abdominal pain and diarrhea. Three weeks ago he drove from Texas to Mexico for family vacation. Temperature is 101. Abdominal exam shows mild tenderness. Sigmoidoscopy demonstrates white-yellow plaques on the colonic mucosa and biopsy shows that the plaques are composed of fibrin and inflammatory cells. Further questioning regarding this patients trip to Mexico is most like to reveal what event?

He required hospitalization and antibiotics for pneumonia Antibiotics disrupt normal intestinal flora which can allow for overgrowth of C. difficile, an aerobic, gram positive, spore forming bacillus. C. Difficile produces 2 toxins that penetrate the colonic epithelial cells leading to watery diarrhea, abdominal cramping and colitis. The presence of pseudomembrane (exudate on colonic mucosa consisting of fibrin and inflammatory cells) is highly suggestive of C. difficile infection

55 year old man with chronic muscle weakness. Has had increasing difficulty walking up stairs and combing hair & lift objects overhead. On exam, he has symmetrical proximal weakness and mild muscle tenderness. No skin rash. Muscle biopsy reveals endomysial mononuclear infiltrate and patchy muscle fiber necrosis. Autoantibody directed against what antigen is most likely to be seen in this patient?

Histidyl-tRNA synthetase Polymyositis is characterized by symmetric proximal muscle weakness. Its associated with antinuclear and anti-Jo-1 (anti-histidyl-tRNA synthetase) autoantibodies. Biopsy shows an endomysial mononuclear inflammatory infiltrate and patchy muscle fiber necrosis. Similar to dermatomyositis but lacks characteristic skin findings. Can occur independently or as a paraneoplastic syndrome (most commonly adenocarcinoma)

Reason serum insulin level rises more with oral glucose vs. IV glucose?

Incretins (GLP-1 and GIP) Incretins are GI hormones produced by the gut mucosa that stimulate pancreatic insulin secretion in response to sugar containing meals. Response is independent of blood glucose levels and occurs before any elevation in blood glucose following a meal.

46 year old man with chest pain is taken to undergo emergent cardiac catheterization. Physician is approached by a distraught woman who says she is his wife and asks about his condition as she is very worried. Most appropriate course of action?

Inform the woman that the patient is stable, but she will have to wait until he can give permission to share any details Under the HIPAA, physicians may disclose patient information to friends and family members only when the patient gives explicit permission or does not object when given a reasonable opportunity When the patient is incapacitated or is not present, basic information can be shared if the physicians professional judgement doing so is in the patients best interest. In this case there is no way of knowing for sure whether this woman is his wife or not, however, she is distressed and leading her to worry for an extended period could cause the patient emotional harm if she is indeed family. Best approach to protect patients privacy is to explain he is stable but that further information can't be shared until he is asked and provides consent Identification or marital status is not required as the patient could not want any information released to his wife. CONSENT from patient is required not proof of identify from wife. No proof is required if patient gives permission to share information.

44 year old female undergoes cervical biopsy which reveals malignant cells containing a double stranded DNA virus that has integrated into the host genome & encodes several proteins including E6 and E7. Mechanism that these viral proteins are involved in patients condition?

Inhibition of cell cycle regulatory proteins HPV oncogenicity relies on the inhibitory effects of viral proteins E6 and E7 on cell cycle regulatory proteins p53 and Rb. This allows cells infected with HPV to undergo unchecked cellular proliferation and evasion of apoptosis, promoting genomic instability and malignant transformation

55 year old male smoker with history of DM and HTN has chest pain that began while watching television. After initial treatment, his chest pain disappears (BP improved, and tachycardia is resolved) but he becomes SOB. Physical exam reveals prolonged expirations and wheezes in the bilateral lung fields. Impairment of what process is responsible for his continued shortness of breath?

Inhibition of the neurotransmitter-receptor interaction in adrenergic synapses. Beta adrenergic receptor blockade reduces BP and cardiac work which is beneficial in the acute treatment of myocardial ischemia. However, when nonselective agents are used, adverse effects like bronchoconstriction & wheezing (dose-dependent) may be seen.

16 year old with confusion, fatigue, and abdominal pain. Has also experienced thirst and pyuria for the past 3 weeks. Urine dipstick positive for glucose and ketones. What factor likely contributed to the development of his condition?

Islet leukocytic infiltration (patient is presenting with DKA) Autoimmune insulitis with progressive beta cell loss is the most common cause of type 1 diabetes mellitus. Although antibodies against islet antigens are detected in a large number of patients, they play a permissive role in disease pathogenesis; destruction of beta cells occurs primarily though CELL MEDIATED immunity. Insulitis (infiltration of islets by inflammatory cells) is more prominent during the early stages of the disease . Patients with T1DM have low circulating levels of islet amyloid polypeptide (secreted with insulin from beta cells) due to destruction of beta cella. Insulin resistance accompanied by relative insulin deficiency is the main cause of type 2 DM. Islet amyloid polypeptide is secreted with insulin from pancreatic beta cells. Amyloid deposition occurs in the islets of patients with T2DM. Visceral fat more strongly related to insulin resistance than subcutaneous fat. Maturity onset diabetes of the young is an AD disease caused by mutations that impair glucose sensing and insulin secretion by pancreatic cells. Presents as NIDDM at a young age (<25) and accounts for few cases of DM

19 year old woman with amenorrhea. Onset of menarche was 12 and she had menses regularly every 25-28 days until last year when her menstrual periods began to be irregular. LMP was 3 months ago. Patient expresses concern about being overweight and asks for suggestions on cutting down her caloric intake. Exercises a ton. Only eats fruits and vegetables. BMI is 18. Patient is thin and has dry skin covered by fine, downy hair. Pregnancy test is negative. Expected LH, FSH, and estradiol hormone levels?

LH- decreased FSH- decreased Estradiol- decreased Distorted body image, inadequate diet, dry skin, and lanugo hair in a female suggest anorexia nervosa. Anorexic females commonly have impaired GnRH release from the hypothalamus, leading to low LH, FDH, and estrogen (functional hypothalamic amenorrhea). Reduced circulating leptin levels (as a result of diminished adipose tissue stores) inhibits pulsatile release of GnRH. Complications include reduced peak bone mass when can lead to early onset osteoporosis

68 year old woman with sudden onset double vision. Woke up with symptoms and noticed they worsen when she looks to the right. When she looks to the right her left eye stays neural and does not move medially/to there right. Lesion at what location is most likely causing the observed ocular finding?

Left dorsal pons (disrupts left MLF resulting in impaired adduction of left eye during right horizontal conjugate gaze & diplopia and horizontal nystagmus of right eye) Patient has internuclear opthalmoplegia which is a disorder of conjugate horizontal gaze in which the affected eye (ipsilateral to the lesion) is unable to adduct and the contralateral eye abducts with nystagmus. Convergence and the pupillary light reflex are preserved. Typically occurs with damage to the medial longitudinal fasciculus which is a paired neural tract that mediates communication between CN III and CN VI nuclei, allowing for coordinated horizontal eye movements. Note: right lateral geniculate nucleus is located in the thalamus and relays visual information to the ipsilateral primary visual cortex & a lesion in this structure would result in contralateral homonymous hemianopsia. Note: superior colliculus controls vertical gaze. Damage to dorsal midbrain in the superior colliculus region causes Parinaud syndrome (upward gaze palsy, absent pupillary light reflex, impaired convergance)

5 week old girl with several hours of fever, irritability, and vomiting. Awoke this morning warm to touch refusing to drink. Temperature is 104.2. Lumbar puncture reveals CSF pleocytosis. Empiric cefotaxime is initiated. However, no clinical improvement is seen. CSF culture yields an organism thats resistant to cefotaxime. What organism is most likely causing patient's infection?

Listeria monocytogenes (resistant to cephalosporins due to altered penicillin binding proteins) Ampicillin is the treatment of choice for Listeria, which is not sensitive to cephalosporins. Listeria most commonly causes disease in those with deficient cell mediated immunity, such as infants or immunocompromised patients. Cephalosporin resistant organisms: Listeria, MRSA, Enterococci, Atypicals (chlamydia, mycoplasma)

Patient dies after a high speed MVC due to massive cerebral edema and brainstem herniation. Autopsy shows enlarged left atrium, increased LV myocardial mass, and increased LV wall thickness. Structural changes observed in this patient's heart are most likely associated with what condition?

Long standing hypertension Concentric hypertrophy is characterized by uniform thickening of the ventricular wall and narrowing of the ventricular cavity due to increased after load (chronic HTN, aortic stenosis). Sarcomeres added in parallel.Patients may develop diastolic dysfunction with left atrial enlargement and CHF due to impaired ventricular compliance and filling Eccentric hypertrophy is characterized by reduced ventricular wall thickness with an associated increase in chamber size due to volume overload. Sarcomeres added in series.

Patient with 4 week history of diarrhea accompanied by occasional abdominal bloating. After basic evaluation, patient is started on a trial of diphenoxylate. Primary target of this drug?

Motility Diphenoxylate is an opioid antidiarrheal drug that binds mu opiate receptors in the gut to slow motility. Overuse can lead to euphoria and physical dependence. To discourage use, its combined with atropine, which induces adverse affects (dry mouth, blurry vision, nausea) if taken in high doses Loperamide is another low potency opioid agonist used in the symptomatic treatment of diarrhea. Common side effects of these drugs: rebound constipation, mild sedation. DONT use to treat diarrhea due to toxin producing or invasive organisms Note: agents that target secretory diarrhea include bismuth subscalicylate, octreotide, and probiotics

60 year old farmer with acute onset confusion, muscle weakness, and difficulty breathing. Has vomited twice and has had 3 loose bowel movements over the last 2 hours. Pupils are constricted, symmetric, and reactive to light. Exam reveals excessive sweating, lacrimation, and wheezing. He is incontinent of urine. IV Atropine is administered. What finding is most likely to persist without additional therapy?

Muscle weakness Organophosphates inhibit cholinesterase in both muscarinic and nicotinic cholinergic synapses leading to decreased Ach degradation and overstimulation of the corresponding receptors. Atropine reverses the muscarinic effects but doesn't prevent the development of nicotinic effects such as muscle paralysis. Pralidoxime is a cholinesterase-reactivating agent that allows for degradation of excess acetylcholine and treats both the muscarinic and nicotinic effects of organophosphates Cholinergic toxicity: Muscarinic effects (DUMBELS): diarrhea/diaphoresis, urination, miosis, bronchospasm/bronchorrhea/bradycardia, emesis, lacrimation, salivation Nicotinic effects: muscle weakness, paralysis, fasciculations

33 year old with infertility. Never been pregnant despite years of unprotected sex with her husband. History of pelvic pain that worsens with menses and is suspected to be due to endometriosis. Multiple flesh colored nodules are present on the pelvic organs, along with thin, filmy adhesions. In addition to having endometrial glands with hemosiderin pigment, one of the biopsies contains simple cuboidal epithelial cells. Most likely site of biopsy?

Ovary The uterine mucosa is covered by endometrium, and abnormal implantation of endometrial glands/stroma is referred to as endometriosis. The ovary is covered by a simple cuboidal epithelium involved in surface repair of defects from ovulation. Endometriosis commonly affects the ovary and results in infertility by disrupting folliculogenesis, oocyte release, and oocyte fertilization. Other frequent sites are fallopian tubes and peritoneum.

62 year old woman with 2mm ST segment elevation in leads v2-v5. Patient is treated with medical management. On fifth day of hospitalization, she dies suddenly despite adequate resuscitation. Most likely cause of death?

Profound hypotension Rupture of the LV free wall is a catastrophic mechanical complication of an ANTERIOR (v2-v5) wall MI (LAD artery occlusion) that usually occurs within the first 5-14 days after MI. During this time period the infarcted myocardium is substantially weakened by coagulative necrosis, neutrophilic and macrophage infiltration, and enzymatic lysis of myocardial connective tissue. Rupture leads to hemoperricardium and cardiac tamponade, causing profound hypotension and shock with rapid progression to pulseless electrical activity and death Note: true aneurysms of the ventricular wall are bound by scarred myocardium that is touch and fibrotic. They are a late complication of large transmural infarcts (>1 month) and can cause mural thrombus, arrhythmias, and HF, but rarely rupture

32 year ld Caucasian male with fever, weight loss, myalgias, abdominal pain. If diagnosed with polyarteritis nodes, what artery with be spared by the disease?

Pulmonary (spares pulmonary arteries and rarely involves the bronchial arteries) Polyarteritis nodosa is a segmental, transmural, necrotizing inflammation of medium to small sized arteries. Renal artery involvement is often prominent. Vessels of the kidneys, heart, liver, and GI tract are most commonly involved in resulting ischemia, infarction, or hemorrhage. Cutaneous manifestations occur in up to 1/3 of patients and include palpable purpura & livedo reticularis.

36 year old woman with sudden right side weakness and speech difficulty. During the last 3 weeks, she has experienced progressive fatigue, malaise and low grade fevers. She had a dental extraction 5 weeks ago which was uncomplicated. Never been previously significantly ill or hospitalized. Never traveled outside the USA. Admitted to hospital but dies 2 hours later. Autopsy shows large, friable, irregular masses attached to the atrial surface of a valve. What underlying condition most likely predisposed this patient to her presenting disease?

Regurgitant mitral valve prolapse Patient had subacute infective endocarditis complicated by embolic stroke. Mitral valve prolapse with regurgitation is the MOST COMMON predisposing condition for native valve infective endocarditis in developed nations. Microscopic deposits of platelets and fibrin occur spontaneously in individuals with valvular disease secondary to endocardial injury from turbulent flow. These deposits then become collided by organisms during episodes of bacteremia (such as dental extraction). Valvular sclerosis and mechanical vales are other common causes in developed nations. Rheumatic heart disease is a frequent cause of infective endocarditis in DEVELOPING nations.

Group A streptococci are treated with chemical to solubilize the cell wall in an experiment and proteins are extracted. EM evaluation of a specific protein shows structural homology with mammalian tropomyosin and myosin. What is the likely function of this bacterial cell wall associated protein?

Resist phagocytosis The major virulence factor of S. progenies is M protein, an alpha-helical coiled, coil protein that shares structural homology with tropomyosin and myosin. It extends from the cell wall and prevents phagocytosis, inhibits complement binding, and mediates bacterial adherence. Serotypes of S. pyogenes are differentiated based on antigenic variation of M protein.

34 year old HIV patient with fever, chills, productive cough, left sided chest pain that worsens with deep breathing. Symptoms began 3 days ago and have progressively worsening. CD4 count was 480 a month ago. Temp is 102. PE shows dullness to percussion, bronchial breath sounds, and crackles over left lower lung. Right lung and cardiac auscultation are normal. Labs reveal elevated leukocyte count with left shift. Organism most likely responsible for his current symptoms?

S. pneumoniae Most common cause of CAP in both HIV infected and non-HIV infected individuals is S. pneumoniae. Risk of invasive pneumococcal disease is significantly increased in patients with HIV regardless of CD4 count because the virus hampers the production of opsonizing antibodies and the recruitment of phagocytes to the area of infection.

17 year old girl diagnosed with appendicitis. Surgical intervention is planned. During surgery, what landmark is most helpful in identifying the appendix?

Teniae coli The teniae coli are 3 separate sooth muscle ribbons that travel longitudinally on the outside of the colon and converge at the toor of the vermiform appendix. If the appendix cannot be identified by palpation during an appendectomy, it can be located by following the teniae coli to their origin at the cecal base.

Presents with escalating fever (with initial diarrhea or constipation), followed by abdominal pain, formation of rose spots on the chest/abdomen, and hemorrhagic enteritis with possible bowel perforation

Typhoid fever (caused by Salmonella Typhi or Paratyphi) Life threatening illness that is not frequently seen in industrialized nations unless there is a recent travel history to endemic areas. Transmission occurs via fecal oral route following ingestion of contained food/water. Salmonella is able to penetrate the gut mucosa via phagocytosis by M cells and bacteria mediated endocytosis by enterocytes. Bacteria can then survive and proliferate in macrophages found in Peyers patches causing severe inflammation and ensuing bacteremia. Patients develop fever with relative bradycardia followed by watery diarrhea OR constipation, abdominal pain, and salmon colored rose spots on chest/abdomen. Widespread dissemination across reticuloendothelial system (liver, spleen, bone marrow) can cause hepatosplenomegaly & ulceration of the Peyer's patches can result in GI bleeding. In severe cases, transmural necrosis can cause intestinal perforation and subsequent polymicrobial peritonitis, sepsis, and death Affected patients are also at risk for becoming chronic carriers, particularly those with cholelithiasis or other biliary tract abnormalities that allow S. type to persist in the bile

Pancreas of a 22 year old Caucasian with recurrent pulmonary infections and finger clubbing shows exocrine gland atrophy and fibrosis. Pancreatic ducts are lined with squamous epithelium with areas of keratinization. Finding of squamous metaplasia in the pancreatic ducts is most likely related to deficiency of?

Vitamin A Recurrent sinopulmonary infections and exocrine gland fibrotic atrophy in a young Caucasian are suggestive of cystic fibrosis. CF can lead to pancreatic insufficiency, fat malabsorption, and a deficiency of vitamins A, D, E, K. Vitamin A maintains orderly differentiation of specialized epithelia, including mucus-secreting columnar epithelia of the ocular conjunctiva, respiratory and urinary tracts, and pancreatic & other exocrine ducts. Avitaminosis A can cause squamous metaplasia of such epithelia to a keratinizing epithelium.

16 year old boy with progressive gait instability, dysmetria and dysarthria for the past several years. Patient enjoys basketball but can't play anymore dye to symptoms. He is worried as his brother died of a neurologic disease at age 21. PE of the LE reveals bilateral motor weakness and loss of DTRs, joint position, and vibration sense. MRI of brain reveals degeneration of the posterior columns and spinocerebellar tracts. What condition has clinical manifestations that closely resemble this patients disease?

Vitamin E deficiency Vitamin E is a lipid soluble vitamin that has antioxidative properties. Neurologic symptoms of vitamin E deficiency closely mimic Friedreich ataxia and include ataxia (due to degeneration of spinocerebellar tracts), loss of position and vibration sense (degeneration of dorsal columns) and loss of DTRs (peripheral nerve degeneration).

Your patient is diagnosed with an STI and admits to having an affair. The following week her husband calls asking for information regarding her health as he is concerned about his physical symptoms. What is most appropriate response?

'If your spouse is a patient here, she would have to provide a release of information' The HIPAA protects health information by requiring verbal or written authorization release of information. Family members who contact the physician for information about their relative shouldn't be given any information about the patient without the patients consent

The center of the airway pressure volume curve is the FRC of the lungs. It identifies the resting state where the air pressure is zero. At the FRC the intrapleural pressure is?

-5 cm H20

Medication that is known to inhibit fungal colony growth by changing the composition of fungal cell membrane & inhibits liver cytochrome P450 dependent metabolism of several drugs?

-Azoles Azoles inhibit the synthesis of ergosterol by the fungal cytochrome P450 enzymes. They also suppress the human P450 system, resulting in many drug drug interactions. Medications such as warfarin, cyclosporine, tacrolimus, and oral hypoglycemia should be avoided (or carefully monitored) in patients receiving azole therapy. Cytochrome oxidase inducers (rifampin, phenytoin, carbamazepine, phenobarbital) increase azole metabolism lowering their concentration in the serum.

General feature that aid in localizing a transverse spinal cord section?

-Moving rostrally, spinal levels have increasing amount of white matter, decreasing amounts of gray matter, and are more oval shaped -Lower cervical and lumbosacral regions have large ventral horns -Thoracic and early lumbar sections contain lateral gray matter horns (interomediolateral cell columns) -Gracile and cuneate fasciculi are present above the T7 spinal level, whereas only the gracile fasiculus is present below this level

In a study its determined that the test result is negative in 95% of patients who do not have the disease. If the new assay is used on 8 samples taken from patients without the dais, whats the probability of all test results coming back negative?

0.95^8 Each of the 8 blood sample results is an independent event (one patients result has no impact on anothers) with a 95% probability of correctly testing negative and a 5% probability of incorrectly testing positive. TO calculate change of all 8 tests being negative multiple the probability of each test resting a negative result: 0.95 x 0.95 x 0.95 ....... aka (0.95^8) If the events are independent, the probability that all the events will turn out the same is the product of the separate probabilities for each event. The probability of at least 1 event turning out differently is given as 1 minus the probability of all the events being the same So for the probability of at least one sample test coming back positive in this question would be (1-.95^8)

Couple undergoes IVF. Father has CF and mother has a sister with CF but doesn't have the disease herself. Neither of the mothers parents are affected. Chance that their child will have CF?

1/3 2/3 chance that mother is a carrier and 1/2 chance that mother will transmit the mutant allele to the child. Father will always contribute a mutant allele. 2/3 x 1/2 = 1/3

Researcher trying to develop a vaccine against HCV. She infects a chimpanzee with HCV of known genotype and subtype. Several weeks later viral RNAs are extracted and their sequences vary significantly from that of the original infecting virus. The genetic instability is most likely due to the lack of what feature during the viral replication process?

3' --> 5' exonuclease activity HCV is genetically unstable because it lacks proofreading 3' --> 5' exonuclease activity in its RNA polymerase. RNA polymerase makes many errors during replication and several dozen subspecies of HCV are typically present in the blood of an infected individual at any one time. Its envelope glycoprotein sequences also contain a hypervariable region prone to frequent genetic mutation.

5' --- ACG- CUA- CCA- UUG- UAA- CAA- GUU- AGC- UAG --- 3' What tRNA anticodon is responsible for adding the last amino acid to this polypeptide during protein translation?

5' CAA 3' Translation of the mRNA template proceeds in the 5' to 3' direction. Because complementary sequences align in antiparallel fashion, during translation tRNA anticodons will be oriented in the opposite 3' to 5' direction. Stop codons (UAA, UAG, UGA) halt protein synthesis by binding release factor, they don't add amino acids to the polypeptide chain

23 year old woman with acute nausea and vomiting. Recently returned form trip to Mexico and then developed nausea and vomiting with bouts of watery diarrhea. Improvement of patients vomiting would be best achieved by a medication targeting what receptor?

5-HT3 receptor 5-HT3 (ondansetron) receptor antagonists are useful in the treatment of visceral nausea due to GI insults such as gastroenteritis, chemotherapy, and general asesthesia. Antihistamines H1 (diphenhydramine, meclizine) and anticholinergics (scopolamine) are recommended for VESTIBULAR nausea & dopamine antagonists (metoclopradmide, prochlorperazine) are used for nausea associated with MIGRAINE

How long does it take ischemic myocardium to stop contracting?

60 seconds Loss of cardiomyocyte contractility occurs within 60 seconds after the onset of total ischemia. When ischemia lasts less than 30 minutes, restoration of blood flow leads to reversible contractile dysfunction (myocardial stunning) with contractility gradually returning to normal over the next hours/days. After 30 minutes of total ischemia, the injury becomes irreversible Note: Under hypoxic conditions, ATP in cardiomyocytes is degraded to ADP, AMP, and adenosine. Adenosine is able to cross the cell membrane and function as a vasodilator when coronary blood flow is insufficient to meet myocardial demand, but persistent ischemia leads to depletion of adenosine.

54 year old man with worsening fatigue, generalized weakness, and unintentional 15 pound weight loss. Has 40 pack year history of smoking. BP 160/100. PE shows darkening of akin in sun exposed areas and scattered ecchymosis on extremities. Mild proximal weakness is presents. Chest imaging reveals an irregular lung mass with mediastinal lymphadenopathy. Most likely responsible for patients current symptoms?

ACTH production from ectopic site Paraneoplastic hypercortisolism most commonly caused by small cell lung cancer, is due to ectopic ACTH secretion. Clinical features include HTN, hyperglycemia, edema, and hyper pigmentation. Unlike nonparaneoplastic Cushing syndrome, central obesity is uncommon and weight loss occurs.

Patient with refractory psoriasis is prescribed a medication that has strong teratogenic potential. What drug?

Acitretin Acitretin is a synthetic retinoid used systemically to treat psoriasis. All systemic retinoids are strongly teratogenic Note: Calcipotriol- vitamin D analog used in topical treatment of psoriasis Sulfalazine- can be used in psoriasis but more often used in psoriatic arthritis (safe in pregnancy but can cause jaundice in breast fed infants) DO NOT use tetracyclines to treat psoriasis- may worsen disease!

39 year old man with month long history of fever, fatigue, joint pain, and urticarial skin rash. Has a history of multiple heterosexual partners. PE reveals moderately enlarged and tender liver. No jaundice. Labs show significant elevations in ALT and AST. Most likely diagnosis?

Acute hepatitis B Acute hepatitis B infection can cause a serum sickness like syndrome with joint pain, lymphadenopathy, and a pruritic urticarial rash. Other features may include RUQ pain, hepatomegaly, and elevated hepatic transaminase levels. Most important early markers are HBsAg and IgM anti-HBc which may be present prior to symptoms or changes in transaminase levels. HBV DNA counts and HBeAg correlate with infectivity. Note: sexual transmission is more common in HBV than HCV. HCV is much more commonly transmitted via IV drugs. HCV is typically asymptomatic.

Laparotopy performed on a 35 year old male with abdominal pain shows chalky white lesions in the mesentery. Histologic evaluation of the lesions reveals fat cell destruction and calcium deposition. Patient most likely suffers from?

Acute pancreatitis In acute interstitial pancreatitis the pancreas is grossly edematous. Focal areas of fat necrosis, calcium deposition, and interstitial edema are seen on LM. In necrotizing (hemorrhagic) pancreatitis, chalky white areas of fat necrosis interspersed with hemorrhage are seen on macroscopic exam

Risk factors for VZV reactivation/shingles?

Advanced age and immunosuppression (HIV)

Oxidase positive, non lactose fermenting, gram negative rod that can cause gastroenteritis, wound infections and bacteremia following exposure to contaminated water

Aeromonas hydrophilia

Decreasing what would increase contraction velocity in an isolated skeletal muscle?

Afterload (force against which the muscle fiber must contract) At constant contractility, the maximal velocity of skeletal muscle contraction increases as after load decreases. The highest contraction velocity is achieved when the after load is zero Note: preload is the initial muscle fiber length prior to contraction; increasing preload increases contractile velocity up to a certain value (beyond this maximum actin/myosin overlap decreases and contractile velocity decreases) Note: passive tension exerted by a skeletal myocyte is positively correlated with its resting length; a decrease in passive tension implies decrease in fiber length and decrease in maximum contraction velocity

35 year old presents with several weeks of occasional hemoptysis, weight loss, and low grade fever. He lost 15 pounds over the last 3 months. Recently immigrated from Central America. CXR reveals an apical lung infiltrate and cavitary lesion. Sputum culture grows acid fact bacilli. Cavitary lesions seen in this patient most likely formed through what pathogenic mechanism?

Aggregations of activated leukocytes The control of M. tuberculosis infection is primarily mediated by activated macrophages which surround extracellular myocobacteria and wall them off within granulomas. The release of digestive enzymes by these cells helps control the infection but leads to damage of the surrounding tissue and the formation of cavitary lung lesions.

63 year old man with acute onset neurologic deficits. Symptoms started shortly after walking up this morning. Imaging shows an infracting involving Broca's area. Most likely finding to be present on patients neurological exam?

Agrammatic speech (slow speech consisting primarily of nouns and verbs) Broca expressive (motor) aphasia is caused by lesions to the inferior frontal gyrus of the dominant (usually left) hemisphere. Its a confluent aphasia usually characterized by agrammatism, intact comprehension, and poor repetition. They can follow commands and are often frustrated as they are aware of their expressive language dysfunction.

26 year old with 3 week history of SOB, cough, and hemoptysis preceded by a URI. PE revels bilateral inspiratory crackles and LE edema. Creatinine is 4.1. UA shows proteinuria and hematuria with dysmorphic red cells. Bilateral pulmonary infiltrates on CXR. Has an increased carbon monoxide diffusing capacity on PFT. Antibodies against what substance is most likely to be associated with patient's condition?

Alpha 3 chain of type IV collagen Goodpasture syndrome is caused by autoantibodies against the alpha 3 chain of type IV collagen in golmerular and alveolar basement membrane (anti-GBM antibodies). Patients typically present with RPGN (nephritic syndrome) & alveolar hemorrhage (SOB, hemoptysis). LM shows glomerular crescent formation and immunofluorescence shows linear deposition of IgG and C3 in the GBM. Hemoglobin in the alveoli leads to increased alveolar oxygen absorption and high carbon monoxide diffusing capacity.

Epinephrine is found to cause a decrease in insulin levels, but after pretreatment what a drug, epinephrine injection causes paradoxical increase in insulin concentration. What receptor does this drug most likely block?

Alpha-2 adrenergic receptors Alpha 2 adrenergic receptors inhibit insulin secretion and beta 2 adrenergic receptors stimulate insulin secretion. Alpha 2 mediated inhibitory effect is generally predominant, causing sympathetic stimulation to lead to an overall inhibition of insulin secretion Histamine H2 receptor stimulation increases insulin secretion

54 year old woman with multiple firm violaceous nodules on her right upper arm. Was diagnosed with right sided breast carcinoma 12 years ago and was treated with radical mastectomy. Extensive axillary lymph node dissection revealed no lymph node metastasis at that time. Post-operatively she developed chronic lymphedema of her right arm. Most likely etiology of her current skin lesions?

Angiosarcoma Axillary lymph node dissection is a risk factor for the development of chronic lymphedema involving the ipsilateral arm. Chronic lymphedema predisposes to the development of angiosarcoma (Stewart-Treves syndrome). Histopathologically, angiosarcoma will show infiltration of the dermis with slit likely abnormal vascular spaces. Prognosis is poor as tumor is usually widespread at time of diagnosis.

28 year old man with 12 hours of severe right knee pain and swelling. No trauma. Right one effusion, erythema and tenderness is present on exam. ROM decreased. Arthrocentesis is performed and synovial fluid shows WBC count of 110,000. Best treatment?

Antibiotics A high synovial fluid leukocyte count (>100,000) and absent crystals on microscopic exam strongly suggest bacterial joint infection. Septic arthritis requires antibiotic treatment to prevent joint destruction, osteomyelitis, and sepsis. Gout/pseudogout have lower WBC counts (less than 100,000).

Most important factor in preventing influenza infection in patients who have prolonged and repeated exposure to sick contacts?

Antibodies against hemagglutinin (can neutralize the virus and prevent its binding to host cells) Note: antibodies against neuraminidase are not the main source of protection against reinfection although they have some protective effect (decrease extent of viral invasion and shedding)

18 year old man with sudden onset right chest pain and dyspnea. Patient was watching TV on the couch when symptoms started. He has main with deep breaths. PE shows a tall, thin patient in acute distress. Smokes cigarettes. Right side of chest is hyperresonant to percussion and lacks audible breath sounds. What condition most likely led to his presentation?

Apical sub pleural blebs Primary spontaneous pneumothorax occurs in patients without preexisting pulmonary disease when large change in the alveolar or intrapleural pressure results in a break in the visceral (ruptured superficial bleb) pleura and air trapping between the pleural spaces. Being tall (more negative intrapleural pressure in the lung apices) and smoking are risk factors.

3 year old boy with recurrent skin rash. Develops erythematous, itching rash on checks, trunk, and arms about 5 times a year in relation to the consumption of certain foods. Patients condition is most likely associated with what disorder?

Asthma Atopic dermatitis (eczema) is a common chronic inflammatory skin disorder of childhood. It presents with pruritus and erythematous, weeping/crusted papules and plaques that occur in response to certain environmental allergens. Its associated with other atopic diseases such as allergic rhinitis and asthma. Caused by mutations affecting proteins such as flaggrin or other components of the epidermis resulting in impairment of the skins barrier function. Affected patients have high serum levels of IgE, peripheral eosinophilia, and high levels of cAMP phosphodiesterase in their leukocytes.

ECG of patient reveals HR of 120 bpm with an irregular rhythm, narrow QRS complexes and no P waves. What physiological factor likely determines the ventricular contraction rate in this patient?

Atrioventricular node refractory period Atrial fibrillation occurs due to irregular, chaotic electrical activity within the atria and presents with absent P waves, irregularly irregular R-R intervals, and narrow QRS complexes. The atrioventricular node refractory period regulates the number of atrial impulses that reach the ventricle and determines the ventricular contraction rate in conditions where the the atria undergo rapid depolarization

24 year old with 2 days of pain in left testes. PE reveals urethral discharge and enlarged tender left testes. LM of a gram stain specimen shows numerous neutrophils but no organisms. What agent would be most effective in treating this patient?

Bacterial protein synthesis inhibitor (azithromycin) Chlamydia trachomatis is a common cause of nongonoccal urethritis and epididymititis in men. Its an obligate intracellular organism that synthesizes an unusual peptidoglycan cell wall (antibiotics that function by inhibiting the peptidoglycan synthesis are not effective). Gram stain typically reveals neutrophils but no organisms. Azithromycin is treatment of choice.

Intussusception most often occurs in children younger than 2 years of age in the region of the ileocecal valve. It manifests with intermittent severe colicky abdominal pain, nausea, vomiting, 'current jelly' stools, and sometimes a palpable mass in the right lower quadrant. How do you diagnose & what is treatment?

Barium enema is diagnostic and may be therapeutic. If it doesn't resolve with barium enema then surgical intervention is mandated

Flumazenil competitively inhibits what drug class at the GABA receptor complex?

Benzos (NOT barbiturates- has no effect on them!)

Subarachnoid hemorrhage presents with a generalized excruciating headache. Its classically described as 'the worst headache of my life'. Usually occurs due to tupture of saccular (berry) aneurysms or arteriovenous malformations. Berry aneurysms of the circle of willis (anterior communicating artery most common site) are associated with ADPKD and Ehlers-Danlos syndrome. Where does blood accumulate in SAH?

Between the arachnoid and pia matter Note: CT scan without contrast is diagnostic & lumbar puncture reveals gross blood or xanthochromia (yellow discoloration of CSF)

Visual defect that occurs with craniopharyngioma?

Bitemporal hemianopia (they are suprasellar tumors that compress on the central portion of the optic chiasm)

Beta-naphthylamine is used in aniline dyes and the rubber industry and is associated with an increased incidence in what type of cancer?

Bladder carcinoma

56 year old who woke up today with facial swelling and difficulty breathing. No itching or skin rash. NO similar symptoms in the past. Began taking Lisinopril 2 months ago for HTN. On exam there is swelling of lips and tongue and mild audible strider without wheezing. Most likely mechanism for patients symptoms?

Bradykinin accumulation Angioedema is a rare and serious side effect of ACE inhibitor therapy. ACE inhibition increases bradykinin levels which increase vascular permeability and lead to hngioedema. Symptoms include tongue, lip, or eyelid swelling and less frequently laryngeal edema and difficulty breathing. ACE inhibitors should be discontinued in such patients. Symptoms typically appear within days of starting therapy but can occurs weeks to years of therapy as well. Note: hereditary C2 esterase deficiency also causes angioedema without a rash but it typically presents in childhood and adolescence.

73 year old with breathing difficulty and increased abdominal girth. Has also had decreased appetite and constipation that has worsened progressively for the past 6 months. On PE abdomen is distended and contender with right adnexal fullness. Microscopic exam of the surgical biopsy specimen of right adnexal mass reveals anaplasia of epithelial cells with invasion into the stroma and multiple papillary formations with cellular atypia. Most likely marker to be elevated in this patient?

CA-125 Epithelial ovarian cancer is the most common ovarian malignancy. Histologic findings include anaplasia of epithelial cells with invasion into the stroma, along with multiple papillary formations with cellular atypia. Produce CA-125 when can be used as serum marker for this condition. Patients with ovarian CA typically present with a pelvic mass, ascites, and peritoneal metastasis that result in decreased appetite, abdominal dissension, and bowel or bladder changes. Nulliparous women are at increased risk due to frequent ovulation resulting in continued disruption and repair of the ovarian epithelium.

Rate of glycogenolysis within myocytes increases several hundred fold during active skeletal muscle contraction. What substance is most likely responsible for increasing the reaction rate during active contraction?

Ca2+ Synchronization of glycogen degradation with skeletal muscle contraction occurs due to release of sarcoplasmic Ca2+ following neuromuscular stimulation. Increased intracellular Ca2+ causes activation of phosphorylase kinase, stimulating glycogen phosphorylase to increase glycogenolysis

Upon administration of a drug intercellular adhesions between epithelial cells are disrupted but the cell adhesions to the basement membranes remain intact. A similar effect is produced when incubated in a medium that has been depleted of calcium ions. The function of what substance is most likely affected by the chemical used in this experiment?

Caherins Cadherins (desmogleins, desmocollins, E-cadherin) are calcium dependent adhesion molecules that bind epithelial cells together within tissues. Cadherin molecules on epithelial cells bind to catherine molecules on adjacent cells to form adherens junctions and desmosomes.

30 year old male with adult polycystic kidney disease presents with sudden headache and confusion. CT shows blood in the subarachnoid space. On 5th day after admission, patient begins to complain of weakness in his right arm and leg. What drug could have prevented patient's neurological sequelae?

Calcium channel blocker Patient likely had a ruptured berry aneurysm, as he has adult polycystic kidney disease. Symptoms of cerebral vasospasm include altered mental status & focal neurological deficits. Substances generated by the degradation of subarachnoid blood clots are believed to be the etiologic agents. Symptoms manifest no earlier than 3 and usually 7-8 days following SAH. CCBs, specifically Nimodipine, can be used to assist in the prevention of cerebral vascular spasm following SAH. This is an alternative use of CCBs.

37 year old man found unresponsive under a tree during a thunderstorm. He's not breathing on scene. Pupils are fixed and dilated. Several cutaneous erythematous marks in a fern leaf pattern (Lichtenberg figures) are seen on LE. Second degree burns are present on both arms. CPR is performed but patient is pronounced dead upon arrival to hospital. Most likely primary cause of his death?

Cardiac arrhythmia Although lightning injuries are rare, they are associated with a 25% fatality rate. 2/3 of lightening related deaths occur within the first hour after injury with fatal arrhythmias and respiratory failure as the most common causes. Patients with minor cutaneous involvement may still have major internal injury after lightening strikes and high voltage electrical content.

56 year old man with chronic cough and fatigue is given supplemental oxygen and his respiratory rate decreases shortly after. Reduced respiratory rate is most likely caused by a sudden decrease in stimulation of what sensory receptors?

Carotid bodies PaCO2 is the major stimulator of respiration in healthy individuals; even a slight increase in PaCO2 results in increased pulmonary ventilation. In patients with COPD, response to PaCO2 is blunted and hypoxemia becomes an important contributor to the respiratory drive. Peripheral chemoreceptors are primarily responsible for sensing arterial PaO2 and can be suppressed with oxygen administration Note: the pulmonary stretch receptors include myelinated and unmyelinated C fibers in the lungs and airways. These receptors regulate the duration of inspiration depending on the degree of lung distension

Vascular malformations composed of abnormally dilated capillaries separated by thin connective tissue septa. Lack of structural support gives them the tendency to bleed, and those that occur in the brain can present with neurologic defects and seizures

Cavernous hemangiomas

32 year old with facial pain and nasal congestion. Was recently discharged after being treated for DKA. On PE right eye movement is moderately restricted and necrotic destruction of nasal turbinates with thin bloody nasal discharge is present. Patient's disease process is most likely to involve what dural sinus?

Cavernous sinus Patient has mucor which can be complicated by cavernous sinus thrombosis. Cavernous sinus receives venous blood from the face via the superior and inferior ophthalmic veins. Spread of infection from the orbit, nose, and paranasal sinuses along this valveless venous system may result in cavernous sinus thrombosis. Lateral gaze palsy due to involvement of CN VI within the cavernous sinus is one of the first signs of this disease

Characterized by downward displacement of elongated cerebellar tonsils through the foramen magnum and into the upper cervical canal. Patients often present during adolescence or adulthood with occipital headache and symptoms of cerebellar dysfunction (dizziness) that may worsen with valsalava

Chiari I malformation

Glucose 6 phosphate dehydrogenase is the rate limiting enzyme in the pentose phosphate pathway, the major source of cellular NADPH. This molecule is necessary for reducing glutathione (protects RBCs from oxidative damage) and for the synthesis of what compounds?

Cholesterol, fatty acids, and steroids

24 year old woman 12 weeks gestation presents for prenatal counseling. She is 3 feet 11 inches tall with short upper and lower extremities but normal torso length. PE is significant for depression of the nasal bridge and a bulging forehead. Her husband is phenotypically normal and has no medical problems. Patient knows there is a 50% chance of passing on her condition to fetus. Abnormality of what cell is responsible for patients features?

Chondrocytes Unlike the process of intramembranous ossification that forms flat bones, endochondral ossification proceeds along a cartilage template and is responsible for the formation of long bones. Achondroplasia is characterized by exaggerated inhibition of chondrocyte proliferation in the growth plates of long bones and manifests with proximal limb shortening, mid face hypoplasia, and macrocephaly. Caused by AD activating point mutation in FGFR3. FGFR3 normally limits chondrocyte proliferation during endochondral ossification.

22 year old woman is 16 weeks pregnancy with her first child. Two months ago she was diagnosed with CMV & recovered with symptomatic treatment and is currently asymptomatic. Patient's unborn fetus is at greatest risk for what eye condition?

Chorioretinitis The most common eye related complication of congenital CMV infection is chorioretinitis. 5% of women develop a primary CMV infection with mononucleosis like illness (fever and mild hepatitis) during pregnancy. Vertical transmission occurs in 1/3 of cases with the highest risk of transmission during 1st trimester. Other complications of CMV transmission are sensorineural deafness, seizures, jaundice, hepatomegaly, splenomegaly, and microcephaly.

35 year old previously healthy woman presents with a painless mass in her neck that has been progressively enlarging over the last several months. Works as a radiation tech and is worried that the radiation exposure has led to the development of cancer. Says her sleep is often disturbed and she feels tired all the time. PE reveals a diffusely enlarged, firm thyroid gland. Biopsy is performed and shows lymphoid germinal centers. Most likely diagnosis?

Chronic autoimune (Hashimoto) thyroiditis Hashimoto thyroiditis is a common cause of hypothyroidism and is characterized by lymphocytic infiltration of the thyroid gland with follicular destruction and germinal center formation. Hurthle cells (enlarged epithelial cells with eosinophilic cytoplasm and prominent nucleoli) are commonly seen.

64 year old man who has chronic low back pain after an injury 8 months ago. Has since used several OTC analgesics and has recently been taking Naproxen daily. On exam he has trace LE edema. BUN is 12. Creatinine is 2. UA show 1+ protein and 3-4 WBCs/hpf. Renal US shows bilateral shrunken and irregular kidneys with few papillary calcifications. Most likely cause of his renal dysfunction?

Chronic interstitial nephritis NSAID associated chronic renal injury is morphologically characterized by chronic interstitial nephritis and papillary necrosis. It is caused by taking large amounts of NSAIDs over extended periods of time. Patients have modest elevation in serum creatinine, mild proteinuria, and evidence of tubular dysfunction (polyuria, nocturne). Microscopic hematuria and sterile pyuria may be seen on UA. NSAIDS uncouple oxidative phosphorylation and cause glutathione depletion with subsequent lipid peroxidation resulting in damage to tubular and vascular endothelial cells. Chronic interstitial nephritis is seen as patchy interstitial inflammation with fibrosis, tubular atrophy, papillary necrosis and scarring, and caliceal architecture distortion. Calcium can deposit in areas of chronic inflammation. NSAIDs also decrease prostaglandin synthesis causing constriction of medullary vasa recta and ischemic papillary necrosis

41 year old man with fever, weight loss, and productive cough. Reports drinking 18 beers a day. PE shows poor dentition. Foul smelling sputum sample is sent for gram stain and culture. CT scan shows air fluid levels. Best antibiotic for this patient's condition?

Clindamycin Alcoholics are more likely than the general population to develop pulmonary infections and abscesses involving combinations of anerobic oral flora (Bacteroides, Prevotella, Fusobacterium, and Peptostreptococcus) and aerobic bacteria. Clindamycin covers most of these organisms as well as aerobic S. pneumoniae and is the antibiotic of choice for treating lung abscesses. CLindamycin binds to 50S ribosomal subunit and disrupts protein synthesis.

Mechanism of action of the drugs used to treat bacterial vaginosis?

Clindamycin- bacteriostatic drug that inhibits protein translation by binding to the 50S ribosomal subunit Metronidozole- bactericidal and damages the DNA of facultative anerobes

Exists in the environment as a mold (with hyphae) that form spores. These spores are inhaled and turn into spherules that contain endospores in the lungs

Coccidioides immitis Endemic to southwest USA and associated with erythema nodosum

43 year old man with low grade fever, chest pain, and a cough productive of blood streaked sputum. Also developed a skin rash on his arms and symmetric joint pains within the past few days. HE is a sales representative and recently traveled to Mexico and South America for business. CXR reveals a thin walled cavity in the right middle lobe. Organism most likely responsible for his symptoms?

Coddidiodes immitis C. immitis is a dimorphic fungus endemic to San Joaquin Valley, southwest US, Mexico, and South America. It most commonly causes asymptomatic infection or mild pneumonia. A rash and symmetric arthralgia can develop. Rarely it can cause formation of a thin walled cavity in the lung leading to hemoptysis in addition to fever and cough

20 year old with osteoporosis. PE shows mild scoliosis. Ophthalmologic exam shows sclerae with visibility of the underlying choroid layers. What protein is most likely improperly synthesized in this patient?

Collagen Osteogenesis imperfect results from mutations that cause defective type I collagen formation. The defining feature of skeletal fragility is highly variable, ranging from multiple fractures at birth to premature osteoporosis. Abnormally thin sclerae and visibility of the underlying choroid (blue sclerae) are common associated findings

Cell types you would expect to see on colposcopy?

Columnar and stratified squamous epithelial cells Cervix has stratified squamous non-keratinized epithelium in the exctocerwix thats continuous with the vagina and simple columnar mucous secreting epithelium in the endocervical canal. The transformation zone contains both epithelial types and is vulnerable to vertical malignancy due to metaplasia. In adolescents, the transformation zone is on the ectocervix and is called an ectropion. The cervix shrinks with age due to decreasing estrogen and the transformation zone recedes into the endocervical canal

54 year old undergoes surgery to remove a thyroid nodule. During surgery, a nearby nerve is injured while ligating an artery entering the superior pole of the thyroid lobe. What muscle is most likely to be denervated as a result of this injury?

Cricothyroid The external branch of the superior laryngeal nerve is at risk of injury during thyroidectomy due to its proximity to the superior thyroid artery and vein. This nerve innervates the cricothyroid muscle. Note: recurrent laryngeal nerve can be injured during thyroidectomy as its near the inferior thyroid artery

62 year old is diagnosed with diffuse large B cell lymphoma and receives combination chemotherapy which leads to rapid lysis of neoplastic cells. She also receives rasburicase prior to and during treatment. This medication helps protect normal organs by what mechanism?

Converting uric acid into more soluble metabolites Tumor lysis syndrome can develop during chemotherapy for cancer with rapid cell turnover (poorly differentiated lymphomas and leukemias), substantial tumor burden, or high sensitivity to chemotherapy. Its characterized by hyperphosphatemia, hypocalcemia, hyperkalemia, and hyperuricemia. Hyperkalemia can produce fatal arrhythmia and uric acid can precipitate in the kidney and cause kidney failure. Prevention often involves hydration and use of hypouricemic agents such as allopurinol or rasburicase. Rasburicase is a recombinant version of irate oxidase, an enzyme present in mammals but not humans, that catalyzes the conversion of uric acid to allantoin which is 10 times more soluble than uric acid

When actively dividing E. coli os exposed to a drug, enzyme mediated removal in the 5' to 3' direction is impaired leading to inhibition of bacterial growth. What enzyme is likely the target of this drug?

DNA polymerase I High fidelity replication is accomplished by the 3' to 5' proofreading exonuclease activity of all 3 prokaryotic DNA polymerases. DNA polymerase I is unique in that its the only prokaryotic polymerase that ALSO HAS 5' to 3' exonuclease activity. This activity functions to remove the RNA primer created by RNA primase and repair damaged DNA sequences

Previously healthy 2 year old boy with fever and mouth pain that began yesterday. Refuses to eat due to the pain. PE reveals swollen gums and vesicular, inflamed lesions on his hard palate and lips. He has enlarged and tender cervical lymph nodes. Description of the virus most likely responsible for his condition?

DNA virus, double stranded, enveloped primary HSV-1 infection in children causes gingivostomatitis (vesicular lesions on the lips and hard palate). HSV-1 and other herpes viruses are double stranded enveloped DNA viruses.

Mechanism of action of Cromolyn/Nedocromil?

Cromolyn and nedocromil are mast cell stabilizing agents that inhibit mast cell degranulation independent of the triggering stimulus. Typically used to prevent acute attacks, rather than to treat acute exacerbations. They are less effective than inhaled glucocorticoids and are considered second line for the treatment of allergic rhinitis and bronchial asthma. Effective for patients with seasonal symptoms, aspirin hypersensitivity, and exercise induced asthma

36 year old with hemoptysis, HTN, and hematuria. Alcohol fixed neutrophils treated with the patients serum demonstrate indirect immunofluorescence. Most likely diagnosis?

Crescentic glomerulonephritis Image shows a positive reaction for c-ANCA. Granulomatosis with polyangitis (Wegner's) is a cause of rapidly progressive (crescentic) glomerulonephritis type 3 (pauci-immune). The symptoms of nephritis are accompanied by signs of upper and lower respiratory tract involvement. Crescents on LM, the absence of deposits on immunoflourescene, and elevated serum c-ANCA are diagnostic.

Patient with rapidly progressive dementia and myoclonic jerks. On microscopic exam, multiple vacuoles are seen in the grey matter of the brain (spongiform encephalopathy). Most likely diagnosis?

Creutzfeldt-Jakob disease Pathogenesis is linked to abnormal prion protein that is resistant to enzymatic digestion by proteases and accumulates

Adding clavulanic acid to amoxicillin treatment serves what purpose?

Decrease amoxicillin cleavage by bacterial cells Clavulanic acid, sulbactam, and tazobactam are beta lactamase inhibitors that expand amoxicillins spectrum of activity to include strains of beta lactamase synthesizing bacteria that are resistant to amoxicillin alone

21 year old man with SOB, fatigue, myalgias, debilitating retroorbital headaches for the past several days. On summer break and came to work at grandparents' dairy farm 3 weeks ago. He cleans out animal waste from the barn. Temperature is 101.8. Bronchial breath sounds heart in the right lower lung. CXR shows right lower and middle lobe consolidation. Liver enzymes are increased and platelets are low. Normal leukocytes. Most likely cause of patient's symptoms?

Coxielli burnetti Q fever is a zoonotic infection that occurs in dark workers exposed to waste from cattle and sheep. Its causative agent is Coxiella brunettei. A patient with exposure to waste from farm animals who develops nonspecific illness (myalgias, fatigue, fever >10 days, rtetroorbital headache) with lobar pneumonia, a normal leukocyte count, thrombocytopenia and increased liver enzymes should be evaluated for acute Q fever infection. Chronic Q fever is fatal if not treated. Its rare and can occur in patients with or without previous symptomatic acute infection. Manifests as infective endocarditis in patients with valvular disease.

What should be suspected in a patient with gram negative sepsis with a prolonged PT and PTT, decreased platelets and fibrinogen levels, and increased D-dimer?

DIC DIC vs. TTP-HUS: DIC: patients bleed, coagulation cascade id activated, PT and PTT are prolonged, low fibrinogen and decreased fibrinogen degredation products TTP-HUS: usually don't bleed, only platelets are activated, normal PT and PTT, normal fibrinogen

32 year old female with claustrophobia is stuck in a malfunctioning elevator. She experiences severe anxiety as well as dizziness, weakness, and blurred vision. Most likely cause of patients symptoms?

Decreased arterial partial CO2 tension Panic attacks are associated with hyperventilation and decreased pCO2. Hypocapnia causes cerebral vasoconstriction and decreased cerebral flow (hence why patients with cerebral edema are often hyperventilated to decrease ICP). pO2 has much less of an influence on the cerebral blood flow. For severe hypoxia (<50 mm Hg O2) there is a rapid increases in cerebral blood flow and ICP. Other vasoactive substances in the systemic circulation don't affect cerebral flood flow because they don't cross the BBB

72 year old female has been suffering from progressive memory loss over the past 2 years. Now she needs help with her daily activities. MRI shows diffuse cortical hippocampal atrophy. What biochemical change most likely contributes to this patient's condition?

Decreased choline acetyltransferase activity in the nucleus basalis In Patients with Alzheimers disease, theres decreased levels of acetylcholine in the nucleus basalis of Meynert and hippocampus. Diminished activity of choline acetyltransferase in these cerebral structures is the cause.

Patient with chronic insomnia. Has had trouble sleeping because he thinks he is being poisoned at night. He says people have tried to poison him by secretly dumping toxic waste in his backyard at night for the past ten years. No history of psychiatric treatment and no other symptoms. He works as a taxi driver and has for many years. Most likely diagnosis?

Delusional disorder Delusional disorder is characterized by greater than or equal to one delusion (persistent, false, fixed beliefs) for at least one month in the absence of other psychotic symptoms. Behavior is not obviously bizarre and functioning isn't significantly impaired apart from the direct impact of the delusions. Patients with delusional disorder have a persistent overriding delusion with a specific theme.

17 year old girl who presents for recurrent episodes in which she 'seems to be in her own world'. Parents say this has happened for the past 2 years but seems to be occurring more frequently since a break up with her boyfriend last month. Patient states that she 'spaces out' easily & often feels like she's in a fog or a dream and feels numb. She says that she sometimes feels as if she's not herself and thinks are happening to someone else. On exam she is mildly anxious with a flat affect. Most likely diagnosis?

Depersonalization/derealization disorder Depersonalization/derealization disorder is a dissociative disorder involving recurrent episodes of feeling detached from ones own body and/or feelings of unreality. In contrast to psychotic disorders, reality testing is intact during these experiences (individual may feel detached from their body but knows its not true)

A protein is isolated from cytoplasmic granules of a certain type of cells. This protein is believed to cause damage to the bronchial epithelium of patients with atopic asthma. Isolated protein is most likely involved in what function?

Destruction of helminths Major basic protein released by eosinophils normally functions to kill helminths. Its also thought to contribute to the bronchial epithelial damage sustained by patients with atopic (extrinsic allergic) asthma Late phase of atopic asthma attack involves mucosal infiltration by eosinophils, basophils, and neutrophils. Eosinophils have bilobed nuclei and are packed with large eosinophilic granules of relatively uniform size. Note: neutrophils- multi lobed nuclei (>3) and relatively pale, heterogeneously staining granules basophils: granules that stain dark clue, are irregularly sized and obscure the nucleus; granules contain heparin, histamine, and slow reacting substance of anaphylaxis (a mixture of leukotrienes)

Cardiovascular manifestations of lupus include accelerated atherosclerosis, small vessel necrotizing vasculitis with fibrinoid necrosis, pericarditis, and Libman Sacks endocarditis (small, sterile vegetations on both sides of the valve). Renal involvement classically manifests as?

Diffuse proliferative glomerulonephritis which is characterized by diffuse thickening of the glomerular capillary walls with wire loop structures on LM

Patient admits to recently using cocaine and attending physician prescribes the patient a beta blocker. Intern recalls that beta adrenergic antagonists can interact poorly with cocaine. Most appropriate action by the intern?

Don't order the medication until discussing concerns with the attending physician and asking how to proceed Physicians are ethically obligated to questions orders that raise concern about potential harm to patients. Issues should initially be discussed directly with the physician who made the order and not involve ancillary staff.

Early lyme disease (days to weeks following exposure) causes flu like symptoms and erythema chronicum migrans. Second stage ( weeks or months later) involves AV block and facial palsy. Late lyme disease (months to years after exposure) can cause chronic asymmetric large joint arthritis and encephalopathy (decreased memory, somnolence, mood changes). Treatment?

Doxycycline or penicillin type antibiotics (Ceftriaxone)

50 year old woman nulliparus without any family history of cancer. Breast exam normal. Undergoes screening mammography which reveals micro calcifications. She undergoes biopsy when reveals ducts distended by pleomorphic cells with prominent central necrosis. Lesion doesn't extend beyond ductal basal membrane. Most likely origin of the lesion?

Duct (C in picture) DCIS is characterized by ducts distended by pleomorphic cells with prominent central necrosis that don't penetrate the basement membrane. Its the precursor to invade ductal carcinoma, the most common type of breast cancer Note: spread of these malignant cells to the nipple through the duct system results in eczematous nipple changes and indicates Paget disease Note: lobular breast tissue can give rise to lobular breast carcinoma that presents as a palpable breast mass with orderly rows of cells organized in single file Note: phyllodes tumors arise from breast stroma and can progress to malignancy; histology shows disorderly, diffusely hyper cellular connective tissue with cellular atypia and lead like projections

Right sided endocarditis involving the tricuspid valve commonly occurs in IV drug uses and is most often due to S. aureus. Specifically it can cause tricuspid regurgitation which is heard best at what location?

Early systolic murmur heard best over left lower sternal border that is accentuated by inspiration Aortic- 2nd right intercostal space RSB Pulmonic- 2nd left intercostal space LSB Tricuspid- 4th left intercostal space lower LSB Mitral- 5th left intercostal space medial to mid clavicular line

Large, multi-country study used population data from 14 countries located within similar altitudes to evaluate association between dietary habits, including vitamin D intake and prevalence of multiple sclerosis. What best describes the design of this study?

Ecological study Unit of analysis in this study is populations not individuals. This is consistent with an ecological study, in which the frequency of a given characteristic (vitamin D intake) and a given outcome (multiple sclerosis) are studied using POPULATION DATA. Ecological studies are useful to generate hypotheses but shouldn't be used to make conclusions regarding individuals within these populations (ecological fallacy) Note: cross sectional surveys evaluate the exposures and outcomes of interest in individuals (not populations) at a given point in time ('snapshot')

A patient with PCOS is at greatest risk of what complication?

Endometrial carcinoma PCOS results from Increased levels of 17 alpha hydroxylase, 17,20 lyase & 3 beta hydroxysteroid dehydrogenase. Excess androgen production prevents development of a monthly dominant follicle resulting in anovulatory cycles. In turn, menstrual irregularity results in decreased progesterone secretion. Progesterone has growth inhibitory effects on the endometrium and counters estrogen driven proliferation. This puts these patients at risk for endometrial hyperplasia and adenocarcinoma & fertility difficulties.

Von Willebrand factor enhances clotting through both augmentation of platelet binding and stabilization of factor VIII. Patients with vWD are deficiency in functional vWF and present with increased bruisability and prolonged mucosal bleeding. Mechanism of desmopressin in improving this condition?

Endothelial release of vWF. Note: although most clotting factors are synthesized in the liver, vWF and factor VIII are produced in endothelial cells

8 year old boy with cystic fibrosis has sputum cultures that grow P. aeruginosa which is resistant to ciproflocaxin with a high minimum inhibitory concentration. Repeat antibiotic sensitivity testing in a growth medium with a higher pH lowers the minimum inhibitory concentration of the drug. What is the most likely mechanism of ciprofloxacin resistance in this bacterial isolate?

Enhanced antibiotic export by the bacterial efflux pumps Bacterial antibiotic efflux pumps are a common cause of drug or multi drug resistance. They actively transport antibiotics out of the bacteria into the external environment using ATP, sodium gradients, or PROTON electrochemical gradients for energy. The reduction in ciprofloxacin minimum inhibitory concentration with a higher pH age indicates that the underlying source of antibiotic resistance was an efflux pump that utilized the proton gradient for energy

Refers to interactions between multiple genes that combine to create a new phenotype or mask/modify the phenotype of one of the genes

Epistasis

Biopsy of breast tissue specimen shows clumps of cells that have positive immunohistochemical staining for cytokeratin. Cells most likely belong to what lineage?

Epithelial Follow up: The cells identified by the specimen immunohistochemical stain are shown to overexposes a 185 kD glycoprotein that spans the cell membrane and has tyrosine kinase activity in the intracellular domain. Most likely function of this protein? Accelerates cell proliferation HER2 oncogene encodes for a transmembrane glycoprotein with intrinsic tyrosine kinase activity and is a member of the family of epidermal growth factor receptors. Overexpression of this protein is associated with a worse prognosis and increased risk of disease reoccurrence. Traztuzumab (monoclonal antibody against HER2) is used to treat women who test positive for overexpression of HER2 oncogene

6 year old girl that has tanner stage 3 breast development and coarse pubic hair. Height is in the 96th percentile. You explain to her mother that although she is tall now, she will ultimately be shorter than average id she doesn't get treatment. Physiologic explanation for this?

Estrogen effect on the long bone epiphyseal plate Sex hormones promote bone growth and epiphyseal plate closure; precocious puberty may result in shorter stature despite an initial growth spurt. Gigantisism is caused by excessive pituitary production of GH; these patients achieve enormous heights because, unlike excessive sex steroids, excessive IGF-1 doesn't lead to premature closure of the epiphysis

Reason for hypoglycemia in alcoholics?

Ethanol inhibits gluconeogenesis and can cause hypoglycemia once hepatic glycogen stores are depleted Metabolism of ethanol by alcohol dehydrogenase and aldehyde dehydrogenase reduces NAD+ to NADH and increases the NADH/NAD+ ratio, inhibiting all other pathways requiring NAD+ including gluconeogenesis. In particular, lactate can't be converted to pyruvate and instead the reaction is driven from pyruvate toward lactate. Also, excess NADH inhibits the conversion of malate to oxaloacetate. Pyruvate and oxaloacetate are both intermediates in gluconeogenesis.

34 year old man with 3 week history of difficulty hearing. Finds it increasingly difficult to tolerate everyday sounds. Complains of ear pain and often avoids public places as a result. Injury to what cranial nerve most likely responsible for his condition?

Facial The stapedius muscle is innervated by the stapedius nerve (a branch of the facial nerve). Paralysis of the stapedius muscle results in hyperacusis (increased sensitivity to sound). Note: tensor tympani functions to contract the TM medially, thereby increasing its tension and dampening sound transmutation; its innervated by CN V3

14 month old boy with recurrent white patches on the inside of his cheeks. Patches have been present 2 days & he has a decreased appetite and is refusing food. Exam shows dysmorphic face and cleft palate. Flow cytometry of patients blood shows a lot CD3 cell fraction but normal CD20 cell fraction. Patients clinical findings are most closely associated with what mechanism?

Failed pharyngeal pouch development DiGeorge syndrome results from failed development of the 3rd and 4th pharyngeal pouches. A hypoplastic or absent thymus results in recurrent Candida infection (thrush) due to impaired cell immunity. Flow cytometry shows low CD3 (T cell) but normal CD20 (B cell) fraction. Results in recurrent bacterial and fungal infections

During collagen synthesis, defective proline hydroxylation leads to?

Failure of triple helix formation and stabilization by pro-alpha chains during collagen synthesis

2 day old healthy infant. Mother has beta thalassemia trait and father has normal hemoglobin electrophoresis. Most likely predominant hemoglobin composition in this patient?

Fetal (composed of 2 alpha and 2 gamma subunits) Hemoglobin F is the predominant hemoglobin type in the second and third trimester (begins at 8 weeks and replaces all hemoglobin by 14, when erythropoiesis in the liver and spleen is established) of pregnancy and during the first few months after both. It consists of two alpha and 2 gamma subunits and has a high affinity of oxygen which facilitates oxygen transport across the placenta to the fetus. Hemoglobin A (consists of two alpha and two beta subunits) is the major hemoglobin in adults. Note: during the first few weeks of embryogenesis, hemoglobin is synthesized by the yolk sac and contains epsilon or zeta globin chains.

3 year old boy that is not able to cooperate with the other kids but likes to play near them. Patient knows his age and gender and speaks in 3 word sentences. Can ride a tricycle but not a bicycle. Doesn't use a spoon or knife but enjoys eating with his hands. Scribbles spontaneously but can't copy a circle. What developmental milestone is likely delayed in this patient?

Fine motor By age three, child is expected to play imaginatively in parallel, speak in simple sentences, COPY A CIRCLE, USE UTENSILS, and ride a tricycle

45 year old man with AF undergoes ECG testing Pre-test QRS is 95 (80-120 is normal) and QT internal is 410 (normal is less than 440). Point at maximally achieved HR shows a QRS complex duration of 125 and a QT interval of 400. Based on these results, which medication is most likely being used to treat his atrial fibrillation?

Flecainide Class I c antiarrhythmics such as Flecainide are potent sodium channel blockers that have increased effect at faster heart rates (use dependence). This makes them more effective at treating tachyarrhythmias, but can also cause prolonged QRS duration (a proarrhythmic effect) at higher heart rates.

32 year old man with fever, night sweats, chills over the last several days. Patient has been using IV drugs because he is stressed out. Further evaluation reveals aortic valve endocarditis with intracardiac abscess and small fistula formation between the aortic root and right ventricle. Doppler ultrasound of the fistula will most likely reveal what blood flow pattern?

Flow from the aortic root to the RV continuously During the cardiac cycle, central aortic pressure is higher than right ventricular pressure during systole and diastole. Consequently, intracardiac fistula between the aortic root and RV will demonstrate a left to right cardiac shunt as blood continuously flows from the aortic root (high pressure) to the RV (low pressure). This can lead to a continuous murmur heard on auscultation

Treatment of Leigonella pneumonia?

Fluoroquinolones (Levofloxacin) or newer macrolides (Azithromycin)

23 year old student collapses and dies suddenly while taking an exam. Myocardial histology reveals myocyte hypertrophy and haphazardly arranged mycocytes and myocyte bundles & prominent interstitial spaces. Cause of patients condition?

Genetic (hypertrophic cardiomyopathy) Extreme myofiber disarray with interstitial fibrosis on cardiac histology strongly suggests HCM. Almost 100% of cases of HCM results from mutations in gene encoding cardiac sarcomere proteins (most commonly beta myosin heavy chain)

Patient infection with a virus in which transmission of the virus was originally thought to occur only through contact with poultry, but during the outbreak sustained human to human transmission. What is most likely responsible for the infectivity of this virus in humans?

Genetic reassortment Influenza epidemics and pandemics are typically caused by reassortment of the RNA segments coding for hemagglutinin or neuraminadase proteins (major antigenic shifts). Orthomyxoviruses contain a segmented genome and HA and NA are coded for separate segments. This allows for genetic reassortment when 2 distinct strains infect the same cell. This can create a novel strain of a virus to which humans are susceptible but have no immunologic resistance. This process can occur between human and animal strains of influenza A virus in avian or swine hosts. Note: antigenic DRIFT refers to point mutations in HA and NA genes that slightly alter the product proteins, allowing them to evade immune recognition and possible increase infectivity of the virus, but a change in species to species transmission represents a major modification in protein structures thats explained by reassortment

34 year old female and her husband are both of normal stature but 2 of their 3 children have developed achondroplasia. No history of achondroplasia on either side of the family. Both affected offspring are found to be heterozygous for the same mutation in the FGFR gene. Most likely explanation for this couple having children with the disorder?

Germline mosaicism Germline mocaicism (presence of multiple, genetically different gamete cell lines) should be considered when a genetic mutation is identified in the offspring but not the parents

Acute hemorrhage into the pituitary gland (pituitary apoplexy) is usually associated with a preexisting pituitary adenoma. It presents acutely with a severe headache, cranial nerve involvement (bitemporal hemianopsia, ophthalmoplegia) and signs of meningeal irritation. Patients often have chronic symptoms associated with the underlying tumor (mild headache, decreased libido). Cardiovascular collapse can occur due to adrenocortical deficiency and therefore administration of what medication is critical to precent life threatening hypotension?

Glucocorticoids Note: the way to differentiate this from subarachnoid hemorrhage is by the presence of bitemporal hemianopia which is only present in pituitary apoplexy & presence of chronic symptoms also helps differentiate

64 year old homeless male hospitalized with abdominal pain. Found to have elevated serum amylase and lipase. Develops acute confusion during hospitalization. Exam shows horizontal nystagmus and bilateral abducens palsy. Brain imaging reveals bilateral lesions in the mammillary bodies and periaqueductal gray matter. What most likely precipitated his condition?

Glucose infusion The infusion of glucose without thiamine in a patient with chronic thiamine deficiency precipitates encephalopathy. Confusion, ataxia, and ophthalmoplegia form the triad of wernike encephalopathy. Hemorrhage into the mammillary bodies is characteristic.

Analysis of a sample shows accelerated cytosine deamination of chromosomal DNA. This damage inmost likely repaired though what enzymatic sequence?

Glycosylase, endonuclease, lyase, polymerase, ligase Base excision repair is used to correct single base DNA defects (depurination, alkylation, oxidation, deamination) induced spontaneously or by exogenous chemicals. In this process, glycosylases remove the defective base, and the corresponding empty sugar phosphate site is cleaved and removed by the action of endonuclease (cleave 5' end) and lyase/phosphodiesterase (cleaves 3' sugar phosphate). DNA polymerase then replaces the missing nucleotide and ligase seals the final remaining nick.

Hormones that function by binding to G protein coupled receptors that activate phospholipase C?

GnRH, TRH, angiotensin II, ADH (V1 receptor)

Gonarrhea can cause urethritis (dysuria and purulent urethral discharge). The NAAT is the diagnostic tool of choice. Gram stain showing what is also diagnostic?

Gram negative intracellular diplococci Note: Dual therapy with ceftriaxone and azithromycin is the first line treatment due to concerns about resistance and chlamidya confection

If acid fast bacteria organisms spread along the poses major muscle by direct extension, the patient is at greatest risk for developing an abscess in what site?

Groin The psoas major muscle originates at the T12-L4 vertebrae, passes below the inguinal ligament and over the wing of the ilium, and attaches to the lesser trochanter of the femur. Spread of pus along the psoas muscle may lead to an abscess formation in the groin and pain referred to that region

An agent leads to increased activity of phosphorylase kinase and phosphenolpyruvate carboxykinase in the liver, resulting in rapid elevation of blood glucose levels. This new agent most likely acts through that proteins in hepatocytes?

Gs protein Glucagon binds to G protein coupled receptors on hepatocytes, activating Gs, which stimulates AC to increase cAMP levels. cAMP activates protein kinase A which leads to the activation of key enzymes in glycogenolysis (glycogen phosphorylase, phosphorylase kinase) and gluconeogenesis (pyruvate carboxylase, PEP carboxykinase)

Patient is started on Dobutamine infusion which provides symptoms relief of her orthopnea and dyspnea. Echocardiogram shows improvement in cardiac contractility. Mechanism underlying patients clinical improvement?

Gs protein adenylate cyclase activation Dobutamine is a beta adrenergic agonist with predominant activity on the B1 receptors and weak activity on B2 and alpha1 receptors. Stimulation of B1 receptors leads to increased production of cAMP and increased cytosolic Ca2+ concentration. This facilitates the interaction between actin and myosin resulting in increased myocardial contractility.

15 year old boy with unexplained erythrocytosis on routine lab analysis. Genetic sequencing is performed and results show single base substitution at amino acid position 82 which impairs the ionic interaction between the beta subunit and 2,3-biphosphoglycerate. As a result of this mutation the patient's hemoglobin will be most similar to what hemoglobin type?

Hemoglobin F 2,3-biphosphoglycerate normally forms ionic bonds with the beta submits of deoxygenated hemoglobin A, facilitating oxygen release into the peripheral tissues. Mutations that result in loss of the 2,3-BPG binding pocket's positive charge cause hemoglobin A to resemble fetal hemoglobin which binds oxygen with a higher affinity due to its inability to interact with 2,3-BPG

32 year old man with several months of fatigue and weight loss. Drinks 2 alcoholic beverages a day and has used illicit drugs in the past. Sexually active with his girlfriend. PE is normal. Liver biopsy is obtained and LM shows large hepatocytes filled with finely, granular, homogenous pale pink cytoplasm. Most likely diagnosis?

Hepatitis B infection Hepatitis B infection causes the hepatocellulr cytoplasm to fill with hepatitis B surface antigen. These inclusions are highly specific for hepatitis B infection and have a filly granular, pale eosinophilic, GROUND GLASS appearance Note: hepatitis c commonly has lymphoid aggregates within the portal tracts and focal areas of macro vesicular steatosis, but ground glass hepatocytes are more specific for hepatitis B

34 year old man with 2 days of anorexia, nausea, and dark colored urine. Recently returned from Mexico. Temperature is 100.2. Mild RUQ tenderness on exam. Most likely to be seen on liver biopsy?

Hepatocyte swelling Presentation with low grade fever, anorexia, nausea, dark colored urine, and RUQ pain is highly suggestive of acute viral hepatitis. HAV is most likely responsible as its the most common cause of acute viral hepatitis in young adults and the patients just returned from an endemic region. Acute hepatitis due to most hepatotropic viruses causes hepatocyte ballooning degeneration and apoptosis with mononuclear cell infiltration. Hepatocyte necrosis- cell swelling and cytoplasmic emptying caused by ATP depletion & disruption of intermediate filament network Hepatocyte apoptosis- cellular shrinkage & nuclear fragmentation with intense eosinophilia (Councilman bodies) caused by mitochondrial oxidative damage

Aldolase B deficiency causes hereditary fructose intolerance. This disease manifests after introduction of fructose into the diet with vomiting and hypoglycemia about 20-30 minutes after fructose ingestion. These infants usually present with?

Hepatomegaly, jaundice, failure to thrive

Cushing syndrome caused by a pituitary adenoma or ectopic (paraneoplastic) ACTH secretion is associated with elevated ACTH levels. How to differentiate between these two causes?

High dose dexamethasone suppresses ACTH and cortisol secretion when its caused by a pituitary adenoma (Cushing disease) but not when its caused by ectopic ACTH secretion (small cell lung cancer)

Patients experiencing a major depressive episode should be carefully screened for what before an antidepressant treatment is started?

History of mania Antidepressant therapy should be avoided in patients with bipolar disorder due to risk of precipitating mania. Physicians should question patients regarding a history of distinct periods of elevated mood and increased energy, decreased need for sleep, hyperactivity, racing thoughts, and uncharacteristic risk taking behavior.

46 year old with cervical carcinoma undergoes hysterectomy and bilateral sapling-oophrectomy. Pelvic lymphadenectomy was preformed during which several enlarged nodes around the pelvic vessels were resected. A week after the surgery, she began experiencing left flank pain that radiates to the groin. Temperature is 97. On PE there is a ballotable left flank mass. What most likely accounts for this PE finding?

Hydronephrosis The ureters run in close proximity to the pelvic lymph nodes and the uterine artery in the female pelvis, which predisposes them to injury during pelvic surgery.

Friedrich ataxia is an autosomal recessive condition that presents during childhood/adolescence with progressive gait ataxia (due to generation of the spinocerebellar tracts) and impaired joint and vibration sense (degeneration of posterior columns and dorsal root ganglia). Other common features include?

Hypertrophic cardiomyopathy (most common cause of death), skeletal abnormalities (kyphoscoliosis, pes cavus), and DM

36 year old with 2 weeks of fatigue and easy bruising. Had a URI recently. No lymphadenopathy or hepatosplenomegaly. Labs reveal pancytopenia and a low reticulocyte count. Peripheral blood smear shows normocytic, normochromic RBCs. Other cells types are also morphologically normal. Bone marrow biopsy would show what pattern?

Hypocellular marrow filled with fat cells and marrow stroma Aplastic anemia causes pancytopenia as the bone marrow is replaced by fat cells and macrow storm. The absence of splenomegaly on exam is characteristic. Bone marrow biopsy is extremely useful in making diagnosis. Aspiration reveals dry tap.

In normal female development, non-fusion of the urethral folds forms the vestibule of the vagina. In the male, non-fusion of the urethral folds causes?

Hypospadias (abnormal opening of the urethra proximal to the glans penis along the ventral shaft of the penis) Note: epispadias is an abnormal opening of the urethra on the dorsal surface of the penile shaft that results from faulty positioning of the genital tubercle in the 5th week of gestation Note: bifed scrotum (two separate sacs) results from malunion of the labioscrotal folds, which form the labia majora in females

5 year old boy with severe recurrent respiratory infections. Its determines that a specific type of bacteria causes his infections and that his T-lymphocytes lack IL-12 receptor. Supplementation with what substance would most likely improve this patient's condition?

IFN-gamma IL-12 stimulates the differentiation of naive T helper cells into Th1 subpopulation. Patients with IL-12 receptor deficiency suffer from severe mycobacterial infections due to the inability to mount a strong cell mediated granulomatous immune response. They are treated with IFN-gamme.

Substance that promotes B-lymphocyte class switching to synthesize IgE?

IL-4 Sensitized Th2 ells secrete IL-4 and IL-13 which together promote B-lymphocyte class switching for IgE synthesis. They also secrete IL-5 which activates eosinophils and promotes IgA synthesis. An excess of these Th2 produced lymphokines may contribute to the pathogenesis of extrinsic allergic asthma. IL-1 is secreted by macrophages to stimulate helper T cells. IL-3 from helper T cells recruits bone marrow stem cells. Interferon gamma from helper T cells functions to activate macrophages. TGF-beta is a growth factor involved in tissue regeneration and repair.

22 year old woman with nosebleed. Similar episode yesterday, but bleeding stopped with prolonged local pressure. Patient has had easy bruising for the past several months. Liver span is 8cm and spleen not palpable. Scattered ecchymosis noted on arms and legs. Marked thrombocytopenia is noted on labs. Normal hematocrit, leukocyte count & differential, fibrinogen level, PT and INR. Most likely primary mechanism causing patients condition?

Immune destruction of platelets Immune thrombocytopenia purpura is characterized by autoimmune destruction of platelets by anti-platelet antibodies (IgG antibodies against GPIIb/IIIa). In children, its acute and self limited, but in adults it runs an insidious and chronic course. Peripheral blood smear with isolated thrombocytopenia and no other platelet abnormalities can help confirm diagnosis (megakaryocytic can sometimes be seen). Treat with immunosuppression (corticosteroids). Autoimmune platelet destruction is a common cause of thrombocytopenia and should be suspected in patients with ecchymosis, petechiae, mucosal bleeding, and no other obvious causes of thrombocytopenia (medications, bone marrow failure).

42 year old man with long history go T1DM presents due to frequent involuntary loss of urine. Has been having difficulty starting and maintaining a urinary stream. In the last few weeks he's had 2 episodes of nocturnal enuresis and multiple daytime episodes of uncontrolled voiding without any sensation of a full bladder. Most likely additional finding to be present in this patient?

Increased postvoid residual volume Patients symptoms are suggestive of overflow incontinence which is due to impaired detrusor contractility or bladder outlet obstruction. Diabetic autonomic neuropathy is common in T1DM and can cause overflow incontinence due to inability to sense a full bladder and incomplete emptying. Postvoid residual testing with US or catheterization can confirm inadequate bladder emptying

Affect of normal saline infusion on a pressure volume loop in the left ventricle on systole and diastole?

Increased ventricular preload & causes a rightward widening of the pressure volume loop

63 year old man diagnosed with DVT of right popliteal vein. A filter placement is planned to prevent embolization. Filter will most likely be placed in what structure?

Inferior vena cava The IVC courses through the abdomen and inferior thorax in a location anterior to the right hand of the vertebral bodies. The renal veins join the IVC at the level of L1/L2, and the common iliac veins merge to become the IVC at the level of L5. IVC filters are placed in patients with DVT who have contraindications to antigoaculation therapy

59 year old man with unintentional weight loss. Reports epigastric discomfort after meals with occasional nausea and that food is not as appetizing as it used to be. Upper endoscopy shows signet ring cells. The lesion most likely demonstrates?

Infiltrative growth within the stomach wall There are 2 morphological variants of gastric adenocarcinoma. The intestinal type forms a solid mass that projects into the stomach lumen and is composed of glandular forming cuboidal or columnar cells. In contrast, diffuse (due to loss of E-cadherin) carcinoma ('leather bottle stomach'/linitis plastica) infiltrates the stomach wall and displays signet ring cells on LM. Prognosis depends on depth of invasion and regional lymph node involvement. Metastasis is often present at the time of diagnosis and most commonly involves the supraclavicular lymph node (Virchow node) and periumbilical nodes (Sister Mary Joseph nodule)

How does Omeprazole inhibiting substance flow across cell membranes?

Inhibits primary active transport Parietal cells release hydrogen ions into the gastric lumen by the means of H/K ATPase which requires hydrolysis of ATP and is therefore an active transport mechanism. Omeprazole and other PPIs suppress the activity of the gastric parietal cell H/K ATPase leading to an increase in the pH gastric lumen

Patient with increased anion gap metabolic acidosis is treated appropriately and within several hours his mental status improves significantly. Labs show an increase in serum bicarbonate and Na+ levels, a decrease in serum osmolality, and a drop in serum K+ level. What treatment was most likely given to this patient?

Insulin and normal saline Based on lab changes, this patient with an increased anion gap metabolic acidosis was most likely suffering from DKA. Patients classically have fruity odor on breath & present with mental status changes, dehydration, abdominal pain, and tachypnea. Labs include hyperglycemia, ketosis, mild hyponatremia, normal or elevated serum K+ (despite a total body deficit), and increased plasma osmolality. Insulin allows cells to use glucose for energy, decreasing lipolysis and production of ketone bodies. Ketones are the principal acid produced in DKA, so decreased ketones will increase serum bicarb. Insulin causes K+ to shift into cells, decreasing serum K+ level. Rehydration helps normalize Na+ and decrease osmolality.

37 year old HIV positive main with confusion and sleep disturbances. He is a known IV drug abuser and tests positive for HBV and HCV. PE shows yellow sclerae, abdominal distension with fluid wave, and ankle edema. Jerky, involuntary hand movements are asks present. What is most likely to improve this patients condition?

Intestinal content acidification Lactulose is useful in treating hepatic encephalopathy because it acidifies the contents of the GI tract converting NH3 to NH4+.

32 year old woman has a mass resected from her liver. Morphology is demonstrated on picture (cavernous blood filled vascular spaces of variable size lined by a single epithelial layer). What is an accurate statement regarding this condition?

It is the most common benign liver tumor Cavernous hemangioma is the most common benign liver tumor typically presenting between ages 30-50. They are congenital malformations that enlarge by ectasia. Microscopically, they consist of cavernous, blood filled vascular spaces of variable size lined by a single epithelial layer. Collagenous scars or fibrous nodules may be seen in associated with thrombosis. Biopsy of a suspected hemangioma is not advisable as it can cause fatal hemorrhage and is of low diagnostic yield.

Patient is started on a medication to improve growth and normalize height. What intracellular pathway is stimulated by the medication used in this patient?

JAK-STAT pathway Growth hormone binds to cell surface receptors, leading to intracellular activation of the JAK-STAT pathway. Cytokines (interferon) and hematopoietic growth factors (erythropoietin, G-CSF) also use this pathway.

Characterized by inability to resist the impulse to steal objects that are of low monetary value or not needed for personal use. Overwhelming feelings of tension or anxiety precede impulses and are relieved with act of theft

Kleptomania

Punch biopsy of a normal appearing epidermis reveals stellar cells with characteristic intracytoplasmic granules having the shape of a tennis racket. These cells demonstrate some myeloid surface markers and can interact closely with T lymphocytes. These cells are referred to as?

Langerhans cells (dendritic cells found in the skin that act as antigen presenting cells; these cells are derived from the myeloid cell line and possess characteristic racquet shaped intracytoplasmic granules known as Birbeck granules)

21 year old suffered an injury during a football and is found to have a fracture of the left 12th rib. What structure is most likely to be lacerated by the fractured bone?

Left kidney (lies immediately deep to the tip of the 12th rib) Note: 9, 10, 11 ribs overly the spleen on the left 8-11th ribs overlie the liver's posterior surface

70 year old man with history of HTN presents with severe mid back pain that started several hours ago. TEE shows a dissection flap in the descending aorta, but no evidence of dissection in the ascending aorta. The dissection flap in this patent most likely originates near what point?

Left subclavian artery (patient has stanford type B aortic dissection) Stanford A- aortic dissections that involve any part of the ascending aorta Stanford B0 all other dissections involving the descending aorta The intimal tear in stanford type A aortic dissection (involving the ascending aorta) usually originates in the sinotubular junction whereas the intimal flap in stanford B aortic dissection usually starts near the origin of the left subclavian artery. Dissections can propagate distally to the thoracoabdominal aorta.

Spontaneous deep intracerebeal hemorrhage is typically caused by hypertensive vasculopathy involving the penetrating branches of them major cerebral arteries. The most frequently affected locations include the basal ganglia (putamen), cerebellar nuclei, thalamus, and pons. Basal ganglia is supposed by what arteries which are small vessel branches off the middle cerebral artery?

Lenticulostriate arteries

36 year old woman presents for follow up of mood disorder. Her depressive and manic episodes have responded well to treatment and she has remained on the same drug regimen for years. However she now has new onset constipation, dry skin and hair loss. She also gained 5 pounds over the last 3 months despite eating healthy. She appears tired but otherwise PE is normal. Diagnosis is established and her condition is attributed to adverse effect from one of her medication. Most likely culprit drug?

Lithium Hypothyroidism nd nephrogenic diabetes insipidis are the most common adverse effects of long term lithium therapy. Serum TSH and renal function (BUN and creatinine) should be monitored routinely. Lithium induced hypothyroidism is treated with levothyroxine and doesn't require discontinuation of lithium therapy

Cross section of a RCA of a 34 year old who died in cardiac intensive care unit shows occlusion by a large atheromatous plaque with a soft yellow lipid core. His father died of a heart attack at age 40. Assuming patients disease had a genetic basis, poor receptor expression in what organ is responsible for his condition?

Liver Familial hypercholesterolemia, one of the most common autosomal dominant disorders, is the result of heterozygous or homozygous LDL receptor gene mutations which cause hepatocyte under expression of functional LDL receptors. This can lead to accelerated atherosclerosis and early onset coronary artery disease

46 year old male with abdominal pain is diagnosed with a rare vascular tumor. This tumor is oftentimes associated with past arsenic or polyvinyl chloride exposure. Immunohistochemical staining of the tumor cells is positive for the CD31 marker. Most likely condition?

Liver angiosarcoma Hepatic angiosarcoma is associated with exposure to carcinogens such as arsenic, thorotrast, and polyvinyl chloride. Tumor cells express CD31 (which is PECAM1), an endothelial cell marker.

25 year old diagnosed with osteosarcoma and has a history of enucleation of his right eye due to retinoblastoma. What mechanism is the most likely underlying cause of his condition?

Loss of heterozygosity Retinoblastoma is a childhood tumor that results from separate mutagenic events that inactivate both copies of the RB1 tumor suppressor gene (one copy on each chromosome 13). Germline mutations in one RB1 gene cause 'hereditary' retinoblastoma which is associated with the development of other primary tumors (such as osteosarcoma) later in life. As a result their cells have the first hit required for tumor formation and an additional somatic mutation (loss of heterozygosity) early in life provides the second hit for malignant transformation

43 year old man with several months of water diarrhea shortly after eating, occasional abdominal discomfort, easy fatiguability, and unintentional weight loss. Has a history of Crohn disease for which he underwent several intestinal resections. PE shows conjunctival pallor. Vibratory and position sense are decreased in LE. Labs show macrocytic anemia and low B12 level. Most likely cause of his presenting symptoms?

Loss of intestinal absorptive area Short bowel syndrome typically occurs in patients with massive small bowel resection and/or Crohn disease due to loss of intestinal absorptive surface area and a decrease in intestinal trasnit time. Patients usually present with postprandial voluminous diarrhea and weight loss due to malabsorption. Loss of functional distal ileum may result in B12 deficiency.

Acidification of lysosomes within APCs is prevented in an experiment. The affected cells show impaired interaction with T lymphocytes upon antigen exposure. Observed effect most likely results from a low cell surface expression of what molecule?

MHC class II MHC class II is expressed on the surface of APCs and functions by presenting antigen that is foreign to the body. This antigen is taken into the APC by phagocytosis/endocytosis and is loaded onto MHC class II within acidified endosomes, and the MHC class II protein antigen complex is then expressed on the cell surface for subsequent interaction with T lymphocytes. Failure to acidify lysosomes would lead to deficient expression of MHC class II bound to foreign antigen and subsequent lack of interaction between APCs and T cells. Note: MHC class I molecules are never processed within acidified lysosomes

Acanthosis nigricans is usually associated with insulin resistance or obesity, but sudden appearance or rapid spread can signal the presence of?

Malignancy in the GI tract or lungs

Proliferation of highly atypical and pleomorphic fibroblasts, histiocyte like cells and bizarre giant cells

Malignant fibrous histiocytoma

34 year old develops severe chest and abdominal pain. He suffered cardiac arrest with pulseless electrical activity and can't be resuscitated. Postmortem exam reveal internal hemorrhage as cause of death. Histochemical evaluation reveals a defect affecting a large extracellular glycoprotein that is normally found in large blood vessels, periosteum and zonular fibers of the lens and functions to form microfibrils by surrounding elastin. Patient suffered from what condition?

Marfan syndrome Marfan syndrome is due to a defect in fibrillin-1 an extracellular glycoprotein that acts as a scaffold for elastin. Its abundant in the zonular fibers of the lens, periosteum and aortic media. Aortic root dilation with dissection and rupture is the most common cause of death.

18 year old suddenly collapses during high school soccer game. He expires before medical team arrives. Had recently experienced occasional exertion related chest discomfort and dyspnea, otherwise no significant medical history. Most likely to be seen at autopsy?

Massive cardiac hypertrophy Hypertrophic cardiomyopathy is the most common cardiovascular cause of sudden cardiac death in athletes less than age 35. Histologic features include massive myocyte hypertrophy predominantly affecting the septal region and myofiber disarray (irregular arrangement of abnormally shaped myocytes with bizarre nuclei and areas of increased connective tissue).

45 year old man with sudden onset vomiting and severe abdominal pain that radiates to his back. AST is 89 and ALT is 31. Serum lipase is markedly increased. Abdominal ultrasound is reveals normal gallbladder and common bile duct. Diagnosis of acute pancreatitis is made and etiology is established. What additional lab finding is most specific for the underlying cause of his pancreatitis?

Mean corpuscular volume of 108 After gallstones, alcohol abuse is the second most common cause of acute pancreatitis. Macrocytosis an AST:ALT ratio >2 are indirect indicators of chronic alcohol consumption. Alcohol related macrocytosis can occur independently of folate deficiency

Median vs. Mode vs. Mean

Mean- add all numbers and divide that sum by the number of observations Median- value located in the center of the dataset (if even amount of numbers add two middle numbers and divide by 2) Mode- most frequent value

21 year old male presents following an episode of syncope that was not provoked by any activity nor preceded by lightheadedness. ECG reburials QT interval prolongation. Assuming this is an inherited condition, the relevant mutation most likely affects what structure?

Membrane potassium channel proteins Unprovoked syncope in previously asymptomatic young person may result from congenital QT prolongation syndrome. The two most important congenital syndromes with QT prolongation, Romano-Ward syndrome (autosomal dominant & no deafness) and Jervell and Lange-Neilsen syndrome (autosomal recessive with neurosensory deafness) are thought to result from mutations in K+ channel protein that contributes to the delayed rectifier current of the cardiac action potential. Both of these conditions predispose to torsades and syncopal episodes & possible sudden cardiac death. Note: HCM can present as syncope in a previously asymptomatic young person, but the syncope is provoked by exertion & QT prolongation is not found in HCM

Common cause of nephrotic syndrome that is associated with solid tumors (lung and colon carcinoma)?

Membranous glomerulonephritis Patients present with edema and weight gain due to severe proteinuria. Histologically, the glomerular capillary loops appear thickened and demonstrate glomerular BM 'spikes' as a result of subepithelial immune complex deposition. The immune complex deposits don't take up methamine silver stain and form pale areas with intervening regions of darkly staining BM that appear as irregular spikes

60 year old man with chest pain. Has had intermittent squeezing, substernal pain over the last 3 days that is now sustained. ECG shows sinus rhythm with 2mm ST elevation in leads II, III, aVF. Cardiac troponin I levels are elevated. Most appropriate treatment for this patient?

Metoprolol Beta blockers are used in acute MI to reduce HR, CO, and myocardial oxygen demand. Non-cardioselective beta blockers (propranolol, nadolol) can trigger bronchospasm in patients with underlying asthma or COPD Cardioselective beta blockers with predominant action on beta 1 receptors are preferred in such patients (metoprolol, atenolol, bisoprolol, nebivolol) Combined beta and alpha receptor blockers (carvedilol, labetalol) are well tolerated and have been used safely in patients with COPD

7 year old boy who had bloody diarrhea last week that has since resolved. Currently, labs show acute renal failure. A CBC shows anemia and thrombocytopenia. Coagulation studies are WNL. Peripheral blood smear shows schistiocytes. What is the most likely cause of his anemia?

Microangiopathic hemolytic anemia Schistiocytes suggest microangiopathic hemolytic anemia (HUS, TTP, DIC, mechanical damage). In childhood, HUS is often preceded by bloody diarrhea. PT and PTT are normal in HUS-TTP but abnormal in DIC.

Fetal ultrasound demonstrates ascites, large bilateral pleural effusions, and a small pericardial effusion. Parents who recently immigrated from Thailand say that both families have a history of blood disorders. Labs confirm diagnosis and parents are counseled regarding the fatal prognosis of the condition. A maternal blood smear is most likely to demonstrate what finding?

Microcytes Patients with alpha thalassemia minor have mild anemia with hypo chromic and microcytic cells. However, offspring of two parents with alpha thalassemia minor are at risk of absent alpha glib production, the most extreme form of alpha thalassemia, which leads to hemoglobin Barts formation (four gamma chains) and death in utero from hydrops fetalis (anasarca, ascites, effusions). The four gamma chain hemoglobin has extremely high oxygen affinity and can't deliver oxygen to tissues.

44 year old man with HIV presents for progressive cognitive decline. He has had worsening memory and difficulty carrying out simple mental tasks for the last few years. He has been noncompliant with HIV treatment. What histopathlogic finding would most likely be seen on this patients brain?

Microglial nodules HIV associated dementia should be suspected in AIDS patients with progressive cognitive decline. Patients typically have features of subcortical dementia: attention/working memory problems, executive dysfunction, slow information processing as HIV affects primarily the subcortical/deep gray matter structures. The characteristic histopathologic finding is microglial nodules, groups of activated macrophages/microglial cells formed around small areas of necrosis that may fuse to form multinucleated giant cells

PGE1 analog that binds to plasma membrane prostaglandin receptors. In the upper GI tract, ti stimulates gastric epithelial mucous production and appears to decreased parietal cell acid secretion. Can be used to help prevent and treat NSAID induced peptic ulcers

Misoprostol

What ultrastructural change most likely indicates irreversible myocardial cell injury?

Mitochondrial vacuolization Appearance of vacuoles and phospholipid containing amorphous densities within mitochondria signifies irreversible injury and implies a permanent inability to generate further ATP via oxidative phosphorylation (cell cannot recover if mitochondria are irreversibly injured). Simple mitochondrial swelling can be associated with reversible cell injury

Synthesized by eosinophils and mast cells, leukotriene D4 plays an important role in bronchial asthma pathogenesis by inducing bronchospasm and increasing bronchial mucous secretion. Anti-leukotriene medication that antagonizes leukotriene D4 activity at the cysteinyl-leukotriene receptor?

Montelukast

44 year old man with depression has not experienced any improvement with antidepressants. Patient declines ECT and asks to try another medication. Physician considers phenelzine. Presence of what additional symptom would male this an appropriate medication for the patient?

Mood reactivity Monoamine oxidase inhibitors (phenelzine, tranylcypromine, selegiline) are particularly useful in patients with treatment resistant and atypical depression. Increased appetite and sleep, leaden paralysis (heavy arms and legs), rejection sensitivity (overly sensitive to slight criticism), and mood reactivity (feeling better in response to positive events) are hallmarks of atypical depression. MAOIs work by inhibiting the oxidative deamination of serotonin, NE, and dopamine. They have severe adverse effects such as serotonin syndrome and hypertensive crisis.

44 year old man with 3 day history of fever, chills, malaise, dyspnea, and cough productive of 'greenish' sputum. CXR shows dense infiltrate in the right lower lobe. What accounts for the color of the patients sputum?

Myeloperoxidase The green discoloration of pus or sputum seen during common bacterial infections is due to the presence of myeloperoxidase, a blue-green heme based enzyme thats released from neutrophil azurophilic granules and forms hypochlorous acid (bleach)

3 month old with 2 days of fever and irritation that progressed to lethargy. Gram negative coccobacilli are isolated from the CSF. Organisms demonstrate little growth on 5% sheep blood agar but grow well when incubated on the same bed alongside S. aureus. Staphylococci promote the growth of these bacteria by supplementing what substance?

NAD+ H. influenzae is a 'blood-loving' organism that requires X (hematin) and V (NAD+) factors for growth. This can be accomplished by growing H. influenzae in the presence of S. aureus and demonstrating the 'satellite phenomenon', where H. influenzae grow only near the beta hemolytic S. aureus colonies that produce the needed X and V factors in sheep blood agar

11 month old boy with irregular jerking movements. Mother states he has looked pale and tired for 2 weeks and has been more fussy and less playful than usual. PE reveals spontaenous burns of nonrhythmic conjugate eye movements in various directions. Patient also has myoclonus involving the trunk and limbs as well as generalized hypotonia. Right sided contender, immobile abdominal mass is palpated. No hepatosplenomegaly, and bowel sounds are normal. CBC reveals anemia and urine is positive for elevated levels of catecholamine breakdown products. Most likely diagnosis?

Neuroblastoma Opsoclonus-myoclonus is a paraneoplastic syndrome associated with neuroblastoma, the most common extracranial solid neoplasm in children. The tumor typically arises from neural crest cells of the adrenal medulla or sympathetic ganglia and presents with an abdominal mass and elevated catecholamine breakdown products. Associated with periorbital ecchymosis and N-myc gene amplification (poor prognosis). Small round blue cells on histology.

32 year old man with sudden onset of severe right flank pain that radiates to the groin. Has had gross hematuria but no fever or dysuria. Imaging shows a stone in the middle of the right ureter. Most likely to be seen on labs?

Normocalcemia, hypercalcuria Calcium stones represent 7-080% of all renal calculi and include calcium oxalate and calcium phosphate stones. Hypercalciuria is the most common risk factor for development of Ca2+ stones in adults. In most patients the hypercalcuria is idiopathic and thought to be due to increased GI absorption, increased mobilization of Ca2+ from bone, or decreased renal tubular Ca2+ reabsorption. These patients remain normocalemic due to intact regulation of serum Ca2+ levels by vitamin D and PTH. Other risk factors for calcium nephrolithiasis that are less common: hyperoxaluria, hyperuricosuria, low urine volume, hypocitrauria

42 year old Caucasian male has been 'losing his temper' and has threatened to kill his wife several times. Wife says he seems to grimace involuntarily and his extremities move without control. Patient's father died of a neurological disorder at 55. Patient most likely has neuron damage in what area?

Nucleus caudatus Los sof neurons in the caudate nucleus and putamen is characteristic of Huntington disease.

Only major derivative of the notochord (mesodermal derived structure) in humans?

Nucleus pulpous of the intervertebral disc

Intraabdominal malignancies can metastazie via lymphatic channels, which travel alongside an organs respective blood supply. The thoracic duct receives all lymphatic draining from the abdominal viscera, and lymph from this duct is sampled by the left supraclavicular node. Enlargement of this node (Virchow's) may signify?

Occult abdominal malignancy

62 year old woman with decreased vision. PE shows right homonymous hemianopia. When light is shone in the left eye, both pupils constrict. When light is immediately moved to the right eye her pupils appear to dilate. Patients symptoms are most likely due to a left sided lesion involving what structure?

Optic tract A lesion of the optic tract can produce contralateral homonymous hemianopia and a relative afferent pupillary defect (Marcus Gunn pupil) in the pupil contralateral to the tract lesion. Pupillary light reflex involves the retina, optic chasm, optic tract fibers, and pretetal nucleus in the midbrain. Optic tract lesions can produce a relative afferent pupillary defect in the pupil contralateral to the tract lesion as the nasal portion of the retina contributes more input to the pretetal nucleus than the temporal portion of the retina. This is observed with the swinging flashlight test because the patients pupils constrict less (and appear to dilate) when a bright light is swung to the pupil contralateral to the lesion Note: lesions in the lateral geniculate nucleus or optic radiation can produce a contralateral homonymous hemianopia, but the pupillary light reflexes would be normal.

Patient with three month history of productive cough, night sweats, and low grade fever. Sputum cultures grow budding yeast that form germ tubes at 37 degrees C. Most likely site of this organism before entering the sputum is?

Oral cavity Candida albicans is a normal inhabitant of the GI tract (including oral cavity) in up to 40% of the population. Thus, its a common contaminant of sputum cultures. The presence of Candida in sputum doesn't indicate disease. In fact, this patients presentation is more consistent with TB.

40 year old man with end stage renal disease due to T1DM is hospitalized for initiation of hemodialysis. Patient develops bleeding around the catheter exit site thats difficult to control. Has not been treated recently with any anticoagulants. Further evaluation would show what lab abnormalities?

PT- normal aPTT- normal Platelet count- normal Bleeding time- prolonged Excessive bleeding is common in patients with significant renal dysfunction due in part to accumulation of uremic toxins in the circulation. These toxins impair platelet aggregation and adhesion resulting in a qualitative platelet disorder characterized by a prolonged bleeding time, normal platelet count, normal PT and normal aPTT. Uremic bleeding can be improved with dialysis as it removes toxins and partial reverses the bleeding abnormality.

65 year old man with non healing ulcer on right foot. PMH significant for recurrent high blood glucose readings over the last several years, but he has failed to comply with appropriate treatment. BMI is 37. PE is significant for bilateral symmetric decreased in vibration over feet and ankle. Most likely to be associated with patient's condition?

Pancreatic amyloid deposition Pancreatic islet amyloid deposition is characteristic of type II diabetes mellitus. Two cardinal defects involved in T2DM are increased insulin resistance and defective insulin secretion (possibly due to amylin deposition) A strong linkage with HLA class II (HLA-DR & HLA-DQ) gene makeup, pancreatic islet infiltration with leukocytes (insulitis) and antibodies against islet antigens are frequently seen in type 1 DM.

Hemophilia is an X linked recessive coagulopathy that presents with intramuscular hemorrhage, hemarthroses, and DELAYED bleeding after procedures (initial platelet plug formation is intact). Lab results show?

Partial thromboplastin time prolongation & other tests of hemostatic function are generally normal

Small for gestational age infant is born prematurely to a 38 year old woman with inconsistent prenatal care. PE shows small head and eyes and cleft lip and palate. There is a round punched out lesion with an overlying thin membrane on the patients scalp. A small membranous sac with a loop of bowel protrudes from patients abdominal midline. Most likely responsible for patients condition?

Patau syndrome Trisonomy 13 usually occurs secondary to meiotic nondisjunction in mothers of advanced maternal age. Key physical findings reflect defective prechordal mesoderm fusion resulting in midline defects (holoprosencephaly, microophthalmia, cleft lip/palate, omphalocels) as well as polydactyly and cutis aplasia

24 year old African American woman presents with husband for prenatal counseling. She has a three year old with sickle cell anemia from a previous marriage. She remarried last year and wants to know changes of their child having sickle cell anemia. Neither she nor her husband have sickle cell anemia. Best initial test that can be offered to this couple?

Paternal hemoglobin electrophoresis Sickle cell anemia is an autosomal recessive hemolobinpoathy. In order for a child to have the disease, both parents must be carriers. We already know mom is a carrier as she has a child with the disease. Hemoglobin electrophoresis can be used to determine the carrier status of a prospective parent who has no history of sickle cell anemia. Abnormal hemoglobin moves at a slower speed than the normal hemoglobin due to the replacement of glutamic acid by valine.

Patient with hyperthyroidism is treated with multiple drugs including propanalol. In addition to its beta adrenergic receptor blocking activity, this drug is likely to decrease?

Peripheral conversion of T4 to T3 Note: hyperthyroidism causes up regulation of beta adrenergic receptor expression leading to increased catecholamine effect manifesting as HTN, palpitations/tachycardia, heat intolerance, tremor, hyperreflexia

12 year old boy diagnosed with Neisseria meningitidis. Parents are concerned about their younger son who shares a bedroom with the patient. Most appropriate management of the patient's household contacts?

Prophylactic rifampin Rifampin is the most frequently used agent for chemoprophylaxis of meningococcal disease (give as soon as possible, preferably within 24 hours of diagnosis). Persons with exposure to the respiratory secretions of a patient with menigiococcal disease require chemoprophylaxis. Vaccination is an important public health strategy but isn't useful for post-exposure prophylaxis & a history of meningococcal vaccination doesn't preclude the need for antibiotic chemoprophylaxis.

37 year old with infertility. A serum analysis shows normal sperm count but completely immobile sperm due to abnormal tail function. What additional finding is most likely associated with this patients condition?

Persistent bronchial dilation Primary ciliary dyskinesia results from an autosomal recessive mutation in the proteins responsible for normal flagellar and ciliary striation and function (dynein, assembly proteins). Clinical manifestations include situs inversus (reversed right/left positioning of internal organs), chronic sinusitis, bronchiectasis (permanent abnormal airway enlargement), and infertility. Triad of situs inversus, chronic sinusitis, bronchiectasis is KARTAGENER syndrome.

8 year old boy with 2 days of worsening headaches, vomiting, and fever. Cries when lights are turned on in the exam room. Temperature is 102.6. PCR of CSF shows a positive sense single stranded RNA virus. Virus is most likely to belong to what family?

Picornavirus Enteroviruses, which are members of the Picornavirdae family, are the most common cause of viral meningitis. All viruses in PIcornavirdiae have a positive sense single stranded RNA genome

Following IV administration, a highly lipophilic drug will be rapidly distributed to organs with high blood flow (brain, liver, kidneys, lungs, heart). The drug is then redistributed to tissues with relatively lower blood flow (skeletal muscle, fat, bone). This accounts for the short duration of action of many commonly used anesthetics, such as?

Propofol

67 year old unresponsive. Developed sudden onset weakness in extremities and face following a heated argument and within minutes became unresponsive. Patient has history of HTN and hasn't been taking his medication regularly. BP is 188/104. PE shows comatose patient with pinpoint pupils and rigid, extended upper and lower extremities. Dies 2 hours later. ICH most likely located in what area?

Pons Pontine hemorrhages typically cause pinpoint pupils, loss of horizontal gaze, quadriparesis, decerebrate posturing, and rapidly evolving coma that culminates in death within hours

17 year old boy sustains posterior displacement of the tibia relative to the femur. What structure is most likely to be injured in this patient?

Popliteal artery Injury to the popliteal artery is the primary concern with bother anterior and posterior dislocations of the knee joint. This vessel is rigidly fixed proximal and distal to the knee joint by the adductor magnus and soleus muscles, respectively, making is susceptible to tearing by traction forces. Note: the tibial nerve courses through the posterior fossa posterior (superficial) to the popliteal artery and is not subject to the same sort of forces as the popliteal artery, although it is vulnerable to penetrating trauma to the popliteal fossa. Popliteal vein courses superficial to the popliteal artery as is also less commonly injured.

6 year old boy presents with difficulty hearing. No ear pain, discharge or URI symptoms. Initial testing suggests bilateral sensorineural hearing loss. Paternal uncle died suddenly at age 12. ENT exam is normal. ECG shows NSR and prolonged QT interval at 520 msec. Echocardiogram shows normal left and right ventricular function without valve disease. Genetic defect affecting what is likely present in this patient?

Potassium channels Jervell and Lange-Nielsen syndrome is an autosomal recessive disorder characterized by profound bilateral sensorineural hearing loss and congenital long QT syndrome, which predisposes to ventricular arrhythmias and sudden cardiac death. Occurs secondary to mutations in genes (KCNQ1, KCNE1) that encode voltage gated K+ channels.

6 year old male with abdominal pain, nausea, and vomiting. He has been urinating more than usual and seems to be breathing heavy. UA is positive for ketones. What pattern of serum abnormalities do you expect?

Potassium- elevated Sodium- decreased Glucose- increased Metabolic derangements associated with DKA include metabolic acidosis, ketonemia, hyperglycemia, hyperkalemia, and hyponatremia. Hyperammonemia can be observed due to muscle degradation. Hyperkalemia occurs because academia drives cells' K+/H+ exchange and insulin's tendency to drive K+ into cells is missing. Total body K+ stores are actually depleted due to urinary losses and GI losses (vomiting). Serum sodium is decreased because of the osmotic activity of glucose and osmotic diuresis. Blood levels of NH3 are decreased due to proteolysis under the influence of counterregularoty hormones such as glucagon, epinephrine, and cortisol

Stages of behavioral change/ transtheorectical model?

Precontemplation- not thinking about behavior modification; denial of the problem Contemplation- thinking about behavior modification; acceptance of the problem Preparation- planning behavior modification Action- putting plan into action Maintenance- maintaining new behavior

Defined as HTN onset after 20 weeks gestation plus proteinuria or signs of end organ damage. End organ damage is due to endothelial damage secondary to the release of inflammatory factors from abnormal placentation

Preeclampsia

Carpal tunnel syndrome is associated with conditions that reduce the carpal tunnel space such as?

Pregnancy (fluid accumulation), hypothyroidism (glocosaminoglycan buildup), DM (connective tissue thickening), rheumatoid arthritis (tendon inflammation), longstanding hemodialysis (deposition of B2 micro globulin- dialysis associated amyloidosis)

8 year old boy with diphtheria. Several days after being admitted to the hospital, he dies of myocarditis and severe HF. What would have prevented his death?

Presence of IgG against circulating proteins Corynebacterium diphtheriae causes diphtheria, an acute bacterial disease that initially infects the oropharynx. The organism is spread by respiratory droplet transmission and causes disease via is A/B exotoxin. The B (B for binding) subunit allows penetrations of the A (A for active) subunit into the cell to inhibit ribosome function. Neural and cardiac toxicity are serious potential sequelae. Immunization with diphtheria toxoid induces production of circulating IgG against the exotoxin B subunit, effectively preventing disease

The vascular endothelium secretes what substance to inhibit platelet aggregation?

Prostacyclin (prostaglandin I2) Prostacyclin is produced by prostaglandin H2 by prostacyclin synthase in vascular endothelial cells. When secreted, it vasodilates, inhibits platelet aggregation, and increases vascular permeability. The functions of prostacyclin oppose those of thromboxane A2. These substances work in concert to ensure vascular hemostasis.

Bacterial cells are lysed and their DNA is extracted and purified. Analysis of the partially replicated DNA fragments shows the presence of uracil. This finings is most likely mediated by what enzyme?

Primase Question describes a scenario in which uracil is found in association with bacterial DNA during prokaryotic DNA replication. In general uracil is only found in RNA so the question essentially asked which enzyme involved in DNA synthesis catalyzes the formation of RNA strands. In prokaryotic DNA replication, primase (a DNA dependent RNA polymerase) is responsible for synthesizing a short RNA primer using the separated strands of DNA at the replication fork as templates.

PCR is performed. The procedure involves heating the solution to 203 degrees followed by cooling the solution to 131. Immediately upon cooling what happens?

Primer binding to DNA strand Thermal cycling PCR involves heating for DNA strand denaturation, coding for primer hybridization, and rewarming for primer extension and DNA synthesis

What two drugs have the highest risk of causing drug induced SLE, which is characterized by development of lupus like symptoms in addition to positive ANA and anti-histone antibodies?

Procainamide and hydralazine Procainamide is metabolized via hepatic acelyation. Individuals who are slow acetylators are at greatest risk for drug induced SLE Note: Unlike with SLE, anti-dsDNA antibodies are rarely seen

Immature defense mechanism involving misattribution of ones unacceptable feelings or thoughts to another person who doesn't actually have them

Projection

39 year old paraplegic with an indwelling bladder catheter is diagnosed with a UTI. Blood cultures grow non-lactose fermenting, oxidase positive, gram negative rods. Most likely pathogen?

Pseudomas aeruginosa Patients with indwelling bladder catheters are at risk for both typical (E. coli, Klebsiella, S. saprophyticus, Proteus) & opportunistic (Pseudomonas, Enteroccoccus, other Staph, fungal) organisms.

38 year old man with exertion dyspnea and cough. Symptoms started 6 months ago and have progressively worsened. PMH includes rheumatoid arthritis. He has taken a medication for RA for many years but stopped a year ago as the drug failed to improve his worsening arthritis. CXR shows dense reticulonodular opacities. Most likely explanation for his pulmonary symptoms?

Pulmonary fibrosis Pulmonary fibrosis presents with gradual onset progressive dyspnea, nonproductive cough, fatigue, eventual weight loss, and bilateral reticulonodular opacities on CXR. Auscultation reveals end inspiratory crackles. PFTs reveal a restrictive pattern with decreased lung volumes, decreased FEV1 and FVC levels, a normal or increased FEV1/FEV ration, and decreased diffusion capacity of carbon monoxide. Progressive fibrosis can lead to mystically dilated bronchioles that later coalesce to form the honeycomb appearance found in advanced disease. Patients with RA can develop interstitial lung disease both from the pulmonary manifestations of the disease itself and from certain therapies (methotrexate, cyclophosphamide, sulfalazine)

34 year old female with fatigue and progressive exertion dyspnea. Lung auscultation is normal. Echocardiogram shows enlarged coronary sinus. What is the most likely cause of the observed finding in this patient?

Pulmonary hypertension Most of the venous drainage from the myocardium traverses the coronary sinus which delivers deoxygenated blood to the right atrium. The coronary sinus communicates freely with the right atrium and will become dilated secondary to any factor that causes increased right atrial pressure. The most common cause is pulmonary hypertension, leading to elevated right heart pressures

Wilson disease can cause cystic degeneration of what brain structure?

Putamen (as well as other basal ganglia structures). Putamen is located medial to the insula and lateral to the globus pallidus on coronal sections

Characterized by intentional and repeated fire setting with no obvious motive; doesn't involve other behaviors seen in conduct disorder (lying, theft, cruelty to others).

Pyromania Note: a separate diagnosis of pyromania is not given when fire setting occurs as a part of conduct disorder

Used focused discussion groups, interviews (structured and semi-structured) and other anthropologic techniques to obtain narrative information that can be crucial in explaining quantitative results

Qualitative studies

6 year old girl fell from a chair onto an outstretched arm. X-rays show a supracondylar humeral fracture with anterolateral displacement of the proximal fracture fragment. What structure is at greatest risk of injury in this patient?

Radial nerve (causes wrist drop due to denervation of the extensor muscles and sensory loss over the posterior forearm/dorsolateral hand) Brachial artery, median nerve, and radial nerve all run anterior to the elbow and may be injured in a supracondylar humeral fracture. The radial nerve is the structure most likely to be injured with lateral displacement of the proximal fracture fragment Note: ulnar nerve runs immediately posterior to the medial epicondyle and can be injured with hyperflexion injuries (falling onto a flexed elbow), causing posterior displacement of the proximal humerus or fracture of the medial epicondyle

Refers to the practice of approximating the odds ratio and relative risk when conducting a case control study for rare diseases. According to the assumption, odds ratio approximates relative risk when disease incidence is low (<10%)

Rare disease assumption

62 year old woman with double vision, especially when reading at bedtime. Problem first started several months ago and was intermittent but has worsened over the last 2 weeks and bothers her constantly. Has difficulty chewing and swallowing foods. Feels better in the morning. Exam shows bilateral ptosis. There is weakness of adduction of the right eye and abduction of the left eye. Muscle strength is 4/5 in proximal muscles in the UE and LE. What changes involving the postsynaptic muscle most likely explains this patients symptoms?

Reduced amplitude of motor end plate potential Myasthenia gravis is an autoimmune disease that causes a decrease in the number of functional acetylcholine receptors within the NMJ. This reduced the number of postsynaptic cation channels that can open in response to acetylcholine, which reduces the amplitude of the motor end plate potential and prevents muscle fiber depolarization

62 year old man with poorly localized intermittent abdominal pain triggered by eating and slowly subsides over the ensuing several hours. Has last 10 pounds over the past 2 months. History of HTN and hyperlipidemia and has smoked 1PPD for 40 years. Abdomen is soft and contender. BP is 175/105. CT reveals one shrunken, atrophic kidney. Most likely condition patient has?

Renal artery stenosis Patient has multiple risk factors for atherosclerosis including advanced age, HTN, hyperlipidemia, and smoking. In such patients, chronic abdominal pain that is postprandial and accompanied by weight loss suffuses intestinal ischemia. These patients are at risk for atherosclerotic involvement of other major vessels such as renal artery stenosis. Marked unilateral kidney atrophy is suggestive of renal artery stenosis. It occurs in elderly individuals due to atherosclerotic changes in the arterial intima or in women of childbearing age due to fibromuscular dysplasia. HTN and an abdominal bruit are often present.

64 year old with painless loss of vision of right eye that began suddenly several hours ago and has persisted. History of CAD, AF< and T2DM. Fundoscopic exam reveals a cherry red macula & a pale retina. Most likely cause of patients loss of vision?

Retinal artery occlusion Acute and painless monocular vision loss is characteristic of central retinal artery occlusion. Vision loss includes the entire vision field and is often permanent. Fundoscopic findings include a pale retina (due to ischemia and edema) & a CHERRY RED macula (fovea and fovela are thin and have a separate blood supply from the choroid artery). Central retinal artery is a branch of the ophthalmic artery which arises from internal carotid artery. Atheroembolism and thromboembolism are most common causes. Predisposing conditions include AF, coronary artery stenosis and vasculitic diseases (GCA). Note: amaurosis fugal is a painless, TRANSIENT, monocular vision loss caused by a small embolus to the ophthalmic artery- doesn't last more than a few seconds

30 year old woman requesting to undergo bilateral tubal ligation a she is tired of using oral contraceptives and does not wish to have children. Patient has been married 10 years and describes the relationship as very stable. Most appropriate response to patient's request for tubal ligation?

Review the risks and benefits of tubal ligation and alternate birth control methods Consent of a married or unmarried significant other is not required for a patient to undergo any type of procedure, including sterilization. Physicians should counsel the patient regarding the risks and benefits of, and alternative to, any procedure or treatment

65 year old man with long history of smoking presents with SOB and chronic mild cough. Symptoms progressed over the last week and became suddenly worse today. Pulse ox shows 86% on room air and he has decreased breath sounds on the right. CXR shows hemithorax opacification on the right with deviation of the trachea toward the opacified side. Most likely cause of paeint's radiographic findings?

Right mainstream bronchus obstruction An obstructive lesion in a mainstem bronchus (such as central tumors in chronic smokers)can prevent ventilation of an entire lung, leading to obstructive atelectasis and complete lung collapse. Characteristic CXR findings include complete pulmonary opacification and deviation of the mediastinum toward the opacified lung Note: tension pneumothorax or a large pleural effusion will cause tracheal deviation away from the affected lung because the excess air or fluid pushes against the mediastinal structures

S. aureus causes ACUTE bacterial endocarditis with rapid onset of symptoms including shaking chills (rigors), high fever, dyspnea on exertion, and malaise. What can it cause in IV drug user?

Right sided endocarditis with septic embolization into the lungs

Histologic appearance of burkitt lymphoma tumor cells?

Round nuclei with basophilic cytoplasm containing prominent lipid vacuoles

22 year old woman with worsening dyspnea. Echocardiography shows a large, pedunculate mass in the left atrium. Histologic analysis will most likely reveal?

Scattered cells within a mucopolysaccharide stroma Myxomas are the most common primary cardiac neoplasm and usually arise with the LA. The tumors typically cause position dependent obstruction of the mitral valve leading to a mid-diastolic murmur and symptoms of decreased CO (dyspnea, syncope). Constitutional symptoms (fever, weight loss) may also be present due to the tumors producing cytokines such as IL-6. Fragments of the tumor can symbolize into the systemic circulation (stroke, acute limb ischemia). These tumors produce a large amount of VEGF contributing to angiogenesis, friability, and hemorrhaging. Histologically the tumors demonstrate scattered cells within a mucopolysaccharide stroma and abnormal blood vessels with hemorrhaging

Minimal chain disease is the most common cause of nephrotic syndrome in children. Systemic T cell dysfunction leads to the production of glomerular permeability factor when causes podocyte foot process fusion and decreases the anionic properties of the GBM. This loss of negative charge leads to ?

Selective albuminuria

Maternal rubella infection produces a low grade fever, a maculopapular rash with cephalocaudal progression, and posterior auricular and suboccipital lymphadenopathy. Most adult women patients develop polyarthritis and polyarthralgia as sequelae. Congenital rubella syndrome is associated with?

Sensorineural deafness, cataracts, and cardiac malformations (PDA)

Patient being considered for long term Amiodarone therapy. What should be tested being initiating this medication?

Serum TSH AmIODarone is 40% IODine by weight. It can cause hypothyroidism due to decreased production of thyroid hormone. Amiodarone can also cause hyperthyroidism due to increased thyroid hormone synthesis or destructive thyroiditis with release of preformed thyroid hormone.

4 year old boy with failure to thrive, SOB, and exercise intolerance. Cardiac exam shows pounding peripheral pulses and palpable thrill over the left upper sternal border. On auscultation, a continuous murmur is best heard over the same region. Surgeon should intervene on a derivative on what embryologic structure?

Sixth aortic arch The ductus arteriosus is derived from the 6th embryonic arch. A PDA causes left to right shunting of blood that an be heard as a continuous murmur over the left infraclavicular region. Indomethacin (PGE2 inhibitor) can be used to close PDA in premature infants, but surgical litigation is often necessary in older patients

24 year old girl with normal physical exam is found to have a factor VIII level 20% of normal. Most likely explanation of this patents bleeding tendency?

Skewed X inactivation To compensate for the additional X chromosome in women, one X chromosome is inactivated in each somatic cell. The section of the inactivated X chromosome is random, but a portion of women may have significantly skewed inactivation making them more susceptible to displaying manifestations of X linked diseases. Normally inactivation shows a 50:50 destruction between the two X chromosomes but some women have ratios that are 70:30 or higher & if these women are heterozygous for an X linked disorder such as hemophilia they are more susceptible to displaying manifestations of the disease.

Single most important intervention to lower the risk of MI?

Smoking cessation Smoking increases the risk of MI by accelerating atherosclerosis, stimulating vasoconstriction, and inducing a prothrombotic state

Major risk factors for esophageal squamous cell carcinoma?

Smoking, excessive alcohol consumption, pewwxistinng esophageal disease (achalasia, caustic injury), high temperature liquid ingestion, and intake of foods containing N-nitroso compounds. Note risk factors for adenocarcinoma are: barrett esophagus, GERD, obesity, tobacco use

Patient treated with a mu opioid analgesic for knee pain. Soon after administration she experiences new onset upper abdominal pain that makes it difficult for her to lie still. An adverse drug effect involving what location is most likely responsible for the patients current condition?

Smooth muscle cells Opioid analgesics can cause contraction of smooth muscles in the sphincter of Oddi leading to increased pressures in the common bile duct and gallbladder. Severe pressure increase can lead to biliary colic

60 year old man with overdose of amitriptyline. He is delirious and sees small animals running around in the corner of the room, appears flushed, and has a brief seizure and becomes unconscious. Both pupils are dilated and equally reactive to light. Skin and MM are dry. ECG shows QRS widening and QTc prolongation. He subsequently dies. Death is most likely related to?

Sodium channel inhibition Most common causes of death in patients with TCA overdose are cardiac arrhythmias and refractory hypotension. Inhibition of fast Na+ channels in cardiac myocytes (and the His Purkinje system) is thought to be the main underlying cellular event

45 year old with history of alcohol abuse and chronic hepatitis C. PE shows spider angiomata, gynecomastia, and distended abdomen. Diffuse muscular wasting is noted. What structure on a normal CT scan would have increased pressure in this patient?

Splenic vein Cirrhosis causes portal hypertension due to distortion of the hepatic vascular bed. The increased pressures in the portal venous system cause splenic vein hypertension and splenomegaly Note: DO NOT CONFUSE IVC with portal vein.

53 year old patient with a pruritic rash of worsening severity involving the posterior thighs for the past 3 weeks. She recently began an exercise program to lose weight and has been applying a topical cream to her thighs and buttocks after work outs. On PE there is an erythematous rash with blisters, ulcers, and weeping drainage involving the posterior thighs bilaterally. Most likely finding on skin biopsy?

Spongiosis Acute allergic contact dermatitis is caused by a type IV (delayed) hypersensitivity reaction to an antigen on the skin surface. Gross findings include erythematous, papulovesicular, weeping lesions. Histology is characterized by spongiosis, an accumulation os edema fluid in the intercellular spaces of the epidermis. With chronic exposure, lesions become less edematous, with thickening of the stratum spinosum (acanthosis) and stratum corneum (hyperkeratosis)

64 year old man with several lesions on his forehead. He has been a gardener for much of his life. On palpation the lesions have a rough, grainy texture. Biopsy reveals atypical keratinocytes with hyperkeratosis and parakeratosis. Patients lesions put him at greatest risk for?

Squamous cell carcinoma Actinic keratoses are small (usually less than 1cm), erythematous epidermal lesions with adherent scale that are the result of chronic sun exposure. Histologic findings include keratinocyte atypic, hyperkeratosis, and parakeratosis. A small percentage of these lesions process to invade squamous cell carcinoma so frequent monitoring is necessary

The most common GI disorder in patients with CF is pancreatic insufficiency. Mutations in the CFTR gene lead to thick, viscous secretions in the lumens of the pancreas resulting in obstruction, inflammation, and subsequent fibrosis. Clinical manifestations include?

Steatorrhea, failure to thrive, and deficiency of fat soluble vitamins. Note: loss of pancreatic tissue in these patients prevents them from developing pancreatitis

72 year old man presents after experiencing an episode of syncope. ECG confirms diagnosis of severe aortic stenosis. Two months later he presents with atrial fibrillation. CXR shows bilateral pulmonary edema. What hemodynamic change is most likely associated with his current condition?

Sudden decerase in left ventricular preload In patients with chronic aortic stenosis and concentric LVH, atrial contraction contributes significantly to LV filling. Loss of atrial contraction due to atrial fibrillation can reduce LV preload and cardiac output sufficiently to cause systemic hypotension. Decreased forward filling pressure of the LV can also result in a back up of blood in the left atrium and pulmonary veins, leading to acute pulmonary edema. As a result, cardioversion is indicated for acute atrial fibrillation in patients with severe aortic stenosis.

37 year old with difficulty sleeping. He has made numerous drips between USA and France for work. Experiences insomnia and daytime sleepiness for several days after these trips but then his sleep improves significantly. What hypothalamic nuclei is most likely responsible for this delayed improvement in his symptoms?

Suprachiasmatic Suprachiasmatic nucleus regulates circadian rhythm. It processes light information received from the retina and relays it to other hypothalamic nuclei and the pineal gland to modulate body temperature and the production of hormones such as cortisol and melatonin. Melatonin supplementation is recommended for treatment of insomnia associated with jet leg

Cells that are exposed to longer ischemic periods are found to have reduced number of ribosomes attached to the endoplasmic reticulum. This structural change is most likely to impair what cellular function?

Synthesis of cell membrane proteins The rough ER is covered with ribosomes and is involved in the transfer of proteins to the cell membrane and extracellular space. The RER is well developed in protein secreting cells. Ribosomes attach to the RER via the translocon, a protein complex containing ribophorins that bind the large 60S unit The smooth ER lacks surface ribosomes and functions in lipid synthesis, carbohydrate metabolism ,and detoxification of harmful substances. Note: free ribosomes remain floating in the cytosol throughout protein synthesis and are responsible for translating proteins found within the cytosol, nucleosol, peroxisome matrix, and nuclear encoded mitochondrial proteins.

Patient with a one week history of dysuria and flank pain develops chills, high fever, and confusion. BP is 80/40 and HR is 120. Extremities are warm and breath is rapid and shallow. Substance most likely responsible for patient's current symptoms?

TNF-alpha TNF-alpha (produced by activated macrophages) is one of the cytokines that induces the systemic inflammatory response. In high concentrations, TNF-alpha causes symptoms of septic shock and cachexia. Other cytokines responsible for inducing the systemic inflammatory response include IL-1 and IL-6.

Patient undergoing treatment with GnRH agonist for prostate cancer with bone metastasis. He reports discomfort at the anterior chest wall. PE shows bilateral mildly tender enlargement of the breast tissue behind the nipple areolar complex. What medication if ignited early could have most effectively prevented this patients breast symptoms?

Tamoxifen Gynecomastia is development of glandular breast tissue in males and is commonly seen in men receiving androgen deprivation therapy for prostate cancer. Tamoxifen inhibits the effect of estrogen in breast tissue and can reduce the risk of gynecomastia in these patients. Note: Bicalutamide- testosterone receptor antagonist used to treat prostate CA (would worsen gynecomastia) Danazol- synthetic steroid with androgenic and antiestrogenic effects (would worsen cancer but improve gynecomastia)

Pill induced esophagitis is commonly seen with?

Tetracycline antibiotics, potassium chloride, bisphosphonates

Thiazide and loop diuretics cause significant volume depletion, activating the RAAS system, which can lead to hypokalemia and metabolic alkalosis (via aldosterone). Difference between these two classes on calcium levels?

Thiazide diuretics are more likely to cause hypercalemia (and HYPONATREMIA), while loop diuretics cause hypocalcemia

70 year old woman with acute right hip after a fall. She was walking in her house when she tripped on a floor rug and fell landing on her hip. Radiographs reveal a right femoral neck fracture. What change in bone structure is most likely responsible for the patients condition?

Trabecular thinning with fewer interconnections Patient has a facility fracture (low intensity trauma that wouldn't cause a fracture in normal bone) likely due to osteoporosis (her risk factors include age, postmenopausal, caucasian). Initially in osteoporosis, bone loss predominately affects trabecular bone, leading to trabeualr thinning and perforation with loss of interconnecting bridges. Over time, cortical bone, which composes most of the appendicular skeleton, also becomes involved. The neck of the femur has components of both trabecular and cortical bone and is a common site of osteoporotic fracture.

8 year old boy with mass in right mandible that has grown rapidly over the past few months. Patient and his family immigrated to the US from East Africa. Exam shows large tumor in right mandible and palpable regional lymphadenopathy. Histopathologic exam shows diffuse infiltrate of lymphoid cell and numerous mitotic figures. Interspersed macrophages surrounded by clear spaces are also seen. The gene translocated in these lymphoid cells produces a protein that is responsible for what function?

Transcription activation Patient likely has endemic African form which is associated with EBV. Burkitt lymphoma is characterized by aggressive rapid growth and a 'starry sky' appearance. Translocation of the c-Myc oncogene on the long chromosome of chromosome 8 with the Ig heavy chain region on chromosome 14 produces nuclear phosphoprotein (c-Myc) that functions as a transcription factor

Investigators isolate mature fibroblasts from the skin of a healthy volunteer. The fibroblasts are infected with a combination of retroviruses containing certain genes that encode human proteins. These modified fibroblasts are subsequently cultured and the colonies obtained from the cultures are found to contain cells rambling pluripotent embryonic stem cells. The genes transferred by the retroviruses in this experiment most likely code for what proteins?

Transcription factors Cellular differentiation is determined by the transcription factor milleu within an individual cell. All nucleated cells in humans possess the entire genome, but tissue specific specific transcription factors allow for only the expression of genes that are relevant for a particular cell type. Transcription factors can also induce cellular dedifferentiation, causing terminally differentiated cells to revert to less specialized forms.

65 year old man with acute onset of severe headache and dizziness. Noncontrast head CT reveals an acute hemorrhage in the cerebellar vermis without mass effect or midline shift. What neurologic finding is most likely to be present during patients PE?

Truncal ataxia Aucte lesions to the cerebellar vermis typically cause truncal and gait ataxia due to impaired maculation of the medial descending motor systems. Involvement of the lower vermis and the flocculonodular lobe causes vertigo/nystagmus due to dysregulation of the vestibular nuclear complex Note: limb dysmetria is associated with lesions affecting the cerebellar hemispheres due to involvement of the lateral descending motor systems; lesions of the cerebellar hemispheres can also cause intention tremor (tremor during goal oriented activity)

44 year old with longstanding history of heartburn presents with worsening substernal burning, particularly at night. Discomfort has been less responsive to OTC antacids. Also now has significant substernal pain after swallowing food, but swallowing doesn't result in cough, choking, or sensation of food getting stuck. Recent worsening of patient's symptoms is most likely related to?

Ulceration New onset odynophagia (painful swallowing) in the setting of chronic GERD should raise suspicion for erosive esophagitis with esophageal ulcers. Diagnosis is made by upper endoscopy.

Patients with diabetes insipidus are unable to concentrate their urine in response to dehydration. Following desmopressin administration during the water desmopressin test what do you expect for the different forms of DI?

Urine osmolality increases to normal in central DI and doesn't change with complete nephrogenic DI

67 year old with 2 week history of decreased vision in right eye that he describes as 'blurry' and 'distorted'. Has been having vision problems over the past year making it difficult to drive and causing him to have to use a bright light to read the newspaper. No history of HTN or DM. Ophthalmologic exam of the right eye shows a grayish discoloration of the macula with areas of adjacent hemorrhage. What should be specifically targeted in this patients condition?

VEGF Wet age related macular degeneration is characterized by retinal neovascularization due to increased VEGF levels. Patients typically have acute vision loss and metamorphosia (distortion of straight lines) with fundoscopy showing a gray-green sub retinal membrane and/or subretinal hemorrhage. Treatment includes smoking cessation and VEGF inhibitor therapy (ranibizumab, bevacizumab) Dry age related macular degeneration is characterized by gradual vision loss in one or both eyes and can cause difficulty with driving/reading. Condition results from chronic oxidative damage to the retinal pigment epithelium and choriocapillaris leading to subretinal inflammation with abnormal ECM formation (BM thickening, confluent drusen). Appears on fundoscopy as subretinal drusen deposits with pigment abnormalities. Progressive ECM accumulation leads to retinal hypoxia stimulating VEGF production and leads to wet age related macular degeneration.

Dissection between the esophagus and the right crus of the diaphragm causes greatest risk of injury to what structure?

Vagus trunks Esophagus passes through the esophageal hiatus of the diaphragm at the level of T10. This opening also contains the anterior and posterior vagal trunks. Esophageal branches of the left gastric vessels also pass through this opening. Note: T8 Vena caval foramen- inferior vena cava (passes though the central tendon of the diaphragm) & terminal branches of right phrenic nerve pass through vena canal foramen T 12 Aortic hiatus (formed by median arcuate ligament of the diaphragm)- aorta, thoracic duct, azygous vein

Why is Heparin safe to use in pregnancy and Warfarin isn't?

Warfarin is a lipophilic molecule (crosses placenta) while Heparin sulfate is a charged molecule and is water soluble ( doesn't cross placenta)

Functional hypothalamic amenorrhea results from the loss of pulsatile GnRH from the hypothalamus and is caused by?

Weight loss, strenuous exercise, systemic illness, or abnormal eating habits. Loss of cyclic gonadotropin release leads to a decreased in LH and FSH secretion from the pituitary which causes low circulating estrogen levels. Its caused by reduced circulating leptin levels as a result of diminished adipose tissue stores.

The I band is the region of the sarcomere containing the Z line and only those sections of the thin (actin) filament that do not overlap with the thick (myosin) filament. How does the length of this region change with relaxation and contraction?

Width of the I band lengthens during muscle relaxation and shortens during contraction Note: the H band (only containing thick filaments) also lengthens as the sarcomere relaxes.

38 year old recently homeless woman and her 35 year old boyfriend present seeking care for her pregnancy at 27 weeks. She brings her 19 year old son with cerebral palsy who hasn't had consistent medical care. Patient explains that family was recently evicted from their apartment for failure to pay rent after her boyfriend lost his job. Which of these individuals is eligible for Medicare coverage?

Woman's disabled son Medicare is a federal socialized medical insurance program that covers individuals 65 and older who have a work history (and paid taxes) and younger individuals with disabilities, ALS, and end stage renal disease Note: mediCAID is a state run medical insurance program that covers the homeless, undocumented immigrants, pregnant women, and low income families

DNA laddering (appearance of DNA fragments in multiples of 180 base pairs on get electrophoresis) is a sensitive indicator of?

apoptosis (programmed cell death)

Antibodies that are specific for SLE?

ds-DNA, however they are only present in 60% of cases so absence of anti-dsSNA doesn't rule out the diagnosis Note: Anti-Sm (anti-smith) are also specific for SLE

In certain cells, glycolysis of a single glucose molecule always yields pyruvate but sometimes generates no net ATP. What cell is this?

erythrocyte 2,3-BPG decreases hemoglobins affinity for oxygen. Therefore, in the presence of lower blood oxygen concentrations, higher 2,3-BPG levels within erythrocytes enable increased oxygen delivery in the peripheral tissues. 2,3-BPG is produced from 1,3-BPG by the enzyme biphosphoglycerate mutase. This reaction bypasses ATP generating steps of glycolysis, causing no net gain in ATP.

Cleavage of proinsulin by endopeptidases in pancreatic beta cell secretory GRANULES yields what two molecules that are stored in the granules until they are secreted in equimolar amounts?

insulin and c peptide

Expected urine values of pH, HCO3-, H2PO4 in a patients with diabetic ketoacidosis?

pH decreased HCO3- decreased H2PO4- increased Urinary acid excretion occurs primarily in the form of NH4+ and titratable acids (H2PO4-). In metabolic acidosis, urinary pH decreases due to increased excretion of free H+, NH4+, and H2PO4-. Bicarbonate is completely reabsorbed from the tubular fluid in acidotic states.

46 year old man with sudden onset right sided sharp chest pain and SOB that began 2 hours ago. Right leg is swollen and tender to palpation. What ABG findings is most likely present in this patient?

pH- 7.51 PaCO2- 31 PaO2- 66 Sudden onset SOB and CP in combination with unilateral leg swelling is strongly suggestive for PE. Significant PE causes hypoxemia due to increased dead space ventilation and worsening ventilation/perfusion mismatch. Hyperventilation leads to hypocapnia with respiratory alkalosis (increased pH, decreased pCO2) which is the typical presentation of acute PE.

24 year old woman is 19 weeks pregnant. US reveals a male fetus with bilaterally enlarged fetal kidneys with diffuse small cysts. Amniotic fluid volume is very low, but there are no other abnormalities. Which will most likely be present in the newborn after delivery?

respiratory distress In its most severe phenotype, autosomal recessive PCKD can be detected on a prenatal sonogram alone with oligohydramnois. Potter sequences (flattened facies, limb deformities, and pulmonary hypoplasia) is caused by oligiohydramnios and is associated with high mortality. Less severe phenotypes often present with hepatic complications (hepatomegaly, portal HTN) and HTN during childhood or early adulthood. ARPKD is caused by mutation in PKHD1 gene that codes for fibrocystin (present in kidney and liver). Clinical findings are renal insufficiency, nephromegaly, HTN. Note: cerebral aneurysms are associated with autosomal DOMINANT PKD- NOT ARPKD.


Kaugnay na mga set ng pag-aaral

Enlightenment and American Revolution

View Set

Chapter 19: PrepU - Nursing Assessment: Hematologic Function

View Set

Health Online- GROWTH, DEVELOPMENT, AND SEXUAL HEALTH-Sexually Transmitted Infections, HIV, and AIDS

View Set

Chapter 41: The animal body and principles of regulation

View Set